Está en la página 1de 229

LENGUA

CASTELLANA Y
LITERATURA
5.º ESO

Solucionario

Juan Carlos Pantoja


Rivero Laura Espí Jimeno
Beatriz González Gallego
Esperanza Mateos Donaire
Emilio Sales Dasí

MADRID · BUENOS AIRES · CARACAS · GUATEMALA · LISBOA · MÉXICO · NUEVA YORK


PANAMÁ · SAN JUAN · BOGOTÁ · SÃO PAULO · AUCKLAND · HAMBURGO · LONDRES
MILÁN · MONTREAL · NUEVA DELHI · PARÍS · SAN FRANCISCO · SÍDNEY · SINGAPUR
SAINT LOUIS · TOKIO · TORONTO
LENGUA CASTELLANA Y LITERATURA
Solucionario 3.º ESO

No está permitida la reproducción total o parcial de este libro, ni su tratamiento informático, ni la


transmisión de ninguna forma o por cualquier medio, ya sea electrónico, mecánico, por fotocopia,
por registro u otros métodos, sin el permiso previo y por escrito de los titulares del Copyright.
Diríjase a CEDRO (Centro Español de Derechos Reprográficos, www.conlicencia.com) si necesita
fotocopiar o escanear algún fragmento de esta obra.

Nota: Este libro se atiene al artículo 32 del derecho de cita de la Ley de Propiedad Intelectual de 1996 (RDLeg
1/1996 de 12 de Abril).

Derechos reservados © 2015, respecto a la primera edición en español, por:

McGraw-Hill/Interamericana de España, S.L.


Edificio Valrealty, 1.a planta
Basauri, 17
28023 Aravaca (Madrid)

Código: 978-00-094-2045-0
Depósito legal: M-18149-2015

Dirección general: Álvaro García


Equipo editorial: Marilia Blanco y Cristina Núñez
Diseño de cubierta: Paula Requena
Diseño interior: Equipo de diseño de McGraw-Hill
Fotografias: 123RF y archivo McGraw-Hill
Maquetación: Diseño y Control Gráfico, S.L.U.
Impreso en:

IMPRESO EN ESPAÑA-PRINTED IN SPAIN


ÍNDICE

UNIDAD PÁGINA

Unidad 1. Los cimientos de tu lengua .................................................................................................................................................... 4

Unidad 2. El mágico poder de las palabras ......................................................................................................................................... 13

Unidad 3. Última hora ................................................................................................................................................................................... 21

Unidad 4. Libertad de expresión...............................................................................................................................30

Unidad 5. Háblame de ti....................................................................................................................................................40

Unidad 6. En la variedad está el gusto...............................................................................................................................49

Unidad 7. Amar, luchar, rezar… ................................................................................................................................................................. 59

Unidad 8. Melibeo soy.......................................................................................................................................................69

Unidad 9. Coged de vuestra alegre primavera..................................................................................................................80

Unidad 10. Aquellos locos tan ingeniosos.........................................................................................................................89

Unidad 11. Entre pícaros y poetas.....................................................................................................................................98

Unidad 12. Todos a una...................................................................................................................................................107

Anexo. Las tipologías textuales en la lengua oral............................................................................................................117

Guías de lectura.........................................................................................................................................................118

Lengua castellana y Literatura. 1.º ESO. Solucionario 3


1 LOS CIMIENTOS DE TU LENGUA

APERTURA DE UNIDAD res vivos en estado salvaje (sin condicionar) y conoce a John
el Salvaje, nacido por el mé todo reproductivo natural. Este
1. Este fragmento explica en qué consiste el cruel espec- personaje visita el «Mundo Feliz» de Bernard y plantea su
táculo que da nombre a la obra. También en este
fragmen- to se explican las razones por las que ha
surgido dicho espectáculo y su finalidad. ¿Quién ha
ideado los Juegos y para qué? ¿Conoces algún castigo
semejante que se haya llevado a cabo en el mundo real
en alguna época?
El Capitolio, que representa a los opresivos gobernantes de
los doce distritos. Estos distritos son los diferentes sectores
en los que se divide y organiza una sociedad clasista. La é lite
de la sociedad vive en el Capitolio.
Los gobernantes del mismo han establecido estos macabros
juegos (que de lú dicos no tienen nada) para recordar a los dis-
tintos distritos que todo intento de sublevarse será frenado
por el Capitolio, como ya ocurrió en otro momento de la histo-
ria. El intento frustrado de los distritos de emanciparse del
Capitolio es un tema tabú pero tiene un recordatorio simbó lico
en los Juegos del Hambre.
Hay algunos ejemplos histó ricos reales en que los invasores
han oprimido al pueblo invadido como, por ejemplo, los roma-
nos que sometieron al martirio a los cristianos. También eran
juegos macabros en los que los cristianos eran arrojados a
las bestias para divertir al pueblo romano.
2. Te proponemos que veas algunos fragmentos de la
versión cinematográfica de esta obra, por ejemplo, el
principio, hasta el momento en que comienzan los
Juegos propia- mente dichos. Después puedes entablar
un debate con tus compañeros en torno a estas
preguntas:
a) Analiza el tipo de sociedad que retrata la escritora.
¿En qué se parece y en qué se diferencia de la tuya
propia?
b) ¿Qué te parece el planteamiento de esta obra?
c) ¿Qué opinas sobre la adaptación al cine de esta parte
de la obra? ¿Crees que existe algún aspecto
mejorable?
d) ¿Qué te parece la idea de emplear un reality show
como castigo?
Respuesta libre.
3. Busca más información sobre la autora de esta obra y su
producción novelística. La idea de una sociedad futura
que plantea su novela recuerda a la que retrata Aldous
Huxley en Un mundo feliz y a una película que aborda los
reality shows: El show de Truman. Busca información
sobre ambas y establece similitudes y diferencias.
Un mundo feliz es la novela más brillante del escritor britá nico
Aldous Huxley. La obra recrea una sociedad del futuro en la
que la tecnología ha avanzado sobre todo en dos direcciones:
la reproducció n humana y la hipnopedia. La combinació n de
ambas da como resultado una sociedad de autó matas deshu-
manizados. El ser humano vive en un mundo mediocre y sin
pasiones, en donde la química ayuda a controlar los estados de
á nimo y donde no hay contacto físico, ni siquiera para procrear.
Esta forma de vida puede parecer en principio una utopía, pero
al final resulta una caricatura de un mundo excesivamente tec-
noló gico y frío. Es, en realidad, una burla y una crítica al
hom- bre moderno. Bernard Max, por su inteligencia, no
responde al condicionamiento psicoló gico a que son
sometidos todos los humanos, de manera que empieza a
pensar por sí mismo y se convierte en un inadaptado social
(un antisistema).
La trama se complica cuando Bernard visita la reserva, junto
con su frívola compañ era, Lenina, lugar en el que habitan se-
LOS CIMIENTOS DE TU 1
punto de vista: la felicidad de este mundo es artificial, una («cabeza») es un apelativo burlesco. Se aprecia en esta cita:
LENGUA
mentira del gobierno, que controla a sus habitantes. Y de cuestión en cuestión, don Eufrasio llegó al punto de
Bernard acaba exiliá ndose y John finalmente sucumbe a los partida necesario para dar un solo paso en firme. Todo esto
placeres del «Mundo Feliz» y acaba suicidá ndose. le ocupó muchos meses, que fueron dilatando el plazo de mi
Entre otros temas, el libro ataca la producció n del muerte.
ensamblaje en línea como humillante, la liberació n de la
moral sexual calificá ndola como una afrenta contra el amor
y la familia, el uso de esló ganes, el concepto de un gobierno
centralizado, y el uso de la ciencia para controlar los
pensamientos y acciones de la gente. Huxley ataca a la
sociedad consumista y capita- lista, una sociedad vacía y
frívola como la de Los Juegos del Hambre, que se deja
controlar y anular por pura comodidad y por una terrible
falta de valores.
El show de Truman es una película estadounidense dirigida
por Peter Weir en la que Jim Carrey encarna al
protagonista y que se estrenó en 1998.
El argumento de esta película gira en torno al tema de un
programa de telerrealidad titulado The Truman Show, su
pro- tagonista, Truman Burbank, vive ajeno a su condició n
de protagonista de este reality show. Las personas que é l
ve, incluso sus familiares, no son quienes é l piensa, sino
actores de televisió n. Ha sido creado para entretener al
pú blico (nació de una madre de alquiler y destinado a vivir
para esta serie). Los autores del guion han manejado
siempre los hilos de su vida, hasta que un hecho accidental
hace sospechar a Tru- man de todo cuanto le rodea.
Finalmente descubre la verdad y, gracias a su astucia, logra
escapar del tirá nico mundo de cartó n-piedra en el que ha
vivido desde siempre.

VIVE LA LECTURA
Comprensión lectora
1. ¿Quién protagoniza este fragmento? ¿Te parece original
el planteamiento del cuento? ¿Por qué?
Un insecto, concretamente una mosca con la peculiar carac-
terística de que tiene inteligencia y ha sido instruida por un
sabio.
2. ¿En qué persona se narra la historia? Indica en qué
párra- fos habla el personaje principal.
Se narra en primera persona, pero no se trata de una primera
persona autobiográ fica sino de un narrador testigo. La
mosca habla en los pá rrafos dos, tres y cuatro, y en estos
pá rrafos es la mosca la que utiliza la primera persona
autobiográ fica.
3. Resume su historia vital.
La mosca nació en la biblioteca del sabio Macrocé falo, quien
la capturó en su infancia y la sometió a mú ltiples
experimentos. Los resultados de todos ellos contradecían las
teorías del sabio y este quiso matar a la mosca por despecho,
pero no encontró el valor necesario y, finalmente, le permitió
la vida y la aleccionó .
4. Explica todos los datos que se ofrecen en la lectura
sobre el sabio Macrocéfalo y describe cómo te lo
imaginas.
Don Eufrasio Macrocéfalo se describe por la mosca má s
como un filó sofo que como un sabio. Lo que parece claro en
este fragmento es que es un hombre indeciso y algo
orgulloso.
5. ¿Crees que este fragmento constituye una crítica o una
alabanza a los sabios? Aporta alguna afirmación del
texto que sirva de ejemplo a tu razonamiento.
Es una crítica a algunos sabios que parecen tener muchos co-
nocimientos pero su ciencia no va má s allá del puro
discurrir. De hecho el apellido Macro- («grande») -céfalo
6. Señala si las siguientes afirmaciones te parecen verda- Batracomiomaquia y la Mosquea, que aluden a los
deras o falsas: batracios y las moscas.
a) Macrocéfalo es el nombre de la mosca que protago-
niza este relato.
b) Don Eufrasio enseña a hablar a la mosca porque se
siente muy solo.
c) La mosca nació en libertad, pero don Eufrasio la atra-
pó para hacer experimentos con ella.
d) La mosca ha leído muchos de los libros de la
biblioteca y es políglota.
e) Don Eufrasio pretendía atravesar la mosca con una
agu- ja, pero sus pensamientos filosóficos le
apartaron de tal decisión.
a) Falsa. c) Verdadera. e) Verdadera.
b) Falsa. d) Verdadera.
7. ¿En qué consiste la teoría cartesiana que Macrocéfalo
que- ría comprobar? ¿Por qué no logra demostrar dicha
teoría?
¿Cómo repercute el hecho de que no pueda comprobar
dicha teoría en la vida de la protagonista?
En que los animales son como má quinas. Porque la propia
inteligencia que empieza a mostrar la mosca, aprendiendo
todo lo que Macrocéfalo le enseñ a, corrobora lo contrario. Su
inteligencia la aleja de la má quina y la acerca al ser humano.
Lo que la salva es que, al parecerse a los humanos en este
aspecto, a don Eufrasio le aborda una duda moral: si tiene o
no derecho a matar a la mosca.

Vocabulario
8. ¿Qué quiere decir la mosca cuando afirma que es
«políglo- ta»? Busca la etimología y el significado de esta
palabra. La palabra políglota viene del griego poliglotos
formada por polis («muchas») y glotos («lengua»). Se refiere
a una persona
–en este caso una mosca– que habla y domina varios idiomas.
9. Explica el significado de la expresión «meter baza» y
aporta sinónimos tanto cultos como coloquiales. Luego,
escribe una oración con cada uno.
Meter baza significa intervenir en una conversació n ajena de
modo inesperado. Sinó nimos cultos: discrepar, interrumpir,
intervenir o entrometerse. Sinó nimo coloquial: meter las
narices.
10. Busca en el diccionario la definición de estas palabras.
Escribe con ellas un pequeño texto fantástico cuya pro-
tagonista sea una hormiga.
quinqué canija baldada
ascendientes suplicio entrañas
Quinqué: lá mpara de mesa alimentada con petró leo y provista
de un tubo de cristal que resguarda la llama.
Ascendientes: padre, madre o cualquiera de los abuelos, de
quienes desciende una persona.
Canija: débil y enfermiza.
Suplicio: lesió n corporal, o muerte, infligida como castigo.
Baldada: cansada, fatigada.
Entrañas: ó rganos contenidos en las principales cavidades del
cuerpo humano y de los animales. Lo má s íntimo y esencial
de un asunto. Lo má s oculto y escondido.

Investigación y redacción
11. Busca información sobre el género de la epopeya y
sobre la Ilíada, a la que hace referencia la mosca. Explica
el para- lelismo que se establece entre esta y la
6 Lengua castellana y Literatura. 3.º ESO. Solucionario
La epopeya es un subgé nero de la narrativa escrito en verso
que cuenta la historia de pueblos y naciones. Las má s antiguas
son la mesopotá mica (Epopeya de Gilgamesh) y las grecola-
tinas (Ilíada, Odisea y Eneida).
La Ilíada es una epopeya griega atribuida al poeta Homero,
que vivió hacia el siglo VIII a. C. Es un extenso poema inte-
grado por má s de quince mil versos y cuya acció n
desarrolla una serie de acontecimientos en torno al asedio
de Troya por parte de los griegos.
El héroe que protagoniza el relato es Aquiles, que abandona el
combate por estar en desacuerdo con el reparto de un
botín. Este solo vuelve a la lucha para vengar la muerte de
su amigo Patroclo, al que ha matado el príncipe troyano
Héctor.
12. En este cuento, Clarín nos relata cómo el narrador (un
es- tudioso que había ido a consultar la biblioteca del
sabio Eu- frasio Macrocéfalo) saca a la mosca de
excursión al campo. Inventa los acontecimientos que
tienen lugar durante dicha excursión y construye así un
final de unas veinte líneas para la historia.
Respuesta libre.
13. ¿Crees que este cuento pertenece al subgénero
narrativo de la fábula? Busca información sobre dicho
género y ofrece tu respuesta razonada.
No. Aunque está protagonizada por un animal y este comparte
con los seres humanos la capacidad para el diá logo y la
inte- ligencia, no tiene una finalidad ejemplar o
ejemplarizante y carece de una moraleja.

ESTUDIO DE LA LENGUA
1. Señala en el siguiente microrrelato qué fonemas están
representados por varias letras:
Está s en casa, y es de noche, apagas la ú ltima luz. Qué extra-
ñ o: de pronto todo desaparece.
Pedro UGARTE
/k/  c (+ a, o, u), k, qu (+ e, i)
/g/  (+ a, o, u), gu (+ e, i), w
/z/  c (+ e, i), z
2. En este otro microrrelato, se menciona una clepsidra y
un cíclope. Identifica el significante, el significado y el
referen- te (aporta una imagen) de cada signo
lingüístico.
Perseguido por tres libé lulas gigantes, el cíclope alcanzó el
centro del laberinto, donde había una clepsidra. Tan sediento
estaba que sumergió irreflexivamente su cabeza en las
aguas de aquel reloj milenario. Y bebió sin mesura ni
placer. Al apu- rar la ú ltima gota, el tiempo se detuvo para
siempre.
Javier PUCHE
Significante: c-l-e-p-s-i-d-r-a.
Significado: reloj de agua.
Referente:

Lengua castellana y Literatura. 3.º ESO. Solucionario 5


Significante: c-i-c-l-o-p-e.
c) bastante decepcionado
Significado: gigante de la mitología griega con un solo ojo.
d) después de las siete
Referente:
e) llueve torrencialmente
a) Nominal. c) Adjetival. e) Verbal.
b) Adjetival. d) Adverbial.
6. Di a qué tipo de sintagma pertenecen los que aparecen
en negrita en los siguientes refranes:
a) Más rápido se coge al mentiroso que al cojo.
b) Al que a buen árbol se arrima buena sombra le cobija.
c) Más vale pájaro en mano que ciento volando.
a) Adverbial y preposicional.
b) Nominal.
c) Preposicional.
7. Indica la modalidad de las siguientes oraciones:
a) Dile a tu hermana que venga a verme.
3. Lee las siguientes frases célebres y realiza el análisis mor- b) ¿Podríamos comer lentejas hoy, por favor?
fológico de las palabras resaltadas en negrita. Aporta c) Los libros son el mejor amigo de los aventureros.
todos los datos que conozcas (tipo de palabra, género, d) No toques esa piedra de ahí.
número, grado, tiempo, etc.). e) ¿Quién de vosotras tiene el retrato? ¿Quién?
a) La diferencia entre estupidez y genialidad es que la a) Apelativa, exhortativa.
genialidad tiene sus límites (Albert Einstein). b) Apelativa, exhortativa. Aunque tiene forma de interroga-
b) Recordar es fácil para el que tiene memoria. tiva, en el fondo es un ruego.
Olvidar es difícil para el que tiene corazón (Gabriel c) Enunciativa afirmativa.
García Márquez).
d) Apelativa, exhortativa.
c) La raza humana tiene un arma verdaderamente efi-
e) Interrogativa.
caz: la risa (Mark Twain).
d) No entres allá de donde no puedas libremente salir 8. Escucha la canción de Pablo Alborán Solamente tú y ana-
(Mateo Alemán). liza la modalidad de cada oración.
• Regálame tu risa (exhortativa).
e) La educación es el arma más poderosa que puedes
usar para cambiar el mundo (Nelson Mandela). • Enséñame a soñar (exhortativa).
a) Diferencia: sustantivo comú n, abstracto, femenino, • Con solo una caricia me pierdo en este mar
singu- lar. Genialidad: sustantivo comú n, abstracto, (enunciativa afirmativa).
femenino, singular. Tiene: verbo de la segunda • Regálame la estrella, la que ilumina esta noche, llena de
conjugació n, tercera persona del singular del presente de paz y de armonía, y te regalaré mi vida (exhortativa).
indicativo. Voz activa. Sus: determinante posesivo, tercera • Haces que mi cielo vuelva a tener ese azul, pintas de
persona del plural. colores mis mañanas solo tú (enunciativa afirmativa).
b) Para: preposició n. El: determinante artículo determina- • Navego entre las olas de tu voz y tú, y solamente tú, haces
do, masculino, singular. Difícil: adjetivo calificativo de que mi alma se despierte con tu luz (enunciativa
una terminació n, singular. Que: conjunció n. Corazón: afirmativa).
sustantivo, comú n, concreto, contable, individual, mascu-
lino, singular. (Aquí está usado en sentido figurado, así • Enseña tus heridas y así las curará. Que sepa el mundo
que es abstracto porque se refiere a sentimientos, que es entero que tu voz guarda un secreto. No menciones tu
abstracto). nombre que en el firmamento se mueren de celos (exhor-
tativa).
c) Humana: adjetivo calificativo, femenino, singular. Ver-
daderamente: adverbio de modo. Eficaz: adjetivo cali- • Tus ojos son destellos, tu garganta es un misterio
ficativo de una terminació n, singular. (enuncia- tiva afirmativa).
d) Allá: adverbio de lugar. No: adverbio de negació n. Sa- 9. Escribe una carta a alguien a quien aprecies mucho. Uti-
lir: verbo de la tercera conjugació n. Forma no personal: liza oraciones de todas las modalidades.
infinitivo. Respuesta libre.
e) Educación: sustantivo comú n, abstracto, femenino, sin-
gular. Poderosa: adjetivo calificativo, femenino, singular. EL TALLER DE LAS PALABRAS
4. Crea un consejo para esquivar la soledad en forma de
oración contundente como las de la actividad 3. Realiza 1. Descompón en monemas las siguientes palabras:
su análisis morfológico completo. reinserción idealizar pájaros
Respuesta libre. juguetón arboledas
5. Indica de qué tipo son los siguientes sintagmas: lavaremos abrasador aniñado
a) el lunes de la próxima semana • re (morfema derivativo, prefijo) -inser- (lexema) -ción
b) cansado de todo (morfema derivativo, sufijo)
• idea- (lexema) -al- (morfema derivativo, sufijo) -izar
(mor- fema derivativo, sufijo)
LOS CIMIENTOS DE TU 1
LENGUA
• pájar- (lexema) -o-s (morfemas flexivos) Puleva AMPA ICONA FM Malena
• juguet (lexema) -ón (morfema derivativo, sufijo aumen- BBVA láser OMS sida Feve
tativo)
• árbol- (lexema) -eda (morfema derivativo, sufijo)
• lav- (lexema) -aremos (morfemas flexivos)
• a- (morfema derivatvo, prefijo) -bras- (lexema) -ador (mor-
fema derivativo, sufijo)
• a- (morfema derivativo, prefijo) -niñ- (lexema) -ado
(mor- fema derivativo, sufijo)
2. Forma nuevas palabras a partir de las que te damos aña-
diendo estos sufijos: -nte, -ero, -dor, -azo, -on.
gol pesca hablar comer
ropa bala acoger casa
presidir sal
• golazo • pescador • hablador • comedor
• ropero • balazo • acogedor • casero
• presidente • salero
3. ¿Qué sufijos de la actividad anterior cambian la categoría
gramatical de la palabra a la que se añaden? Pon otros
ejemplos.
-dor (orar, orador), -nte (pender, pendiente).
4. Añade uno de estos prefijos a las siguientes palabras
para formar su antónimo: i-, in-, anti-, des-.
responsable congelante descriptible
habitado empleo
oportuno confiado lógico
• irresponsable • anticongelante • indescriptible
• deshabitado • desempleo • inoportuno
• desconfiado • iló gico
5. Forma palabras con los lexemas y morfemas derivativos
siguientes, indicando qué procedimiento has seguido en
cada caso.
boca- compra- -gras- -legal -ista
des- para- -alm- -tensión -ado
hipo- a- -caid- -añ- -ar
cumple- en- -grand- -venta -ada
i- perr- -calle -ito -era

• hipotensió n • ilegal • desalmado


• compraventa • perrito • bocacalle
• cumpleañ era • agrandar • paracaidista
• engrasada
6. Escribe juntos o separados los siguientes términos unidos
por composición:
cartón + piedra para + choques
perro + lobo recién + casado
guía + catálogo coche + patrulla
medio + día sofá + cama
• cartó n piedra • perro lobo
• catá logo guía • mediodía
• parachoques • recié n casado
• coche patrulla • sofá cama
7. Explica qué significan estas siglas y acrónimos:

Lengua castellana y Literatura. 3.º ESO. Solucionario 7


1 LOS CIMIENTOS DE TU LENGUA
• Puleva (acró nimo de Pura Leche de Vaca) a) Déjame que te diga una cosa: así no llegarás a
• AMPA (siglas de Asociació n de Madres y Padres de ninguna parte, Óscar.
Alum- nos) b) Los íberos llegaron a la península ibérica antes de la
• ICONA (acró nimo de Instituto Nacional para la invasión romana.
Conserva- ció n de la Naturaleza)
• FM (siglas de Frecuencia Modulada)
• Malena (acró nimo de Magdalena o María Elena)
• BBVA (unió n de las siglas BBV, Banco Bilbao-Vizcaya
y
Argentaria)
• lá ser (acró nimo compuesto por las siglas: light
amplifica- tion by stimulated emision of radiation,
es decir, amplia- ció n de la luz por emisió n estimulada
de radiació n)
• OMS (siglas de Organizació n Mundial de la Salud)
• sida (siglas de síndrome de inmunodeficiencia
adquirida)
• Feve (acró nimo de Ferrocarriles de Vía Estrecha)

EL RINCÓN DE LA NORMA
1. Añade tilde donde sea necesario. Si dudas, consulta
el diccionario.
Corrige
dijeron medula damelo margen arbol
telegrafo jamas Oscar iran cogi
carcel sali ambar patena ambiguo
• dijeron • mé dula • dá melo • margen • á rbol
• telégrafo • jamá s • Ó scar • irá n • cogí
• cá rcel • salí • á mbar • patena •
ambiguo
2. Lee detenidamente el siguiente fragmento y añade
tilde donde corresponda:

—Es enorme… —murmure—. ¿Qué hay al otro lado? Corrige


—Unos dicen que el oceano no se acaba nunca —
repuso Erik—; otros aseguran que al final del
horizonte se en- cuentra la magica isla de Hy Brasil, y
hay quien afirma que la Tierra es redonda como una
naranja y que más alla del mar se encuentran las
Indias, Catay y Cipango. Pero lo unico cierto es que
nadie lo sabe, porque nadie ha ido alli para
comprobarlo.
Sonrei ante la absurda idea de que la Tierra fuera
redon- da —de ser asi, los que estan abajo se caerían,
¿no es cierto?—.
Cé sar MALLORQUÍ: La
catedral.

—Es enorme… —murmuré —. ¿Qué hay al otro lado?


—Unos dicen que el océano no se acaba nunca —repuso
Erik—; otros aseguran que al final del horizonte se
encuentra la má gica isla de Hy Brasil, y hay quien afirma
que la Tierra es redonda como una naranja y que má s allá
del mar se en- cuentran las Indias, Catay y Cipango. Pero
lo ú nico cierto es que nadie lo sabe, porque nadie ha ido
allí para comprobarlo.
Sonreí ante la absurda idea de que la Tierra fuera
redonda
—de ser así, los que está n abajo se caerían, ¿no es cierto?
—.
3. Explica la regla de acentuación que se cumple en las
pa- labras acentuadas:

8 Lengua castellana y Literatura. 3.º ESO. Solucionario


a) Déjame (palabra esdrú jula) que te diga una cosa: así (agu-
es la consecuencia final de un dañ o causado por hepatopatía
da acabada en vocal) no llegará s a ninguna parte, Óscar
cró nica. Las consecuencias dependen fundamentalmente del
(llana que acaba en consonante diferente a n o s).
grado de funcionalidad que el hígado pueda conservar a pesar
b) Los íberos (esdrú jula) llegaron a la península de la alteració n histoló gica.
(esdrú jula) ibérica (esdrú jula) antes de la invasión
(aguda acabada en n) romana. En este texto falla la adecuació n. El tema es lo suficientemen-
te serio para ser tratado con un médico especialista, no con
4. Busca errores de acentuación en tu entorno: carteles, un auténtico desconocido. Ademá s sería recomendable tratar
anun- cios, rótulos o luminosos. Anota cinco de ellos y el asunto en persona, no en un chat. El contexto situacional
ponlos en común con tus compañeros. no es el má s adecuado porque las tecnologías nos acercan
Respuesta libre. la informació n, pero en muchos casos esa informació n no es
fiable y hay que filtrar los datos que obtenemos. Por otro lado,
5. Separa en sílabas estas palabras y coloca la tilde donde emisor y receptor no comparten el mismo conocimiento del
sea necesario: caso, pues la informació n se presenta sin una contextualiza-
ció n previa. No sabemos los síntomas, la edad del paciente del
Corrige que se habla, qué pruebas se le han realizado ni los resultados
huespedsombrio ruido Luis cacatua
aeronave doscientos de las mismas. Es imprudente adelantar un diagnó stico en una
situació n tan inconcreta.
buho mienteme vagais acuatico
Receptor y emisor no comparten el mismo registro y segura-
heroico cooperacion llovia mente no tienen el mismo grado de conocimientos mé dicos.
conociais Uruguay cienaga Por eso la primera persona que interviene escribe trankilos
nauseabundo saliesen construid y dice que el término cirrosis hepática le suena muy feo (ad-
verbializació n del adjetivo). Se ve que es una persona con un
• hué s-ped • som-brí-o • ae-ro-na-ve nivel cultural bajo y ningú n conocimiento de medicina. No es
• dos-cien-tos • bú -ho • rui-do adecuado que la persona que contesta use tantos tecnicismos
del campo de la medicina. Ademá s no es coherente porque
• va-gá is • a-cuá-ti-co • he-roi-co si la persona que pregunta no entiende lo que es cirrosis
• Luis • co-o-pe-ra-ció n • llo-ví-a hepática tampoco sabrá qué significa tejido hepático, he-
• co-no-cí-ais • ca-ca-tú -a • U-ru-guay patopatía crónica o funcionalidad, por no decir alteración
histológica.
• cié-na-ga • nau-se-a-bun-do • mién-te-me
• sa-lie-sen • cons-truid Tampoco es coherente por parte del emisor que pida que
alguien (pronombre indefinido) conteste a su duda, dando a
6. Conjuga en tu cuaderno estos tiempos de los verbos ver, entender que da igual quién sea y el grado de conocimiento
oír, aplaudir: condicional simple, pretérito imperfecto de que tenga sobre el tema. Y resulta incoherente que pida esta
indicativo y presente de subjuntivo. informació n a quien quiera contestarla para ganar tranqui-
Condicional simple: vería, oiría, aplaudiría. lidad. Por ú ltimo, tampoco es coherente que si la persona
que pregunta busca tranquilidad, la persona que contesta lo
Preté rito imperfecto de indicativo: veía, oía, aplaudía. haga con tecnicismos incomprensibles para cualquier perso-
Presente de subjuntivo: vea, oiga, aplauda. na ajena a la medicina, pues es normal que se intranquilice en
7. Subraya y clasifica los diptongos, triptongos e hiatos de lugar de encontrar la serenidad que busca. La ú ltima incohe-
rencia de este «médico» es que recomienda calma (aunque
este texto:
no contribuye decisivamente a ella), hasta que un médico
Apelo al lector, para que él diga si hay indicios de demencia en profesional interprete los resultados. Da a entender con ello
el estilo o en el mé todo de la historia que ha estado leyendo. que él no es el má s indicado para contestar porque no es ese
Soy un hombre joven encerrado en el cuerpo de un viejo. Pero profesional.
a todo el mundo le resulta increíble la innegable realidad de
este hecho. 2. Busca en chats, foros, blogs o mensajes de texto de móvi-
les otros textos que no posean alguna de las
H. G. WELLS: El caso del difunto señor Elvesham.
propiedades.
Apelo al lector, para que él diga si hay (diptongo) indicios (dip-
Respuesta libre.
tongo) de demencia (diptongo) en el estilo o en el método de la
historia (diptongo) que ha estado leyendo. Soy (diptongo) 3. Explica por qué canal crees que se difundiría este peque-
un hombre joven encerrado en el cuerpo de un viejo ño texto y cuál es su intención comunicativa:
(diptongo). Pero a todo el mundo le resulta increíble (hiato)
Con motivo de la jornada de desinfecció n prevista para hoy,
la innegable realidad (hiato) de este hecho.
las instalaciones de la piscina cubierta permanecerá n cerra-
das hasta mañ ana jueves. Muchas gracias y disculpen las
LA FACTORÍA DE TEXTOS molestias.
1. Observa el siguiente texto extraído de un chat y señala Canal oral o escrito. Segú n la intenció n comunicativa es
cuál de las propiedades del texto crees que falla aquí. informativo.
—A mi suegro le han hecho una serie de pruebas y en una de 4. Escribe un texto que posea adecuación, coherencia y co-
ellas han determinado que tiene cirrosis hepá tica. ¿Alguien hesión y que se adapte a las siguientes características: su
podría explicarme qué es eso para que nos quedemos tranki- forma será la de una narración, su intención
los? Me suena muy feo. comunicativa consistirá en informar, debe pertenecer al
—En todo caso hay que mantener la calma hasta que un médi- ámbito periodís- tico y su canal de comunicación debe
co profesional interprete los resultados. Os explico en qué con- ser escrito.
siste la enfermedad: la cirrosis que afecta al tejido hepá tico
Respuesta libre. Debe tenerse en cuenta que lo que se está
pidiendo al alumno es que redacte una noticia de un suceso
con la finalidad de informar, no de opinar.
ACTIVIDADES FINALES • Es un sintagma.
Repasa lo que has aprendido
1. Lee el siguiente texto y resuelve las cuestiones que se
proponen a continuación.
Encima de todo había un papel enrollado. Cuando lo desen-
rollaron, vieron que estaba escrito con una caligrafía difícil
de descifrar. Sin embargo, Annika dijo que podría entenderlo.
—Lé enoslo —le pidió David.
Y ella empezó :
«Hoy es 30 de junio de 1763. Acabo de oír al reloj de abajo dar
las ocho».
Annika enmudeció y miró a David. Tambié n hoy era 30, y
aca- baban de oír, igualmente, dar las ocho en el reloj de
abajo…
Joná s abrió unos ojos como platos. ¡Eso sí que era sorprenden-
te! Conectó el magnetofó n. ¡Había que leer el escrito en voz
alta para grabarlo! El papel temblaba en las manos de
Annika. Lo agarró fuertemente y prosiguió leyendo:
«Estoy sentada frente a la ventana. Es el tiempo en que flo-
recen los tilos, y quisiera abrir la ventana y sentir el olor de
las flores… Pero ya no me quedan fuerzas. Sé que no viviré
mucho tiempo. Aunque eso no me preocupa.
»Delante de mí, encima de la mesa, tengo el estuche que hizo
para mí Andreas cuando cumplí catorce añ os. Cuando con-
templo mis ojos y veo mi cara en el espejo que hay en la
parte interior de la tapa, pienso y deseo que se queden ahí
y que se encuentren con los ojos de aquel que un día
encontrará este estuche y lo abrirá . ¡Có mo me gustaría mirar
esos ojos, y conocer el corazó n de esa persona y sus
sentimientos!».
María GRIPE: Los escarabajos vuelan
al atardecer (Editorial SM).
a) Indica a qué niveles de la lengua pertenecen las si-
guientes unidades:
• La novela de la que se ha extraído este fragmento.
• Los fonemas que componen la palabra encima.
• Este fragmento de la novela.
• Annika.
• Léenoslo.
• Unos ojos como platos.
b) Busca en el texto un sintagma de cada tipo.
c) Analiza morfológicamente todas las palabras del pri-
mer párrafo.
d) Descompón en monemas las siguientes palabras: de-
senrollaron, descifrar, caligrafía, enmudeció, sorpren-
dente, florecen, sentimientos.
e) ¿Qué categoría o categorías gramaticales están re-
presentadas en la expresión Léenoslo? Justifica tu res-
puesta.
f) Busca en el texto dos oraciones enunciativas, dos ex-
clamativas y una desiderativa.
g) ¿Constituye este un buen ejemplo de texto? ¿Reúne
las propiedades necesarias? Razona tu respuesta y
aporta ejemplos claros.
a) • Es un texto.
• Son fonemas.
• Es un texto.
• Es una palabra.
• Es una oració n.
b) Respuesta libre.
c) Encima (adverbio), de (preposició n), todo (pronombre in-
definido), había (verbo), un (determinante artículo inde-
terminado, masculino, singular), papel (sustantivo comú n,
concreto, contable, masculino, singular), enrollado (par-
ticipio funcionando como adjetivo). Cuando (adverbio), lo
(pronombre), desenrollaron (verbo), vieron (verbo), que
(conjunció n) estaba (verbo), escrito (participio, adjetivo)
con (preposició n) una (determinante artículo indetermi-
nado, femenino, singular) caligrafía (sustantivo comú n,
concreto, incontable, femenino, singular) difícil (adjetivo
calificativo, femenino, singular) de (preposició n) desci-
frar (verbo). Sin embargo (locució n conjuntiva), Annika
(sustantivo propio) dijo (verbo) que (conjunció n) podría
entender (verbo) lo (pronombre enclítico).
d) • desenrollaron: des- (morfema derivativo, prefijo) -en-
(morfema derivativo, prefijo) -roll- (lexema) -aron (mor-
femas flexivos).
• descifrar: des- (morfema derivativo, prefijo) -cifr- (le-
xema) -ar (morfema derivativo, sufijo).
• caligrafía: cali- (lexema griego, hermoso) -grafía (le-
xema griego, escritura).
• enmudeció: en- (morfema derivativo, prefijo) -mud- (le-
xema) -ec- (morfema derivativo, sufijo) -ió (morfemas
flexivos).
• sorprendente: sorprend- (lexema) -ente (morfema deri-
vativo, sufijo).
• florecen: flor- (lexema) -ec- (morfema derivativo, sufijo)
-en (morfemas flexivos).
• sentimientos: senti- (lexema) -miento- (morfema deri-
vativo, sufijo) -s (morfema flexivo).
e) Está n representadas el verbo (lee) y los pronombres (nos
y lo).
f) Enunciativas: Encima de todo había un papel enrollado.
Cuando lo desenrollaron, vieron que estaba escrito con
una caligrafía difícil de descifrar.
Exclamativas: ¡Eso sí que era sorprendente! ¡Había que
leer el escrito en voz alta para grabarlo!
Desiderativa: ¡Cómo me gustaría mirar esos ojos, y cono-
cer el corazón de esa persona y sus sentimientos!
g) Sí, es un verdadero texto. Es adecuado porque tiene en
cuenta al lector y va distribuyendo la informació n de ma-
nera que se genere el suspense propio de una novela de
misterio, como es esta.
También es adecuado el registro culto y el léxico que utili-
za para el canal escrito propio de la transmisió n de obras
literarias.
La coherencia se ha conseguido desarrollando la narra-
ció n en torno a un tema y unas claves. Ademá s el texto
está dotado de una estructura clara, en donde las ideas se
suceden de una forma ló gica, progresiva y cohesionada.
Esto ú ltimo se consigue gracias al uso de los pronombres,
que evitan la repetició n de conceptos, el uso de determi-
nantes posesivos y la sustitució n léxica.
2. Explica la regla de acentuación que se cumple en las si-
guientes palabras: difícil, léenoslo, empezó, enmudeció,
Jonás, sí, magnetofón.
• difícil: es llana y acaba en consonante distinta de n o s.
• léenoslo: sobresdrú jula (se acentú a siempre).
• empezó: es aguda y acaba en vocal.
• enmudeció: ídem.
• Jonás: es aguda y acaba en s.
• sí: es una palabra monosílaba. Lleva tilde para diferenciar-
se de la conjunció n homó nima si. 8. Inventa frases como eslogan para promocionar los
siguien- tes productos.
• magnetofón: es aguda y acaba en n.
Respuesta libre.
3. Define con tus palabras cada una de las unidades de la
lengua representativas de cada nivel y pon un ejemplo 9. Indica a qué tipo de sintagma corresponden los
tomado del texto. siguientes y marca su núcleo:
Respuesta libre. a) algunas tardes de verano
4. Escucha atentamente una noticia en un informativo de b) felizmente casados
televisión. Después resume su contenido e indica de qué c) bastante lejos del parque
tipo de texto se trata según su forma, intención comuni- d) cada día de lluvia
cativa, ámbito de uso y canal razonando tus
e) caminó lentamente
afirmaciones. Haz lo mismo con el texto de Los Juegos
del Hambre que abre la unidad. f) en la estación de tren
Respuesta libre. g) tus ojos azules
5. Lee el siguiente poema y responde a las cuestiones que a) algunas tardes de verano (SN)
se formulan sobre él: b) felizmente casados (SAdj)
La palabra c) bastante lejos del parque (SAdv)
La palabra pregunta y se contesta d) cada día de lluvia (SN)
tiene alas o se mete en los tú neles
e) caminó lentamente (SV)
se desprende de la boca que habla
y se desliza en la oreja hasta el tímpano f) en la estación de tren (SPrep, no tiene nú cleo propio, pero
estación es nú cleo del SN que hay dentro).
la palabra es tan libre que da pá nico
divulga los secretos sin aviso g) tus ojos azules (SN)
e inventa la oració n de los ateos 10. Escribe los siguientes tipos de textos haciendo uso de la
es el poder y no es el poder del alma
modalidad indicada en cada caso:
y el hueso de los himnos que hacen patria […]
a) Una pequeña nota para hacer un pedido en la que
la palabra es correo del amor
pero tambié n es arrabal del odio uses el modo imperativo.
golpea en las ventanas si diluvia b) Una reclamación a Papá Noel porque te ha regalado
y el corazó n le abre los postigos un objeto absurdo en la que uses la modalidad inte-
y ya que la palabra besa y muerde rrogativa.
mejor la devolvemos al futuro c) Unas instrucciones de uso de un exprimidor en las
Mario BENEDETTI: Inventario cuatro. que uses el modo imperativo.
a) ¿Cómo describe el autor la naturaleza de las d) Una pequeña narración o microrrelato en el que uses
palabras? la modalidad enunciativa.
¿Te da la sensación de que las siente como algo débil e) Una pequeña carta expresando tus deseos de que un
o como algo poderoso? enfermo se recupere en la que uses oraciones desi-
b) Inventa tu propia definición en prosa o en verso sobre derativas.
la palabra. f) Un mensaje publicitario en el que uses la modalidad
c) Busca en el poema tres sustantivos, cinco verbos, dos exhortativa.
adjetivos, dos preposiciones y dos conjunciones. Respuesta libre.
a) La palabra se define como algo vivo e inquieto, como una
MIRA A TU ALREDEDOR Y…
realidad contradictoria pero con mucho poder. Es algo má -
gico. Se siente en todos los versos como algo poderoso, … ve más allá
capaz de cualquier imposible.
1.El siguiente texto es una exposición literaria sobre cómo
b) Respuesta libre. subir escaleras. Léelo con atención y observa si se
c) Sustantivos: palabra, alas, túneles. cumplen en él y de qué manera las propiedades textuales
Verbos: pregunta, contesta, tiene, mete, desprende. (adecua- ción, coherencia y cohesión).
Adjetivos: libre, ateos. Las escaleras se suben de frente, pues hacia atrá s o de
costado resultan particularmente incó modas. La actitud
Preposiciones: en, de.
natural con- siste en mantenerse de pie, los brazos colgando
Conjunciones: o, y. sin esfuerzo, la cabeza erguida aunque no tanto que los ojos
6. Observa las siguientes imágenes extraídas de la prensa dejen de ver los peldañ os inmediatamente superiores al que se
y formula en un sintagma nominal un título que resuma pisa, y res- pirando lenta y regularmente. Para subir una
la información: escalera se co- mienza por levantar esa parte del cuerpo
situada a la derecha abajo, envuelta casi siempre en cuero o
Respuesta libre. gamuza, y que salvo excepciones cabe exactamente en el
7. Los sintagmas que has formulado en la actividad anterior escaló n. Puesta en el pri- mer peldañ o dicha parte, que para
¿constituyen enunciados u oraciones? Justifica tu res- abreviar llamaremos pie, se recoge la parte equivalente de la
puesta. izquierda (tambié n llamada pie, pero que no ha de
confundirse con el pie antes citado), y llevá ndola a la altura
Respuesta libre.
del pie, se le hace seguir hasta colocarla
en el segundo peldañ o, con lo cual en este descansará el pie, texto poé tico y, segú n el canal que utiliza, se trata de un
y en el primero descansará el pie. (Los primeros peldañ os texto escrito.
son siempre los más difíciles, hasta adquirir la coordinació n Sí hay adecuació n, porque los textos poéticos suelen
necesaria. La coincidencia de nombre entre el pie y el pie hace
difícil la explicació n. Cuídese especialmente de no levantar
al mismo tiempo el pie y el pie). Llegando en esta forma al
se- gundo peldañ o, basta repetir alternadamente los
movimientos hasta encontrarse con el final de la escalera. Se
sale de ella fá cilmente, con un ligero golpe de taló n que la
fija en su sitio, del que no se moverá hasta el momento del
descenso.
Julio CORTÁ ZAR: Historias de cronopios y famas.
Aparentemente no se cumplen todas las propiedades textua-
les. La adecuació n parece no cumplirse por el hecho de que
el emisor (el autor) no tienen en cuenta los conocimientos
sobre el mundo que posee el receptor (el lector), pues todo el
mundo sabe subir una escalera. Parece no tener en cuenta
que cual- quier lector sabe que eso que va envuelto en cuero o
gamuza es un pie, etc. ¿Pero es cierta esa falta de conciencia
sobre lo que el lector sabe? Evidentemente, representa un
juego literario al que el autor recurre en busca de
provocació n y originalidad.
Lo mismo ocurre con la coherencia. En principio no parecen
coherentes las recomendaciones absurdas (por ser demasia-
do cotidianas) para subir las escaleras, como hacerlo de pie,
subiendo primero el pie derecho a un peldañ o y luego el iz-
quierdo, etc. Pero la literatura goza de una libertad especial
en este aspecto, igual que el arte. Pues de estas
incoherencias (coherentemente hiladas unas a otras) se
desprende el humor que pretende el texto.
Por otro lado, dichas incoherencias hacen que un objeto co-
tidiano resulte má gico y recié n creado ante nuestros ojos.
Sobre todo ayuda a este efecto el final del mismo: Se sale de
ella fácilmente, con un ligero golpe de talón que la fija en su
sitio, del que no se moverá hasta el momento del descenso.
A pesar de todo ello, la cohesió n está perfectamente conse-
guida por el uso de varios mecanismos, entre ellos, los mar-
cadores textuales que dotan al texto de un orden y una ló gica
interna de la que parecía carecer en principio.
2. Los textos literarios incumplen a menudo las
propiedades del resto de los textos en beneficio de la
expresividad. Tratan con ello, como el texto de la
actividad anterior, de crear sorpresa y belleza. Lee el
siguiente texto, y explica si te parece un mensaje
coherente y por qué.
Guitarra
Habrá un silencio verde
todo hecho de guitarras
destrenzadas.
La guitarra es un pozo
con viento en vez de agua.
Gerardo DIEGO: Imagen.
a) Analiza de qué tipo de texto se trata desde el punto
de vista de su forma, su intención comunicativa y el
canal de comunicación que utiliza. De acuerdo con
todo ello, explica si existe la propiedad de adecua-
ción en el texto.
b) ¿Qué relación observas entre el poema y la imagen
de la guitarra en cuanto a su parecido con la realidad?
c) Investiga sobre el surrealismo y sus manifestaciones
literarias y pictóricas. ¿Qué papel cumple la coheren-
cia en los textos surrealistas?
a) Desde el punto de vista de su forma, se trata de un
poema. Por su intenció n comunicativa se trata de un
transmitirse por escrito y su intenció n comunicativa cuan- do en un mundo global / el buscar comida en otra tierra
es la de producir belleza. te convierte en ilegal; / cuando la ley de extranjería te atrapa
b) Ni el texto ni la guitarra reflejan la realidad de una sin motivo / y la hipocresía tapa sus ojos y sus oídos; /
manera transparente, es decir, no imitan la realidad
tal y como es, sino que descomponen dicha realidad y
la reinventan, de manera que no parece existir una
asociació n directa entre el poema o la pintura y la
realidad a la que se refiere. En ambas creaciones
parece romperse la coherencia porque se transgrede
la ló gica, pero la originalidad de ambas obras de artes
consiste en seguir expresando sentimientos y
emociones a pesar de esa transgresió n.
c) Uno de los movimientos europeos de vanguardia que se
dieron en el periodo de entreguerras (1910-1940) y que
fue menos pasajero que el resto. Este movimiento
artístico (primero pictó rico y luego literario) supuso un
cambio radical en la concepció n del papel del arte y
del trabajo del artista. El paso decisivo en su creació n lo
dará André Breton, tras leer a Sigmund Freud.
El surrealismo no se presenta solo como una
renovació n estética, sino que pretende ser una
revolució n integral. Su lema sería la frase de Rimbaud:
«transformar la vida», y en ese sentido se produce un
«encuentro» entre las doctrinas de Freud y las de Marx.
Para los surrealistas, la vida es la cara má s gris de la
rea- lidad; hay que conquistar una verdadera vida,
acceder a una realidad má s alta y para ello es necesaria
también una liberació n del lenguaje, hay que sacarlo
de la ló gica. En un poema surrealista, se entremezclan
objetos, conceptos y sentimientos que la razó n
mantendría separados; aparecen asociaciones libres e
inesperadas de palabras, metá foras insó litas,
imá genes oníricas y hasta delirantes. Ante un poema
así, el lector no «comprende» pero puede recibir
fuertes impactos que modifican su estado de á nimo y
sus- citan en é l oscuras emociones.
Son cultivadores del surrealismo en Españ a autores
como Juan Larrea, Salvador Dalí, Luis Buñ uel y a su
influjo se de- ben libros fundamentales como Sobre los
ángeles de Rafael Alberti o Poeta en Nueva York de
Federico García Lorca.
3. A veces, la naturaleza y el arte se combinan. Observa
la siguiente imagen y escribe una pequeña historia
fantástica con ella que cumpla las tres propiedades
textuales. Pon a trabajar a tu imaginación.
Respuesta libre.

… encuentra la clave
1. El siguiente texto es un fragmento de un rap de
Chojin en el que colaboran otros famosos raperos
como Kase O, Lírico, El Langui o Nach. Léelo o
escúchalo en YouTube tecleando: El Chojin. «Rap
contra el racismo», y realiza las siguientes actividades
que se te plantean:
Kase O
Alto, bajo, feo, guapo, negro, blanco, ¿qué má s da? /
Dentro de cien añ os todos calvos bajo tierra, ¿va? / ¿No has
probado nunca a conocer a un extranjero? / Fíjate en
los niños: ellos saben de qué va este juego / y es que la
raza humana es un crisol. / El que no pueda ver belleza en
esto no mere- ce ver el sol. / Paso el relevo al compañ ero,
para este mundo nuevo / es el triunfo del amor contra el
miedo.
Nach
Cuando la bestia racista siente rabia y muerte; / cuando la
fobia se contagia y hierve acusá ndote de no ser igual; /
racismo y marginació n, cuando solo ven la piel / y se olvidan de las personas.
de mirar al corazón. Respuesta libre.
a) ¿Este texto es una manifestación de la lengua o del
lenguaje? Explica tu respuesta.
b) Explica por qué llevan tilde las palabras marginación
y acusándote. Luego, escribe tres palabras llanas que
necesiten tilde.
c) Explica cuál es el significado, el significante y el refe-
rente de la palabra relevo. ¿Está usada esta palabra
en sentido literal? Justifica tu respuesta apoyándote
en la información que encuentres en el diccionario.
d) Indica la modalidad de las oraciones que aparecen en
negrita.
e) Este texto es pura poesía. Señala en él algunos
recursos literarios (metáfora, comparación, etc.) que
conozcas.
a) De la lengua, ya que se trata de una manifestació n con-
creta del lenguaje (capacidad lingü ística que poseen todos
los seres humanos). Se trata de una manifestació n de la
lengua castellana un tanto especial, pues, en oposició n
a lo que ocurre con el lenguaje, que todos poseemos, no
todos los seres humanos poseemos la capacidad de usar la
lengua para hacer un texto literario. Es una
manifestació n de lengua, que equivale al idioma, porque el
lenguaje tiene una naturaleza social; entendemos el texto
los hablantes de una misma lengua, en este caso, el
castellano.
b) Marginación: es aguda y acaba en n. Acusándote: es es-
drú jula y las palabras esdrú julas se acentú an siempre.
c) Significante: r-e-l-e-v-o. Significado: acció n de reemplazar
un corredor a otro de su mismo equipo en el momento
de recibir el relevo.
Referente:

No. Está empleado en un sentido metafó rico. Pasarle el


relevo al compañ ero quiere decir que le cede el turno de
palabra porque ambos está n juntos en esta causa. Le
cede la palabra para que é l tambié n luche contra el
racismo.
d) • ¿No has probado nunca a conocer a un extranjero?
(interrogativa).
• Fíjate en los niños: ellos saben de qué va este juego
(exhortativa y enunciativa).
• El que no pueda ver belleza en esto no merece ver el sol
(enunciativa).
• La hipocresía tapa sus ojos y sus oídos (enunciativa).
• Se olvidan de mirar al corazón (enunciativa).
2. Busca otros temas musicales que transmitan este mismo
mensaje y anota la letra de dos.
Respuesta libre.
3. Inventa señales de prohibición en contra del racismo y
traduce su significado en forma de oraciones
imperativas.
Ejemplo:
Prohibidas las conductas racistas que dañen el corazón
4. Inventa enunciados y oraciones con mensajes en contra
del racismo.
Indica cuáles de esos enunciados son oraciones y cuáles
son frases.
Respuesta libre.
5. Ahora escribe un texto en el que mantengas el tema de
los enunciados del ejercicio anterior. Luego, crea un
cartel apropiado que acompañe al texto. Te ofrecemos
este de Nelson Mandela para que te sirva de
inspiración.

«NADIE nace odiando a otra persona por el color de su piel, o su origen, o su religión.
La gente tiene que aprender a odiar, y si ellos pueden aprender a odiar, también se les puede enseñar a amar; el amor llega má

Respuesta libre.
6. Busca información sobre la figura de Nelson Mandela y
elabora un resumen de su biografía con los datos más
inte- resantes que hayas encontrado.
Nelson Mandela (Umtata, Transkei, 1918-Johannesburgo,
2013). Político sudafricano que lideró los movimientos con-
tra el apartheid. Prisionero durante 27 añ os (1963-1990)
en penosas condiciones, el gobierno de Sudá frica recha-
zó todas las peticiones para ponerlo en libertad. Nelson
Mandela se convirtió en un símbolo de la lucha contra el
apartheid dentro y fuera del país, en una figura legendaria
que representaba la falta de libertad de todos los negros
sudafricanos.
En 1984 el gobierno intentó acabar con tan incó modo mito,
ofreciéndole la libertad si aceptaba establecerse en uno de los
bantustanes a los que el régimen había concedido una
ficció n de independencia; Mandela rechazó el ofrecimiento.
Durante aquellos añ os, su esposa Winnie simbolizó la
continuidad de la lucha, alcanzando importantes posiciones
en el Congreso Nacional Africano. El ferviente activismo de
Winnie no estu- vo libre de escá ndalos; añ os después, ya en
los 90, se vería envuelta en un polémico juicio en el que fue
acusada de ase- sinato, si bien salió absuelta.
Finalmente, Frederik De Klerk, presidente de la Repú blica
por el Partido Nacional, cedió ante la evidencia y decidió
abrir el camino para desmontar la segregació n racial,
liberando a Mandela en 1990 y convirtiéndole en su principal
interlocutor para negociar el proceso de democratizació n.
Mandela y De Klerk compartieron el Premio Nobel de la Paz
en 1993.
Las elecciones de 1994 convirtieron a Mandela en el primer
presidente negro de Sudá frica; desde ese cargo puso en
mar- cha una política de reconciliació n nacional,
manteniendo a De Klerk como vicepresidente y tratando de
atraer hacia la participació n democrá tica al díscolo partido
Inkhata, de ma- yoría zulú . Una película muy recomendable
del cineasta es- tadounidense Clint Eastwood, Invictus
(2009), reflejaría con bastante fidelidad el Mandela de
aquellos añ os; su apoyo a una selecció n nacional formada
por blancos durante la Copa Mundial de Rugby de 1995,
celebrada en Sudá frica, muestra su empeñ o en integrar la
minoría blanca y la mayoría negra sir- viéndose de aquel
acontecimiento deportivo y su firme volun- tad de construir
una nació n para todos los sudafricanos, sin distinció n de
raza.
EL MÁGICO PODER DE LAS PALABRAS 2
APERTURA DE UNIDAD rumores y supersticiones sobre los riesgos que entrañ a su uso:
1.Los integrantes de la «Granja Animal» escribieron sobre puede electrocutar a sus portadores o atraer los rayos.
un muro las normas que todos deberían acatar tras 5. ¿Dónde se puede adquirir el Baby H. P.?
expulsar a su dueño. En las buenas tiendas.
a) ¿Cómo intentaron hacer bien visible su mensaje? 6. Comprueba si los siguientes enunciados son verdaderos
b) ¿Crees que el rudimentario procedimiento que ellos (V) o falsos (F):
em- plearon podría tener algún tipo de relación con
los usa- dos por los modernos medios de V F
comunicación de masas? ¿Cuál? a) El Baby H. P. lo pueden utilizar todos
a) Sobre una pared negra escribieron en letras blancas muy los miembros de una familia.
grandes su mensaje.
b) Aunque los modernos medios de masas emplean recursos b) Se recomienda que los niños se lo pongan
más sofisticados, en el fondo los animales hicieron uso para dormir.
de procedimientos con un evidente poder visual: así el c) El aparato jamás puede actuar como
contraste cromá tico entre el color de la pared y el de las un pararrayos.
letras, mientras que el tamañ o de estas ú ltimas nos remite
al papel importante que tiene la tipografía, por ejemplo, d) La energía transformada en electricidad
en la prensa. puede venderse.
2. Imagínate por un instante que los habitantes de la e) El aparato tiene un modelo único, por lo
«Gran- ja Animal» tuviesen las mismas habilidades que no es adecuado para todos los niños.
racionales de los humanos y desearan difundir los
mandamientos entre toda su población. a) Falso. c) Falso. e) Falso.
a) Aparte del código lingüístico, ¿a qué otros códigos y b) Verdadero. d) Verdadero.
medios de comunicación podrían acudir para transmi-
tir su mensaje? Vocabulario
b) ¿Podrían conseguir su objetivo fácilmente? ¿Por qué? 7. El kilovatio es una medida de potencia eléctrica. Para
Respuesta libre. medir otras magnitudes se emplean diferentes unidades.
Relaciona cada unidad con su correspondiente magnitud:
VIVE LA LECTURA intensidad de corriente • • pascal
Comprensión lectora longitud • • kelvin
1.En el relato, Arreola recurre al lenguaje publicitario. masa • • ohmio
¿Qué marcas típicas de este tipo de discurso resistencia eléctrica • • kilogramo
reconoces en el texto? ¿Quién se supone que es el
emisor y quién su destinatario? presión • • amperio
Elementos típicos del lenguaje publicitario son el contenido temperatura • • metro
descriptivo del texto y el predominio de la funció n apelativa • intensidad de corriente  amperio
del lenguaje en un intento de convencer al destinatario. Esta
• longitud  metro
instancia se corresponde con cualquier ama de casa, a la que
se dirige con un usted (fó rmula de respeto) el emisor que, en • masa  kilogramo
este caso, se identifica con un anunciante. • resistencia eléctrica  ohmio
2. ¿Qué producto se anuncia? ¿En qué lugar del texto se • presió n  pascal
describen sus características físicas? ¿Dónde se explica • temperatura  kelvin
su funcionamiento? 8. ¿Por qué dice el texto que el Baby H. P. es lucrativo?
En el texto se anuncia el Baby H. P., un aparato que actú a Explí- calo en tu cuaderno.
como depó sito de la energía que «producen» los niñ os con
El aparato es lucrativo porque permite obtener unas ganan-
su actividad física diaria. Sus características físicas y su
cias o beneficios, ya que permite ahorrar en el recibo de la
funcio- namiento se detallan en el segundo pá rrafo.
electricidad e incluso vender la energía sobrante.
3. ¿Por qué dirías que la historia tiene un carácter 9. En los mensajes publicitarios se busca interesar al consu-
fantástico? La historia tiene un cará cter fantá stico porque midor por un producto cuyas virtudes se destacan me-
plantea una posibilidad bastante inverosímil, convirtiendo a diante adjetivos. Anota las palabras que desempeñen esa
los niñ os en una especie de má quinas. Con ello, el autor función en el texto.
parodia no solo
Las virtudes del producto se destacan mediante adjetivos
el lenguaje publicitario, sino que lanza sus dardos contra una
como: maravilloso Baby H. P., cómodos cinturones, preciosa
sociedad cada vez má s tecnificada.
alcancía o fuente generosa; pero también a partir del signi-
4. ¿Qué beneficios puede tener para las familias la compra ficado de formas verbales como: revolucionar, contribuye a,
del producto descrito? En cambio, ¿en qué momento ad- se economizan y se obsequian.
vierte el autor de los peligros que se derivan de su uso? 10. ¿Qué extranjerismos y términos cultos emplea el autor?
Con la adquisició n del aparato, el agobiante ajetreo de los ni- Como extranjerismos: Leyden (por Leiden, universidad ho-
ñ os deja de ser un quebradero de cabeza para convertirse en landesa) y récord. Como términos cultos, de los indicados en
fuente de ahorro econó mico para las familias. En cambio, en el el glosario: motriz, alcancía o lucrativo.
penú ltimo pá rrafo, se alude como de pasada a los supuestos

Lengua castellana y Literatura. 3.º ESO. Solucionario 13


Investigación y redacción e) París, la capital de Francia
11. En este cuento el autor alerta sobre los peligros del f) cuatro horribles chaquetas sin solapas
desa- rrollo tecnológico, pero también sobre cómo la
publicidad puede resultar engañosa. A partir de estas
ideas:
a) Elabora una redacción dando tu punto de vista sobre
el tema: «¿Es el hombre actual esclavo de la
técnica?».
b) En grupos, seleccionad varios anuncios publicitarios
para debatir en el aula si la información que
transmiten es fiable o engañosa.
Respuesta libre.

ESTUDIO DE LA LENGUA
1. Escribe estos sustantivos en masculino y femenino e indi-
ca cómo se marca el género en cada caso:
diablo astronauta serpiente
padrino héroe emperador
escritor tigre yerno
avestruz equilibrista profeta
• diablo: pasa a femenino como diablesa (cambio en termina-
ció n).
• astronauta: mismo lexema para masculino y femenino, es
el determinante el que precisa el género.
• serpiente: cambia el género añ adiendo macho o hembra.
• padrino: pasa a femenino como madrina (cambio en el
lexema).
• héroe: pasa a femenino como heroína (cambio en termina-
ció n).
• emperador: pasa a femenino como emperatriz (cambio en
terminació n).
• escritor: pasa a femenino como escritora (cambio en termi-
nació n).
• tigre: pasa a femenino como tigresa (cambio en
terminació n).
• yerno: pasa a femenino como nuera (cambio en el lexema).
• avestruz: cambia el género añ adiendo macho o hembra.
• equilibrista: mismo lexema para masculino y femenino, es
el determinante el que precisa el género.
• profeta: pasa a femenino como profetisa (cambio en termi-
nació n).
2. Forma el plural de los siguientes sustantivos:
raíz crisis jueves lápiz
árbol tren naranja peral
reactor ayuntamiento ciudad fórceps
• ayuntamiento, naranja: ayuntamientos, naranjas (se
añ a- de –s).
• árbol, tren, peral, reactor ciudad: árboles, trenes,
perales, reactores, ciudades (se añ ade –es).
• raíz, lápiz: raíces, lápices (se añ ade –es y la z pasa a c).
• crisis, jueves, fórceps: misma forma para singular y plural
(el nú mero lo señ ala el determinante).
3. Analiza los siguientes sintagmas nominales, identificando
todos sus elementos constituyentes:
a) dos botellas de agua
b) aquellas hermosas pinturas
c) mi primo Arturo
d) la casa amarilla de la esquina
g) una banalidad de película
h) nuestros primeros días juntos
i) la casa de los espíritus
a) Det + N + CN (enlace + té rmino-SN [N])
b) Det + Ady + N
c) Det + N + Aposició n especificativa
d) Det + N + Ady + CN (enlace + té rmino-SN [Det + N])
e) N + aposició n explicativa (Det + N + CN [enlace +
término- SN {N}])
f) Det + Ady + N + CN (enlace + té rmino-SN [N])
g) Det + N + CN (enlace + té rmino-SN [N])
h) Det + Det + N + Ady
i) Det + N + CN (enlace + té rmino-SN [Det + N])
4. Descubre la palabra sustantivada en las siguientes oracio-
nes e indica si la sustantivación es ocasional o
permanente:
a) Nos intriga el mañana.
b) Le daremos el pésame a su abuelo.
c) Era el hazmerreír de la clase.
d) Solo los ricos envidian nuestra felicidad.
e) En la plaza se oyó un olé de júbilo.
a) mañana: sustantivació n permanente.
b) pésame: sustantivació n permanente.
c) hazmerreír: sustantivació n permanente.
d) ricos: sustantivació n ocasional.
e) olé: sustantivació n ocasional.
5. Además de los numerales cardinales que aparecen en el
ladillo Recuerda, existen otros tipos de numerales.
Escribe en tu cuaderno un breve listado de ordinales,
multiplicati- vos y partitivos.
Ordinales: primero, segundo, tercero, cuarto, quinto, sexto,
séptimo, octavo, noveno, décimo, undécimo, duodécimo…,
vigésimo…, trigésimo…, cuadragésimo…, quincuagésimo…,
sexagésimo…, septuagésimo…, octogésimo…, nonagésimo…,
centésimo, etc.
Multiplicativos: doble, triple, cuádruple, quíntuple, séxtu-
ple, séptuple, óctuple, nónuplo, décuplo, undécuplo, duodé-
cuplo, terciodécuplo…, céntuplo, etc.
Partitivos: medio o mitad, tercio, cuarto, quinto, sexto o sei-
savo, séptimo, octavo, noveno, décimo, undécimo u onceavo,
duodécimo o doceavo, decimotercero o treceavo…, vigésimo
o veinteavo, etc.
6. Señala el sujeto en las siguientes oraciones:
a) Yo te encontré en la discoteca.
b) A tu padre le disgusta mi ropa.
c) Todas tus libretas las escondieron ellos.
d) Hicimos el trabajo entre mis compañeras y yo.
e) El río Pisuerga pasa por Valladolid.
f) Llegaron hasta los corredores más débiles.
g) De la pared colgaban varios cuadros.
h) Trajo el camarero dos botellas de vino.
i) Usted le hablará con palabras amables.
a) Yo.
b) mi ropa.
c) ellos.
d) entre mis compañeras y yo.
e) El río Pisuerga.
f) hasta los corredores más débiles.
• río: corriente de agua; ría: desembocadura del río por don-
g) varios cuadros. de penetran corrientes marinas.
h) el camarero. • música: arte de combinar los sonidos; músico: compositor
i) Usted. o intérprete.
7. Reescribe en tu cuaderno el siguiente fragmento, resta- • leño: trozo de un á rbol; leña: conjunto de trozos de troncos
y ramas de á rboles.
bleciendo los sujetos que se han omitido en él:
• barco: embarcació n de mayor tamañ o, movido a motor o
Scrooge cerró la ventana e inspeccionó la puerta por la que
vela; barca: embarcació n pequeñ a.
había entrado el espectro. Tenía la llave echada con doble
vuelta, tal como la había cerrado é l con su propia mano, y • peso: cantidad que pesa algo; pesa: pieza para calcular el
los cerrojos estaban intactos. Trató de decir: «¡Paparruchas!», peso o para hacer mú sculos.
pero se detuvo en la primera sílaba. • el coma: estado de inconsciencia; la coma: signo de
Charles DICKENS: Canción de Navidad. puntuació n.
Respuesta libre. 10. Clasifica los siguientes nombres, atendiendo a su signi-
8. Identifica el tipo de impersonalidad que se da en los si- ficado.
guientes casos: Ejemplo: alambrada  concreto, común, contable, colec-
a) En mi pueblo no hay piscina. tivo.
b) Se invitó al presidente a la celebración. bofetada ajuar Rusia agua
libro aceite amor vajilla
c) Se espera un gran porcentaje de aprobados.
jauría televisor salto Andes
d) El verano pasado granizó con gran virulencia.
control Asturias trampolín equipo
e) Ordenaron un registro de la casa.
• bofetada: concreto, comú n, contable, individual.
f) Me llamaron al teléfono varias veces. • ajuar: concreto, comú n, contable, colectivo.
g) Es pronto para la comida. • Rusia: concreto, propio, topó nimo.
h) Durante la excursión hizo un tiempo apacible. • agua: concreto, comú n, de materia.
i) Habrá mucha colada en la lavadora. • libro: concreto, comú n, contable, individual.
j) Se trataba de tu amigo. • aceite: concreto, comú n, de materia.
k) A partir de mañana anochecerá una hora más tarde. • amor: abstracto.
a) Impersonal gramaticalizada. • vajilla: concreto, comú n, contable, colectivo.
b) Impersonal con se. • jauría: concreto, comú n, contable, colectivo.
c) Impersonal con se. • televisor: concreto, comú n, contable, individual.
d) Impersonal de fenó meno meteoroló gico. • salto: concreto, comú n, contable, individual.
e) Impersonal ocasional. • Andes: concreto, propio, topó nimo.
f) Impersonal ocasional. • control: abstracto.
g) Impersonal gramaticalizada. • Asturias: concreto, propio, topó nimo.
h) Impersonal gramaticalizada. • trampolín: concreto, comú n, contable, individual.
• equipo: concreto, comú n, contable, colectivo.
i) Impersonal gramaticalizada.
j) Impersonal con se. 11. Transforma los siguientes titulares y eslóganes
k) Impersonal de fenó meno meteoroló gico.
publicita- rios en oraciones con sujeto y predicado:
a) Gran expectación ante la llegada del Real Madrid.
9. Explica la diferencia de significado existente entre los
pares de sustantivos que se enumeran (las ilustraciones b) Ferrero Rocher, la expresión del buen gusto.
de la parte inferior te serán de ayuda): c) Maduro, preocupado por la caída del petróleo.
cesto-cesta olivo-oliva madera- d) El número de jóvenes: en máximos en el mundo y
madero ramo-rama huerto-huerta míni- mos en España.
el corte-la corte río-ría música- e) Opel Astra. Ingeniería alemana de futuro.
músico leño-leña f) Al menos seis muertos en un ataque a una escuela pú-
blica de Roma.
barco-barca peso-pesa el coma-la coma
g) Móviles con tapa: ¿el regreso?
• cesto-cesta: recipientes de mimbre, con mayor altura del
cesto que la cesta. h) Un juguete, una ilusión.
• olivo: á rbol; oliva: aceituna, fruto del olivo. a) La llegada del Madrid provocó una gran expectación.
• madera: materia; madero: tronco de un á rbol sin las ramas b) Ferrero Rocher es la expresión del buen gusto.
o tabló n. c) Maduro está preocupado por la caída del petróleo.
• ramo: conjunto de ramas y flores combinadas ornamental- d) El número de jóvenes está en máximos en el mundo y
mente; rama: cada una de las partes que nacen del tronco en mínimos en España.
de las plantas. e) La ingeniería alemana de futuro se llama Opel Astra.
• huerto: terreno pequeñ o y acotado; huerta: extensió n f) Un ataque a una escuela pública de Roma ha provocado
de terreno de regadío para cultivo de verduras y á rboles al menos seis muertos.
frutales. g) Los móviles con tapa regresan a las tiendas.
• el corte: herida o interrupció n o vergü enza; la corte: lugar h) Un juguete equivale a una ilusión.
o població n donde vive el rey con sus servidores.
12. Identifica los sintagmas nominales que funcionan como aguafiestas, aguador, aguadilla, aguachirle,
sujeto en las siguientes oraciones. A continuación, aguamanos,
analiza la estructura de dichos sintagmas. ¿Has
encontrado algu- na palabra sustantivada actuando
como núcleo? Anótala en tu cuaderno.
a) Ayer cumplió años mi tía Julia.
b) Seis atracadores encapuchados asaltaron la sucursal
del banco.
c) Recibirá un trofeo impresionante el ganador de la
carrera.
d) En el referéndum ganó por mayoría el sí.
e) A todos nos encanta el parque de atracciones Port-
Aventura.
f) Dublín, capital de Irlanda, acogerá un gran concierto
de música electrónica.
g) El frotar se va a acabar.
a) mi tía Julia: Det + N + Aposició n especificativa
b) Seis atracadores encapuchados: Det + N + Ady
c) el ganador de la carrera: Det + N + CN (enlace + término-
SN [Det + N]). Sustantivació n de ganador.
d) el sí: Det + N. Sustantivació n de sí.
e) el parque de atracciones PortAventura: Det + N + CN (en-
lace + té rmino-SN [N]) + Aposició n especificativa
f) Dublín, capital de Irlanda: N + Aposició n explicativa (N
+ CN [enlace + término-SN {N}])
g) El frotar: Det + N. Sustantivació n de frotar.
13. En las siguientes oraciones, sustituye los sintagmas nomi-
nales por un pronombre e indica de qué tipo es cada
uno.
a) Ningún asistente quiso firmar la petición.
b) Estas rosas huelen estupendamente; aquellas rosas
no tanto.
c) Dame un regaliz a mí y a ella dale dos regalices.
d) ¿Estudiamos en mi habitación o en tu habitación?
a) Nadie quiso firmarla. Ningún asistente: nadie (indefini-
do); la petición: la (personal á tono).
b) Estas huelen estupendamente; aquellas no tanto. Es-
tas rosas: estas (demostrativo); aquellas rosas: aquellas
(demostrativo).
c) Dame uno a mí y a ella dale dos. Un regaliz: uno
(numeral cardinal); dos regalices: dos (numeral
cardinal).
d) ¿Estudiamos en la mía o en la tuya? Mi habitación: la
mía (posesivo); tu habitación: la tuya (posesivo).

EL TALLER DE LAS PALABRAS


1. Con ayuda de un diccionario, forma las familias léxicas
de las siguientes palabras. Anota en tu cuaderno el signi-
ficado de aquellas que no conozcas: poesía, mano,
salvar, caballo, figura.
• caballo: caballete, caballico, caballito, caballuelo, caballe-
ría, caballero, caballeresco, caballeroso, caballeriza.
• figura: figuració n, figuradamente, figurado, figurante, fi-
gurar, figurativo.
• mano: manita, manopla, manosear, manido, manecilla.
• poesía: poeta, poetisa, poético, poéticamente.
• salvar: salvació n, salvado, salvador, salvaguardia, salvo,
salvavidas, salvedad.
2. En la familia léxica de agua encontramos, entre otras,
las palabras que te ofrecemos a continuación: desagüe,
aguacero, aguardiente. Di cuáles de ellas se han formado
por derivación y cuáles por composición. Puedes usar un
diccionario y anotar en tu cuaderno los significados.
• desagüe: derivada; «conducto por donde sale el agua».
• aguafiestas: compuesta; «persona que estropea una
diversió n».
• aguador: derivada; «persona que transporta o vende agua».
• aguadilla: derivada; «breve zambullida».
• aguachirle: compuesta; «alimento líquido sin sustancia».
• aguamanos: compuesta; «agua que sirve para lavar las
manos».
• aguacero: derivada; «lluvia abundante».
• aguardiente: compuesta; «bebida alcohó lica de mucha
graduació n».
3. Reescribe las siguientes oraciones sustituyendo las ex-
presiones o palabras en negrita por alguna palabra que
pertenezca a la familia léxica de caja. Utiliza el
diccionario:
a) Eduardo estaba enamorado de la chica que cobra a
los clientes que guardan cola en el supermercado.
b) No creo que sea posible meter esta vela en el
cande- labro.
c) Estoy aprisionado entre la mesa de estudio y la pared;
no puedo ni moverme.
d) Alicia compró un paquete de tabaco, pero luego lo
quemó, porque, en realidad, quería dejar de fumar.
e) Eladio se quedó sin dinero y tuvo que ir a sacar más a
la máquina que había en el exterior del banco.
a) cajera c) encajonado e) cajero
b) encajar d) cajetilla
4. Lee el siguiente fragmento de un poema de Jorge Man-
rique (c. 1440-1479) y busca en él palabras de una misma
familia léxica. Indaga en el significado exacto de cada una
de ellas dentro del poema:
Diciendo qué cosa es amor
Es amor fuerza tan fuerte una porfía forzosa
que fuerza toda razó n; que no se puede vencer,
una fuerza de tal suerte, cuya fuerza porfiosa
que todo seso convierte hacemos má s poderosa
en su fuerza y afició n; queriéndonos defender.
• fuerza: como sustantivo en es amor fuerza, pero como ver-
bo («forzar, obligar, presionar») en fuerza toda razón. De
la misma familia léxica, los adjetivos fuerte y forzosa.
• porfía (sustantivo) y porfiosa (adjetivo) con el
significado de «lucha obstinada».
5. Di si las siguientes listas de palabras forman campos se-
mánticos o asociativos y justifica tu respuesta:
a) primavera, verano, otoño, invierno
b) rosa, florecimiento, verdor, prado, abril
c) lámpara, linterna, faro, foco, flexo
d) examen, biblioteca, gimnasio, título
e) motocicleta, automóvil, tractor, bicicleta
f) nieve, frío, oscuridad, helada
g) pared, muro, verja, cerca
a) campo semá ntico e) campo semá ntico
b) campo asociativo f) campo asociativo
c) campo semá ntico g) campo semá ntico
d) campo asociativo
Las palabras de los campos asociativos no comparten semas.
6. En los casos en los que has reconocido un campo semán-
5. Elige ahora entre sino y si no para completar las
tico, elabora una tabla de semas como la que has visto
siguientes oraciones:
más arriba.
a) Puedes quedarte a dormir________te apetece andar
Respuesta libre.
hasta tu casa.
EL RINCÓN DE LA NORMA b) Mira,________quieres entenderlo, peor para ti.
c) Este era mi__________levantarme al amanecer para
1. Pon las tildes diacríticas que faltan en las siguientes ora-
ciones: cultivar la tierra.
d) No quiero comer acelgas,________coliflor, que me gus-
Corrige ta más.
a) si no c) sino
a) No se como poner en marcha la tostadora. b) si no d) sino
b) El te era para mi, mas se lo bebió el: ¡que 6. Por último, completa estas oraciones con asimismo, así
rápido fue! mismo o a sí mismo:
c) Si te hubiera hecho caso, aun estaríamos a) Déjalo________, ya lo recogeremos mañana.
esperan- do a Gerardo en la puerta de tu casa. b) La situación es comprometida, pero Ricardo no tiene
d) Mira que cara de imbécil se te ha quedado por qué castigarse________.
cuando has visto que tengo razón. c) Pintaremos la casa;________construiremos un establo
e) ¿Cuanto hace que no se nada de ti? para nuestro caballo.
f) Tu has dado un grito mas agudo que el. d) Todos los días se habla________en voz alta; todos le
toman por loco.
a) No sé cómo poner en marcha la tostadora. a) así mismo c) asimismo
b) El té era para mí, mas se lo bebió él: ¡qué rápido fue! b) a sí mismo d) a sí mismo
c) Si te hubiera hecho caso, aún estaríamos esperando a
Gerardo en la puerta de tu casa. LA FACTORÍA DE TEXTOS
d) Mira qué cara de imbécil se te ha quedado cuando has
visto que tengo razón. 1. Clasifica los programas de algún canal televisivo público
según su finalidad. ¿Has encontrado espacios educativos
e) ¿Cuánto hace que no sé nada de ti?
y culturales? ¿Cuáles?
f) Tú has dado un grito más agudo que él.
Respuesta libre.
2. Teniendo en cuenta lo que acabas de aprender sobre la 2. Compara el lugar que una misma noticia de actualidad
tilde diacrítica, razona por qué no es fiable el corrector ocupa en diversas portadas de la prensa diaria y en los
ortográfico de un ordenador y por qué hemos de revisar espacios informativos de radio y televisión. ¿A qué puede
lo que escribimos. obedecer que los distintos medios le concedan distinta
Los correctores ortográ ficos, si no llevan incorporado un co- importancia a un mismo hecho?
rrector gramatical, no advierten de muchos errores. Aun así, Respuesta libre.
difícilmente detectará los acentos diacríticos, ni tampoco, las
palabras homó nimas o las palabras con doble acentuació n. 3. Define los conceptos de desinformación, contrainforma-
El corrector ortográ fico se guía a través del diccionario que ción y telebasura.
lleva incorporado. Sin embargo, tal repertorio no distingue • Desinformación: control intencionado y, por tanto, mani-
cuando dos té rminos se escriben igual pero pertenecen a pulació n de la informació n que se suministra, omitiendo
categorías gramaticales distintas o incluso se diferencian datos importantes para mantener a los destinatarios en
semá nticamente. una situació n de desconocimiento e ignorancia.
3. Construye oraciones con las siguientes palabras, respe- • Contrainformación: mecanismo contrario a la desinforma-
ció n, consistente en ofrecer una informació n independien-
tando la tilde diacrítica: adonde/adónde; como/cómo;
te y con un enfoque reivindicativo. Es responsabilidad de
cual/cuál; cuando/cuándo; cuanto/cuánto; grupos opuestos al poder político o econó mico. También
donde/dónde; que/qué; quien/quién. se usa el término de contrainformació n para aludir a la
Respuesta libre. informació n falsa que difunde un Estado o un grupo de
4. En las siguientes oraciones incluye porque, porqué, por poder para confundir a sus oponentes.
qué y por que, según convenga: • Telebasura: distingue a un tipo de programas televisivos
de muy baja calidad y con dominio del sensacionalismo.
a) Terminé cansado_________había estado estudiando
toda la noche. 4. Lee un fragmento del artículo de M. García Terán: «Los
adolescentes crearon un extraño lenguaje para chatear»,
b) Nunca sabré________tuviste que tomar esa decisión La Nación (25 de julio de 2004):
tan arriesgada. Advertencia: si usted es un purista del lenguaje, lo que sigue
c) He aquí la explicación de todo, el________de su extra- puede ponerle los pelos de punta. Si no lo es, o está
ño comportamiento. dispuesto a continuar, trate de descifrar este mensaje:
d) Estuvimos velando toda la noche________tuvierais re- «Akabo d ygr a ksa, tki n un rto. slmos mñ na? b.:-)». ¿Un
traductor? No lo dude, llame a su hijo.
suelto el problema cuando os despertarais.
a) porque c) porqué
¿Te parecen descabellados los temores de la autora? Jus-
tifica tu respuesta con datos o argumentos convincentes.
b) por qué d) por que
Respuesta libre.
5. Ahora te toca ser corrector. Formados unos grupos de ción Se hace merecedor del desprecio? ¿Por qué?
trabajo, se elaborará un listado semanal con todas aque-
llas incorrecciones ortográficas que se hayan detectado
en una selección de periódicos y en diversos espacios
televisivos, y se propondrán las soluciones convenientes.
Respuesta libre.

ACTIVIDADES FINALES
Repasa lo que has aprendido
1. Lee este fragmento del artículo de opinión de Luis C.
Pérez, «La violencia creciente contra la discapacidad»,
aparecido el 25 de enero de 2008 en el diario Levante.
En las ú ltimas semanas han saltado a las pá ginas de los dia-
rios noticias muy inquietantes sobre hechos violentos diri-
gidos contra personas con discapacidad, que han tenido la
circunstancia agravante de ser difundidos y publicitados por
Internet.
Distintas personas con discapacidad en Madrid y en el País
Vasco, por citar los casos más recientes, han sido objeto de
agresiones, precedidas de humillaciones y tratos crueles, por
parte de grupos de desaprensivos a los que la discapacidad de
la víctima anima a atacar. Pero la agresió n no acaba ahí, sino
que, elevada a la categoría de espectá culo, ha sido grabada
por medio de telé fonos mó viles y videocá maras, y exhibida
obscenamente a travé s de la red.
Estos hechos tan graves indican un estado mental aú n vigente
en ciertas personas que siguen considerando la discapacidad
como una circunstancia que rebaja al que la presenta, y en
tanto que inferior, se hace merecedor del desprecio, de la burla
y hasta de la violencia. Como su vida, por portar ese
elemento de diversidad, vale menos, se convierte a las
personas con discapacidad en víctimas propiciatorias de la
violencia […].
a) ¿Qué finalidad preside el artículo?
b) ¿Su vocabulario pertenece a un nivel estándar? ¿Por
qué?
c) Imagínate que tú quieres tratar el mismo asunto del
artículo en un foro de Internet. Reescribe de forma
resumida el fragmento.
d) ¿A qué clase pertenecen los sustantivos discapacidad,
gravedad y violencia? Asimismo, teniendo en cuenta
su significado, ¿en qué grupos situarías los
sustantivos teléfonos y videocámaras?
e) En principio, ¿a qué categoría gramatical pertenecen
las palabras desaprensivos y merecedor? Sin
embargo,
¿como qué otra clase gramatical actúan en el texto?
¿Ante qué fenómeno nos hallamos, pues?
f) Analiza los sintagmas. Luego indica qué sintagmas
pre- posicionales aparecen entre ellos:
• distintas personas con discapacidad
• los casos más recientes
• víctimas propiciatorias de la violencia
g) Localiza el sujeto en las oraciones:
• En las últimas semanas han saltado a las páginas
de los diarios noticias muy inquietantes.
• La discapacidad de la víctima anima a atacar.
• La agresión no acaba ahí.
h) ¿Qué tipo de impersonalidad se da en la oración Se
convierte a las personas con discapacidad en víctimas
propiciatorias de la violencia? ¿Es impersonal la ora-
a) El autor del artículo condena e invita a tomar conciencia
sobre la gravedad de los ú ltimos episodios de violencia
contra personas discapacitadas.
b) El registro está ndar es el característico de los textos perio-
dísticos y debe su empleo al intento de que los
contenidos sean entendidos por cualquier lector: para
llegar a la gran mayoría.
c) Respuesta libre.
d) Los sustantivos discapacidad, gravedad y violencia son
abstractos. Por su parte, teléfonos y videocámaras son
concretos, comunes, contables e individuales.
e) Las palabras desaprensivos y merecedor pertenecen a la
categoría de los adjetivos. En el texto, sin embargo, no
actú an como adyacentes directos del sustantivo, sino que
ocupan una funció n nuclear dentro de sus respectivos
sintagmas. De ahí que hayan experimentado un proceso
sustantivador.
f) • Ady + N + CN (enlace + té rmino-SN [N]). SPrep: con
discapacidad
• Det + N + Ady-SAdj (Det + N)
• N + Ady + CN (enlace + té rmino-SN [Det + N]). SPrep:
de la violencia
g) Sujeto:
• noticias muy inquietantes
• La discapacidad de la víctima
• La agresión
h) Impersonal con se. El sujeto de la oració n Se hace merece-
dor del desprecio es muy genérico. Alude a la persona que
padece alguna discapacidad.
2. A partir de la siguiente imagen, diseña una campaña pu-
blicitaria a favor de la integración de las personas con
discapacidad. Crea seis eslóganes diferentes y a conti-
nuación divídelos en sintagmas y analízalos.
Respuesta libre.
3. Francisco de Quevedo enriqueció el léxico castellano
con curiosos vocablos, como demostraría en el célebre
retrato del dómine Cabra, incluido en el capítulo IV de
su relato picaresco El Buscón y que ahora reproducimos.
Léelo atentamente.
É l era un clé rigo cerbatana, largo solo en el talle, una cabeza
pequeñ a […]. Los ojos, avecindados en el cogote, que parecía
que miraba por cué vanos; tan hundidos y obscuros, que era
buen sitio el suyo para tiendas de mercaderes; la nariz, entre
Roma y Francia, porque se le había comido de unas bú as de
resfriado […]. Mirado de media abajo, parecía tenedor, o com-
pá s con dos piernas largas y flacas […]. Traía un bonete los
días de sol, ratonado, con mil gateras y guarniciones de
grasa; era de cosa que fue pañ o, con los fondos de caspa. La
sotana, segú n decían algunos, era milagrosa, porque no se sabía
de qué color era. Unos, viéndola tan sin pelo, la tenían por de
cuero de rana; otros decían que era ilusió n; desde cerca
parecía negra, y desde lejos, entre azul […]; no tenía cuello ni
puñ os; lacayuelo de la muerte. Cada zapato podía ser tumba de
un filisteo. Pues
¿su aposento? Aun arañ as no había en él; conjuraba los ratones,
de miedo que no le royesen algunos mendrugos que guarda-
ba; la cama tenía en el suelo, y dormía siempre de un lado,
por no gastar las sábanas; al fin, era archipobre y
protomiseria.
a) Explica a qué familias léxicas pertenecen las palabras
avecindados, ratonado, milagrosa y protomiseria.
A continuación, escribe al menos dos términos más de
cada familia.
b) En el texto aparecen palabras pertenecientes a los
campos semánticos del calzado y de los muebles.
Localízalas, descomponlas en semas y haz una lista sobrevivirá.
con otras palabras de cada campo semántico. Indica
los dis- tintos semas de cada una.
c) En el texto de Quevedo aparecen las palabras el y él.
Localízalas, explica por qué una de las dos lleva tilde
e indica cuál es la función de cada una de ellas. ¿Para
qué sirve entonces la tilde diacrítica?
a) • avecindados: vecino, vecindad, vecindario.
• ratonado: rata, rató n, ratonera, ratonar.
• milagrosa: milagro, milagrero, milagrería, milagrear.
• protomiseria: miseria, mísero, miserable, miserear.
b) Respuesta libre.
c) La palabra él es pronombre personal tó nico. Como tal, en
fíl era, actú a como sujeto. Por su parte, el término el es un
artículo determinado que funciona como determinante,
acompañ ando al sustantivo en el talle, el cogote. La tilde
diacrítica sirve para diferenciar entre palabras que se es-
criben igual pero que pertenecen a distintas categorías
gramaticales.

Recuerda lo que ya sabías


4. Lee el siguiente fragmento de la novela de Joan
Manuel Gisbert El misterio de la isla de Tökland:
El presidente, con la firmeza de quien decide aplicar una tá c-
tica aceptando de antemano los riesgos que conlleva,
remató :
—Mi ú nico secreto es el laberinto, el má s grande conglome-
rado de misterios de la Historia. ¿No le parece eso bastante?
Solo podrá afirmar que conoce mi secreto quien consiga atra-
vesarlo. Todavía está por ver si usted será capaz de dar con
la salida. Basta ya de inú til discurseo. Dispó ngase a iniciar el
recorrido. Las mil trampas que le aguardan no admiten nin-
guna demora. A partir de este momento, cada minuto cuenta.
El mar está al acecho, redobla su oleaje, desde aquí lo siento.
Si llegamos tarde nada de esto sobrevivirá .
a) ¿Por qué podría afirmarse que el fragmento que aca-
bas de leer es un texto? Justifica tu respuesta.
b) Localiza en el texto una oración enunciativa
afirmativa y otra negativa; luego, encuentra una
interrogativa y otra imperativa.
c) Realizando los cambios oportunos, a partir de Mi
único secreto es el laberinto, redacta una oración
exclamativa, una dubitativa y otra desiderativa.
d) Separa los distintos monemas que integran cada una
de estas palabras: firmeza, conlleva, salida, recorrido,
aguardan, redobla, oleaje, sobrevivirá.
e) Explica el uso de la tilde en las palabras táctica,
Historia, remató, único, está, misterios, inútil,
dispóngase, iniciar, aquí, jamás.
a) El fragmento en cuestió n es un conjunto de enunciados que
mantienen entre sí relaciones de adecuació n, coherencia
y cohesió n, e integra un texto al transmitir un contenido
con significado ló gico. Fijé monos, por ejemplo, có mo el
primer pá rrafo se adecua a las características de la narra-
ció n mediante el uso de los verbos en pretérito perfecto
sim- ple. Asimismo, existe una cohesió n léxica, pues en
un texto donde la intriga es lo primordial ya en el primer
pá rrafo se usa el té rmino riesgos, en el segundo se alude
al inmediato inicio de un recorrido y a las trampas que
pueden toparse los personajes. De forma similar, la urgencia
temporal se revela notable a partir de vocablos o
expresiones como demora, cada minuto cuenta y
b) • Enunciativa afirmativa: El mar está al acecho. nosamente a sus mú ltiples ocupaciones y negocios, estaban
• Enunciativa negativa: Las mil trampas […] no
admi- ten ninguna demora.
• Interrogativa: ¿No le parece eso bastante?
• Imperativa: Dispóngase a iniciar el recorrido.
c) • Exclamativa: ¡Mi único secreto es el laberinto!
• Dubitativa: Posiblemente mi único secreto es el
laberinto.
• Desiderativa: ¡Ojalá mi único secreto fuese el laberinto!
d) • firmeza: firm- (lexema) -eza (morfema derivativo
sufijo)
• conlleva: con- (morfema derivativo prefijo) -llev-
(lexe- ma) -a (morfema flexivo)
• salida: sal- (lexema) -ida (morfema derivativo sufijo)
• recorrido: re- (morfema derivativo prefijo) -corr-
(lexe- ma) -ido (morfema derivativo sufijo)
• aguardan: a- (morfema derivativo prefijo) -guard-
(lexe- ma) -an (morfema flexivo)
• redobla: re- (morfema derivativo prefijo) -dobl- (lexema)
-a (morfema flexivo)
• oleaje: ole- (lexema) -aje (morfema derivativo sufijo)
• sobrevivirá: sobre- (morfema derivativo prefijo)
-viv- (lexema) -irá (morfema flexivo)
e) • táctica: se acentú a como todas las esdrú julas.
• Historia: no se acentú a al ser llana y acabar en vocal.
• remató: se acentú a como aguda acabada en vocal.
• único: se acentú a como todas las esdrú julas.
• está: se acentú a como aguda acabada en vocal.
• misterios: no se acentú a al ser llana y acabar en -s.
• inútil: se acentú a al ser llana y terminar en
consonante distinta de n o s.
• dispóngase: se acentú a como todas las esdrú julas.
• iniciar: no se acentú a al ser aguda y acabar en
conso- nante distinta de n o s.
• aquí: se acentú a como aguda acabada en vocal.
• jamás: se acentú a como aguda acabada en -s.

MIRA A TU ALREDEDOR Y…
… ve más allá
1.El 30 de octubre de 1938 se emitió en la radio, en forma
de noticiario, la adaptación que Orson Welles había
realizado de la novela La guerra de los mundos. En el
programa, de casi una hora de duración, se narraba una
invasión alie- nígena. Como muchos oyentes no habían
escuchado la introducción donde se explicaba que todo
era una drama- tización, en Estados Unidos se desató el
pánico colectivo al darse como cierta la invasión del
planeta.
LOCUTOR 1.— La Columbia Broadcasting System y sus
emisoras asociadas presentan a Orson Welles y el Teatro
Mercurio del Aire en… La Guerra de los Mundos, de H.
G. Wells.
[CONTROL. Sintonía de entrada (versión corta de 22
seg.) hasta final].
LOCUTOR 1.— Señ oras y señ ores, a continuació n el director
del Teatro Mercurio del Aire y protagonista de esta
emisió n… Orson Welles.
ORSON WELLES.— Sabemos ahora que, en los primeros añ os
del siglo XX, nuestro planeta estaba siendo observado
muy atentamente por inteligencias superiores a las del
hombre, aunque tambié n tan mortales como las
nuestras.
Sabemos ahora que mientras los hombres se dedicaban afa-
siendo examinados y estudiados tan minuciosamente como de las nuevas tecnologías.
el hombre mismo hace con un microscopio cuando examina
los microbios que se concentran y multiplican dentro de
una gota de agua.
[…] Individuos plenamente convencidos de su dominio
sobre este pequeñ o planeta del sistema solar, que, por
casualidad, o mejor dicho, por designio divino, el hombre
ha heredado, escapando, así, de la misteriosa oscuridad del
tiempo y del espacio.
Sin embargo, a través del inmenso universo, mentes que
son a nuestras mentes como las nuestras lo son a las de
las bestias de la jungla, inteligencias poderosas, frías y
carentes de sentimientos, contemplaban con envidia nues-
tro planeta Tierra. Seres que lentamente, pero con mucha
seguridad, preparaban un plan contra nosotros.
Fue en el añ o 39 del siglo XX cuando llegó la gran
desilusió n. A finales del mes de octubre sucedió lo
inesperado. Los nego- cios estaban en su mejor momento y
el miedo a la guerra se había alejado. Gran nú mero de
personas encontraba trabajo nuevamente, y las ventas en
los comercios se disparaban.
Era la noche del 30 de octubre. La agencia de noticias
Crossley estimó que unos treinta y dos millones de personas,
en todo el país, tenían, en ese instante, conectada la radio.
a) Recaba información sobre el argumento de la novela
de H. G. Wells La guerra de los mundos y elabora un
breve resumen.
b) ¿A qué género narrativo pertenece la obra? Enumera
otros títulos literarios de la misma índole que se
hayan adaptado al cine. ¿Dónde y cuándo transcurren
tales historias?
c) Lo sucedido tras la emisión del guion de Orson Welles
demuestra el poder de sugestión de los medios de
comunicación. En cambio, Julio Verne escribió rela-
tos cuyo contenido resultaba fantástico en su época y
luego se hizo realidad. Ilustra este curioso fenómeno
con dos novelas del escritor francés.
d) A veces, la realidad supera a la ficción. Recuérdales a
tus compañeros terribles catástrofes que hayan
pasado a la gran pantalla con un éxito clamoroso. ¿En
qué género cinematográfico incluirías cada uno de los
títulos citados?
a) Respuesta libre.
b) La actividad está orientada a que el alumno reconozca el
papel de la narrativa de ciencia ficció n como suministra-
dora de argumentos al cine. Pero se puede ampliar a
otros géneros narrativos como el fantá stico, épico, etc.
c) y d) Respuesta libre.

… encuentra la clave
1. La rápida implantación de las redes sociales y su uso
masivo han suscitado distintas polémicas que plantean la
necesidad de un control y un uso responsable.
a) De entre los inconvenientes que se enumeran a conti-
nuación, extraídos del blog Psicología Para Vivir Mejor
(psicologiavivirmejor.blogspot.com.es), indica los que
te resultan más preocupantes y argumenta por qué:
• Adicción. Se estima que hay un 6 % de adictos a In-
ternet. La red consume una gran cantidad de
tiempo y el exceso de información produce
sobrecarga, di- ficultando la discriminación de
contenidos y el equi- librio en el tiempo utilizado.
• Fracaso escolar entre los adolescentes por el abuso
• Creación de una identidad ficticia gracias al
anonima- to, lo cual promueve el aislamiento social
al reducir los contactos en vivo y sustituirlos por los
virtuales.
• Falta de privacidad. Según la Agencia Española de
Protección de Datos, el 43 % de los usuarios tiene
configurado su perfil de forma que todo el mundo
puede verlo; eso hace que la privacidad pueda ver-
se en peligro. Resulta especialmente preocupante
en el caso de los menores.
• Falta de control de datos. Otras personas pueden
apropiarse de lo que publicamos, incluso puede ha-
ber robos de identidad.
• Riesgos por interactuar con extraños. Los niños y
adolescentes pueden resultar víctimas de pedófi-
los, abusadores sexuales, personas antisociales y
delincuentes de todo tipo.
b) ¿Es posible sentirse solo aunque una persona tenga
muchos amigos en la cuenta de su red social? ¿Cómo
explicarías esta aparente contradicción?
c) Sin embargo, si eres partidario de las nuevas tecno-
logías, ¿cuáles consideras sus principales ventajas?
Anótalas en tu cuaderno.
d) En otro sentido, el uso indiscriminado de la informa-
ción suministrada por Internet puede fomentar prácti-
cas como la piratería. Define los conceptos de plagio,
derecho de autor y propiedad intelectual.
e) ¿Por qué no resulta ético plagiar el trabajo de otras
personas?
a), b) y c) Respuesta libre.
d) Plagio: copia intencionada de una obra ajena para ofrecer-
la como propia.
Derecho de autor: en inglés, copyright, hace referencia a
los derechos morales y econó micos que le corresponden
a un autor como artífice de cualquier obra, ya sea litera-
ria, musical, artística o científica. Su importancia es tal
que está n contemplados en la Declaració n Universal de
los Derechos Humanos.
Propiedad intelectual: la posee el creador sobre cualquier
obra o invenció n surgida de su mente, y engloba tanto a
obras artísticas como a inventos que pasan a ser de uso
habitual.
e) Respuesta libre.
2. Observa la siguiente imagen. Pertenece a una campaña
contra la piratería del Ministerio de Cultura.
a) ¿Qué argumentos esgrime en contra de la piratería?
b) ¿Qué medidas crees que se podrían proponer para
contrarrestarla?
c) ¿Cómo se puede conseguir que la gente cobre con-
ciencia sobre la importancia de proteger el trabajo de
creación?
d) ¿Alguna vez has pirateado algo? ¿Por qué?
a) La imagen subraya la importancia que tienen las ideas en
nuestra vida cotidiana, no solo como fuente de placer y
disfrute (las leemos, las cantamos, bailamos), sino como
instrumento del progreso humano: hacen crecer y vivir y
tener nuevas ideas. Si nosotros recurrimos a la prá ctica
de la piratería y robamos, estamos poniendo en riesgo la
supervivencia de la creatividad y con ella de un patrimo-
nio humano cuyas virtudes se han enfatizado. Con ello,
la imagen implica al destinatario en una cuestió n que nos
afecta a todos y no solo a los creadores.
b), c) y d) Respuesta libre.
ÚLTIMA HORA 5
APERTURA DE UNIDAD produjo, a su entender, la resaca.

1. El texto es un fragmento de la famosa novela de terror


Drácula, publicada en 1897. Lo que acabas de leer refle-
ja el momento en que Jonathan Harker, uno de los per-
sonajes principales, ve por primera vez a Drácula en el
ambiente lúgubre de la cripta de su castillo. ¿Con qué
expresiones consigue el autor crear el ambiente terrorí-
fico que domina la escena?
Hay dos oraciones dentro de este fragmento que
contribuyen claramente a crear la atmó sfera de terror. La
primera es la siguiente: No sabría decir si estaba muerto o
dormido: tenía los ojos abiertos y como petrificados, aunque
sin la vidriosi- dad de la muerte. La incertidumbre de si el
conde está muerto o dormido, unida a esos ojos petrificados
y a la expresió n vidriosidad de la muerte generan el espanto
en el lector. Del mismo modo, cuando al final del texto
leemos: Al ir a regis- trarle vi que sus ojos sin vida,
aunque estaban apagados y no se daban cuenta de mi
presencia, tenían tal expresión de odio, que hui de aquel
lugar, vemos también có mo se produce un nuevo momento de
terror, ya que los ojos sin vida le mues- tran al narrador el odio
que aquel ser a medio camino entre la vida y la muerte le
profesa.
2. De esta novela se han hecho varias versiones cinemato-
gráficas. Ved en clase alguna escena que recree el mo-
mento recogido en el texto y luego, en grupos de cuatro,
comentad las semejanzas y diferencias que hayáis encon-
trado. Para ello, organizaos así:
a) Poned en común vuestras ideas en el grupo.
b) Elegid un portavoz. Este expondrá en voz alta las con-
clusiones.
c) A continuación, se abrirá un debate sobre las dos ver-
siones, la de la novela y la de la película.
d) Valorad ambas versiones y mostrad vuestras prefe-
rencias razonadamente y respetando los turnos de
palabra.
Respuesta libre.
3. En grupos de cuatro, contad cada uno una historia de
terror. A continuación, elegid la que más os ha gustado;
su autor se la contará a la clase.
Respuesta libre.
4. Elige la historia que más te ha gustado de entre las que
han contado tus compañeros y redáctala con tus
palabras en tu cuaderno.
Respuesta libre.

VIVE LA LECTURA
Comprensión lectora
1. ¿Por qué los sueños de Santiago Nasar, relatados en el
primer párrafo, pueden considerarse un mal presagio?
Aparecen una serie de elementos negativos o, al menos, poco
agradables, como sentirse salpicado de cagada de pájaros o
volar en un avió n de papel de estaño. Esto ú ltimo da una
idea de fragilidad muy marcada, como si fuera imposible no
estre- llarse volando en ese avió n. También contribuye a lo
negativo la expresió n no había tenido ningún augurio aciago,
ya que esta confianza de la madre de Santiago parece
subrayar que preci- samente sí había algo aciago, terrible en
los sueñ os de su hijo.
2. ¿A qué atribuye Santiago Nasar el dolor de cabeza con
el que se levantó aquella mañana?
A la fiesta de bodas en la que estuvo la noche anterior, que le

Lengua castellana y Literatura. 3.º ESO. Solucionario 21


5 ÚLTIMA HORA
3. Explica la relación entre la muerte anunciada del cuidado en esa ma- ñ ana que fue terrible para é l.
persona- je y la percepción de sus vecinos de que «era
un tiempo fúnebre, con un cielo turbio y bajo y un
denso olor de aguas dormidas». ¿Qué valor simbólico
les atribuyes a los elementos enumerados (tiempo
fúnebre, cielo turbio y bajo, aguas dormidas)? Presta
especial atención a los adjetivos.
Muy a menudo asociamos el tiempo meteoroló gico con
nues- tros estados de á nimo, por lo que aquí, el tiempo
nublado, lluvioso y desapacible de esos días parece tener
una relació n con la tristeza, tan cercana a la muerte. Las
expresiones que emplea el autor contribuyen a esa sensació n
de muerte, sobre todo a través de los adjetivos: el primero,
fúnebre, remite di- rectamente a ella, los demá s subrayan la
oscuridad y la tris- teza: cielo turbio, aguas dormidas. No
es difícil relacionar estas sensaciones con la muerte.
4. ¿Quién crees que puede ser el narrador? ¿Tendrá algún
tipo de relación con Santiago Nasar? Razona tus
respuestas.
El narrador parece ser un amigo o conocido cercano de
San- tiago Nasar, que cuenta la historia de primera mano,
como observador presente en ella. Esto lo podemos deducir
de que afirma haber estado en la misma fiesta que el
protagonista (Yo estaba reponiéndome de la parranda de
la boda…). Ade- má s, cuando las campanas tocaron a
rebato, é l pudo oírlas mientras se encontraba pasando la
noche con María Alejan- drina Cervantes, por lo tanto vivía
en el mismo pueblo que Nasar. También conocía a la madre
de este, quien le contó los sueñ os premonitorios que no
supo interpretar.
5. Explica la expresión «era un atuendo de ocasión», según
el contexto en el que se encuentra.
La expresió n atuendo de ocasión aparece en el siguiente frag-
mento, ahora omitido en la lectura: «Santiago Nasar se puso
un pantaló n y una camisa de lino blanco, ambas piezas sin
al- midó n, iguales a las que se había puesto el día anterior
para la boda. Era un atuendo de ocasió n. De no haber sido
por la llegada del obispo se habría puesto el vestido de
caqui y las botas de montar con que se iba los lunes a El
Divino Rostro, la hacienda de ganado que heredó de su
padre». Quiere decir que era una forma de vestir ocasional,
con la que no solía ataviarse.
6. Sabiendo desde el principio del texto que van a matar al
protagonista, ¿cómo interpretarías la siguiente
oración?:
«Siempre dormía como durmió su padre, con el arma
es- condida dentro de la funda de la almohada, pero
antes de abandonar la casa aquel día le sacó los
proyectiles y la puso en la gaveta de la mesa de noche».
¿Tiene un com- ponente irónico?, ¿marca la fatalidad
del destino? Razona tu respuesta.
En efecto, la oració n tiene un tono iró nico, ya que si no hubiera
descargado la pistola, tal vez la hubiera llevado luego en el
bolsillo cuando salió por la mañ ana, antes de que lo
mataran. El destino del protagonista parece estar marcado,
pues a la incapacidad de su madre de interpretar los sueñ os
que tuvo hay que unir tambié n la casualidad de que,
precisamente esa noche, no dejara cargada la pistola.

Vocabulario
7. ¿Qué valor tiene en el texto el adverbio tampoco con
el que comienza el segundo párrafo?
El adverbio tampoco sirve para enlazar lo que se cuenta en el
primer pá rrafo con lo que se cuenta en el segundo, y,
desde el punto de vista del contenido, suma una
circunstancia má s que impidió que Santiago Nasar tuviera
22 Lengua castellana y Literatura. 3.º ESO. Solucionario
8. Busca los adjetivos que hay en el primer párrafo y explica e) Me gustan los ojos azules de Sara.
luego qué sensaciones producen en el lector (positivas, f) El pobre anciano estaba muy cansado de la vida.
negativas, de esperanza, de inquietud, de alegría, de tris-
teza, etc.).
Los principales adjetivos son tierna, feliz, ingrato y aciago.
Como vemos, mientras los dos primeros comunican una sensa-
ció n agradable, placentera y tranquila, los dos ú ltimos contras-
tan con ellos por comunicar todo lo contrario: lo
desagradable y lo terrible. La felicidad que muestra el estado
de á nimo del protagonista chocará con la inquietud y los malos
presagios que parecían indicar sus sueñ os y que nadie fue
capaz de descifrar.
9. Busca en el diccionario el significado de las siguientes
palabras del texto y escribe luego una oración con cada
una de ellas: estragos, parranda, destazado, presagio.
Estragos: ruina, dañ o, asolamiento.
Parranda: juerga bulliciosa, especialmente la que se hace
yendo de un sitio a otro.
Destazado: hecho pedazos.
Aciago: infausto, infeliz, desgraciado, de mal agü ero.
10. En el siguiente fragmento cambia las palabras mar-
cadas en negrita por otras que tengan el mismo significa-
do o equivalente. Si no encuentras una palabra, puedes
emplear también una expresión que signifique lo mismo:
Tenía una reputación muy bien ganada de intérprete certera
de los sueñ os ajenos, siempre que se los contaran en
ayunas, pero no había advertido ningú n augurio aciago en
esos dos sueñ os de su hijo, ni en los otros sueñ os con á rboles
que é l le había contado en las mañ anas que precedieron a su
muerte.
Tenía una fama muy bien ganada de descifradora certera
de los sueñ os de los otros, siempre que se los contaran antes
de comer, pero no había percibido ningú n presagio aciago
en esos dos sueñ os de su hijo, ni en los otros sueñ os con
á rboles que é l le había contado en las mañ anas que
antecedieron a su fallecimiento.

Investigación y redacción
11. Busca información acerca de la novela a la que
pertenece el texto y sobre su versión cinematográfica.
Recoge los datos en tu cuaderno: director de la película,
fecha de estreno, actores principales, resumen del
argumento, etc.
Respuesta libre.
12. Compara los argumentos de la novela y la película y haz
una lista con las diferencias que hayas encontrado.
Respuesta libre.
13. Investiga sobre las características del realismo mágico e
indica si encuentras alguna en el fragmento que acabas
de leer.
Respuesta libre.

ESTUDIO DE LA LENGUA
1. En las siguientes oraciones señala el sintagma adjeti-
val (SAdj) e identifica su estructura en función de sus
componentes:
a) Tenían la cabeza llena de fantasías.
b) La madre de Diana es una mujer muy alta.
c) Pintaron el coche de amarillo chillón.
d) El café estaba demasiado caliente y nos quemamos la
lengua.
g) Los niños estaban hartos de papilla.
h) El color azul marino es el preferido de Noelia.
i) Alfonso está bastante satisfecho de las notas de su
hijo.
a) Tenían la cabeza llena de fantasías.
Adj + SPrep
b) La madre de Diana es una mujer muy alta.
Adv + Adj
c) Pintaron el coche de amarillo chillón.
Adj + Adj
d) El café estaba demasiado caliente y nos quemamos la
lengua.
Adv + Adj
e) Me gustan los ojos azules de Sara.
Adj
f) El pobre anciano estaba muy cansado de la vida.
Adv + Adj + SPrep
g) Los niños estaban hartos de papilla.
Adj + SPrep
h) El color azul marino es el preferido de Noelia.
Adj + Adj
i) Alfonso está bastante satisfecho de las notas de su
hijo.
Adv + Adj + (SPrep + SPrep)
2. En las oraciones anteriores hay algunos cuantificadores.
Localízalos y escríbelos en tu cuaderno.
Cuantificadores: muy alta, demasiado caliente, muy can-
sado, bastante satisfecho.
3. Clasifica los siguientes adjetivos según sean de una o dos
terminaciones en cuanto al género:
vulgar anatómico terrible
atroz inquieto malvado
absurdo medieval maloliente
salvaje imprudente inca
rapaz indiferente estudioso

Una terminación Dos terminaciones


vulgar anatómico, -a
terrible inquieto, -a
atroz malvado, -a
medieval absurdo, -a
maloliente estudioso, -a
salvaje
imprudente
inca
rapaz
indiferente

4. Forma oraciones con los adjetivos de la actividad ante-


rior, poniendo especial atención a la concordancia entre
estos y el sustantivo al que califiquen. Para ello modifica
el género y el número tantas veces como sea necesario.
Respuesta libre.
5. En las siguientes oraciones identifica la función que des-
empeña el sintagma adjetival:
a) Los jugadores volvieron derrotados del campeonato.
b) Contemplábamos el cielo rojizo del atardecer. e) mucho
c) Caminar es bueno para la salud. Adv
d) Las luces de la habitación estaban apagadas. f) bien / ayer
e) En casa comprobamos que habíamos comprado los Adv / Adv
yogures caducados.
9. Analiza las funciones que desempeñan los sintagmas ad-
a) Complemento predicativo (derrotados).
verbiales de la actividad 8.
b) Adyacente (rojizo).
a) Complemento circunstancial de modo.
c) Atributo (bueno para la salud).
d) Atributo (apagadas).
b) Complemento circunstancial de modo.
e) Complemento predicativo (caducados). c) Complemento circunstancial de tiempo / Complemento
circunstancial de tiempo / Complemento circunstancial
6. Di cuál es el adjetivo positivo que corresponde a los si- de lugar.
guientes superlativos: libérrimo, nobilísimo, felicísimo,
agra- dabilísimo, sapientísimo, fortísimo, altísimo, d) No hay SAdv.
celebérrimo. Puedes ayudarte de un diccionario. e) Complemento circunstancial de cantidad.
libérrimo libre f) Complemento circunstancial de modo / Complemento cir-
cunstancial de tiempo.
nobilísimo noble
felicísimo feliz 10. Localiza en las siguientes oraciones los adverbios de can-
agradabilísimo agradable tidad:
sapientísimo sabio a) A Julia le gusta bastante el arroz con leche.
fortísimo fuerte b) Tuvieron bastante tiempo para hacer el trabajo.
altísimo alto
c) Había muchas ovejas pastando en aquel prado.
celebérrimo célebre
d) Corrieron mucho y acabaron muy cansados.
7. Transforma los adjetivos positivos extraídos de la activi- e) Se queja demasiado demasiadas veces.
dad anterior en comparativos de inferioridad, de supe- a) bastante
rioridad y de igualdad, y construye una oración con cada d) mucho / muy
uno de ellos.
e) demasiado
Inferioridad: menos libre que, menos noble que, menos feliz
que, menos agradable que, menos sabio que, menos fuerte 11. En las siguientes oraciones, identifica las locuciones ad-
que, menos alto que, menos célebre que. verbiales, sustitúyelas por un adverbio y di luego qué
Superioridad: más libre que, más noble que, más feliz que, función sintáctica desempeñan:
más agradable que, más sabio que, más fuerte que, más alto a) El hijo de Paqui no tiene ni un pelo de tonto.
que, más célebre que.
b) Los especuladores gastaron el dinero a manos
Igualdad: tan libre como, tan noble como, tan feliz como, tan
llenas en los años de abundancia económica.
agradable como, tan sabio como, tan fuerte como, tan alto
como, tan célebre como. c) El instituto en el que estudio está donde da la vuelta
Respuesta libre. el aire.
8. Indica la estructura de los sintagmas adverbiales que d) Los invitados al banquete comían a dos carrillos, sin
apa- recen en las siguientes oraciones: hablar con sus compañeros de mesa.
a) Tiraron bien fuerte y consiguieron abrir la puerta. e) Tus problemas con la informática me traen sin
b) Sospecho que tu amiga Carla me mira mal. cuidado, querido.
c) Mañana será tarde para viajar tan lejos. f) En el cumpleaños de Sara lo pasamos de maravilla.
d) Es el payaso más gracioso del mundo. g) Revisamos a conciencia todo el texto, por si había al-
e) Hemos trabajado mucho para superar esta prueba gún error.
tan dura. h) A ti te salen las tortitas de rechupete.
f) Los niños se portaron bien en la fiesta celebrada ayer. i) Se leyó la historia de cabo a rabo.
a) bien fuerte j) Acabé los deberes de matemáticas en un periquete.
Adv + Adv (adjetivo adverbializado). a) El hijo de Paqui no tiene ni un pelo de tonto.
b) mal El hijo de Paqui es muy listo.
Adv
La locució n compone el predicado, lo mismo que la alterna-
c) Mañana / tarde / tan lejos tiva que proponemos. Ni un pelo de tonto es complemento
Adv / Adv / Adv + Adv directo; muy listo, atributo.
d) Es el payaso más gracioso del mundo. b) Los especuladores gastaron el dinero a manos llenas en
No hay sintagma adverbial; tan solo un adverbio (más) los años de abundancia económica.
complementando como cuantificador a un adjetivo Los especuladores gastaron el dinero abundantemente
(gracioso). en los años de abundancia económica.
Complemento circunstancial de modo.
c) El instituto en el que estudio está donde da la vuelta
el aire. EL TALLER DE LAS PALABRAS
El instituto en el que estudio está muy lejos. 1.Averigua, con ayuda de un diccionario, la procedencia de
Complemento circunstancial de lugar. los siguientes préstamos léxicos y anótala en tu cuaderno
d) Los invitados al banquete comían a dos carrillos, sin
junto con su significado: holding, bigote, mejillón,
hablar con sus compañeros de mesa. capricho, blues, garaje, pizzicato, alcoba, affaire, pirsin,
biombo, alio- li, paparazzi, abertzale, vigía, zaguán, café,
Los invitados al banquete comían mucho, sin hablar con
sus compañeros de mesa.
viaje, rueca, chef, bandeja, grafiti, élite, footing, brindis,
huracán, esqui- rol, glamour, beicon, bigudí, chirimbolo,
Complemento circunstancial de cantidad.
chamizo, acequia, caramelo, tomate, bajel, albariño,
e) Tus problemas con la informática me traen sin cuidado, zurrón, guiñol, chao.
querido.
• holding: anglicismo. Sociedad que controla las acciones de
Tus problemas con la informática me importan poco, un grupo de empresas.
querido. • bigote: germanismo. Pelo que crece sobre el labio superior.
Complemento circunstancial de cantidad (poco). • mejillón: lusismo. Un tipo de molusco.
f) En el cumpleaños de Sara lo pasamos de • capricho: italianismo. Determinació n que se hace de ma-
maravilla. En el cumpleaños de Sara lo pasamos nera arbitraria, sin razonar mucho.
muy bien. Complemento circunstancial de modo. • blues: anglicismo. Mú sica de origen africano propia de los
g) Revisamos a conciencia todo el texto, por si había algún Estados Unidos.
error. • garaje: galicismo. Lugar cerrado donde aparcar un vehículo.
Revisamos muy bien todo el texto, por si había algún • pizzicato: italianismo. Té cnica para tocar el violín y otros
error. instrumentos de cuerda, tirando con los dedos de estas
cuerdas, como si se pellizcaran.
Complemento circunstancial de modo.
• alcoba: arabismo. Dormitorio.
h) A ti te salen las tortitas de rechupete.
• affaire: galicismo. Negocio o asunto poco lícito.
A ti te salen las tortitas estupendamente.
• pirsin: anglicismo. Perforació n en una parte del cuerpo
Complemento circunstancial de modo. distinta de la oreja.
i) Se leyó la historia de cabo a rabo. • biombo: lusismo. Mampara mó vil para separar espacios.
Se leyó la historia completamente. • alioli: catalanismo. Ajo y aceite, un tipo de salsa hecha
Complemento circunstancial de modo. con esos ingredientes y huevo.
j) Acabé los deberes de matemáticas en un periquete. • paparazzi: italianismo. Fotó grafo, por lo general de la
Acabé los deberes de matemáticas inmediatamente. prensa rosa.
Complemento circunstancial de tiempo. • abertzale: vasquismo. Patriota, nacionalista.
• vigía: lusismo. Persona que vigila, generalmente desde un
12. Explica qué significan las siguientes locuciones
faro o una atalaya elevada.
adverbiales y construye una oración con cada una de
• zaguán: arabismo. Portal de una casa.
ellas. Si lo nece- sitas, puedes utilizar un diccionario; en
este caso, busca la palabra más importante de la locución • café: italianismo. Bebida que se elabora con la infusió n de
(pierna, ojo, etc.): a pierna suelta, en un abrir y cerrar de la planta del mismo nombre.
ojos, a tontas y a locas, por un tubo, a la chita callando, • viaje: catalanismo. Acció n de desplazarse de un lugar a otro.
de memoria, en mi vida, a chorros, en un santiamén, a lo • rueca: germanismo. Instrumento para hilar.
bestia, al pie de la letra, de primera mano. • chef: galicismo. Cocinero, jefe de cocina.
• a pierna suelta: con absoluta tranquilidad. Se emplea con • bandeja: lusismo. Recipiente plano que sirve para trans-
el verbo dormir. portar bebidas o comidas.
• en un abrir y cerrar de ojos: con gran rapidez, de manera • grafiti: italianismo. Letrero o dibujo que, por lo general, se
inmediata. hace en las paredes.
• a tontas y a locas: sin pensarlo demasiado, hecho de ma- • élite: galicismo. Grupo selecto de personas.
nera irreflexiva. • footing: galicismo de procedencia inglesa. Carrera a velo-
• por un tubo: en abundancia. cidad moderada.
• a la chita callando: de manera disimulada, procurando que • brindis: germanismo. Acció n de brindar, beber expresando
no se entere nadie. algú n deseo.
• de memoria: sin olvidar nada, sin error. • huracán: americanismo. Viento muy fuerte.
• en mi vida: nunca, jamá s. • esquirol: catalanismo. Persona que trabaja durante una
jornada de huelga, con intenció n de perjudicar esta.
• a chorros: de manera abundante, derrochando.
• glamour: galicismo. Encanto sensual.
• en un santiamén: inmediatamente.
• beicon: anglicismo. Panceta ahumada.
• a lo bestia: sin límites, de forma desmesurada.
• bigudí: galicismo. Pequeñ o instrumento utilizado para en-
• al pie de la letra: palabra por palabra, sin dejarse nada sin sortijar el cabello.
hacer o decir.
• chirimbolo: vasquismo. Objeto de forma extrañ a que no
• de primera mano: sin intermediarios. Se suele emplear tiene un nombre concreto.
junto al sustantivo información.
• chamizo: galleguismo. Refugio hecho con ramas y hojas
de á rboles.
• acequia: arabismo. Conducto de agua para el riego de las
• michelín: marca de neumá ticos. Tambié n pliegues que se
huertas.
forman en alguna parte del cuerpo, sobre todo la tripa,
• caramelo: lusismo. Golosina hecha con azú car fundido y por semejanza con la figura que publicita esos neumá ticos.
endurecido. Causas psicoló gicas.
• tomate: americanismo. Fruto de la tomatera. • enchufar: conectar un enchufe a la red elé ctrica. También
• bajel: catalanismo. Barco. colocar a una persona en un puesto de trabajo por amis-
• albariño: galleguismo. Vino blanco afrutado que se elabora tad, familiaridad o cualquier otra causa irregular. Causas
en Galicia. psicoló gicas.
• zurrón: vasquismo. Bolsa grande hecha de cuero o de otro 4. Busca eufemismos para los siguientes tabúes: negro
material. (raza), ciego, parir, basurero, retrete, morir, huelga, viejo
• guiñol: galicismo. Teatro de títeres. (persona), mear, vagabundo, criada, culo, gordo, loco,
• chao: italianismo. Adió s. se utiliza como saludo. mendigo.
2. Los siguientes anglicismos son innecesarios. Escribe una • negro: de color, moreno, afroamericano.
palabra española que pueda sustituirlos: business, spon- • ciego: invidente, deficiente visual.
sor, pin, doping, hall, póster, mailing. • parir: dar a luz, alumbrar.
• business: negocio. • basurero: estació n de residuos só lidos urbanos.
• sponsor: patrocinador. • retrete: vá ter, aseo, bañ o, servicio.
• pin: insignia. • morir: fallecer.
• doping: dopaje.
• huelga: conflicto laboral.
• hall: vestíbulo, recibidor.
• viejo: anciano, persona mayor.
• póster: cartel.
• mear: orinar.
• mailing: buzoneo, correo.
• vagabundo: sin techo, sin hogar.
3. Con ayuda de un diccionario, explica razonadamente qué
tipos de cambios semánticos se han producido en las si- • criada: empleada de hogar, asistenta.
guientes palabras: retrete, pringar, portátil, pluma, ven- • culo: trasero, pompis.
tana, afroamericano, satélite, embarcar, centro de salud, • gordo: grueso, rellenito.
cabina, techo, michelín, enchufar. • loco: enfermo mental.
• retrete: pasa de referirse a un cuarto apartado para reti- • mendigo: sin techo, sin hogar.
rarse y estar en soledad, a un cuarto para realizar las ne-
cesidades fisioló gicas. Causas histó ricas. 5. Las siguientes palabras han experimentado algún tipo de
• pringar: impregnar algo de una sustancia pegajosa o prin- cambio semántico. Construye oraciones con ellas en las
gosa. De ahí pasa a significar comprometer a alguien en que se perciban las diferencias entre sus significados y
algo poco agradable. Causas psicoló gicas. ex- plica de qué tipo son los cambios producidos:
• portátil: objeto que se puede llevar de un lugar a otro. Ac- caballero, navegar, azafata, portal, chato, vinilo, red,
tualmente un tipo de ordenador que se puede transportar gorila.
con facilidad. Por el contexto. Respuesta libre.
• pluma: cada una de las piezas que cubren el cuerpo de 6. Intenta explicar el porqué de las siguientes etimologías
las aves. Pasó a significar instrumento para escribir con populares: mondarina (por mandarina), destornillar-
tin- ta, por fabricarse los primeros con estas piezas. se (por desternillarse), andalias (por sandalias),
Causas histó ricas.
alicóptero (por helicóptero), esparatrapo (por
• ventana: abertura en un muro de un edificio. Ahora, tam- esparadrapo), alqui- lino (por inquilino), lenteojos (por
bié n, espacio delimitado en la pantalla de un ordenador.
anteojos), doble dosis (por sobredosis), doctor Rino (por
Causas histó ricas.
otorrino).
• afroamericano: causas sociales. Es un eufemismo para
evitar la palabra negro, de connotaciones racistas, consi- Respuesta libre.
derada un tabú .
EL RINCÓN DE LA NORMA
• satélite: cuerpo celeste que gira alrededor de un planeta.
Por extensió n, persona que depende de otra y que 1. Explica por qué colocamos la diéresis en pingüino o pira-
siempre anda tras ella. Causas psicoló gicas. güismo y no en guarro o piragua.
• embarcar: subir a un barco o a un avió n para realizar un La diéresis se pone cuando los grupos gue y gui deben pro-
viaje. Tambié n implicar a alguien en algo que puede ser nunciarse distinguiendo la u y la i por separado, lo cual no
poco grato o costoso. Causas psicoló gicas. afecta nunca a los grupos con las vocales a y o (gua, guo). Por
• centro de salud: causas sociales. Eufemismo que pretende eso sí se pone en pingüino y piragüismo, pero no en guarro
evitar palabras como hospital o ambulatorio, que se con- y piragua.
sideran tabú es por su relació n con la enfermedad. 2. Estas palabras designan especialidades médicas: odon-
• cabina: recinto pequeñ o para usos diversos. También, entre tología, ginecología, cardiología, urología. ¿Qué regla
otros muchos significados, lugar desde el que se ortográfica exige aquí usar la g? ¿Y en los numerales or-
proyectan las películas en el cine. Causas histó ricas.
dinales: vigésimo, trigésimo, etc.?
• techo: cubierta de un edificio. Se entiende también como
Odontología, ginecología, etc. se escriben con g porque la re-
el puesto más alto logrado por alguien en un determinado
gla dicta que sea así con las secuencias gia o gio, lleven tilde
trabajo. Causas psicoló gicas.
o no la lleven.
También se escriben con g las palabras terminadas en -gé-
simo, -gésima y -gesimal. De ahí que los numerales de esta
actividad deban escribirse con g.
3. Resuelve en tu cuaderno estos revoltigramas: con mayor frecuencia, de la casa al contenedor amarillo. Los
QIEPAJUE:  _ Q_____J _ hogares españ oles reciclaron el 56,6 % de las 656.000 toneladas
ONOSGITACO:  C _ _ _ _ G _ _ S _ puestas en el mercado en 2013, una cantidad que convierte al
país en el segundo de la Unió n Europea que má s recicla el
XODEAREAG:  E _ _ G _ _ _ _ _ material por habitante, segú n el informe que este martes
QIEPAJUE  EQUIPAJE presentó Cicloplast.
ONOSGITACO  CONTAGIOSO Los españ oles separaron 371.218 toneladas de plá sticos el añ o
pasado, un 3,7 % más que el añ o anterior y tres veces más que
XODEAREAG  EXAGERADO hace una década. Con este volumen se llenarían 28 estadios
4. Completa en tu cuaderno las palabras siguientes con g o del tamañ o del Santiago Bernabéu.
j y forma con ellas diversas oraciones: Cada ciudadano recicló 7,7 kilogramos del material, una cifra
va_illa dili_ente pota_e pá_ina que se encuentra por encima de la media europea —de 7,1 ki-
logramos por habitante— y por delante de países como Reino
sumer_ir ru_ido analo_ía esp_ismo
Unido, Italia, Bé lgica y Finlandia. Alemania encabeza la lista
tatua_e an_inas extra_eron relo_ería con 12 kilogramos por habitante.
co_era in_erto hemorra_ia ori_en A pesar de los resultados, Españ a «no puede dormirse», ma-
pilla_e farin_e _imnasia falan_e tizó Teresa Martínez, directora general de Cicloplast. Si bien
el país «supera y duplica» la meta de reciclado establecida
ve_etal su_eto re_ente _eneral por la directiva europea en 2008, que se sitú a en el 22,5 %,
• vajilla • diligente • potaje • página Bruselas ya ha marcado un nuevo objetivo para 2025: un
• sumergir • rugido • analogía • espejism 60 % de reciclado de plá sticos y cero residuos reciclables en
o los vertederos.
• tatuaje • anginas • extrajeron • relojería El País, 30 de julio de 2014.
• cojera • injerto • hemorragia • origen En la noticia podemos distinguir bien las tres partes princi-
• pillaje • faringe • gimnasia • falange pales que la componen:
• vegetal • sujeto • regente • general • El titular: Los hogares españoles, los segundos en reciclaje
5. Completa las siguientes palabras con la grafía correcta: de plástico en la Unión Europea.
her_í_ora, ama_ilidad, _i_liotecario, _íbora, absor_er, • La entradilla: Separaron el 56,6 % de las 656.000 toneladas
absol_ er, _icepresidenta, _iodegradable, _ete, e_idente, puestas en el mercado en 2013, según Cicloplast.
é_ano, disol_er, aperci_ir, contri_uir, mantu_imos,
• El cuerpo: el resto de la noticia.
_ipartidista, andu_o, tu_ieron, _enefactor, í_amos,
estudia_a, _ioquími- ca, decisi_o, bre_e. 2. ¿Cuáles de los factores de importancia (actualidad, proxi-
• herbívora • amabilidad • bibliotecario midad, magnitud e interés humano) están presentes en
la noticia anterior? Razónalo.
• víbora • absorber • absolver
El principal factor de importancia en este caso tal vez sea el
• vicepresidenta • biodegradable • vete
de la proximidad, ya que habla del porcentaje elevado y
• evidente • ébano • disolver satis- factorio obtenido por Españ a en el reciclaje de
• apercibir • contribuir • mantuvimos plá sticos. La noticia nos toca de cerca pues dice algo positivo
• bipartidista • anduvo • tuvieron sobre nuestro país. También es importante el interés humano,
ya que mues- tra la buena conciencia ecoló gica de un elevado
• benefactor • íbamos • estudiaba
porcentaje de españ oles. Incluso podríamos decir que es
• bioquímica • decisivo • breve actual, ya que en los tiempos que corren se le da mucha
6. Copia en tu cuaderno el siguiente dictado. Pon atención importancia al reciclaje de residuos contaminantes.
a las grafías b/v: 3. ¿Crees que se respetan en esta noticia las características
a) Es obvio que van a disolver el gobierno a causa de lingüísticas propias del género? ¿Por qué? Analiza cada
la evasión del vicepresidente. una de ellas.
b) Si escribes con letra tan grande, no cabe todo el Respuesta libre.
tema de biología. 4. Busca información en Internet sobre el tema del recicla-
c) Me han atribuido una burla incisiva que yo no tuve je y elabora un reportaje en el que incluyas estadísticas,
a bien realizar. imágenes y otros datos de interés.
d) Elvira anduvo toda la víspera atareada preparando Respuesta libre.
un bizcocho nuevo. 5. ¿Te parece que es importante reciclar? Elabora una re-
Respuesta libre. dacción en la que manifiestes tu opinión.
Respuesta libre.
LA FACTORÍA DE TEXTOS
1. Lee la noticia e identifica las partes que la ACTIVIDADES FINALES
componen. Los hogares españoles, los segundos Repasa lo que has aprendido
en reciclaje de plástico en la Unión Europea 1.El siguiente texto es el primer capítulo de Platero y yo, de
Separaron el 56,6 % de las 656.000 toneladas puestas en Juan Ramón Jiménez. Léelo atentamente y responde des-
el mercado en 2013, segú n Cicloplast. pués a las actividades.
Botellas de agua y refrescos, envases de comida o detergen- Platero es pequeñ o, peludo, suave; tan blando por fuera, que
tes, bolsas, bandejas, tarrinas… Los plásticos van, cada vez se diría todo de algodó n, que no lleva huesos. Solo los
espejos
de azabache de sus ojos son duros cual dos escarabajos de
c) Se trata de un comparativo de igualdad, ya que muestra
cristal negro.
la cualidad de la ternura de Platero asociada a la de un
Lo dejo suelto, y se va al prado, y acaricia tibiamente con niñ o y la equipara con este.
su hocico, rozá ndolas apenas, las florecillas rosas, celestes
d) Se trata de un adjetivo explicativo, ya que va delante del
y gualdas… Lo llamo dulcemente: «¿Platero?», y viene a mí
sustantivo y aporta una informació n que no es imprescin-
con un trotecillo alegre que parece que se ríe, en no sé qué
dible para entender la oració n. Sí es un epíteto, puesto que
cascabeleo ideal…
la gota de miel, en sí misma, es cristalina, transparente.
Come cuanto le doy. Le gustan las naranjas mandarinas, las
e) Adverbios: tan (adverbio de cantidad, cuantificador),
uvas moscateles, todas de á mbar; los higos morados, con su
solo (adverbio de modo, complemento circunstancial de
cristalina gotita de miel…
modo), tibiamente (adverbio de modo, complemento
Es tierno y mimoso igual que un niñ o, que una niñ a…; pero circunstan- cial de modo), apenas (adverbio de negació n,
fuerte y seco por dentro, como de piedra. Cuando paso sobre complemen- to circunstancial de modo), dulcemente
é l, los domingos, por las ú ltimas callejas del pueblo, los hom- (adverbio de modo, complemento circunstancial de modo),
bres del campo, vestidos de limpio y despaciosos, se quedan dentro (adverbio de lugar, complemento circunstancial de
mirá ndolo: lugar), cuando (ad- verbio de tiempo, complemento
—Tien’ asero… circunstancial de tiempo).
Tiene acero. Acero y plata de luna, al mismo tiempo. f) Todas es un pronombre indefinido, ya que está sustituyen-
a) Busca los adjetivos que hay en el texto y clasifícalos do a las uvas moscateles que acaba de citar.
según sean de una o dos terminaciones en cuanto al 2. En el siguiente fragmento de la égloga I de Garcilaso de la
género. Vega aparecen varios epítetos. Localízalos y explica
qué crees que aportan al contenido del texto.
Una terminación Dos terminaciones
Por ti el silencio de la selva umbrosa,
por ti la esquividad y apartamiento
del solitario monte me agradaba;
por ti la verde hierba, el fresco viento,
b) Di qué funciones desempeñan los sintagmas adjetiva- el blanco lirio y colorada rosa
les siguientes: y dulce primavera deseaba.
• Platero es pequeño Selva umbrosa, solitario monte, verde hierba, fresco
• escarabajos de cristal negro vien- to, blanco lirio, colorada rosa, dulce primavera.
• lo dejo suelto Los epítetos, marcados en negrita, repiten todos cualidades
que ya está n presentes en los sustantivos a los que acompa-
• un trotecillo alegre ñ an, por lo que su aportació n al texto es el color, el ambiente,
c) ¿Qué grado del adjetivo está presente en la siguiente lo sensitivo: todos ellos refuerzan el significado de los sustan-
frase: «Es tierno y mimoso igual que un niño»? tivos e invitan al lector a disfrutar con los sentidos del paraje
Razona tu respuesta. que se describe.
d) Por su posición y su significado, ¿qué tipo de adjetivo 3. Abusar de los adverbios terminados en -mente resulta
es cristalina en la expresión «con su cristalina gotita poco elegante y denota pobreza léxica. Sustituye los del
de miel»? ¿Crees que podría ser un epíteto? ¿Por siguien- te texto por otros que no tengan esa
qué? terminación o por expresiones equivalentes:
e) Localiza en el texto los adverbios presentes, di de qué Te diría que prá cticamente recientemente hemos comprado
tipo son y analiza sus funciones. el coche que ú ltimamente llevamos a casi todos los viajes,
f) En el sintagma todas de ámbar, ¿qué tipo de palabra porque en él se va perfectamente y divinamente, sobre todo
cuando ves a muchos conductores que caprichosamente te
es todas? Explica por qué.
pasan velozmente y tú , tranquilamente, sigues a tu ritmo,
a) tan ricamente, sin necesidad de conducir locamente, que hay
Una terminación Dos terminaciones muchos que probablemente piensan que la carretera es mis-
suave pequeño mamente un circuito.
rosas peludo Te diría que apenas hace unas semanas hemos comprado
celestes blando el coche que en los últimos días llevamos a casi todos los
gualdas duros
viajes, porque en él se va muy bien y muy a gusto, sobre todo
cuando ves a muchos conductores que por capricho te
alegre negro
pasan a gran velocidad y tú , tan tranquilo, sigues a tu ritmo,
ideal morados tan campan- te, sin necesidad de conducir a lo loco, que hay
fuerte cristalina muchos que tal vez piensan que la carretera es sin más un
tierno circuito.
mimoso 4. Lee la siguiente noticia y resuelve luego las actividades:
seco Una pitón de un metro lleva cuatro días suelta por
despaciosos Zaragoza
La serpiente se escapó de un piso el lunes, cuando su
b) • Atributo.
dueñ o salió a comprarle comida.
• Adyacente.
El Ayuntamiento emite un mensaje de tranquilidad por-
• Complemento predicativo.
que el animal no es peligroso.
• Adyacente.
El propietario ha presentado la correspondiente denuncia de
extravío.
Una pitó n de un metro de largo, que al parecer se escapó de • Cuá ndo: en el momento en que Cristian Gó mez salió de
un piso de Zaragoza el pasado lunes cuando su dueñ o salió a su casa para comprar un rató n para la comida de la
comprarle comida, anda suelta por la ciudad desde entonces, serpiente.
sin que por el momento se haya encontrado rastro del
animal ni se conozca su paradero.
Cristian Gó mez, de 32 añ os, ha informado a EFE de que sa-
lió de su casa un momento «para ir comprar un rató n a la
serpiente» y, en ese intervalo de tiempo, «debió de salir por
un respiradero del terrario, con tan mala suerte que tenía la
ventana de la cocina abierta y se escapó ».
Después de dos horas buscá ndola por el piso, alertó a la Po-
licía Local y a los Bomberos de Zaragoza para decir que su
pitó n se había escapado.
Hasta allí se desplazaron los efectivos de los Bomberos y de
la Policía Local, que estuvieron revisando terrazas del
inmueble y algunos rincones del edificio, pero no han
encontrado nada.
Fuentes del Ayuntamiento de Zaragoza han emitido un men-
saje de tranquilidad a la població n porque el animal no es
peligroso.
www.elmundo.es, 1 de agosto de
2014.
a) ¿Cuáles de los factores principales de una noticia
(actua- lidad, proximidad, magnitud e interés humano)
ves pre- sentes en ella? Razona la respuesta y ten en
cuenta el medio de comunicación en el que se ha
difundido la noticia.
b) ¿Te parece que esta noticia tiene la suficiente
relevan- cia para figurar en un periódico de tirada
nacional? ¿Por qué?
c) Analiza en el texto las características lingüísticas
(obje- tividad, claridad, concisión y corrección).
d) ¿Te parece que el titular está bien planteado? Razona
tu respuesta.
e) ¿Están presentes en la entradilla las respuestas a las
seis preguntas básicas que deben resolverse en ella?
Copia en tu cuaderno las preguntas y sus respuestas.
a) La noticia relata un suceso menor que, a lo sumo, podría
tener el interé s de la proximidad si el perió dico se difun-
diera solo en Zaragoza, pero al ser una noticia en un medio
nacional y digital, casi carece de relevancia en este sentido.
Tal vez podamos percibir el interés humano, ya que tanto el
dueñ o de la serpiente como la policía local, los bomberos
y el ayuntamiento, han actuado para proteger a la
població n.
b) Respuesta libre.
c) Todas estas características está n presentes en el texto. La
objetividad no se puede poner en duda, ya que el autor
de la noticia se limita a informar sin hacer ningú n
comentario sobre lo que cuenta. La claridad se percibe en
que tras la lectura del texto somos capaces de imaginar
perfectamente la situació n y no nos queda ninguna duda
sobre los hechos narrados. También está presente la
concisió n, pues la noti- cia se cuenta con brevedad y con
las palabras exactas, sin dar rodeos innecesarios que
dificulten la comprensió n del suceso. Por ú ltimo, la
correcció n se percibe en los pá rrafos del texto, bien
estructurados, en los que la informació n va progresando
y se expresa lo acaecido de forma cronoló gica.
d) Respuesta libre.
e) • Qué : una serpiente pitó n se ha escapado de la casa de
su amo.
• A quién: le sucede el hecho a Cristian Gó mez, el
propie- tario de la serpiente pitó n.
• Dó nde: en Zaragoza, más concretamente en el Mi amado, las montañ as,
domicilio de Cristian Gó mez. los valles solitarios nemorosos,
• Por qué: porque la ventana de la cocina estaba las ínsulas extrañas,
abierta y la serpiente pudo salir por ella. los ríos sonorosos,
el silbo de los aires amorosos,
• Có mo: abandonando el terrario por un respiradero.

Recuerda lo que ya sabías


5. En el texto de Platero y yo (actividad 1) aparecen los
sus- tantivos trotecillo y florecillas. ¿Qué tipo de
morfemas se han empleado en su formación? ¿Qué
aportan desde el punto de vista del significado?
En los sustantivos trotecillo y florecillas se emplean
morfe- mas apreciativos, má s concretamente diminutivos,
que desde el punto de vista del significado, y en el
contexto en que se si- tú an, aportan un tono cariñ oso y de
delicadeza, má s allá de su significado de algo más
pequeñ o.
6. En la siguiente oración identifica los distintos
sintagmas que la componen y señala sus partes: «Los
espejos de aza- bache de sus ojos son duros».
Los espejos de azabache de sus ojos son duros:
• Los espejos de azabache de sus ojos (SN: Det + N +
SPrep
+ SPrep).
• son duros (SV: V + SAdj).
7. Di cuál es el sujeto en el fragmento de la égloga I de
Garci- laso de la actividad 2 y explica sus
características.
El sujeto es la primera persona del singular, yo, tal y
como deducimos de los verbos: agradaba y deseaba. Se
trata de un sujeto elíptico u omitido, ya que no está
presente en el texto pero se puede identificar con
facilidad.
8. Divide las siguientes palabras en monemas y explica
de qué tipo son en función de la combinación de
estos: pe- ludo, cascabeleo, despaciosos.
Peludo, cascabeleo, despaciosos.
• peludo: pel-udo (derivada).
• cascabeleo: cascabel-eo (derivada).
• despaciosos: despac-iosos (derivada).
9. Analiza la expresión «¿Platero?». ¿Crees que es una
ora- ción? ¿Podrías asimilarla a alguna de las
modalidades ora- cionales que conoces? Ten presente
el contexto.
La expresió n ¿Platero? no es en sentido estricto una
oració n, pero sí es una frase, ya que, aunque no tiene
verbo, sí expresa un significado completo, pues
entendemos una pregunta en la que se pueden haber
eliminado algunas palabras, como por ejemplo: Platero,
¿vienes? En este sentido podríamos asimilar la expresió n
al grupo de las interrogativas.

MIRA A TU ALREDEDOR Y…
… ve más allá
1. Una de las funciones principales del uso de los
adjetivos con un fin artístico en literatura es la de
servir de adorno, embellecer aquello que se dice con
palabras que pueden ser innecesarias (o no
imprescindibles), pero que despier- tan la sensibilidad
del lector y sirven de recreo a los sen- tidos. Lee el
siguiente fragmento del Cántico espiritual de san
Juan de la Cruz y fíjate en los adjetivos que te
marcamos en negrita:
la noche sosegada
montañ as; de estos mansos alcores y terreros, desde donde
en par de los levantes de la aurora,
se divisa un camino que va en zigzag hasta un riachuelo. Las
la mú sica callada,
auras marinas no llegan hasta esos poblados pardos de casu-
la soledad sonora,
chas deleznables, que tienen un bosquecillo de chopos junto
la cena que recrea y enamora.
al ejido. Desde la ventanita de este sobrado, en lo alto de la
Nuestro lecho florido, casa, no se ve la extensió n azul y vagarosa; se columbra allá
de cuevas de leones enlazado, en una colina una ermita con los cipreses rígidos, negros, a los
en pú rpura teñ ido, lados, que destacan sobre el cielo límpido. A esta olmeda que
de paz edificado, se abre a la salida de la vieja ciudad no llega el rumor rítmico
de mil escudos de oro coronado. y ronco del oleaje; llega en el silencio de la mañ ana, en la paz
Como ves, todos ellos dan color al texto, pero no son azul del mediodía, el cacareo metá lico, largo, de un gallo, el
absolutamente necesarios para su comprensión. Sí lo golpear sobre el yunque de una herrería. Estos labriegos se-
cos, de faces polvorientas, cetrinas, no contemplan el mar: ven
son para conmovernos, para hacernos disfrutar con su
la llanada de las mieses, miran sin verla la largura monó tona
sonoridad y con su significado. Algo parecido ocurre con de los surcos en los bancales. Estas viejecitas de luto, con sus
ciertos adornos que se emplean en la música y que tie- manos pajizas, sarmentosas, no encienden cuando llega el cre-
nen como principal intención decorarla. Uno de ellos es pú sculo una luz ante la imagen de una Virgen que vela por
el trino, que consiste en alternar de manera rápida dos los que salen en las barcas; van por las callejas pinas y
notas cercanas. Tiene una especial relevancia en la voz, tortuosas a las novenas, miran al cielo en los días
sobre todo si esta es aguda (soprano, tenor). Escucha el borrascosos y piden, juntando sus manos, no que se
aria de la Reina de la Noche, de la ópera La flauta aplaquen las olas, sino que las nubes no despidan granizos
mágica de Wolfgang Amadeus Mozart (1756-1791), asoladores.
interpretada por la soprano Graciela Armendáriz (teclea José MARTÍNEZ RUIZ, AZORÍN: Castilla.
en YouTube a) Date un paseo por tu ciudad o tu pueblo y observa
«Graciela Armendariz» «Reina de la Noche»), así como la bien todo lo que te encuentres. Después, haz una
canción Días de verano, del grupo Amaral (búscala con redac- ción en la que mantengas un tono parecido al
su título en YouTube o Spotify). Presta atención a la voz y del texto precedente, poniendo atención al uso de los
escucha los trinos. Su finalidad es, como la de los adjeti- adjetivos.
vos, recrear nuestros sentidos.
Actividad sin respuesta. Ejercitar y recrear los sentidos.
b) Busca en Internet imágenes de paisajes pintados por
algunos pintores y observa detenidamente sus
2. Con la misma finalidad decorativa que acabamos de detalles. Elige una de estas imágenes y redacta una
comentar, los adjetivos se pueden emplear en la descrip- descripción de lo que ves en ella. Te sugerimos
ción, y esta nos la encontramos tanto en la literatura algunos paisajistas españoles que fueron
como en la pintura. Lee el siguiente fragmento de la contemporáneos de Azorín: Joa- quín Sorolla, Ignacio
novela Flor de mayo (1895), de Vicente Blasco Ibáñez de Zuloaga, Ricardo Arredondo, Aureliano de Beruete.
(1867-1928), y compara lo que nos dicen los adjetivos
con el cuadro de Joaquín Sorolla (1863-1923) Sacando la c) ¿Por qué crees que en el texto se le da tanta impor-
barca (1916), que te adjuntamos a continuación del texto: tancia al contraste entre el paisaje castellano y el
mar?
Algunas «parejas» habían de aguardar en seco hasta el día si-
guiente, y para tirar de ellas entraban olas adentro los ¿Te parece que el paisaje que nos rodea puede influir
bueyes de la Comunidad de Pescadores, hermosos animales, en nuestro carácter y en nuestros estados de ánimo?
rubios y blancos, enormes como mastodontes, moviéndose Razona tus respuestas.
con una pesada majestad y agitando su enorme papada con d) El texto de Azorín contiene una serie de sustantivos
la altivez de un patricio romano. que se refieren a elementos del paisaje. Cópialos en
Estas yuntas, que hundían la arena bajo sus pezuñ as y de un tu cuaderno y busca en un diccionario el significado
tiró n arrastraban las barcas má s grandes, guiá balas Chepa. de los que no conozcas. Asocia luego estos
Los bueyes descritos por Blasco Ibá ñ ez tienen las mismas sustantivos a tu experiencia con el paisaje de tu
características que los que ha dibujado Sorolla: hermosos, tierra: ¿te parece que son todos aplicables o para
rubios, enormes… Tambié n se mueven con la misma describirla necesita- rías otros?, ¿cuáles?
lentitud que parecen mostrar los de la pintura. Y sobre todo,
vemos có mo realizan la misma acció n en ambos casos: sacar Si tu tierra es Castilla, valora la descripción que hace
una barca del mar. Azorín en función de tu propia experiencia: ¿crees
Lo que en Blasco Ibá ñ ez son adjetivos, en Sorolla son que es acertada?, ¿se parece la Castilla que él
colores, formas y luz solar. describe a la que tú conoces?
e) Observa los adjetivos que emplea Azorín y di qué sen-
… encuentra la clave saciones producen. Redacta de nuevo la descripción
1. El paisaje de tu tierra es una referencia de tu propia vida. con adjetivos opuestos a los que usa él.
Te ofrecemos un fragmento de un texto de Azorín (1873- a) Respuesta libre.
1967) en el que describe Castilla por medio del contraste
b) Respuesta libre.
con las tierras que están junto al mar:
c) Respuesta libre.
No puede ver el mar la solitaria y melancó lica Castilla. Está
muy lejos el mar de estas campiñ as llanas, rasas, yermas, d) Sustantivos que se refieren a elementos del paisaje: mar,
polvorientas; de estos barrancales pedregosos; de estos te- campiñas, barrancales, terrazgos, quiebras, alcores, ria-
rrazgos rojizos, en que los aluviones torrenciales han abierto chuelo, poblados, casuchas, bosquecillo, ejido, colina, er-
hondas mellas; de estas quiebras aceradas y abruptas de las mita, cipreses, olmeda, llanada, surcos, bancales.
Respuesta libre.
e) Respuesta libre.
4 LIBERTAD DE EXPRESIÓN

APERTURA DE UNIDAD el mundo la conocía como Mediamujer, las calles de mi


pueblo).
1. En el texto se habla de la imagen que tienen de España
en algunos países extranjeros. Valora los argumentos
que maneja el periodista del que habla el autor para
justificar la pregunta de si «España había superado sus
demonios antiguos, las pasiones y los enfrentamientos
de otro tiem- po». ¿Crees que son suficientes para
caracterizar a los españoles o son solo tópicos? Razona
tus respuestas.
Respuesta libre.
2. La película La vaquilla (1985), de Luis García Berlanga,
na- rra un episodio que pudo ocurrir durante la Guerra
Civil española, en un tono en el que se mezcla la comedia
con la crítica sociopolítica. En ella se nos presenta la
lucha de los dos bandos enfrentados, llena de
contradicciones y con un claro simbolismo al final. Ved la
película y después organizad un debate sobre la realidad
española.
Respuesta libre.
3. En grupos de cuatro, buscad información sobre momen-
tos de la historia de España en los que pueda ser posible
hablar de enfrentamientos, pasiones extremas y fanatis-
mos. Cada grupo se ocupará de un momento histórico
(la época de la Inquisición, el reinado de Fernando VII,
la Guerra Civil de 1936, la Transición, etc.). Después, un
portavoz de cada grupo expondrá ante los demás las
con- clusiones. Finalmente, se promoverá un debate en
el que se valorarán los temas citados arriba e intentaréis
sacar conclusiones.
Respuesta libre.

VIVE LA LECTURA
Comprensión lectora
1. Explica el significado de la expresión «avanzando como
si pisaran huevos». ¿Crees que es una expresión culta o
coloquial? ¿Por qué?
La expresió n avanzando como si pisaran huevos significa que
caminaban muy despacio. Es una expresió n de uso coloquial,
pues suele emplearse, por lo general, en la conversació n infor-
mal. El tono có mico y la expresividad de la frase refuerzan su
utilizació n familiar.
2. ¿Qué quiere decir la frase «escupía sus dos apellidos»?
¿El verbo escupir está empleado en sentido recto o figu-
rado? Razona la respuesta.
Escupía sus dos apellidos significa que los decía como con
rabia, con malas maneras, para dejar claro cuá les eran esos
dos apellidos. El verbo escupir, aquí, está usado en sentido
figurado, pues no se refiere a su significado recto de arrojar
saliva por la boca, sino a lo que acabamos de decir arriba:
ha- blar como escupiendo, con desprecio, arrojando las
palabras como si fueran escupitajos.
3. ¿Por qué, según el narrador, nunca podría Marisol
quitar- se el mote que le había puesto Cuelloduro?
Segú n el narrador, nadie había conseguido nunca, en su pue-
blo, liberarse de un mote bien puesto, como lo era el de las
mediasmujeres, a su entender.
4. ¿Qué relación tiene el narrador con los personajes de los
que habla? Explícalo razonadamente.
Tanto con Cuelloduo como con las hermanas Rodríguez Peñ al-
va, la relació n del narrador es principalmente la de ser vecinos
del mismo pueblo, tal y como nos dice este (en el pueblo todo

30 Lengua castellana y Literatura. 3.º ESO. Solucionario


LIBERTAD DE EXPRESIÓN 4
Con las dos mujeres, ademá s, le unía una relació n má s de hoja muy estrecha y aguda. La autora utiliza la palabra
cerca- na, ya que, segú n cuenta, el padre de estas era el jefe como
del suyo.
5. Al leer el siguiente fragmento, ¿qué idea nos hacemos
de los padres de Marisol y Sonsoles? Describe su
persona- lidad a partir de lo que aquí se nos dice:
«Sonsoles y Ma- risol, nombres cuidadosamente
escogidos en el santoral para distinguirlos de los que se
repetían en las muchachas corrientes».
Al leer esta oració n tenemos la sensació n de que los padres de
las mediasmujeres eran personas que pretendían
mostrarse diferentes a los demá s, sin duda por algú n
complejo de supe- rioridad que les hacía creerse má s
importantes que el resto de los habitantes del pueblo.
Posiblemente fueran personas superficiales, que daban má s
importancia a las apariencias que a la gente, sobre todo si
esta procedía de un grupo social inferior al suyo.
6. Explica de manera razonada el porqué del mote de las
dos hermanas.
Segú n cuenta el propio narrador, una de ellas era fea pero
tenía un cuerpo bonito, mientras la otra, má s baja de estatura,
era muy guapa, a pesar de que su cuerpo era poco
agraciado. Por eso Cuelloduro les puso el mote, porque
juntando la mitad de cada una de las hermanas, se formaba
una mujer hermosa (una tía buena, en palabras de
Cuelloduro).
7. «Marisol apretaba los párpados con tanta fuerza
como si alguien le hubiera exprimido un limón encima
de los ojos». ¿Qué valores expresivos tiene esta
oración? ¿Qué crees que aporta a la descripción que
está haciendo el narrador?
La expresividad de la oració n es muy grande, ya que si a al-
guien le echaran zumo de limó n en los ojos, el escozor que esto
le produciría le llevaría, tal vez, a cerrarlos con fuerza, para
aliviar la sensació n desagradable. Almudena Grandes muestra
con esta expresió n el desagrado que le producía a Marisol
oír su mote. Tambié n define al personaje como una persona
iras- cible y poco amiga de bromas, con lo que el lector se
hace una idea bastante aproximada de la personalidad de
esta joven.

Vocabulario
8. En el texto hay varias palabras compuestas, búscalas
y di cuál es su composición. ¿Crees que de esa manera
se refuerza el significado por medio de este recurso?
¿Por qué?
Palabras compuestas: mediamujer (mediasmujeres), Cuello-
duro, culibajo.
• Mediamujer: media + mujer (determinante + sustantivo).
• Cuelloduro: cuello + duro (sustantivo + adjetivo).
• culibajo: culo + bajo (sustantivo + adjetivo).
El uso de la composició n ofrece muchas posibilidades expresi-
vas, al reducir a una sola palabra lo que en su origen son
dos. De esta manera se crea una nueva palabra. Por
ejemplo, con el mote del tabernero, de las palabras
independientes cuello y duro (que pueden emplearse juntas
como complemento la segunda de la primera, pero escritas de
manera separada), extraemos una sola que nos ofrece, de
manera muy grá fica, la realidad que quiere representar.
9. Busca en el diccionario la palabra estilete y explica luego
qué quiere decir en el contexto en que la sitúa la
autora:
«zapatos de tacón altísimo, estiletes pensados para las
lisas aceras de las ciudades».
Un estilete, segú n el diccionario de la RAE es un puñ al
Lengua castellana y Literatura. 3.º ESO. Solucionario 31
metá fora para referirse a los tacones de los zapatos de las Respuesta libre.
dos hermanas, que, de tan finos como eran, se asemejaban
mucho a estiletes. Aprovecha también para decir que este tipo
de tacones era excesivo para las calles mal empedradas de
su pueblo, y criticar así la afectada y artificial elegancia de
las mediasmujeres.
10. ¿Cómo entendemos el adjetivo afilado en relación con el
mote («además de afilado, era certero»)?
Afilado es sinó nimo de agudo, y aquí tenemos que entender-
lo en este sentido; es decir, el mote, en opinió n del narrador,
estaba muy bien ideado y era muy ingenioso, perfecto casi.
11. Busca y copia en tu cuaderno los adjetivos que
aparecen en el primer párrafo del texto y analiza su
significado se- gún su posición y según la intención del
narrador.
• tardes tontas: por su posició n, es un adjetivo especifica-
tivo, que pretende diferenciar con claridad unas tardes de
otras. Tontas con el sentido de aburridas, monótonas.
• las dos muy tiesas: también este adjetivo es especificativo,
por su posició n posterior al nombre (en este caso el pronom-
bre dos, que sustituye a las hermanas). La autora marca con
este adjetivo la altivez y soberbia de las mediasmujeres.
• a duras penas: la expresió n es una locució n adverbial, que
equivale a dificultosamente, por lo que el adjetivo forma
parte de dicha locució n y no admite que lo analicemos por
separado.
• tacón altísimo: una vez má s un adjetivo especificativo,
situado detrá s del nombre. En este caso es destacable,
igualmente, el uso del superlativo absoluto (-ísimo), con
el que la autora quiere ponderar la altura de los tacones
que llevaban Sonsoles y Marisol.
• lisas aceras: aquí, en cambio, tenemos un adjetivo expli-
cativo, colocado delante del sustantivo para matizar, de
pasada, una cualidad de este que no se considera impres-
cindible para la comprensió n de lo que se quiere decir: solo
con referirse a las aceras ya intuimos el suelo má s regular
de la ciudad con respecto al empedrado del pueblo.
• chinos redondos: adjetivo especificativo, que persigue
de- finir có mo eran esos chinos, nombre con el que se refiere
la autora a las piedras de las calles de Fuensanta de
Martos.
• rosa pálido: adjetivo especificativo que matiza la tonali-
dad rosa de la rebeca.
• azul celeste: exactamente lo mismo que en el caso anterior,
variando el color y el matiz.
• rebeca de punto fino: también este es un adjetivo especi-
ficativo que pretende recalcar la labor primorosa con que
estaban tejidas las rebecas de las dos hermanas.
• perlas diminutas: como en el caso anterior, la pequeñ ez
expresada por el adjetivo especificativo aporta un refina-
miento a las mujeres, cuyas perlas no eran toscos y grandes
diamantes ostentosos, sino delicadas piedras diminutas.
• barbilla alta: adjetivo especificativo con el que se nos quie-
re dar a entender, una vez má s, la soberbia de las
llamadas mediasmujeres, que adoptaban esa postura de
altivez ante los demá s habitantes del pueblo.

Investigación y redacción
12. Fuensanta de Martos refleja la vida de cualquier pueblo
durante la posguerra. A partir del texto, haz una redac-
ción en la que describas cómo crees tú que debía de ser
el pueblo en aquel tiempo.
13. Busca información sobre la posguerra española y
sobre la vida en los pueblos, y compara tus hallazgos
con lo que se nos dice en el texto.
Respuesta libre.
14. La costumbre de poner motes está muy enraizada en los
pueblos de España. Pregunta a tus padres o a amigos
que conozcan de cerca la vida en una pequeña
población y elabora una lista de los motes que allí se
usan.
Respuesta libre.

ESTUDIO DE LA LENGUA
1. Identifica el sintagma verbal en las siguientes oraciones
y a continuación separa el lexema y los morfemas del
verbo:
a) Todas las tardes, Elvira y yo paseábamos por la
orilla del mar.
b) Encontró las llaves de su casa en el contenedor
de basura orgánica.
c) Los bomberos apagaron el fuego en menos de
quince minutos.
d) Nos gusta mucho la paella valenciana.
e) ¿Recibiste la carta que te envié?
El predicado se marca en negrita:
a) Todas las tardes, Elvira y yo paseábamos por la
ori- lla del mar.
• paseá bamos: pase-á ba-mos.
b) Encontró las llaves de su casa en el contenedor
de basura orgánica.
• encontró : encontr-ó .
c) Los bomberos apagaron el fuego en menos de
quince minutos.
• apagaron: apag-aron.
d) Nos gusta mucho la paella valenciana.
• gusta: gust-a
e) ¿Recibiste la carta que te envié?
• recibiste: recib-iste.
• envié : env-ié .
2. En las siguientes oraciones localiza las perífrasis y
clasifí- calas según su significado:
a) Alfonso acaba de dejar Internet y ya está zapeando
en el televisor.
b) Iba a saltar desde el trampolín, pero ella se puso
a gritar y me hizo bajar.
c) Dieron las tres y debí marcharme: ni se ponían a
comer ni me invitaban.
d) Los cuatro amigos se echaron a descansar cuando
tu- vieron hechas todas sus tareas.
e) Andan diciendo que te vas a casar.
Las perífrasis aparecen marcadas en negrita:
a) Alfonso acaba de dejar Internet y ya está zapeando
en el televisor.
• acaba de dejar: aspectual perfectiva (acabar de +
infinitivo).
• está zapeando: aspectual durativa (estar + gerundio).
b) Iba a saltar desde el trampolín, pero ella se puso
a gritar y me hizo bajar.
• iba a saltar: aspectual incoativa (ir a + infinitivo).
• se puso a gritar: aspectual incoativa ( ponerse a +
infinitivo).
c) Dieron las tres y debí marcharme: ni se ponían a co-
mer ni me invitaban. b) Alba me informó de su decisión de no acudir a la
reu- nión.
• debí marcharme: modal de obligació n (deber +
infinitivo). c) La carta fue firmada por todos los asistentes.
• se ponían a comer: aspectual incoativa ( ponerse a + d) Encontramos abiertas las ventanas de nuestra casa.
infinitivo). e) Me habló de su relación con Marisa en la cafetería de
d) Los cuatro amigos se echaron a descansar cuando tuvie- la estación.
ron hechas todas sus tareas.
f) Esteban abría las nueces con un martillo.
• tuvieron hechas: aspectual perfectiva (tener +
participio). g) Carolina es la amiga íntima de Lola.
e) Andan diciendo que te vas a casar. a) • por el suelo: complemento circunstancial de lugar.
• andan diciendo: aspectual durativa (andar + gerundio). • todos los juguetes de Luisito: complemento directo.
• te vas a casar: aspectual incoativa (ir a + infinitivo). b) • me: complemento indirecto.
3. Indica si las siguientes oraciones tienen complemento • de su decisión de no acudir a la reunión: complemento
directo, complemento indirecto o ambos, y delimítalos. de régimen.

a) Implicamos a María en la organización de la fiesta. c) • por todos los asistentes: complemento agente.
d) • abiertas: complemento predicativo.
b) Enrique contó a Silvia una historia de terror.
• las ventanas de nuestra casa: complemento directo.
c) Desde el barco vimos a lo lejos las luces del puerto
de Valencia. e) • me: complemento indirecto.
d) En la piscina había muchas avispas alrededor de la • de su relación con Marisa: complemento de régimen.
ducha. • en la cafetería de la estación: complemento circunstan-
cial de lugar.
e) Amelia habló a Joaquín de sus problemas.
a) A María: complemento directo. f) • las nueces: complemento directo.

b) A Silvia: complemento indirecto. Una historia de terror: • con un martillo: complemento circunstancial de ins-
complemento directo. trumento.

c) Las luces del puerto de Valencia: complemento directo. g) • la amiga íntima de Lola: atributo.
d) Muchas avispas: complemento directo. 6. Indica cuáles de las oraciones de la actividad anterior son
e) A Joaquín: complemento indirecto.
transitivas y explica por qué.
Las oraciones transitivas son las que llevan complemento di-
4. Reconoce en las siguientes expresiones los casos de
recto, por lo que, de las anteriores, pertenecen a esa
laísmo, loísmo y leísmo que estén presentes y escríbelas categoría las siguientes:
luego correctamente:
• Tiraron por el suelo todos los juguetes de Luisito.
Corrige • Encontramos abiertas las ventanas de nuestra casa.
• Esteban abría las nueces con un martillo.
a) A Sergio le he visto en la estación.
7. Transforma en pasivas las oraciones que acabas de selec-
b) Dalos la merienda a los niños. cionar en la actividad anterior. ¿Puedes poner en pasiva
c) La compré unas flores por su cumpleaños. el resto? ¿Por qué?
d) No las digas nada de lo que te conté. • Todos los juguetes de Luisito fueron tirados por el suelo.
e) Dilos que se callen de una vez. • Las ventanas de nuestra casa fueron encontradas abiertas
f) A mis hijas las gustan mucho los caramelos. por nosotros.
• Las nueces eran abiertas por Esteban con un martillo.
a) A Sergio le he visto en la estación.
El resto de las oraciones no se pueden poner en voz pasiva,
• leísmo: lo correcto es lo he visto. porque no tienen complemento directo, y este es imprescindi-
b) Dalos la merienda a los niños. ble para realizar el proceso, pues es el que pasa a convertirse
• loísmo: lo correcto es dales la merienda. en sujeto paciente.

c) La compré unas flores por su cumpleaños. 8. Di razonadamente si las siguientes oraciones son reflexi-
• laísmo: lo correcto es le compré unas flores.
vas o recíprocas:
d) No las digas nada de lo que te conté. a) El gato se rascaba una oreja con la pata derecha.
• laísmo: lo correcto es no les digas nada. b) Lucía y yo nos miramos a los ojos.
e) Dilos que se callen de una vez. c) Mis hermanos se pelean constantemente.
• loísmo: lo correcto es diles que se callen. a) Esta oració n es reflexiva, ya que la acció n realizada por
f) A mis hijas las gustan mucho los caramelos. el sujeto (el gato) repercute sobre él mismo (rascarse).
• laísmo: lo correcto es les gustan mucho. b) Esta oració n es recíproca, ya que se produce un
intercam- bio de acciones entre los componentes del
5. Analiza los complementos del verbo que hay en las si- sujeto; en este caso Lucía y yo intercambiamos
guientes oraciones: miradas.
a) Tiraron por el suelo todos los juguetes de Luisito. c) Esta oració n es recíproca, pues se produce tambié n un
in- tercambio: uno pelea contra el otro y viceversa.
9. Indica cuáles de las siguientes oraciones son transitivas
EL TALLER DE LAS PALABRAS
y explica por qué:
a) Teníamos la cabeza llena de fantasías. 1. Relaciona los verbos siguientes con las expresiones
que figuran más abajo: fijarse, desempeñar, declarar,
b) A Pilar le gusta mucho el arroz con leche.
proporcionar.
c) Leí la increíble noticia en el periódico de mi barrio.
a) dar información c) decir la verdad
d) Antonio se compró un libro muy antiguo en una
b) tener un cargo d) tener una meta
librería de San Sebastián.
a) dar información: proporcionar.
e) El tren llegó con media hora de retraso.
b) tener un cargo: desempeñ ar.
f) Los jardines de Aranjuez son un lugar de gran belleza. c) decir la verdad: declarar.
g) Fuimos a Angola con un programa de intercambio. d) tener una meta: fijarse.
h) Ganarás el pan con el sudor de tu frente. 2. Señala los casos de deixis en el siguiente fragmento e in-
i) Explícame otra vez el teorema. dica si son anáforas o catáforas:
j) No entiendo a las personas pesimistas. Kim meditó un momento. Tres añ os antes hubiera sacado todo
Las siguientes oraciones son transitivas porque tienen un com- el provecho posible de su situació n y hubiera continuado su
plemento directo (marcado en negrita): camino como si tal cosa; pero el respeto que le manifestaba
el jat le probaba que era ya un hombre. Ademá s, él también
• Teníamos la cabeza llena de fantasías.
había tenido fiebres una o dos veces y las conocía lo bastante
• Leí la increíble noticia en el periódico de mi barrio. para reconocer la desnutrició n cuando la veía.
• Antonio se compró un libro muy antiguo en una librería Rudyard KIPLING: Kim.
de San Sebastián.
Aparecen tres casos de deixis textual y todas ellas son aná-
• Ganarás el pan con el sudor de tu frente. foras, ya que el elemento al que se refieren va delante: el
• Explícame otra vez el teorema. res- peto que (se refiere al respeto), las conocía (se refiere
• No entiendo a las personas pesimistas. a las fiebres, citadas poco antes) y cuando la veía (se
refiere a la desnutrición).
10. Clasifica las siguientes oraciones en pasivas reflejas e im-
personales y explica por qué: 3. Sustituye por un sinónimo el término en negrita:
a) Desde mi ventana se contempla un paisaje ma- a) Obtuvo el premio mediante muchos engaños.
ravilloso. b) Nuestros rivales eran equipos muy entrenados.
b) No se ven bien las estrellas con este cielo tan nu- c) Se demoró en el pago del alquiler.
blado. d) Los perros son animales leales.
c) En este país se lee poco. e) Siempre nos miraba con desdén.
d) Se siente, pero este año no habrá regalos para los f) Tarzán creció en plena selva.
ga- nadores del concurso. a) muchas mentiras
e) Se publicaron unos artículos muy interesantes en la b) oponentes, contrarios, enemigos
revista del instituto. c) se retrasó
f) Se come mucho en las cenas con amigos. d) fieles
g) En equipo se trabaja de maravilla. e) desprecio
h) Con este receptor no se escuchan bien algunas emi- f) se crio, se desarrolló
soras. 4. ¿Son sinónimos totales los verbos amar y querer? ¿Y gló-
i) Se busca cocinero con experiencia. bulos rojos y hematíes? ¿Por qué?
a) Pasiva refleja, porque lleva un sujeto paciente: un Los verbos amar y querer no son sinó nimos totales, porque,
paisaje maravilloso. aunque pueden ser intercambiables perfectamente en algunos
b) Pasiva refleja, porque lleva un sujeto paciente: las contextos (amo a Celia, quiero a Celia), no lo pueden ser en
estrellas. otros (quiero un coche, *amo un coche).
c) Impersonal con se, ya que no existe ningú n tipo de Sin embargo, glóbulos rojos y hematíes sí son sinó nimos tota-
sujeto, ni activo ni paciente. les, pues son siempre intercambiables: hematíes es la forma
d) Impersonal con se, ya que no existe ningú n tipo de científica de referirse a los glóbulos rojos.
sujeto, ni activo ni paciente. 5. Explica la diferencia de matiz (culto o familiar, más o me-
e) Pasiva refleja, porque lleva un sujeto paciente: unos nos intenso, etc.) entre estas parejas de sinónimos:
artí- culos muy interesantes. prisión-chirona obedecer-acatar
f) Impersonal con se, ya que no existe ningú n tipo de ruido-estruendo difunto-finado
sujeto, ni activo ni paciente.
golpe-cachete finca-bien inmueble
g) Impersonal con se, ya que no existe ningú n tipo de
• Prisión es má s formal, mientras que chirona es coloquial.
sujeto, ni activo ni paciente.
• Obedecer es de uso comú n, tambié n má s coloquial, frente
h) Pasiva refleja, porque lleva un sujeto paciente: algunas
a acatar, que parece má s propio del lenguaje formal de
emisoras.
las leyes.
i) Pasiva refleja, porque lleva un sujeto paciente: cocinero
• Mientras que ruido es una palabra de uso comú n, estruen-
con experiencia.
do parece tener un matiz má s culto, tal vez literario.
• Difunto es una palabra de uso familiar, frente a finado que
es, sin duda, una palabra má s culta. 3. Busca en un diccionario etimológico de dónde vienen
las palabras hambre, harina, hermoso, hijo, hoja, hongo y
• Golpe muestra un uso menos coloquial que cachete.
humo. ¿Explica su etimología el hecho de que actualmen-
• Si finca se asocia con un uso familiar, bien inmueble está te se escriban con h?
má s en la línea de lo mercantil y de lo legal.
• hambre: del latín vulgar famis.
6. Escribe los antónimos de las siguientes palabras e indica
• harina: del latín farina.
de qué tipo de antonimia se trata:
• hermoso: del latín formosus.
mortal alegre femenino nuera pequeño
• hijo: del latín filius.
subir negro bueno sobrino fuera
• hoja: del latín folia.
• mortal. Antó nimo: inmortal. Complementario.
• hongo: del latín fungus.
• alegre. Antó nimo: triste. Gradual.
• humo: del latín fumus.
• femenino. Antó nimo: masculino. Complementario.
El hecho de que todas estas palabras procedan del latín y co-
• nuera. Antó nimo: suegra o suegro. Recíproco. menzaran con f- en esta lengua es la explicació n de que en
• pequeño. Antó nimo: grande. Gradual. españ ol se escriban con h-, ya que al pasar del latín a esta
• subir. Antó nimo: bajar. Complementario. ú ltima, la f- se transformó en h-.
• negro. Antó nimo: blanco. Gradual. 4. Escribe dos palabras derivadas de hacer, de hombre y de
• bueno. Antó nimo: malo. Gradual. hierba. ¿Conservan la h?
• sobrino. Antó nimo: tío o tía. Recíproco. • hacer: hacedero, hacedor.
• fuera. Antó nimo: dentro. Complementario. • hombre: hombría, hombruno.
• hierba: herbolario, herbívoro.
7. Con los prefijos a-, des-, in- forma el antónimo de estas
palabras y escribe una oración con cada uno: En estas palabras y en sus derivadas se mantiene la h- porque
proceden de palabras latinas que la tenían tambié n.
coser digno orientada moral creíble
5. Escribe tres palabras que comiencen por cada uno de los
normal tejer hacer cauto real siguientes elementos compositivos: hecto-, helio-,
• coser: descoser hemo-, hemi-, hepta-, homo-, hiper-, hipo-.
• digno: indigno • hecto-: hectó metro, hectolitro, hectogramo.
• orientada: desorientada • helio-: heliocéntrico, heliotropo, helioscopio.
• moral: inmoral • hemo-: hemorragia, hemorroide, hemodiá lisis.
• creíble: increíble • hemi-: hemiciclo, hemisferio, hemistiquio.
• normal: anormal • hepta-: heptaedro, heptasílabo, heptá gono.
• tejer: destejer • homo-: homonimia, homosexualidad, homó fono.
• hacer: deshacer • hiper-: hiperó nimo, hipermercado, hipersensible.
• cauto: incauto • hipo-: hipocausto, hipogeo, hipodérmico.
• real: irreal 6. Escribe h donde corresponda:
Respuesta libre. __olgado __ormona __oval
__orfanato __eterogéneo __alde__uela
EL RINCÓN DE LA NORMA
__omeopatía __invernar __emorragia
1. Escribe en tu cuaderno las siguientes oraciones que te __ermandad __oquedad __allazgo
dictará tu profesor:
__ogareño __echado __umedecer
a) Las hienas son animales muy huidizos, en cuanto • hueco: oquedad, oquedal, oqueruela.
hue- len un olor extraño huyen.
b) En el hemiciclo se halló la solución al problema de
la habitabilidad humana.
c) Los huesos de los santos se hallan en un osario me-
dieval.
d) ¡Hala!, ya me has echado encima la salsa de las
habas.
e) Voy a hacer más habitable este hogar.
f) Ha acudido a la consulta de un homeópata porque
tiene una hernia lumbar.
Respuesta libre.
2. Escribe tres derivados de hueso, de huérfano, de huevo
y de hueco que no lleven h.
• hueso: osamenta, osario, ó seo.
• huérfano: orfanato, orfandad, orfelinato.
• huevo: oval, ovoide, ovario.
• holgado • hormona • oval
• orfanato • heterogéneo • aldehuela
• homeopatía • invernar • hemorragia
• hermandad • oquedad • hallazgo
• hogareño • echado • humedecer
7. Copia en tu cuaderno las siguientes oraciones que te
dic- tará tu profesor:
a) Oyó que yendo hacia la plaza mayor de Uruguay
había un gran bullicio.
b) En esa proyección subyace un problema apabu-
llante.
c) No creo que vaya esta tarde a arreglar la valla de
los caballos.
d) No te escabullas a la hora de regar el bonsái.
e) No llegó un solo convoy, sino dos, e iban cargados
de cigarrillos.
Respuesta libre.
8. Completa con y o ll según corresponda: pú blicas, fundaciones, organismos, agencias patronales, me-
bu__icio convo__ atribu__era __ate dios de comunicació n pú blicos, etcé tera, suman muchos mi-
les de personas con sueldos a cargo de las arcas pú blicas. El
__ave cosi__a __emen co__ado problema es complicado, porque, paradó jicamente, son ellos
__ermo mau__ar deca__ó pro__ecto mismos los que deben tomar la decisió n de reducir ese nú mero
sobredimensionado.
Bomba__ le__es dis__unción pro__ector
Necesitamos una nueva generació n de políticos que incluyan
pi__o __ema ca__ena in__ección en el programa de su partido las reformas que se necesitan
• bullicio • convoy • atribuyera • yate hacer […]. Para ello se necesita voluntad de diá logo para
con- seguir un amplio consenso.
• llave • cosilla • Yemen • collado
André s SANJURJO MARTÍNEZ, Ferrol, A Coruñ a,
• yermo • maullar • decayó • proyecto
en El País, 20 de julio de 2014.
• Bombay • leyes • disyunción • proyector
a) El texto muestra el descontento de un ciudadano con
• pillo • yema • cayena • inyección
el elevado número de políticos que hay en España. ¿Te
pa- rece que habrá mucha gente que comparta sus
LA FACTORÍA DE TEXTOS ideas?
1. A continuación te ofrecemos una columna de opinión. ¿Qué opinión te merece a ti lo que él plantea? Razona
Léela con atención y responde luego a las cuestiones tus respuestas.
que te planteamos: b) Escribe tú una carta al director sobre un tema que te
Cualquiera que lleve hoy un perió dico bajo el brazo no es preocupe, manteniendo una extensión similar a la
que esté mal informado, pero da la sensació n de estar que te ofrecemos.
viviendo la realidad del día anterior. […] Aparte de eso, el a) y b) Respuesta libre.
perió dico que uno lleva bajo el brazo define ideoló gicamente
al lector. […] Así sucedía también cuando en la Repú blica 3. Algunos diarios digitales permiten la lectura en línea de
cada diario era el estandarte de una bandería política, de la sus editoriales. Lee algunos de ellos y comenta luego los
lucha de clases, incluso de un pensamiento religioso o aspectos más destacados de cada periódico: ideas políti-
anticlerical. Durante la larga ceniza de la posguerra el cas, preocupaciones sociales, visión de España, etc. Pue-
perió dico llegaba al pueblo en el renqueante autobú s de des consultar los editoriales de El Mundo (elmundo.es/
línea o en el correo ordinario, solo unos pocos ejemplares, opinion.html) y de El País (elpais.com/elpais/opinion.
que leía gente muy significada […]. Sobre un velador del café
html).
y en la barbería quedaba el diario deportivo un poco
grasiento despué s de haber pasa- do de mano en mano. […] Respuesta libre.
Al llegar la democracia la prensa escrita se adaptó a la
libertad y cada diario se acomodó de nuevo a la manera de ACTIVIDADES FINALES
ser y de pensar de sus lectores. Pero con la revolució n digital
hoy la prensa de papel siempre es la de ayer y encima el Repasa lo que has aprendido
perió dico […] que el ciudadano compra en el quiosco es un 1. El siguiente texto pertenece al capítulo XVII del libro pri-
gesto ideoló gico que lo delata. No sucede así con la tableta mero del Amadís de Gaula y describe un combate entre
digital. Picoteando en el teclado del portá til con los dedos en dos caballeros. Léelo y responde luego a las preguntas:
el metro, en el tren, en una terraza al sol, nadie a tu lado
puede saber si eres de derechas o de izquierdas. Leer el Y metiendo mano a las espadas se dejaron a él ir muy brava-
perió dico de papel se va a convertir en el futuro en una ex- mente. Amadís metió mano a su espada como aquel que era
quisitez para estetas. Mientras todas las noticias en el digital de gran corazó n, y dejose a ellos ir muy sañ udo por los haber
son ya las de mañ ana, tampoco está tan mal ser un quitado de su batalla y lo acometían tan malamente, e hirió
ciudadano elegantemente inactual. al uno de ellos por cima del yelmo de tal golpe, que le alcanzó
en el hombro que las armas con la carne y huesos fue todo
Manuel VICENT, en El País, 11 de marzo de 2012. corta- do hasta descender la espada a los costados; así
a) Valora la actualidad de la columna. ¿Crees que el quedá ndole el brazo colgado cayó del caballo abajo; y dejose
tema del que trata se ajusta a los tiempos en los que ir a los dos que le herían bravamente y dio al uno por el yelmo
vivi- mos? ¿A qué tipo de personas piensas que va tal golpe, que se lo hizo saltar de la cabeza, y la espada
descendió al pescuezo y cortole todo lo má s de é l, y cayó el
dirigido el artículo? Razona tus respuestas.
caballero. El otro, que esto vio, comenzó de huir contra
b) ¿Está presente en el artículo el criterio de subjetivi- donde viniera. Amadís, que lo vio en caballo corredor y que
dad? Explica con tus palabras cuál es la opinión del se alejaba, dejó de lo seguir y tornó a Gandalín.
autor acerca del tema que plantea y cómo la percibi- Garci RODRÍGUEZ DE MONTALVO: Amadís de Gaula (1507)
mos al leer el texto. (texto modernizado).
c) Expresa ahora tu opinión y redacta un artículo en el a) Busca en el texto los verbos en forma conjugada y
que plantees lo que piensas sobre la prensa. analízalos morfológicamente. ¿Qué tiempos predo-
a), b) y c) Respuesta libre. minan? ¿Crees que son adecuados para la narración?
2. Lee esta carta al director y contesta a las cuestiones: ¿Por qué?
En Españ a, segú n las ú ltimas informaciones, tenemos so- b) Haz una lista con los verbos que indican acción y ana-
bredimensionado el nú mero de políticos: entre diputados, liza qué aportan a la descripción del combate.
senadores, diputados autonó micos, diputados provinciales, c) ¿Qué formas no personales están presentes en el
miembros de los Gobiernos, concejales, asesores, así como texto? Cópialas en tu cuaderno y di de qué manera
cargos de designació n política que trabajan en empresas
contribuyen a hacer más intensa la acción.
d) ¿Qué crees que significa la expresión dejose ir que Todas estas formas no personales, unidas a las conjugadas
se usa varias veces en el texto? ¿Es una perífrasis? que hemos analizado arriba, ofrecen un enorme dinamis-
¿Por qué? mo a la acció n que se está narrando, dando una impre-
a) Verbos: dejaron, metió, era, dejó, acometían, hirió, al- sió n de continuo movimiento y manteniendo la atenció n
canzó, fue, cayó, herían, dio, hizo, descendió, cortó, vio, del lector ante lo trepidante de los hechos contados. Los
comenzó, viniera, alejaba, tornó. infinitivos muestran acciones que parecen estar siempre
• dejaron: tercera persona del plural del pretérito perfec- en curso, mientras los gerundios indican claramente la
to simple del verbo dejar. duració n de estas y los participios el resultado ú ltimo de
algunas de ellas.
• metió: tercera persona del singular del pretérito perfec-
to simple del verbo meter. d) La expresió n dejose ir significa que se dirigió voluntaria-
mente a un lugar concreto. Es una perífrasis, ya que
• era: tercera persona del singular del pretérito imperfec- indica una ú nica acció n y podríamos sustituirla por se
to de indicativo del verbo ser. dirigió.
• dejó: tercera persona del singular del pretérito perfecto
2. Transforma en pasivas las siguientes oraciones siempre
simple del verbo dejar.
que sea posible. En el caso de que alguna no se pueda
• acometían: tercera persona del plural del pretérito im- transformar, explica por qué:
perfecto de indicativo del verbo acometer.
a) Comían con ganas un bocadillo de chorizo.
• hirió: tercera persona del singular del pretérito perfec-
to simple del verbo herir. b) Las ventanas del aula estaban abiertas.
• alcanzó: tercera persona del singular del pretérito per- c) A Carmen le gustan mucho los helados de limón.
fecto simple del verbo alcanzar. d) Ángel y Ana corrieron el maratón de Boston.
• fue: tercera persona del singular del pretérito perfecto e) Mantuvimos el fuego encendido toda la noche.
simple del verbo ir.
• cayó: tercera persona del singular del pretérito perfecto a) Un bocadillo de chorizo era comido con ganas por ellos.
simple del verbo caer. b) No puede ponerse en pasiva porque no tiene complemento
• herían: tercera persona del plural del pretérito imper- directo.
fecto de indicativo del verbo herir. c) No puede ponerse en pasiva porque no tiene complemento
• dio: tercera persona del singular del pretérito perfecto directo.
simple del verbo dar. d) El maratón de Boston fue corrido por Ángel y Ana.
• hizo: tercera persona del singular del pretérito perfecto e) El fuego fue mantenido encendido por nosotros toda la
simple del verbo hacer. noche.
• descendió: tercera persona del singular del pretérito 3. Haz un análisis sintáctico de las oraciones de la actividad
perfecto simple del verbo descender. anterior.
• cortó: tercera persona del singular del pretérito perfecto a) Comían con ganas un bocadillo de chorizo.
simple del verbo cortar.
V CCM CD
• vio: tercera persona del singular del pretérito perfecto
simple del verbo ver. b) Las ventanas del aula estaban abiertas.
• comenzó: tercera persona del singular del pretérito per- Sujeto V Atributo
fecto simple del verbo comenzar. c) A Carmen le gustan mucho
• viniera: tercera persona del singular del pretérito im- CI CI V CCC
perfecto de subjuntivo del verbo venir.
los helados de limón.
• alejaba: tercera persona del singular del pretérito im-
Sujeto
perfecto de indicativo del verbo alejar.
• tornó: tercera persona del singular del pretérito perfec- d) Ángel y Ana corrieron
to simple del verbo tornar. Sujeto V
En general predomina el preté rito perfecto simple, pero el maratón de Boston.
tambié n hay varios preté ritos imperfectos, tanto de indi- CD
cativo como de subjuntivo. Estas formas verbales que ex-
presan el pasado son ideales para la narració n de e) Mantuvimos el fuego encendido
sucesos (reales o ficticios), porque el narrador se V CD CPred
encuentra en el presente y echa la vista atrá s para contar toda la noche.
lo que ya ha ocurrido.
CCT
b) Verbos que indican acció n: acometían, hirió, alcanzó,
herían, dio, hizo, descendió, cortó, comenzó, viniera, 4. Detecta y corrige los casos de laísmo, leísmo y loísmo
alejaba, tornó. presentes en las siguientes oraciones y explica por qué
Muchos de estos verbos indican movimiento, por lo que son incorrectos. Atención, no todas las oraciones contie-
se muestra el dinamismo del combate (descendió, comen- nen errores.
zó, viniera, alejaba, tornó). Otros indican directamente Corrige
acciones bé licas y por ello contribuyen a dar viveza al
a) La dediqué un poema a mi novia.
combate (acometían, hirió, alcanzó, herían, dio, cortó).
c) Formas no personales: metiendo, ir, haber quitado, cor-
b) No los des dinero, que son unos derrochones.
tado, descender, quedando, colgado, saltar, huir, seguir. c) A Elvira la quiero mucho.
d) Dilas que mañana vendremos temprano.
e) Los vi alejarse en la oscuridad.
Corrige • curar: herir (herían). Recíproco.
• cobardemente: bravamente. Gradual.
f) Te dejo el disco, pero no me le destroces, que
te conozco. • ascender: descender. Complementario.
g) No la gusta que la mires de esa manera. 7. Lee el siguiente fragmento de un editorial de El País, del
h) Me le encontré en el parque y le dejé las cosas día 20 de agosto de 2014, y contesta luego a las
claras. cuestiones:
i) La miro y no la reconozco. Las protestas y disturbios que se han producido tras la muerte
de un joven negro por disparos de un policía en la localidad
a) La dediqué un poema a mi novia. estadounidense de Ferguson, en el estado de Misuri, han saca-
• laísmo: utiliza el pronombre de complemento directo la do a la luz de nuevo las tensiones raciales en Estados Unidos
en lugar del de complemento indirecto, le. que, aunque teó ricamente se han resuelto en las leyes, siguen
siendo una de las grandes asignaturas pendientes de la prin-
b) No los des dinero, que son unos derrochones.
cipal potencia mundial. […]
• loísmo: utiliza el pronombre de complemento directo los
Muy lejos queda ya 2008, cuando Obama se convirtió en el pri-
en lugar del de complemento indirecto, les.
mer presidente negro en la historia de EE. UU. y académicos,
c) A Elvira la quiero mucho. soció logos y periodistas abusaron de la expresió n América
• Oració n correcta. posracial como un reflejo de que el país norteamericano ha-
d) Dilas que mañana vendremos temprano. bía dejado atrá s una era de discriminació n y prejuicios. El mi-
crocosmos de Ferguson sirve, sin embargo, como ejemplo de
• laísmo: utiliza el pronombre de complemento directo que hay una sospechosa desproporció n en las actuaciones
las en lugar del de complemento indirecto, le. contra la comunidad negra. Mientras el 65 % de la localidad de
e) Los vi alejarse en la oscuridad. poco más de 21.000 habitantes es negra, el 94 % de los policías
• Oració n correcta. son blancos. El 84 % de los automó viles que estos agentes de-
tienen para solicitar la documentació n está n conducidos por
f) Te dejo el disco, pero no me le destroces, que te conozco.
negros y el 92 % de los detenidos son negros. A estos datos
• leísmo: utiliza el pronombre de complemento indirecto hay que sumar un 21 % de familias que viven bajo el umbral
le en lugar del de complemento directo, lo. de pobreza y una comunidad en la que no resulta excepcional
g) No la gusta que la mires de esa manera. que se obstaculice la integració n negra por un sentimiento
• laísmo: utiliza el pronombre de complemento directo la discriminatorio que viene de muy lejos. Esa situació n, con
en lugar del de complemento indirecto, le. El segundo variantes, se repite desigualmente a lo largo y ancho de la
la está usado correctamente. geografía de EE. UU.
h) Me le encontré en el parque y le dejé las cosas claras. a) ¿Cuál es la opinión del periódico sobre el tema que
• leísmo: utiliza el pronombre de complemento indirecto trata el editorial? ¿Se percibe con claridad? ¿Por qué?
le en lugar del de complemento directo, lo. El segundo b) ¿Te parece que la acumulación de datos es-tadísticos,
le está usado correctamente. que sirve para dar objetividad, puede estar usándose
i) La miro y no la reconozco. aquí, paradójicamente, para reforzar la opinión subje-
• Oració n correcta. tiva del diario? Razona tu respuesta.
5. Distingue si las siguientes oraciones son reflexivas, recí- c) Tras leer el editorial, ¿te formas tú una opinión sobre
procas, pasivas reflejas o impersonales. Justifica tu res- el problema del racismo en Estados Unidos? ¿Qué
puesta en cada caso: pien- sas al respecto? Puedes tener en cuenta tus
a) Clara y Vicente se aman con pasión. conoci- mientos sobre el país a partir de películas o
b) Se veía perfectamente el destrozo que hizo la series de televisión.
tormenta en la huerta. a), b) y c) Respuesta libre.
c) Se lavó los dientes después de comerse un plato de
cochinillo asado. Recuerda lo que ya sabías
d) Los jugadores se cambiaron las camisetas al final 8. Con los datos que te ofrece el editorial que acabas de
del partido. leer, elabora una noticia en la que estén presentes las
características propias de este género, que aprendiste
a) Oració n recíproca, ya que entre los sujetos hay un inter-
cambio de acciones, en este caso se aman el uno al otro. en la unidad anterior.
b) Pasiva refleja. Aparte de la presencia del pronombre se, Respuesta libre.
la oració n tiene un sujeto paciente: el destrozo que la tor- 9. Busca en el texto del Amadís de Gaula de la actividad 1
menta hizo en la huerta. los adjetivos y los adverbios que estén presentes y di
c) Es una oració n reflexiva, ya que la acció n que realiza el luego qué aportan a la descripción del combate que se
sujeto recae sobre é l mismo. narra.
d) Oració n recíproca, ya que entre los sujetos hay un inter-
Adjetivos: gran, sañudo, corredor. Adverbios: bravamente,
cambio de acciones, en este caso se cambian las
malamente, así, abajo, donde.
camisetas unos a otros.
El adjetivo sañudo aporta ferocidad al caballero y hace que
6. Busca en el texto de la actividad 1 antónimos de las la acció n narrada sea má s intensa. Por su parte, los
siguien- tes palabras y di qué tipo de antonimia se da en adverbios indican circunstancias que redundan en esa
cada caso: misma fiereza del combate, como es el caso de bravamente
curar cobardemente ascender (que aparece dos veces) o malamente. El resto de adjetivos
y adverbios no aportan nada especial al desarrollo del
combate.
10. Analiza los sintagmas nominales presentes en el primer Escribe luego
párrafo del editorial de la actividad 7 e indica los comple-
mentos del núcleo, cuando los haya. ¿Qué otros tipos de
sintagmas están presentes en el fragmento?
Las protestas y disturbios que se han producido tras la
muerte de un joven negro por disparos de un policía en la
localidad estadounidense de Ferguson, en el estado de Misu-
ri, han sacado a la luz de nuevo las tensiones raciales en
Estados Unidos que, aunque teóricamente se han resuelto en
las leyes, siguen siendo una de las grandes asignaturas
pendientes de la principal potencia mundial.
• Los sintagmas nominales aparecen marcados en negrita:
las protestas y disturbios funciona como sujeto del verbo
han sacado; las tensiones raciales, complemento directo
de han sacado; una de las grandes asignaturas pendien-
tes, complemento directo de siguen siendo.
• En el fragmento hay también sintagmas preposicionales
(tras la muerte, de un joven negro, por disparos de la
prin- cipal potencia mundial, etc.). Encontramos
igualmente sin- tagmas verbales (han sacado a la luz..., se
han resuelto en las leyes..., siguen siendo...).
11. Investiga sobre el significado de los siguientes
latinismos y anótalo en tu cuaderno:
a) in dubio pro reo e) in vino veritas
b) panem et circenses f) ex cathedra
c) delirium tremens g) corpore insepulto
d) alter ego h) ab intestato
a) in dubio pro reo: al haber dudas en un juicio se debe actuar
en beneficio del reo.
b) panem et circenses: pan y circo. Se emplea para referirse
a la manera de entretener al pueblo para que no proteste
y se conforme con lo que tiene.
c) delirium tremens: estado de delirio provocado por la in-
gesta excesiva de alcohol.
d) alter ego: otro yo. Se emplea para referirse a alguien que
ocupa a veces el lugar de uno o con quien este se siente
muy identificado.
e) in vino veritas: en el vino está la verdad. Se utiliza para
referirse al estado que provoca el vino en quien lo bebe,
que lo lleva a decir las verdades sin reservas.
f) ex cathedra: desde la cá tedra. Se emplea para aludir a
alguien que habla o se expresa como si fuera el más
sabio, como si hablara desde la cá tedra, como un
catedrá tico.
g) corpore insepulto: cuerpo sin sepultar. Se usa para refe-
rirse a funerales u homenajes que se hacen a los difuntos
antes de que sean enterrados.
h) ab intestato: sin haber hecho testamento. Es una fó rmula
judicial que sirve para expresar que una determinada per-
sona ha muerto sin hacer testamento.

MIRA A TU ALREDEDOR Y…
… ve más allá
1. Ya sabemos que los verbos se utilizan, a menudo, para
narrar las acciones y pueden generar un gran dinamismo
que hace la lectura más rápida e intensa. La misma fun-
ción que los verbos la realiza la cámara en el cine,
cuando actúa como narradora de escenas de acción. Ve
un frag- mento de la película En busca del arca perdida
(1981), de Steven Spielberg. Teclea en YouTube «En
busca del arca perdida» «El despotricador cinéfilo».
una redacción en la que narres la escena que has visto, atención del lector? ¿Qué imagen nos dan de la
empleando todos los verbos necesarios para recrear la sociedad de su tiempo?
intensidad de la acción. a), b) y c) Respuesta libre.
Respuesta libre.
2. Observa la imagen que reproduce la escultura Apolo y
Dafne de Gian Lorenzo Bernini (1598-1680) y lee luego
el soneto XIII de Garcilaso de la Vega (c. 1499-1536).
Valora la expresión de movimiento que producen los
verbos en el poema y compárala con el dinamismo de la
escultura. Describe la escultura explicando de qué modo
se plasman en ella los distintos movimientos recogidos
en el poema.
A Dafne ya los brazos le crecían
y en luengos ramos vueltos se mostraban;
en verdes hojas vi que se tornaban
los cabellos que el oro escurecían;
de á spera corteza se cubrían
los tiernos miembros que aun bullendo estaban;
los blancos pies en tierra se hincaban
y en torcidas raíces se volvían.
Aquel que fue la causa de tal
dañ o, a fuerza de llorar crecer
hacía
este á rbol, que con lá grimas regaba.
¡Oh miserable estado, oh mal tamañ o,
que con llorarla crezca cada día
la causa y la razó n por que lloraba!
Garcilaso DE LA VEGA
Respuesta libre.
3. Observa estas tiras gráficas de El Roto y Forges y
explica qué quieren expresar sus autores. Valora el uso
de la ironía:
Respuesta libre.

… encuentra la clave
1. La prensa escrita comenzó su andadura en el siglo xvıı,
pero se desarrolló, sobre todo, a partir de la segunda
mitad del xıx, con el auge de la industrialización. Te
invita- mos a que te des un paseo por la prensa histórica
a través de Internet. Busca en alguna de las siguientes
páginas algún periódico antiguo de tu provincia o de tu
región y realiza las actividades que te proponemos
después:
• Biblioteca Virtual de Prensa Histórica:
prensahistori- ca.mcu.es
• Hemeroteca digital de la Biblioteca Nacional:
hemero- tecadigital.bne.es
a) Selecciona dos periódicos de épocas diferentes
(por ejemplo, finales del siglo xıx y la década de
los años treinta del siglo xx) y elabora un esquema
con las sec- ciones que hay en cada uno.
Compáralos entre sí y después con un periódico
actual.
b) Busca en esos mismos periódicos las secciones de
opi- nión y lee algún artículo. ¿Qué diferencias ves
entre este y los artículos de opinión actuales?
Analízalas.
c) Una curiosidad de la prensa antigua son los anuncios
publicitarios. Léelos en varios periódicos y haz una
rela- ción de productos anunciados. ¿Son los mismos
que ahora? ¿De qué manera tratan de llamar la
2. Te ofrecemos a continuación el artículo 20 de la Cons- 5. Solo podrá acordarse el secuestro de publicaciones, gra-
titución española de 1978, que trata sobre la libertad de baciones y otros medios de informació n en virtud de
expresión. Léelo y contesta luego a las cuestiones que reso- lució n judicial.
figuran al final:
a) Analiza el uso de las formas verbales conjugadas en
1. Se reconocen y protegen los derechos: el texto. ¿Qué tiempos predominan? ¿Qué personas
a) A expresar y difundir libremente los pensamientos, gramaticales? ¿Por qué crees que se usan estos y no
ideas y opiniones mediante la palabra, el escrito o cual- otros?
quier otro medio de reproducció n.
b) Fíjate ahora en las formas no personales y observa su
b) A la producció n y creació n literaria, artística, científica
uso. ¿Cuáles aparecen? ¿Qué función desempeñan?
y té cnica.
c) A la libertad de cá tedra.
c) Reflexiona sobre el contenido de este artículo 20.
¿Crees que contribuye a una mejor convivencia entre
d) A comunicar o recibir libremente informació n veraz por
los ciudadanos? ¿Por qué?
cualquier medio de difusió n. La ley regulará el derecho
a la clá usula de conciencia y al secreto profesional en a) En el texto hay pocas formas verbales conjugadas, pero de
el ejercicio de estas libertades. entre ellas predomina el uso del futuro imperfecto de indi-
cativo (regulará, garantizará, podrá), que proyecta hacia
2. El ejercicio de estos derechos no puede restringirse me-
ese tiempo las acciones que la ley tendrá que juzgar y anali-
diante ningú n tipo de censura previa.
zar. Hay algú n presente de indicativo (reconocen,
3. La ley regulará la organizació n y el control parlamenta- protegen), que tiene la utilidad de indicar de forma neutra
rio de los medios de comunicació n social dependientes del y duradera los preceptos contenidos en la ley. El uso
Estado o de cualquier ente pú blico y garantizará el acceso exclusivo de la tercera persona garantiza una neutralidad y,
a dichos medios de los grupos sociales y políticos signifi- por lo tanto, una equidad que deben estar siempre
cativos, respetando el pluralismo de la sociedad y de las presentes en la ley.
diversas lenguas de Españ a.
b) En cuanto a las formas no personales, la que predomina
4. Estas libertades tienen su límite en el respeto a los dere- es el infinitivo, que tiene un valor intemporal con una pro-
chos reconocidos en este Título, en los preceptos de las yecció n a un presente que podríamos llamar continuo, con
leyes que lo desarrollen y, especialmente, en el derecho al lo que parece garantizarse la pervivencia de los preceptos
honor, a la intimidad, a la propia imagen y a la protecció n contenidos en este artículo (expresar, difundir,
de la juventud y de la infancia. comuni- car, acordar...).
c) Respuesta libre.
5 HÁBLAME DE TI

APERTURA DE UNIDAD leyes que rigen la realidad, hasta hacer del mundo algo iló gico

1.Según Susie, la narradora, hay muchas personas que es-


criben un diario. ¿Conoces a alguien que comparta esta
inquietud? ¿Por qué le dedican parte de su tiempo a
esta actividad?
Respuesta libre.
2. De acuerdo con Susie, ¿qué se puede contar en un
diario?
¿Es posible que ella consiga hacerse famosa con el relato
de sus historias? ¿Por qué?
Para Susie, en un diario tiene cabida todo lo que se quiera
contar y le pase a uno por la cabeza. No podrá hacerse famo-
sa con su diario, porque, como solo quiere contar su verdad,
prefiere mantenerlo en secreto.
3. El último párrafo del texto plantea la cuestión de la
pri- vacidad. ¿Consideras que la sociedad actual
respeta la privacidad del individuo? ¿Y los jóvenes?
Esta actividad y la siguiente está n destinadas a la reflexió n y
el debate, y deberían servir para alertar al alumnado para
que tomara conciencia de los peligros reales que existen en la
socie- dad actual de la informació n. Por eso, cabe poner el
é nfasis en có mo el desarrollo de las nuevas tecnologías
responde, en parte, a la obtenció n de informació n privada
que podrá ser utilizada por empresas o por el Estado para sus
propios fines. Asimismo, debería destacarse la necesidad de
una conducta responsable en las redes sociales para que los
más jó venes no pusieran sus datos personales al alcance de
cualquier persona o entidad.
4. Con la colaboración de algunos de tus compañeros, en-
tablad un debate en el aula sobre la privacidad. Seguid
estos pasos:
a) En grupos de tres o cuatro, elaborad un guion en el
que respondáis a las siguientes cuestiones:
• ¿Cómo afectan los medios de comunicación y las
redes sociales a la privacidad de cada uno? Identi-
ficad programas televisivos que atenten contra ese
derecho.
• ¿Qué consecuencias tiene para el individuo que
se hagan públicos sus secretos? ¿Conocéis algún
ejemplo?
• ¿A qué mecanismos podemos recurrir para prote-
ger nuestra vida privada y para escapar a posibles
manipulaciones?
b) Elegid un moderador que asigne los turnos de palabra
y regule las intervenciones. Luego, entablad el debate
respetando los turnos de palabra y las indicaciones
del moderador.
Véase respuesta a actividad anterior.

VIVE LA LECTURA
Comprensión lectora
1.¿Qué le sorprende al narrador de la civilización del pla-
neta Soror? Uno de los tópicos literarios más antiguos es
el del «mundo al revés». Explica en qué consiste y, a con-
tinuación, señala cómo se plasma en el texto.
Al personaje-narrador le sorprende y aterroriza có mo se han
intercambiado los papeles y los simios someten a los humanos.
Esta extraordinaria inversió n se ajusta al tó pico del «mundo
al revés», reconocible, cuando menos, desde el escritor griego
Hesíodo y, segú n el cual, se han alterado completamente las

40 Lengua castellana y Literatura. 3.º ESO. Solucionario


HÁBLAME DE TI 5
e incomprensible. Esa es la impresió n que a Ulises le magnética donde se registran grabaciones
provoca, por ejemplo, el hecho de que los gorilas sepan independientes.
hablar y expre- sen sentimientos humanos.
2. Dentro de la sociedad de los simios, ¿qué especie se
com- porta con mayor brutalidad? ¿Crees que existe
algún mo- tivo que justifique dicha conducta? ¿Cuál?
Dentro de la sociedad de los simios, son los gorilas,
identifica- dos como cazadores, quienes muestran un porte
más aristo- crá tico y se comportan con mayor brutalidad.
Dicha conducta podría justificarse por los há bitos de su
oficio, pero, sobre todo, porque pertenecen a la especie con
mayor fortaleza física.
3. ¿Qué gestos utiliza el autor para animalizar a los seres
hu- manos?
Los humanos no solo son cazados con trampas como los
ani- males, sino que su miedo a los simios es de tal calibre
que el pá nico les lleva a morder, a moverse frené ticamente,
a patear con sus cuatro extremidades, rechinar los dientes
lanzando espuma por la boca y morder las cuerdas de la red
que les atrapa.
4. ¿En qué se diferencia Mérou de sus compañeros de
jaula? A diferencia de los otros humanos, Mérou puede
usar un len- guaje articulado, posibilidad que sorprende a
los simios hasta
el punto de interpretarlo como una amenaza.
5. ¿Por qué podrías decir que la situación en que se
halla el narrador es desesperada? Imagínate que eres el
pro- pio Mérou. ¿De qué modo intentarías hacerte
entender y doblegar la violenta hostilidad de los
simios?
Aparte de estar desarmado y a merced de sus
carceleros, el protagonista no sabe có mo hacer frente a una
situació n que le resulta increíble. Ahora le toca al alumno,
imaginar el modo en que el personaje podría afrontar el
problema que se le plantea.

Vocabulario
6. Escribe palabras que pertenezcan a la misma familia
léxi- ca de matanza, servidores, ojeadores y brutalidad.
• matanza: matar, matadero matador, matanza, matarife.
• servidores: servir, servicial, servicio, servidumbre, servil.
• ojeadores: ojeada, ojear, ojeo, ojo.
• brutalidad: brutal, bruto, embrutecer.
7. Con la ayuda de un diccionario, define el término
para- doja. Luego, escribe tres oraciones con dicho
sustantivo.
Podemos definir la paradoja como un contradicció n aparente
entre dos cosas o ideas, pero tambié n como una afirmació n
inverosímil que se presenta con apariencia de verdadera.
Se- rían sinó nimos contradicción o absurdo.
8. ¿Qué distintos significados puede tener la palabra
pista? Escribe una oración con cada uno de ellos.
• Rastro que deja cualquier criatura en el lugar por donde
ha pasado.
• Conjunto de señ ales que puede conducir a la
averiguació n de algo.
• Lugar habilitado para la prá ctica de deportes u otras
actividades.
• Terreno acondicionado para el despegue y aterrizaje
de aviones.
• Camino o autopista.
• Cada uno de los espacios paralelos de una cinta

Lengua castellana y Literatura. 3.º ESO. Solucionario 41


9. Explica el significado de las expresiones: «tenían aire de
h) Unos edificios me parecen muy grandes, otros me pa-
aristócratas» y «se debatían con desesperación».
recen pequeños.
Tenían aire de aristócratas significa, en el texto, que los
a) Nexo: o; disyuntiva excluyente.
gorilas se comportaban como aristó cratas, tal era su poder.
Se debatían con desesperación significa que los humanos lu- b) Nexo: o; disyuntiva excluyente.
chaban y forcejaban por escapar desesperadamente de una c) Nexo: esto es; explicativa.
situació n muy adversa. d) Nexo: ya… ya; distributiva.
10. ¿Cuáles de los siguientes sustantivos podrían reem- e) Nexo: o sea; explicativa.
plazar al sustantivo tumulto en la expresión «Sin prestar f) Nexo: sino; adversativa.
atención alguna a este tumulto»?
g) Nexo: ni; copulativa.
escándalo multitud motín h) Elementos correlativos: Unos… otros; distributiva.
jaleo gentío alboroto
2. Coloca el nexo correspondiente e indica qué relación
tranquilidad aglomeración desorden coordinante se da en estas oraciones compuestas:
En el contexto, podrían reemplazar al sustantivo tumulto los
a) Unos coches se quedaron sin gasolina,____tenían lleno
sustantivos escándalo, motín, jaleo, alboroto y desorden.
el depósito.
Investigación y redacción b) Dentro de mil años el cosmos se deshará,_____llegará
11. Ayudándote de alguna enciclopedia o de Internet, a su fin.
resume el desenlace de la inquietante aventura de Ulises c) El niño se enfadó,_____, perdió los papeles.
Mérou en el planeta Soror. d) No me dejaban salir,____escrutar tras la ventana.
Mérou consigue que una pareja de científicos simios le con-
e) Los turistas llegaron todos empapados,_____tres.
sidere como criatura inteligente procedente de otro planeta.
Parece adaptarse a su nueva situació n y vive como un simio f) Vino pronto_____exhibió sus habilidades.
más, hasta que deja embarazada a Nova y eso despierta los g) ¿Te sirvo croquetas_____prefieres patatas?
recelos de las autoridades de Soror, que temen que los hijos del
a) otros; distributiva.
protagonista sean tan inteligentes como él y pongan en peligro
su dominio. Ayudado por Zyra y Cornellius, Ulises huye con b) o sea, esto es, es decir; explicativa.
Nova y su hijo recié n nacido para embarcarse en una nave c) o sea, esto es, es decir; explicativa.
es- pacial con direcció n a la Tierra. A su llegada, descubre d) sino; adversativa.
que el planeta Soror no es otro que la misma Tierra, aunque, a
e) excepto, salvo; adversativa.
causa de un cataclismo ató mico, la era humana se vio
reemplazada por la era de los simios. Así pues, la peripecia f) y; copulativa.
de Mérou es un viaje en el tiempo, dos mil añ os despué s de g) o; disyuntiva excluyente.
la gran tragedia. 3. Sustituye las palabras marcadas en negrita por una
12. A partir del nombre del protagonista de la novela, es propo- sición. ¿De qué tipo de subordinación hablamos
po- sible establecer una curiosa relación con un célebre en cada caso?
texto épico homérico. ¿Cuál? ¿Qué puntos en común a) Nos gusta tu tranquilidad.
tienen ambas historias? b) Se desplazarán hasta tu casa.
El nombre del protagonista de la novela de Pierre Boulle nos
remite al Ulises de la Odisea de Homero. Además de la coinci- c) El gato extraviado no tenía dueño.
dencia onomá stica, ambos personajes son viajeros que sufren d) Le aterraba la tormenta.
una serie de aventuras de carácter fantá stico en lugares donde e) La muchacha se despidió a la madrugada.
imperan unas leyes distintas a las de la realidad conocida.
f) Os encontrasteis con el jugador lesionado.
13.La novela de Pierre Boulle ha sido adaptada al cine en
varias ocasiones. Investiga sobre el tema e indica las más g) Dime la hora.
famosas. Sin embargo, en tales películas se ofrece una Responder siguiendo los ejemplos siguientes:
versión bastante modificada de la historia compuesta por a) Nos gusta que seas una persona tranquila. Subordina-
el escritor francés. ¿Qué cambios sustanciales en el argu- da sustantiva.
mento adviertes entre el texto literario y sus b) Se desplazarán hasta el lugar en que vives. Subordina-
adaptaciones para la gran pantalla? da adjetiva.
Respuesta libre. c) El gato que andaba extraviado no tenía dueño. Subor-
dinada adjetiva.
ESTUDIO DE LA LENGUA d) Le aterraba que hubiera rayos. Subordinada sustantiva.
1. Subraya el nexo e indica el tipo de coordinación que se e) La muchacha se despidió cuando anocheció. Subordi-
establece en cada caso: nada sustantiva adverbial.
a) ¿La capital de Alemania es Bonn o es Berlín? f) Os encontrasteis con el jugador que estaba lesionado.
Subordinada adjetiva.
b) Devuélveme el dinero o no vengas más.
g) Dime qué hora es. Subordinada sustantiva.
c) Lamentaba su ausencia, esto es, lo echaba de menos.
4. Escribe cinco oraciones compuestas que contengan una
d) Ya me susurraba al oído, ya reía estrepitosamente. proposición sustantiva cada una que realice la función
e) Hay mucha cola, o sea, tardaré en pagar. soli- citada: sujeto, complemento directo, atributo,
f) No era ambidiestro, sino zurdo. complemento de régimen, complemento del nombre.
g) No quiere a su madre, ni tampoco a sus amigos. Respuesta libre.
5. Completa estas secuencias con el nexo oportuno: d) ya que los conocía muy bien: causal.
a) No dijeron____iban a venir a la fiesta.
b) Le preguntaron____estaba satisfecho.
c) Mi padre ignora____he conseguido aprobar.
a) si; pero caben otras alternativas como cuántos, cuándo,
cómo.
b) si; pero con cambio de significado cabe la opció n por qué.
c) cómo
6. Subraya las proposiciones subordinadas, indicando
cuáles son sustantivas y cuáles adjetivas.
a) Tenía la esperanza de que ganaran el premio.
b) Estoy junto a la mujer que extravió la cartera.
c) La gente, que miraba el cuadro, sonreía.
d) Es imposible que lo castiguen por eso.
e) Quiero saber cuándo será tu cumpleaños.
f) Deseo que me compren un perro.
g) Te enojaste con el muchacho al que saludamos.
h) Hallaron la cabaña donde el pirata dormía.
a) de que ganaran el premio: subordinada sustantiva.
b) que extravió la cartera: subordinada adjetiva
especificativa.
c) que miraba el cuadro: subordinada adjetiva explicativa.
d) que lo castiguen por eso: subordinada sustantiva.
e) cuándo será tu cumpleaños: subordinada sustantiva.
f) que me compren un perro: subordinada sustantiva.
g) al que saludamos: subordinada adjetiva especificativa.
h) donde el pirata dormía: subordinada adjetiva
especificativa.
7. Localiza el antecedente de las subordinadas adjetivas de
la actividad anterior y trata de identificar la función
sintáctica que desempeña el nexo relativo.
b) antecedente: mujer; el nexo funciona como sujeto.
c) antecedente: gente; el nexo funciona como sujeto.
g) antecedente: muchacho; el nexo funciona como comple-
mento directo.
h) antecedente: cabaña; el nexo funciona como complemento
circunstancial de lugar.
8. Subraya las subordinadas adverbiales e indica de qué
tipo es cada una:
a) Trajeron el cuadro para que lo colgaras en la pared.
b) Terminó el examen, por tanto, pudo salir al patio.
c) Los nómadas cabalgan hacia donde los llevan sus
camellos.
d) No le engañaron en absoluto, ya que los conocía
muy bien.
e) Con tal de que asistan, recibirán un magnífico regalo.
f) Aunque era rico, siempre iba mal vestido.
g) Realizarán el examen según les he dicho.
h) Ese plato tiene más calorías que todos los demás.
i) El ladrón estaba tan rabioso que extravió su botín.
j) La expedición prosiguió su ruta cuando amaneció.
k) Estoy descontento porque me has mentido.
l) Estoy satisfecho, por tanto, no comeré más.
a) para que lo colgaras en la pared: final.
b) por tanto, pudo salir al patio: consecutiva.
c) hacia donde los llevan sus camellos: de lugar.
e) Con tal de que asistan: condicional.
f) Aunque era rico: concesiva.
g) según les he dicho: de modo.
h) más calorías que todos los demás: comparativa de
supe- rioridad.
i) que extravió su botín: consecutiva ponderativa.
j) cuando amaneció: de tiempo.
k) porque me has mentido: causal.
l) por tanto, no comeré más: consecutiva.
9. En este fragmento hay varias oraciones compuestas.
Iden- tifica el tipo de coordinadas y subordinadas que
intro- ducen los nexos destacados y señala las
proposiciones respectivas:
El mundo era armó nico para Ramó n en aquel momento,
hasta que en la esquina, la del Payaso Fofó con la avenida,
descubrió entre el pú blico a las cuatro hembras de sus
ojos, que le salu- daban enternecidas, con esa sonrisa que se
dedica a los niñ os cuando hacen una travesura perdonable.
A Ramó n se le heló la canció n en los labios, pero venció la
ligera incomodidad interior para saludarlas con su mano-
tijera, y volvió a su ver- dadera naturaleza, la de hué rfano
de por vida, hijo pó stumo, niñ o eterno, aunque a
principios de añ o fuera a cumplir ya diecisé is añ os. Su
descenso al lado salvaje de la vida había durado menos
que un viaje en ascensor.
Elvira LINDO: El otro
barrio.
• que le saludaban enternecidas: subordinada adjetiva
explicativa.
• que se dedica a los niños: subordinada adjetiva especi-
ficativa.
• cuando hacen una travesura perdonable: subordinada
adverbial de tiempo.
• pero venció la ligera incomodidad interior: coordinada
adversativa.
• y volvió a su verdadera naturaleza, la de huérfano de
por vida, hijo póstumo, niño eterno: coordinada
copulativa.
• aunque a principios de año fuera a cumplir ya
dieciséis años: subordinada adverbial concesiva.
• menos que un viaje en ascensor: subordinada
adverbial comparativa de inferioridad.
10. Escribe una continuación para la historia de la actividad
anterior en la que emplees al menos una proposición
su- bordinada adverbial impropia de cada tipo.
Respuesta libre.
11.¿De qué tipo son las siguientes oraciones
compuestas por coordinación? Señala cuál es su nexo:
a) Algunos vecinos no estaban de acuerdo, pero
acepta- ron la decisión de la mayoría.
b) Ni yo sé hacer el trabajo ni el sueldo me parece
sufi- ciente.
c) Todos los miércoles voy al gimnasio y al día
siguiente me siento fenomenal.
d) Ya los perros aullaban aterrados, ya parecían
estar conformes.
e) La concursante hizo un doble salto, o sea, se jugó
el todo por el todo.
a) Nexo: pero; adversativa.
b) Nexo: ni; copulativa.
c) Nexo: y; copulativa.
d) Nexo: ya… ya; distributiva.
e) Nexo: o sea; explicativa.
12. Lee el comienzo de un cuento de Carmela Greciet y ana- ralektryonoptekephalliokigklopeleiolagoiosiraiobaphetra-
liza sus oraciones. Sepáralas en simples y compuestas. ganopterygon.
Indica de qué tipo son estas últimas: • 163 letras: krungthepmahanakhonmonrattanakosin
Mi madre se entretuvo con el mó vil en el momento en que Mahintharayutthaya MahadilokphopNoppharatrat-
íbamos a entrar en el ascensor para bajar al parque y no chathaniburiromudomratchaniwetmahasathanAmonphi
pudo evitar quedarse paralizada en el rellano cuando las - manawatansathit
puertas se cerraron conmigo dentro. Sakkathattiyawitsanukamprasit, nombre original de
Aparecí unos pisos más arriba, donde me esperaba una señ ora la ciudad tailandesa de Bangkok que significa ciudad de
a la que nunca antes había visto y que me gritaba enfadada á ngeles, la gran ciudad, la ciudad de joya eterna, la ciudad
en […] lengua ucraniana. Con el vé rtigo de mi primer viaje en impenetrable del dios Indra, la mag- nífica capital del mundo
solitario, no tuve fuerzas para llevarle la contraria e hice lo dotada con nueve gemas precio- sas, la ciudad feliz, que
que sus gestos, con el lenguaje universal de las madres, me abunda en un colosal Palacio Real que se asemeja al
indicaban: «Entra pa casa». domicilio divino donde reinan los dioses reencarnados, una
Se esbozan las proposiciones coordinadas y subordinadas: ciudad brindada por Indra y construida por Vishnukam.
• en que íbamos a entrar en el ascensor: subordinada adje- • 130 letras, en sueco: nodöstersjökustartilleriflygspa-
tiva especificativa. ningssimulatoranläggningsmaterielunderhallsuppfölj-
• para bajar al parque: subordinada adverbial final. ningssy-stemdiskussionsinläggsförberedelsearbeten, con
el significado de artillería de la costa norte del Bá ltico,
• y no pudo evitar…: coordinada copulativa.
construcció n de un simulador de vuelo, sistemas de moni-
• quedarse paralizada en el rellano: subordinada sustantiva torizació n y mantenimiento y preparació n de pó steres de
de complemento directo. comunicació n.
• cuando las puertas se cerraron conmigo dentro: subordi- • 85 letras: taumatawhakatangihangakoauauotamateatu-
nada adverbial de tiempo. ripukakapikimaungahoronukupokaiwhenuaki-tanatahu,
• donde me esperaba una señora…: subordinada adjetiva en maorí, para referirse a un topó nimo: la cumbre donde
explicativa. Tamatea, el hombre de grandes rodillas, el escalador de
• a la que nunca antes había visto y que me gritaba montañ as, el que se traga la tierra cuando viaja, toca la
enfada- da en […] lengua ucraniana: dos subordinadas flauta nasal a su amada.
adjetivas especificativas unidas mediante coordinada 2. Mediante un anagrama, obtén un adjetivo calificativo a
copulativa. partir de la palabra argentino, y un sustantivo a partir del
• para llevarle la contraria: subordinada adverbial final. adverbio armoniosamente.
• e hice…: coordinada copulativa. argentino: tangerino; armoniosamente: enamoramientos.
• lo que sus gestos […] me indicaban: subordinada sustan- 3. ¿Qué tienen de especial las frases Se van sus naves y
tiva de complemento directo. No di mi decoro, cedí mi don?
La ú nica oració n simple es «Entra pa casa». Las frases Se van sus naves y No di mi decoro, cedí mi don
son palíndromos porque se leen igual hacia delante que hacia
EL TALLER DE LAS PALABRAS atrá s.
1. Investiga en Internet cuáles son las cinco palabras más 4. Lee atentamente el siguiente abecegrama de
largas del mundo. Francisco Briz Hidalgo. ¿Puedes deducir a través de él
• 1185 letras: acetylseryltyrosylserylisoleucylthreonylseryl- qué es un abecegrama? Defínelo en tu cuaderno.
prolylserylglutaminylphenylalanylvalylphenylalanylleu- Anoche brillaron cerca, chispeantes, dos estrellas fugaces;
cylserylserylvalyltryptophylalanylaspartylprolylisoleu- gravitaban hermosas iluminando juntas kilométricos luga-
cylglutamylleucylleucylasparaginylvalylcysteinylthreon- res; llevaban mágicos negros ñ ublos; originaban planetas
ylserylserylleucylglycylasparaginylglutaminylphenyla- que relucían surcando tenues universos…, vertiendo wolfra-
lanylglutaminylthreonylglutaminylglutaminylalanylar- mio, xenó n y zafiros.
ginylthreonylthreonylglutaminylvalylglutaminylgluta- Un abecegrama es un texto integrado por palabras que se
minylphenylalanylserylglutaminylvalyltryptophyllysyl- disponen en orden alfabé tico, de modo que la primera empe-
prolylphenylalanylprolylglutaminylserylthreonylvalylar- zará por a, la segunda por b y así sucesivamente hasta llegar
ginylphenylalanylprolylglycylaspartylvalyltyrosyllysyl- a la palabra final que empezará con z.
valyltyrosylarginyltyrosylasparaginylalanylvalylleucylas-
partylprolylleucylisoleucylthreonylalanylleucylleucylgly- 5. Ahora inventa tú un abecegrama y escríbelo en tu cua-
cylthreonylphenylalanylaspartylthreonylarginylaspara- derno.
ginylarginylisoleucylisoleucylglutamylvalylglutamylaspa- Respuesta libre.
raginylglutaminylglutaminylserylprolylthreonylthreonyla- 6. En la literatura española de los siglos xvı y xvıı los
lanylglutamylthreonylleucylaspartylalanylthreonylargin- abece- gramas recibieron el nombre abecé y fueron
ylarginylvalylaspartylaspartylalanylthreonylvalylalanyli-
utilizados por escritores como Cervantes o Lope de
soleucylarginylserylalanylasparaginylisoleucylasparagin-
ylleucylvalylasparaginylglutamylleucylvalylarginylgly-
Vega. En el Quijote aparece un abecé muy famoso,
cylthreonylglycylleucyltyrosylasparaginylglutaminylas- donde se explican las cua- lidades que debe tener un
paraginylthreonylphenylalanylglutamylserylmethionylse- buen enamorado. Búscalo en Internet y cópialo en tu
rylglycylleucylvalyltryptophylthreonylserylalanylprolyla- cuaderno.
lanylserine, en inglés para referirse a un término químico. «É l es, segú n yo veo y a mí me parece, agradecido, bueno, ca-
• 182 letras, inventada por el escritor griego Aristó fanes ballero, dadivoso, enamorado, firme, gallardo, honrado, ilus-
para referirse a una comida imaginaria: lopadotemacho- tre, leal, mozo, noble, honesto, principal, quantioso, rico y las
selachogaleokranioleipsanodrimhypotrimmatosilphiopa- eses que dicen, y luego, tá cito, verdadero. La x no le cuadra,
raomelitokatakechymenokichlepikossyphophattoperiste- porque es letra á spera; la y ya está dicha; la z, zelador de tu
honra» (Quijote, Primera parte, cap. XXXIV).
7. Lee el siguiente acertijo y resuélvelo mediante un ana- Algunos ejemplos de calambur:
grama:
• Y lo es, y lo es. Quien no lo adivine tonto es: hilo.
Este enigma está basado
• Si el Rey no muere, el Reino muere.
en los días de la semana,
se trata de averiguar, • Yo loco, loco, y ella loquita: Yo lo coloco y ella lo quita.
mejor hoy que mañ ana, • ¡Ave!, Cé sar de Roma: A veces arde Roma.
la solució n adecuada: • Alberto Carlos Bustos: Al ber tocar los bustos.
¿qué día de la semana
se oculta con anagrama • ¿Por qué lavo la rueda? ¿Por qué la bola rueda?
en una de estas palabras?
EL RINCÓN DE LA NORMA
sá bado: basado.
8. Escribe en tu cuaderno cinco palíndromos. 1.En estas oraciones, hay comas mal utilizadas. Explica en
qué consiste el error en cada caso y corrígelo (presta es-
Ejemplos de palíndromos: allá, arañara, nadan, radar,
pecial atención a los casos en que debas sustituir por
rayar, reconocer, seres, somos, etc.
punto y coma):
9. Busca los famosos versos acrósticos que aparecen en La
Celestina y escribe en tu cuaderno la información que Corrige
está oculta en ellos. a) Mi hermana, tiene ya veinte años.
«El bachiller Fernando de Rojas acabó la comedia de Calysto
b) Ha llovido toda la mañana, las calles están llenas
y Melybea y fue nascido en la puebla de Montalbá n».
de charcos.
10. Busca en el diccionario el significado de las siguientes pa-
c) Hoy tenemos que comprar tela, hilo, alfileres, pre-
labras. ¿Son homónimas o polisémicas?
parar los disfraces de la fiesta, redactar las invita-
a) cavo (verbo cavar) y cabo ciones, la lista de los que vendrán y sus teléfonos.
b) botar y votar d) Estimados amigos, os envío estas líneas para da-
c) vela (verbo velar) y vela ros una noticia inquietante.
a) Homó nimas. e) Se han agotado las entradas del concierto; de la
b) Homó nimas. función de teatro y de la exposición.
c) Homó nimas.
a) Mi hermana tiene ya veinte años. No debe usarse la coma
11. Escribe una oración con cada una de las palabras entre el sujeto y el predicado.
de la actividad anterior. b) Ha llovido toda la mañana; las calles están llenas de
Respuesta libre. charcos. Se usa punto y coma porque el tono al final de la
12. Explica el significado de estas palabras e indica si se trata frase es descendente. Ademá s, entre las dos proposiciones
de palabras homógrafas u homófonas: media una relació n de causa-efecto subordinante.

a) haya (sustantivo) y haya (verbo haber) c) Hoy tenemos que comprar tela, hilo, alfileres; preparar
los disfraces de la fiesta; redactar las invitaciones, la
b) a y ha lista de los que vendrán y sus teléfonos. Se usa el punto y
c) arroyo y arrollo (verbo arrollar) coma porque hay enumeraciones con comas en el interior
d) basto y vasto de los elementos enumerados.
a) haya (sustantivo: á rbol) y haya (verbo haber): homó grafas. d) Estimados amigos: os envío estas líneas para daros una
noticia inquietante. En los encabezamientos de las cartas
b) a (preposició n) y ha (verbo haber): homó fonas. hay que usar dos puntos y no la coma.
c) arroyo (sustantivo: riachuelo) y arrollo (verbo arrollar):
e) Se han agotado las entradas del concierto, de la función
homó fonas.
de teatro y de la exposición. No puede haber punto y coma
d) basto (sustantivo: palo de la baraja o adjetivo: grosero) y entre los términos enumerados.
vasto (adjetivo: extenso): homó fonas.
2. Coloca en este fragmento de Eloísa está debajo de un
13. Construye una oración en la que utilices correctamente almendro, de Enrique Jardiel Poncela, los puntos, las ma-
los siguientes homófonos: yúsculas y los signos de interrogación y exclamación.
a) a ver y haber
Corrige
b) así mismo y a sí mismo
c) hasta y asta PRÁ XEDES.— se puede sí, porque no hay nadie que no hay
nadie bueno; hay alguien, pero como si no hubiera na-
d) agito y ajito die hola qué hay qué haces aquí perdiendo el tiempo,
e) botar y votar no tú dirá s que no, pero yo digo que sí qué ah bueno,
f) combine y convine por eso… que por qué vengo porque me lo han man-
dado quién la señ ora mayor.
Respuesta libre.
14. El calambur es un recurso retórico que consiste en usar PRÁ XEDES.— ¿Se puede? Sí, porque no hay nadie. ¿Qué no hay
palabras de fonética parecida, aunque con significado nadie? Bueno; hay alguien, pero como si no hubiera
dis- tinto (homófonos). Por ejemplo: oro parece, plata nadie.
¡Hola! ¿Qué hay? ¿Qué haces aquí? Perdiendo el tiempo,
no es,
¿no? Tú dirá s que no, pero yo digo que sí. ¿Qué? ¡Ah! Bueno,
¿qué es? Plátano. Anota tú otros ejemplos de por eso… ¿Que por qué vengo? Porque me lo han mandado.
calambures que conozcas o invéntate alguno.
¿Quié n? La señ ora mayor.
3. Incorpora los dos puntos siempre que sea necesario y
ACTIVIDADES FINALES
explica por qué los has empleado:
Repasa lo que has aprendido
Corrige 1. Lee este apartado, extraído de un manual de formación
a) Eusebio me respondió «Eres muy atrevida». vial cuyo estudio es necesario para la obtención del per-
b) Sus preferencias eran la buena comida, la música miso de conducción.
disco y la lectura. Obligaciones del conductor que adelanta:

c) Apenas lloró todavía le quedaban lágrimas. Antes de adelantar, el conductor debe comprobar que puede
hacerlo sin ningú n riesgo y advertirlo con suficiente ante-
d) Estos son los artículos rebajados bañadores, cha- lació n. Asimismo, deberá comprobar que el carril que va a
lecos, pareos y gafas de sol. ocupar está libre y tener cuidado de no poner en peligro ni
e) Apreciado cliente la nueva oferta telefónica que le entorpecer la marcha a los vehículos que se aproximen de
proponemos le dejará sin habla. frente ni a ningú n otro usuario de la vía; en caso contrario,
no se debe adelantar.
a) Eusebio me respondió: «Eres muy atrevida». Se reprodu- Ademá s no se debe acercar demasiado al vehículo que va a
cen las palabras dichas por otra persona. ser adelantado ni quedarse demasiado lejos.
b) Sus preferencias eran la buena comida, la música disco Durante el adelantamiento, el conductor que adelanta debe
y la lectura. No se incluyen si no hay elemento anticipador. circular a una velocidad notoriamente superior a la del ve-
c) Apenas lloró: todavía le quedaban lágrimas. En proposi- hículo que pretende adelantar y dejar, entre ambos, una sepa-
ciones yuxtapuestas. ració n lateral suficiente para realizar el adelantamiento con
seguridad.
d) Estos son los artículos rebajados: bañadores, chalecos,
Después de adelantar, se debe poner el intermitente derecho
pareos y gafas de sol. En enumeraciones con un término
y regresar al carril derecho de forma gradual y sin obligar al
anticipador (artículos rebajados).
vehículo adelantado a modificar su trayectoria o velocidad.
e) Apreciado cliente: la nueva oferta telefónica que le pro- Manual Matfer
ponemos le dejará sin habla. En encabezamiento de carta
para dirigirse a un tú . a) El texto está basado en varios decretos legislativos so-
bre tráfico y circulación de vehículos de motor.
LA FACTORÍA DE TEXTOS ¿Cómo se pone de manifiesto su carácter
prescriptivo? Anota en tu cuaderno las palabras que
1. Una receta de cocina puede considerarse como un texto lo evidencien.
donde se dan unas instrucciones muy precisas. Invéntate b) Subraya los conectores y expresiones que
un texto de este tipo para elaborar un plato exquisito. contribuyen a ordenar minuciosamente la exposición,
Respuesta libre. e indica qué idea introduce cada uno de ellos.
2. Imagina que quieres pedirle a la directora de tu instituto c) ¿Qué semejanzas y diferencias encuentras entre este
que os permitan utilizar el patio por la tarde, después del texto y, por ejemplo, un escrito con instrucciones
horario escolar. Escribe una instancia siguiendo la estruc- para montar un mueble?
tura expuesta más arriba. d) En el fragmento aparecen diversas oraciones
Respuesta libre. compues- tas. Localiza en él los siguientes tipos de
3. El siguiente fragmento pertenece a Los sufrimientos del proposiciones:
joven Werther, de J. W. Goethe, una novela decisiva en • Una coordinada copulativa negativa.
el surgimiento del Romanticismo alemán. Gran parte de • Una subordinada adverbial de modo.
la obra está integrada por una sucesión de cartas. Sin • Una subordinada adverbial final.
embargo, en la que aquí se reproduce faltan algunos ele- a) El cará cter prescriptivo del texto se concreta desde su mis-
mentos típicos de la estructura clásica de las cartas per- mo encabezamiento, donde se alude a las obligaciones o
sonales. ¿Cuáles? Ayuda al protagonista a reescribir su normas a las que deberá atender el conductor. La exposi-
epístola para que se ajuste al esquema estudiado. ció n sigue un orden claro: antes de adelantar, durante el
4 de diciembre adelantamiento y después de adelantar. Predomina, ade-
má s, en el texto, la modalidad imperativa, materializada,
Te lo ruego… Mira, estoy acabado, ya no aguanto má s. Hoy
segú n se verá en actividades posteriores, en el empleo de
estaba sentado a su lado, estaba sentado; ella, al piano,
las perífrasis verbales de obligació n y el uso del se con
tocan- do toda clase de melodías, y todas ellas ¡con qué
valor impersonal.
expresió n! […] A mí se me saltaban las lá grimas. Me incliné y
cayó ante mi vista su anillo de desposada, se me saltaban las b) Aparte de los conectores mencionados en el ejercicio ante-
lá grimas. Y de repente le vino a la mente la vieja melodía, la rior que ordenan temporalmente los tres momentos de que
melodía celestial, así, de repente, y he aquí que por mi alma consta cualquier adelantamiento: antes, durante y después,
pasa un sentimiento de consuelo y un recuerdo del pasado, de el texto emplea también conectores de contraste, como
los tiem- pos en que yo había oído esta canció n. en caso contrario, y de adició n como asimismo y
además, para contemplar todas las precauciones que
En el fragmento reproducido faltan elementos típicos de la debe tomar el conductor a la hora de realizar el citado
estructura clá sica de la carta personal como el encabezamien- adelantamiento.
to, la introducció n y la despedida. Son los aspectos que el
alumnado deberá tener en cuenta para completar la epístola c) Ambos son textos prescriptivos con una finalidad prá c-
y, por tanto, la actividad. tica, pues a fin de obtener un determinado objetivo se
organizan mediante una enumeració n ordenada de pasos
que seguir. Coinciden tambié n en el uso de la modalidad
imperativa, siendo su principal diferencia el empleo de un
léxico específico que varía segú n el tema tratado.
d) • No se debe acercar demasiado al vehículo […] ni que- • tendrá lugar
darse demasiado lejos.
• Sin obligar al vehículo adelantado…
• Para realizar el adelantamiento con seguridad.
2. Clive Staples Lewis fue un prestigioso profesor universi-
tario que también alcanzó el éxito como escritor de no-
velas infantiles y juveniles. El texto que sigue pertenece
a El caballo y el muchacho, uno de los siete libros que
integran la famosa serie de Las crónicas de Narnia. Léelo
y reescribe los tres primeros enunciados para
convertirlos en una carta personal.
Shasta no sentía el menor interé s por lo que estaba situa-
do al sur de su hogar porque en una o dos ocasiones ha-
bía estado en el pueblo con Arsheesh y sabía que allí no
había nada interesante. En el pueblo solo encontraba a otros
hom- bres que eran iguales a su padre: hombres con largas
tú nicas sucias, zapatos de madera con las puntas vueltas
hacia arriba, turbantes en las cabezas y el rostro barbudo,
que hablaban entre sí muy despacio sobre cosas que
parecían aburridas. Sin embargo, sí le atraía en gran medida
todo lo que se encontraba al norte, porque nadie iba jamás en
aquella direcció n y a él tam- poco le permitían hacerlo. Cuando
estaba sentado en el exterior remendando redes, y totalmente
solo, a menudo dirigía ansio- sas miradas en aquella
direcció n. No se veía nada, a excepció n de una ladera
cubierta de hierba que se alzaba hasta una loma baja y, más
allá , el cielo y tal vez unas cuantas aves en é l.
Respuesta libre.
3. Vuelve a leer el texto de C. S. Lewis y resuelve las siguien-
tes tareas:
a) Indica la función sintáctica que desempeña el nexo en
la proposición subordinada adjetiva que eran iguales
a su padre.
b) ¿Qué tipo de subordinación se da en la proposición
que allí no había nada interesante? ¿Qué función
desempeña esta subordinada con respecto al verbo
sabía?
c) En eran iguales a su padre, el verbo ser posee su habi-
tual naturaleza atributiva. En cambio, en otros
contextos se requiere de una mayor precisión léxica
para recono- cer su valor predicativo. Sustituye en
estas oraciones el verbo ser por alguno de los
siguientes: suceder, alcan- zar, tener lugar y
pertenecer.
• El aterrizaje será en media hora.
• Ese coche fue de mi prima.
• Todo fue tan breve que ni siquiera nos besamos.
• En el accidente la cifra de heridos fue de cien.
d) Las palabras sombreadas en negrita en el texto de la
acti- vidad 2 desempeñan una función deíctica, puesto
que se refieren y sustituyen a otros vocablos o
expresiones que han aparecido previamente. ¿A qué
categoría gramatical pertenece cada una de estas
palabras? ¿Qué tipo de deixis textual ejemplifican?
a) El relativo que funciona como sujeto en la proposició n
su- bordinada adjetiva.
b) … que allí no había nada interesante: subordinada sus-
tantiva en funció n de complemento directo.
c) Junto con la sustitució n del verbo ser por los mencionados,
el alumno podrá realizar algunos mínimos cambios en las
oraciones para hacerlas gramaticales:
• perteneció a g) El condicional simple del verbo salir.
• sucedió h) El futuro simple del verbo andar.
• alcanzó
d) • allí: adverbio de lugar, sustituye a en el pueblo.
• que: pronombre relativo, sustituye a otros hombres.
• él: pronombre personal tó nico, sustituye a Shasta.
• le: pronombre personal á tono, sustituye a Shasta.
• aquella: determinante demostrativo, se refiere a lo
que está al norte, lejos del hogar de Shasta.
Todas las palabras citadas son casos de deixis textual
ana- fó rica, porque el elemento sustituido va antes del
vocablo que lo sustituye.
4. Completa las siguientes oraciones con punto, dos
puntos, coma, punto y coma, signos de interrogación
o de excla- mación donde sea necesario:
a) Oye__ Sergio estás seguro de lo que has dicho__
b) Los ríos con los nombres más bonitos son tres__ el
Lim- popo__ el Zambeze__ y el Amazonas__
c) Ayer me compré__ un bañador__ que ya me hacía
falta__ unas sandalias __que las que tengo están
muy usadas__ y unas gafas de sol__
d) __Socorro__ auxilio__ me ahogo__
a) Oye, Sergio, ¿estás seguro de lo que has dicho?
b) Los ríos con los nombres más bonitos son tres: el
Limpo- po, el Zambeze y el Amazonas.
c) Ayer me compré un bañador, que ya me hacía falta;
unas sandalias, que las que tengo muy usadas; y
unas gafas de sol.
d) ¡Socorro, auxilio, me ahogo!

Recuerda lo que ya sabías


5. El reglamento precedente de circulación nos permite
re- pasar los tiempos verbales. Encuentra en el texto
de la ac- tividad 1 una forma verbal en cada uno de
estos tiempos:
• presente de indicativo
• futuro de indicativo
• presente de subjuntivo
• está, pretende
• deberá
• aproximen
6. Conjuga en todas sus personas gramaticales los
siguien- tes tiempos verbales:
a) El pretérito perfecto simple de indicativo del
verbo
poner.
b) El pretérito imperfecto de subjuntivo del verbo
saber.
c) El presente de indicativo del verbo dormir.
d) El pretérito pluscuamperfecto de indicativo del
verbo
proveer.
e) El presente de subjuntivo del verbo ir.
f) El presente de imperativo del verbo ir.
a) puse, pusiste, puso, pusimos, pusisteis, pusieron.
MIRA A TU ALREDEDOR Y…
b) supiera o supiese, supieras o supieses, supiera o supiese,
supiéramos o supiésemos, supierais o supieseis, supie- … ve más allá
ran o supiesen. 1. Cuando se habla de famosos diarios personales, se nos
c) duermo, duermes, duerme, dormimos, dormís, duermen. viene de inmediato a la memoria la amarga experiencia
d) había proveído, habías proveído, había proveído, relatada en unos cuadernos por Ana Frank. Durante la
había- mos proveído, habíais proveído, habían Segunda Guerra Mundial, los ejércitos alemanes invadie-
proveído. ron Holanda. Ana pertenecía a una familia judía y estuvo
e) vaya, vayas, vaya, vayamos, vayáis, vayan. escondida con ella en una casa de Ámsterdam, entre los
f) ve (tú), id (vosotros). años 1942 y 1944, hasta que se descubrió su escondrijo y
fue trasladada a un campo de concentración nazi. Desde
g) saldría, saldrías, saldría, saldríamos, saldríais,
su encierro escribió cosas como esta:
saldrían.
h) andaré, andarás, andará, andaremos, andaréis, andarán. Jueves, 19 de noviembre de 1942

7. Enumera las perífrasis modales de obligación y posibili- dad A Kitty


que encuentres en el texto de la actividad 1, distin- guiendo No hay noche en que los coches militares verdes o grises no
en cada caso entre los verbos auxiliares y los auxiliados. recorran la ciudad; los alemanes llaman a todas las puer-
tas para dar caza a los judíos. Si los encuentran, embarcan
inmediatamente a toda la familia; si no, llaman a la puerta
siguiente. Los que no se ocultan no escapan a su suerte. En
Perífrasis modal de
• puede (auxiliar) hacer (auxiliado) ocasiones, los alemanes se dedican a eso sistemá ticamen-
posibilidad
te, lista en mano, golpeando a las puertas tras las cuales,
• debe (auxiliar) comprobar piensan, les aguarda un rico botín. A veces se les paga un
(auxiliado) rescate, a tanto por cabeza, como en los mercados de escla-
• deberá (auxiliar) comprobar vos de antañ o. Es demasiado trá gico para que tú lo tomes a
(auxiliado) broma. Por la noche, veo a menudo desfilar a esas caravanas
• debe (auxiliar) adelantar de inocentes, con sus hijos llorando, arrastrados por algunos
Perífrasis modal de (auxiliado)
brutos que los azotan y los torturan hasta hacerlos caer. No
obligación • debe (auxiliar) acercar (auxiliado) respetan a nadie, ni a los viejos, ni a las criaturas, ni a las
• debe (auxiliar) circular (auxiliado) mujeres embarazadas, ni a los enfermos: todos deben tomar
• (debe) (auxiliar) dejar (auxiliado) parte en esa ronda de la muerte.
• debe (auxiliar) poner (auxiliado)
Ana FRANK: Diario.
• (debe) (auxiliar) regresar
(auxiliado) a) Aunque ha habido estudiosos que han dudado sobre
8. Indica la función sintáctica que desempeñan los sintag- la autenticidad de este diario y han puesto en duda la
mas destacados: autoría de Ana, difícilmente puede discutirse el valor
• advertirlo con suficiente antelación informativo de su relato.
• [el carril] está libre • ¿Qué se conoce como el «holocausto judío»?
• entorpecer la marcha • ¿Por qué el régimen hitleriano persiguió a ese
• regresar al carril derecho de forma gradual pueblo?
• dejar una separación lateral suficiente • Su «antisemitismo» ¿era un caso de «racismo» o de
«xenofobia»?
• puede hacerlo sin ningún riesgo
• Con la ayuda de un diccionario, define los tres tér-
• entorpecer la marcha a los vehículos
minos entrecomillados.
• lo: complemento directo; con suficiente antelación: com-
b) ¿Conoces algún conflicto bélico internacional reciente
plemento circunstancial de modo.
que haya surgido por motivos raciales o religiosos?
• libre: atributo. ¿Cuáles?
• la marcha: complemento directo.
c) Investiga para qué servía este símbolo durante la
• de forma gradual: complemento circunstancial de modo. dicta- dura nazi y quiénes debían llevarlo. Averigua
• una separación lateral suficiente: complemento directo. también si existían otros símbolos para marcar a otras
• sin ningún riesgo: complemento circunstancial de modo. personas y cuáles eran.
• a los vehículos: complemento indirecto. d) En España también los judíos fueron víctimas de dra-
9. Explica el significado de los siguientes prefijos y escribe al máticas persecuciones y matanzas. ¿En qué época
menos dos palabras que los contengan: hecto-, hetero-, tuvo lugar tal fenómeno? Justifica tu respuesta con
he- mato-, helio-, hidra-, homo-. datos históricos.
• hecto-: cien; hectárea, hectogramo, hectolitro, hectómetro. e) ¿Qué semejanzas adviertes entre la narración de
Ana y una noticia periodística? Convierte el relato
• helio-: sol; heliocéntrico, helioterapia.
de Ana en una noticia. No olvides que debes ser obje-
• hemato-: sangre; hematoma, hematosis, hematocrito. tivo y veraz.
• hetero-: diferente, otro; heterodoxia, heterogéneo. a) • El «holocausto judío» fue el terrible genocidio, durante
• hidro-: agua; hidroavión, hidroeléctrico, hidrosfera. la Segunda Guerra Mundial, de seis millones de judíos
• homo-: mismo, igual; homófono, homogéneo, homologar. por parte del gobierno nazi de Hitler.
• El ré gimen hitleriano persiguió al pueblo hebreo am-
2. A los efectos de la presente Declaració n, se entiende por
pará ndose en la creencia de que los alemanes pertene-
«intolerancia y discriminació n basadas en la religió n o
cían a una raza superior, la aria, y, como los judíos
las convicciones» toda distinció n, exclusió n, restricció n
eran una etnia inferior, merecían morir por ello. Los
o preferencia fundada en la religió n o en las convicciones
nazis también consideraban inferiores a otras razas
y cuyo fin o efecto sea la abolició n o el menoscabo del
como los gitanos u otros pueblos eslavos como rusos y
reconocimiento, el goce o el ejercicio en pie de igualdad
polacos.
de los derechos humanos y las libertades fundamentales.
• Su «antisemitismo» es un caso de racismo.
a) ¿Con qué tipo de textos relacionarías el artículo? ¿Por
• Antisemitismo: actitud contraria y hostil frente a la raza
y cultura hebreas; racismo: aversió n a otros grupos
qué?
é t- nicos; xenofobia: odio u hostilidad a los b) ¿Se respetan los principios enunciados en el escrito
extranjeros. en España? En el caso de no ser así, ¿qué medidas
b) Los conflictos bélicos por cuestiones raciales siguen es- adop- tarías para evitar cualquier tipo de
tando, desafortunadamente, a la orden del día. En discriminación? Haz tus propias propuestas.
muchas ocasiones, a las diferencias raciales se le suman a) El fragmento reproducido es relacionable con los regla-
discre- pancias religiosas y conflictos econó micos. mentos. Se inscribiría dentro del á mbito jurídico, junto a
Tomando como referencia algunos de los casos que se los textos prescriptivos que se consideran como normas
citan, podría animarse al alumnado a profundizar en tan de obligado cumplimiento en los lugares donde ha sido
trá gicos epi- sodios: Rodhesia, apartheid sudafricano, aprobada la citada Declaració n. De hecho, al igual que
conflicto de los Balcanes, genocidio de Ruanda. en los textos legales, el artículo va dividido, a su vez, en
c) A los judíos se les marcaba con la insignia amarilla o es- otros dos subapartados donde se precisa el alcance de los
trella de David sobre un fondo amarillo y con la palabra distintos casos contemplados por el ó rgano legislador.
Jude (judío). En los campos de concentració n, los presos
b) Respuesta libre.
eran clasificados tambié n mediante una simbología que
se puede consultar en el siguiente enlace: wikipedia. 2. Imagínate que te conviertes en un personaje de una serie
org/wiki/Sistema_de_marcado_en_los_campos_de_ televisiva que relata los problemas de una comunidad
concentraci%C3%B3n_nazis de vecinos.
d) La hostilidad hacia los judíos en la península ibérica alcan- De pronto, pasas a ser el presidente de la finca y tienes
zó , durante la Edad Media, periodos de auténtico terror. que resolver sendos problemas urgentes: las goteras del
A grandes rasgos, deben señ alarse estos momentos.
ático y el cambio de las puertas de los ascensores. Como
• En la época visigoda, la conversió n del rey Recaredo al necesitas recaudar fondos, redacta una convocatoria
catolicismo condujo a que ya en el siglo VII, durante el
para citar a tus vecinos a una reunión inmediata.
reinado de Sisebuto, se endureciesen las leyes antijudías.
Recuerda acompañarla del orden del día
• En el siglo XV, el antisemitismo se cebó contra los judíos correspondiente. Para ello, quizá te resulte interesante
conversos. A raíz de las crisis econó micas y políticas que
buscar en Internet modelos de convocatorias.
tuvieron lugar entre 1449 y 1474, hubo revueltas contra
los conversos en varias ciudades, siendo la más cruenta Respuesta libre.
la de 1449 en Toledo. 3. En un partido de fútbol el árbitro no solo actúa como
• En las Cortes de Toledo de 1480, se obligó a los judíos juez, sino que, después de terminado el enfrentamiento,
a vivir en barrios separados o juderías. deja constancia de las principales incidencias en unas
• A través de la Inquisició n, los Reyes Cató licos decidieron actas. Estas sirven, por ejemplo, para que los comités de
expulsar a los judíos de Andalucía en 1483. Los disci- plina deportiva sancionen a los jugadores
mismos monarcas publicaron el decreto de expulsió n amonestados. Busca en Internet el acta arbitral del
de dicho pueblo a finales de abril de 1492, conquistada partido de vuelta de la Supercopa de España de 2014
ya Granada.
para tomarla como modelo, ejerce tú también como
e) Lo dicho en su diario por Ana puede considerarse como árbitro y registra lo sucedido en un partido en el que tú
una informació n verosímil y de rabiosa actualidad en el
mismo hayas participado o en uno que hayas visto.
momento de la escritura (signifíquese el uso de los verbos
en presente), que pueden interesar a un pú blico mayorita- Respuesta libre.
rio. Esas son las similitudes con una noticia periodística 4. Los anuncios publicitarios están plagados de eslóganes
que deberían servir de acicate para realizar la actividad que incorporan proposiciones subordinadas adverbia-
propuesta a continuació n. les. Con ellas se remarca el motivo, la finalidad y otras
… encuentra la clave circunstancias que justifican la conveniencia de adquirir
determinado producto. Así, podemos encontrarnos con
1.La Asamblea General de las Naciones Unidas proclamó el subordinadas del tipo «Porque yo lo valgo» (causal) o «Si
25 de noviembre de 1981 la resolución 36/55, con la bebes, no conduzcas» (condicional). Sirviéndote de Inter-
Decla- ración sobre la eliminación de todas las formas de net, elabora un mural o dossier donde aparezcan
intole- rancia y discriminación fundadas en la religión o eslóganes publicitarios que te permitan ejemplificar los
las convic- ciones. De ella entresacamos este artículo: distintos tipos de subordinación adverbial.
Artículo 2
Respuesta libre.
1. Nadie será objeto de discriminació n por motivos de reli-
gió n o convicciones por parte de ningú n Estado, institu-
ció n, grupo de personas o particulares.
EN LA VARIEDAD ESTÁ EL 6
GUSTO
APERTURA DE UNIDAD ¿Para qué sirven? ¿Qué tipo de indicaciones da el autor a
través de ellas?
1. El texto que acabas de leer es el inicio de un entremés
muy divertido de los hermanos Álvarez Quintero. Busca
en Internet información sobre qué tipo de obra teatral es
un entremés y sobre los autores de este.
El entremés es una pieza có mica breve, y fue uno de los
géne- ros má s populares durante el Siglo de Oro.
Generalmente se intercalaba entre las distintas jornadas
(actos) de la repre- sentació n. Varios eran los tipos de
entremeses, destacando los que satirizan acerca de las
costumbres sociales y los en- tremeses cantados y bailados,
mucho má s creativos y fuera de convenciones.
Los hermanos Á lvarez Quintero (Serafín y Joaquín), se-
villanos, escribieron la mayor parte de su producció n li-
teraria a finales del siglo XIX y principios del XX. Fueron
ambos poetas, narradores, periodistas, pero, sobre todo
comedió grafos. Son exponentes principales de un teatro po-
pular, costumbrista, basado en el sainete, el gé nero chico
o la comedia.

2. En este fragmento aparecen dos recursos muy típicos del


teatro: la acotación y el monólogo. Señala qué partes co-
rresponden a cada uno de ellos y qué información nos
dan.
Las acotaciones son las partes inicial y final, que está n es-
critas en cursiva. La primera nos informa de dó nde se va a
desarrollar la escena y nos describe có mo es Martirio, su
protagonista. La ú ltima nos indica la entrada en escena de
su novio, Juliá n.
El monó logo es toda la intervenció n de Martirio, en la que esta
nos hace saber sus intenciones de reñ ir esa tarde con su novio.
3. ¿Te llama la atención la forma de expresarse de Martirio?
¿Por qué? ¿Podemos deducir de qué zona de España
es este personaje gracias a la variante geográfica del
caste- llano que utiliza?
Deducimos que Martirio es andaluza, debido a su forma ca-
racterística de hablar.
4. Ved ahora este entremés representado en teatro. Fíjate
en la comicidad de la escena y en la manera de
expresarse de ambos personajes y anota los rasgos que
te parezcan más llamativos. Para ver la escena, teclea en
YouTube «María Vidal y Paco Valladares» «Ganas de
reñir».
Respuesta libre.

VIVE LA LECTURA
Comprensión lectora
1. ¿Cómo reacciona la estanquera ante el intento de
atraco?
¿Se lo esperaban los atracadores?
Reacciona resistié ndose, dando voces y pidiendo auxilio. Los
atracadores no se esperaban esta reacció n.
2. ¿Qué rasgos de la personalidad de los personajes pode-
mos deducir de sus intervenciones?
Los atracadores, pese a su papel de «malos», tienen mucha
gracia y sentido del humor, sobre todo Tocho. Leandro es
algo má s violento y agresivo. La abuela es muy expresiva y
tiene mucho cará cter, no se deja amedrentar por los
atracadores ni mucho menos.
3. Fíjate en la información que nos dan las acotaciones.

Lengua castellana y Literatura. 3.º ESO. Solucionario 49


6 EN LA VARIEDAD ESTÁ EL GUSTO
Las acotaciones nos dan informació n sobre los Escé ni- cas y de la Mú sica) que depende del Ministerio de
movimientos de los personajes, lo que hacen y có mo Educació n, Cultura y Deporte. Creada por Adolfo Marsillach
reaccionan ante la situació n que se produce en el estanco. en 1986, es
4. La escena supuestamente es dramática, ya que se trata
de un atraco violento con un arma de fuego. Pero en
ella se trasluce cierta comicidad. ¿Dónde puedes
observarla?
La comicidad de la escena se trasluce sobre todo en el len-
guaje, con expresiones como se lanzan al lío, en un tris, un
pistolón de aquí te espero, esto es un atraco, como en el
cine, la mando al otro barrio, etc. Es especialmente patente
en la manera de hablar de los atracadores.
5. ¿Cuál de los dos atracadores crees que lleva las
riendas de la situación, Tocho o Leandro? ¿Por qué?
En cierto modo Leandro, porque aunque sea Tocho el que
ini- cia el atraco, despué s Leandro le da ó rdenes, y su tono
es má s cortante y agresivo.

Vocabulario
6. El texto está lleno de expresiones coloquiales, propias
de un registro informal de la lengua. Localízalas y
escríbelas en tu cuaderno. ¿Son apropiadas para un
contexto comu- nicativo formal?
Se lanzan al lío, en un tris, un pistolón de aquí te espero, la
pasta o la mando al otro barrio, pa la compra, unos gritos
que pa qué, se arranca por peteneras, a duras penas, la
cosa se pone negra, le saco las tripas al aire a ventilarse,
será animal, podemos echar un tute si cuadra…
Estas expresiones no serían adecuadas para un contexto co-
municativo formal.
7. Dichas expresiones coloquiales nos dan una pista sobre
el nivel social y cultural de los atracadores. ¿A qué clase
social crees que pertenecen?
Los atracadores parecen ser de un nivel sociocultural bajo.
8. Aparecen también algunas expresiones propias de la
jerga juvenil, es decir, la manera de hablar propia de los
jóvenes, como por ejemplo, decir la pasta para referirse
al dinero. Escribe en tu cuaderno otras expresiones que
consideres propias del lenguaje que usáis los jóvenes.
Respuesta libre.
9. ¿Encuentras en el texto algún vulgarismo o alguna inco-
rrección? Señala dónde y di cuál sería la forma
correcta.
Pa en lugar de para, la pego un tiro en lugar de le pego un
tiro.
10. En la primera acotación del fragmento se dice que
Tocho
«se busca los duros» para, supuestamente, pagar el
paque- te de tabaco. ¿Recuerdas tú qué era un duro?
¿Sabrías decir a cuánto equivale en céntimos de euro?
Un duro equivaldría a 0,03 céntimos de euro.

Investigación y redacción
11. En la biografía de José Luis Alonso de Santos, el autor
de esta obra, se señalaba que fue durante unos años
director de la Compañía Nacional de Teatro Clásico.
Busca infor- mación en Internet sobre dicha compañía,
por ejemplo, procura averiguar cuáles han sido sus
mayores éxitos o quién la dirige en la actualidad.
La Compañ ía Nacional de Teatro Clá sico es una unidad de
producció n del INAEM (Instituto Nacional de la Artes

50 Lengua castellana y Literatura. 3.º ESO. Solucionario


la institució n de referencia en la recuperació n, preservació n,
e) Variedad geográ fica, se expresan de acuerdo a la zona de
producció n y difusió n del patrimonio teatral anterior al si-
la que proceden.
glo XX, con especial atenció n al Siglo de Oro y a la prosodia
del verso clá sico. f) Variedad social (pertenencia a un grupo) y también
de situació n (ya que se trata de una situació n formal y
En sus casi treinta añ os de historia la Compañ ía ha llevado a acadé mica).
cabo má s de ochenta y cinco producciones que han viajado
no solo por todo el estado españ ol, sino tambié n por muchos g) Variedad de situació n (ya hemos visto como ese mismo pro-
lu- gares de Europa y Amé rica, afianzando con su presencia fesor se expresa de forma diferente en otras situaciones).
en ellos la voluntad de difusió n de nuestro patrimonio 2. Escribe una conversación en la cual dos jóvenes empleen
dramá tico. una variedad social propia de su edad (lenguaje juvenil)
La directora de escena Helena Pimenta dirige la Compañ ía y, a continuación, la misma conversación entre dos
desde septiembre de 2011. adultos empleando la lengua estándar. Cada una de ellas
12. Inventa y escribe una pequeña escena teatral que conti- deberá constar, como mínimo, de sesenta palabras.
núe la que acabas de leer. Debes conservar en escena a Respuesta libre.
Tocho, Leandro, la Abuela y Ángeles. Intenta mantener 3. ¿Alguien de tu entorno cercano emplea alguna variedad
el tono y la manera de hablar de los personajes. Deberá geográfica o social de la lengua? Si es así, explica cuál es
constar, al menos, de cien palabras. dicha variedad y cuáles son sus peculiaridades más des-
Respuesta libre. tacadas.
13. Busca información en la red acerca de la adaptación al Respuesta libre.
cine de la obra teatral La estanquera de Vallecas (direc- 4. Lee los siguientes textos, identifica el dialecto del que se
ción, producción, actores principales…). A continuación trata y señala sus rasgos más característicos:
ved la versión cinematográfica del fragmento que habéis a) Y xunt a ello taba una mociquina tamién muy guapa; vistía
leído. Teclead en YouTube «La estanquera de Vallecas de blancu, quey dixo: «¿Qué te gusta má s de tou?». El mozu
1987». Luego, comentad si os imaginabais así la escena. quedó se clisá u n’ello, yen vez ce icíi, «gú stesme tú »,
La estanquera de Vallecas es un película de 1987 dirigida por dixoy:
Eloy de la Iglesia, basada en la obra teatral homó nima de «anquel anfiler dioru».
José Luis Alonso de Santos (1981). Fue producida por Ega b) L’ochetibo de o premio Pirene ye promober un millor
Medios Audiovisuales. conoximiento de a reyalidá pirinenca en toz os suyos
En el reparto tenemos a Emma Penella como doñ a Justa (la aspeutos. Pueden optar a o premio Pirene os treballos
Abuela), a José Luis Gó mez como Leandro, a José Luis pe- riodisticos (de prensa, arradio u telebisió n) que seigan
Manza- no como Tocho y a Maribel Verdú como Á ngeles. es- tatos publicatos u esparditos entre l’1 de chulio de 2009
e o 30 de chunio de 2010 e que faigan referenzia á
cualsiquier aspeuto rilazionato con os Pirineyos.
ESTUDIO DE LA LENGUA
a) Se trata de un texto perteneciente al dialecto astur-leonés.
1. Explica razonadamente a qué tipo de variedades corres- Lo sabemos por los diminutivos en -ina (mociquina), el
pondería la lengua empleada en las siguientes cierre de vocales finales (blancu, tou, mozu, dioru), o la
colocació n de los pronombres (quedóse, gústesme).
situaciones:
b) Texto perteneciente al dialecto navarro-aragoné s, como
a) Dos presos hablando con expresiones propias de la podemos observar en la diptongació n de vocales
cárcel. (seigan, faigan), o el mantenimiento de la f- inicial latina
b) Un grupo de andaluces empleando el dialecto propio (faigan).
de su zona. 5. Investiga en la red cuáles fueron los primeros testimo-
c) Unos jóvenes en una fiesta expresándose de acuerdo nios escritos en lengua navarro-aragonesa, y de qué si-
con su edad. glo datan, y escribe la información más relevante en tu
cuaderno.
d) Un joven en una entrevista de trabajo utilizando un
Parece que los primeros testimonios escritos del navarro-ara-
registro formal.
gonés datan del siglo XI y proceden de las glosas emilianen-
e) Los actores de una telenovela venezolana. ses, anotaciones manuscritas en los má rgenes de un có dice
latino, compuestas en el monasterio riojano de San Millá n de
f) Un profesor dando una conferencia a un grupo de la Cogolla.
alumnos universitarios.
6. Vuelve a leer el fragmento del entremés con el que se
g) El mismo profesor charlando distendidamente con abre la unidad, Ganas de reñir, y comenta los rasgos dia-
sus amigos. lectales del andaluz que observes.
a) Variedad social, se expresan así porque pertenecen a un Aparecen rasgos propios del andaluz como el seseo (grasia,
determinado grupo (en este caso, marginal). cabeza, durse, resibí), la confusió n entre -r- y -l- (durse, sir-
b) Variedad geográ fica, porque depende de la zona en la ba), la pé rdida de -d- intervocá lica (pescao, to), pé rdida de
que se habla. consonantes finales (señó, reñí, tomá, é, durá, resibí, tené,
sorbé) o el yeísmo (ayí).
c) Variedad social, ya que se expresan de acuerdo a su edad
y a su grupo. 7. Copia este mapa de España en tu cuaderno. A continua-
d) Variedad de situació n, ya que depende de la situació n co- ción, marca en él, con diferentes colores, las zonas donde
municativa (ese mismo joven en otro contexto se expre- se hablan los distintos dialectos del castellano.
saría de forma diferente). No te olvides de incluir, además, una leyenda en la que
expliques qué color corresponde a cada dialecto.
Mapa con los dialectos del castellano:
con ellas, aprenderían a valorarlas y a la larga

Zona
deCastellano
influencia del Zona de
leonés influencia del aragonés

central

Extremeño

Murciano
Dialectos septentrionales del español
Andaluz Variables del español habladas en los territorios del gallego, vasco y catalán

Canario

SeseoDialectos meridionales
Ceceodel español

desaparecerían ciertas actitudes heredadas».


8. Define en tu cuaderno los siguientes conceptos: lengua Respuesta libre.
estándar, diversidad lingüística, variedades de la lengua, 11. Investiga en Internet cuáles son las principales
variedad geográfica o dialecto. lenguas in- dígenas que se hablan en América. Resume la
Lengua está ndar: es la lengua comú n que todos los hablantes información que consideres más importante en tu
de una lengua deben conocer para entenderse entre sí. Es la cuaderno.
lengua del á mbito académico y de los medios de comunica-
Las lenguas indígenas de Amé rica son aquellas habladas en
ció n, y debe respetar una serie de reglas y normas.
el continente americano desde los primeros asentamientos
Diversidad lingü ística: son las diferencias en las maneras de humanos hasta la llegada de los colonizadores europeos, afri-
expresarse de los hablantes de una misma lengua. canos y asiá ticos.
Variedades de la lengua: son aquellas diferencias Aunque muchas de ellas se han extinguido, aú n se siguen ha-
ocasionadas por factores geográ ficos, sociales o de situació n, blando má s de mil lenguas indígenas, pertenecientes a diver-
y que origi- nan la diversidad lingü ística. sas familias.
Variedad geográ fica o dialecto: es la que depende de la zona Las principales lenguas indígenas son:
en la que se hable una determinada lengua. • Quechua: de 9 a 14 millones de hablantes.
9. Busca en Internet información sobre los guanches, los • Guaraní: de 7 a 12 millones. Aunque algunos apuntan una
pri- mitivos habitantes de las islas Canarias. Copia dicha cifra de entre 15 y 22 millones de hablantes.
infor- mación en tu cuaderno y resúmesela a tus • Aimara: de 2 a 3 millones.
compañeros de forma oral. • Nahua: de 1,3 a 5 millones.
Los guanches eran los antiguos aborígenes que habitaban • Maya: de 900.000 a 1,2 millones. Las lenguas de la familia
en la isla de Tenerife, antes de la conquista de las islas por maya son habladas por entre 2 y 7 millones de personas.
la Corona de Castilla en 1496. Por extensió n este té rmino se
ha aplicado tambié n a los aborígenes del resto de las islas 12. Busca en el Diccionario de la lengua española (lema.rae.
Canarias. Poblaron las islas durante casi dos mil añ os hasta es) el significado de los siguientes términos que provie-
que fueron sometidos por las tropas castellanas. Se dedicaban nen de lenguas indígenas, y anótalo en tu cuaderno junto
fundamentalmente al pastoreo y a una agricultura bá sica de a la lengua de la que proceden:
cereales. choclo cacao chocolate
10. Lee este fragmento de un texto publicado en la canoa alpaca vicuña
sección de cultura del ABC, que expone la opinión
cóndor tomate puma
acerca de la diversidad lingüística de una profesora del
Instituto de Lengua, Literatura y Antropología del CSIC, guacamayo coyote cigarro
Pilar García Mouton. Después argumenta si estás o no • choclo (del quechua): mazorca tierna de maíz.
de acuerdo con sus afirmaciones. • canoa (del taíno): embarcació n de remo muy estrecha, nor-
García Mouton considera «una pena que se haya politizado malmente de una pieza y sin diferencia de forma entre proa
tanto todo lo relacionado con el legado lingü ístico, que de- y popa.
bería considerarse patrimonio de todos, una riqueza cultu- • cóndor (del quechua): ave rapaz de poco más de un
ral». En su opinió n, convendría seguir la recomendació n de metro de longitud y tres de envergadura, y plumaje fuerte
la Unesco de escolarizar a los niñ os en su lengua materna de color negro azulado. Habita en los Andes y es la mayor
y luego ir añ adiendo otras a sus conocimientos. «Y tambié n de las aves que vuelan.
convendría que a los niñ os españ oles se les enseñ ase desde
• guacamayo (del taíno): ave de América, especie de papa-
pequeñ os que, ademá s de la suya, hay otras lenguas en su
gayo, del tamañ o de una gallina y fuertes colores.
entorno, y que aprendiesen rudimentos de esas otras lenguas
a través de canciones, adivinanzas… Así se familiarizarían • cacao (del nahua): á rbol de América cuyo fruto brota di-
rectamente del tronco y las ramas y que se emplea como
principal ingrediente del chocolate.
• alpaca (del aimara): mamífero rumiante, de la misma fa-
milia que la llama, propia de amé rica meridional y muy
apreciado por su pelo, que se emplea en la industria
textil.
• tomate (del nahua): fruto de la tomatera, que es una
baya casi roja, de superficie lisa y brillante, en cuya pulpa
hay numerosas semillas, algo aplastadas y amarillas.
• coyote (del nahua): especie de lobo que se cría en México
y otros países de Amé rica, de color gris amarillento y del
tamañ o de un perro mastín.
• chocolate (del nahua): pasta hecha con cacao y azú car mo-
lidos, a los que generalmente se añ ade canela o vainilla.
• vicuña (del quechua): mamífero rumiante del tamañ o
del macho cabrío, pero con cuello má s largo y erguido,
cabeza má s redonda y sin cuernos, orejas puntiagudas y
derechas y piernas muy largas. Vive salvaje en manadas en
los Andes del Perú y de Bolivia, y se caza para aprovechar
su velló n, que es muy apreciado.
• puma (del quechua): felino americano de unos 180 cm de
• mazo: mucho. (Jerga juvenil).
longitud, de color rojizo o leonado uniforme, que vive en
serranías y llanuras. • traqueotomía: abertura que se hace artificialmente en la
trá quea para impedir en ciertos casos la sofocació n de los
• cigarro (del maya): rollo de hojas de tabaco, que se enfermos. (Jerga profesional, de la medicina).
enciende por un extremo y se fuma por el opuesto.
• trullo: cá rcel, presidio. (Jerga marginal).
13. Corrige los vulgarismos que encuentres en las siguientes
oraciones. ¿A qué nivel de la lengua pertenecerían? ¿Qué 15. Señala cuáles de estos enunciados te parecen propios
información nos dan acerca del grado de formación de de un registro formal y cuáles de uno informal, argumen-
los hablantes que los emplean? tando por qué:
a) No se entera de la misa la media, está empanado.
Corrige b) Lamentamos profundamente el malentendido
a) Amos, que no se ha enterao de na de lo que le he origina- do por la falta de comunicación.
contao. c) ¿Me haría usted el favor de alcanzarme aquel libro de
b) Mi agüela hace unas almóndigas pa chuparse los la estantería?
dedos. d) Te he dicho mil veces que no seas tan plasta.
c) Ayer andamos ventidós kilómetros y estoy muy e) A causa de una mala gestión de los fondos de la
cansao. em- presa, se han visto obligados a realizar
d) No me dijistes que la Marta y el Rubén se reajustes de plantilla.
habían arrejuntao. f) No tiene un pelo de tonta, está siempre a la que salta.
e) Me se ha estropeado el CD que me regalastes. g) Voy a darme un garbeo por ahí, estoy hasta el gorro
f) Joder, si te gusta tanto el vestido, cómpratele. de estar metido en casa.
g) Se me ha infestado la herida y mi madre me h) Le rogaría que nos confirmase la recepción de la do-
ha echado agua oxigenada. cumentación en un plazo razonable.
a) Registro informal, ya que emplea té rminos coloquiales y
a) Vamos, que no se ha enterado de nada de lo que le he expresiones hechas.
contado.
b) Registro formal. El lé xico es más cuidado, la sintaxis más
b) Mi abuela hace unas albó ndigas para chuparse los dedos. elaborada.
c) Ayer anduvimos veintidó s kiló metros y estoy muy c) Registro formal, ya que emplea un lé xico cuidado, y
cansado. utiliza el usted.
d) No me dijiste que Marta y Rubén se habían ido a vivir d) Registro informal, utiliza expresiones coloquiales como
juntos. plasta.
e) Se me ha estropeado el CD que me regalaste. e) Registro formal, con un léxico cuidado, una sintaxis
elabo- rada, uso de eufemismos (reajustes de plantilla
f) Si te gusta tanto el vestido, có mpratelo.
en lugar de despidos).
g) Se me ha infectado la herida y mi madre me ha echado
f) Registro informal, ya que emplea frases hechas y coloquia-
agua oxigenada.
les (un pelo de tonta, a la que salta).
Todas estas incorrecciones pertenecen al nivel vulgar de la g) Registro informal. Utiliza expresiones coloquiales (gar-
lengua y nos indican un nivel sociocultural bajo. beo, hasta el gorro).
14. Busca el significado de las siguientes palabras en el h) Registro formal. Emplea un léxico cuidado, una sintaxis
diccio- nario e indica si pertenecen a una jerga elaborada y hace uso de la forma de cortesía usted.
profesional, juvenil o marginal: molar (verbo), diurético, 16. Escribe un diálogo entre dos amigos que emplean un re-
faca, pasma, cogorza, sumario, colonoscopia, mazo, gistro coloquial o informal. A continuación, escribe ese
traqueotomía, trullo. mismo diálogo entre dos interlocutores que no tienen
• molar: gustar, resultar agradable o estupendo. (Jerga confianza entre ellos, por lo que emplean un registro
juvenil). mucho más formal. Cada uno deberá constar, al menos,
• diurético: que tiene virtud para aumentar la excreció n de de sesenta palabras.
orina. (Jerga profesional, de la medicina). Respuesta libre.
• faca: cuchillo corvo. (Jerga marginal).
EL TALLER DE LAS PALABRAS
• pasma: policía. (Jerga marginal y por extensió n, jerga
juvenil). 1.Escribe el significado denotativo de las siguientes
• cogorza: borrachera. (Jerga juvenil). palabras: instituto, nieve, agosto, piscina, pueblo y
ordenador. A con- tinuación, anota en tu cuaderno qué
• sumario: conjunto de actuaciones encaminadas a preparar
el juicio criminal, haciendo constar la perpetració n de los significados connota- tivos tienen para ti esas mismas
delitos con las circunstancias que puedan influir en su cali- palabras.
ficació n, determinar la culpabilidad y prevenir el castigo de Respuesta libre.
los delincuentes. (Jerga profesional, del campo del 2. Compara con un compañero las connotaciones que tie-
derecho y la justicia). nen para vosotros los términos de la actividad 1. ¿Podéis
• colonoscopia: exploració n del interior del colon mediante explicar las discrepancias?
un colonoscopio. (Jerga profesional, de la medicina).
Respuesta libre.
3. En algunas culturas y religiones, las palabras tienen un • dar en el clavo: acertar. Has dado en el clavo con la solu-
determinado significado connotativo. Busca información ción a ese problema.
sobre cuál es el significado connotativo de vaca para los • estar a dos velas: estar sin dinero. A finales de mes estoy
hindúes, de cerdo para los musulmanes y de cordero siempre a dos velas, no puedo permitirme salir a cenar.
para los cristianos.
• quedarse en blanco: no saber qué decir, no acordarse de
Para los hindú es las vacas son animales sagrados, que deben lo se iba a decir. Me quedé en blanco en el examen oral de
ser venerados y respetados. italiano.
Para los musulmanes el cerdo es un animal impuro y por lo • ser coser y cantar: ser muy sencillo. Esta tarea es coser y
tanto no consumen su carne. cantar, no tiene ninguna complicación.
Para los cristianos el cordero simboliza a Jesucristo (el Cor- • hincar los codos: estudiar. Estuvo hincando los codos toda
dero de Dios). la noche para el examen de álgebra.
4. Piensa en alguna palabra que esté muy marcada conno- • empinar el codo: beber alcohol. Aquel hombre ha empinado
tativamente en la sociedad en la que vives, y explica por demasiado el codo y va dando tumbos.
qué. • echar una mano: ayudar. fíchame una mano con las ma-
Respuesta libre. letas, no puedo subirlas solo.
5. Completa los refranes y explica su significado: • morder el polvo: ser derrotado. En el primer asalto el con-
trincante mordió el polvo.
a) A buenas horas…
• dar el brazo a torcer: ceder. No voy a dar mi brazo a torcer
b) Se coge antes a un mentiroso… en ese asunto, sé que tengo razón.
c) A caballo regalado… • vivir del cuento: vivir sin trabajar, valié ndose de la ayuda
d) A falta de pan… de otros. Juan no trabaja, vive del cuento.
• estar en las nubes: estar distraído. Mateo siempre está en
e) En boca cerrada…
las nubes, no se entera de nada.
f) Quien a buen árbol se arrima… • tener mala pata: tener mala suerte. ¡Qué mala pata tengo!
g) Aunque la mona se vista de seda… Se me ha estropeado el coche otra vez.
a) A buenas horas, mangas verdes. (Se usa cuando alguien 8. Escribe en tu cuaderno tres refranes, tres proverbios y
llega demasiado tarde). tres frases hechas. A continuación inventa situaciones en
b) Se coge antes a un mentiroso que a un cojo. (Si mientes, las que podría ser apropiada su utilización.
tarde o temprano te acaban descubriendo). Respuesta libre.
c) A caballo regalado, no le mires el diente. (Si te regalan 9. Os sorprenderá descubrir la cantidad de refranes en es-
algo, acé ptalo sin poner pegas).
pañol que versan sobre un mismo tema. Dividid la clase
d) A falta de pan, buenas son tortas. (Cuando no tenemos en dos grupos, la mitad de vosotros se encargará de
posibilidad de elegir, hay que conformarse con lo que se buscar en Internet refranes que tengan como
tiene).
protagonistas a los gatos; la otra mitad, refranes que
e) En boca cerrada no entran moscas. (A veces es mejor es- traten sobre perros. Una vez hecha la recopilación,
tar callado para no decir alguna imprudencia). ponedla en común ante el resto de compañeros. El
f) Quien a buen árbol se arrima, buena sombra le cobija. profesor se encargará de llevar la cuenta. ¿Qué grupo ha
(Si tienes buenas amistades, estas te ayudará n cuando lo conseguido reunir más refranes?
necesites).
Respuesta libre.
g) Aunque la mona se vista de seda, mona se queda. (Por
mucho que se intente disimular con ropas o maquillajes, EL RINCÓN DE LA NORMA
la fealdad no se puede esconder).
1. Justifica el uso de rayas en el texto de Laura Gallego.
6. Di si estás o no de acuerdo con los siguientes proverbios Las rayas en este texto sirven para marcar las intervenciones
y justifica tu respuesta: de los personajes, así como para incluir aclaraciones dentro
a) Quien bien te quiere te hará llorar. del diá logo.
b) La probabilidad de hacer el mal se encuentra cien ve- 2. Añade los paréntesis o las comas que consideres oportu-
ces al día; la de hacer el bien, una vez al año. nos en el siguiente texto:
c) El infierno está lleno de buenas intenciones y el Corrige
cielo de buenas obras.
Tristá n e Iseo es una leyenda del ciclo artú rico sobre el
d) La palabra es plata y el silencio es oro. rey Arturo que relata los amores entre Iseo apodada la ru-
e) Si lloras por no haber visto el sol, las lágrimas no te bia y el caballero Tristá n de Leonís. «La rubia» es el apodo
dejarán ver las estrellas. que el autor utiliza para distinguirla de su homó nima la de
Respuesta libre. las blancas manos. La trama de raigambre posiblemente
vikinga la constituye el triá ngulo amoroso formado por
7. Explica el significado de las siguientes frases hechas, y Tristá n y ambas damas.
después escribe en tu cuaderno una oración con cada
una de ellas: dar en el clavo, estar a dos velas, quedarse Tristá n e Iseo es una leyenda del ciclo artú rico (sobre el rey
en blanco, ser coser y cantar, hincar los codos, empinar el Arturo), que relata los amores entre Iseo (apodada la rubia) y el
caballero Tristá n de Leonís. La «rubia» es el apodo que el autor
codo, echar una mano, morder el polvo, dar el brazo a
utiliza para distinguirla de su homó nima (la de las blancas ma-
tor- cer, vivir del cuento, estar en las nubes, tener mala nos). La trama (de raigambre posiblemente vikinga) la constitu-
pata.
ye el triá ngulo amoroso formado por Tristá n y ambas
damas.
3. Escribe una breve autobiografía en la que uses paréntesis
b) ¿Quién es el emisor de cada texto? ¿Se puede señalar
para ofrecer detalles o aclaraciones.
en ellos la presencia de un narrador? Indica cuál es en
Respuesta libre. cada caso.
4. Escribe correctamente los siguientes compuestos (con
c) En cuanto a la intención comunicativa de ambos emi-
o sin guion): sores, ¿es la misma? En caso contrario, ¿en qué se
a) obsesivo compulsivo diferencian?
b) cabeza rapada d) Observa el código de ambos textos y explica las dife-
c) telefonía móvil rencias que encuentres.
d) físico químico a) Es literario el primero de ellos, ya que posee una finalidad
e) peruano ecuatoriano eminentemente estética, intenta crear belleza a través de
las palabras.
f) histórico artístico
b) En el primero de ellos el emisor es Chejov. Ademá s hay
g) carril bici un narrador omnisciente en tercera persona.
a) obsesivo-compulsivo En el segundo el emisor es el autor del texto (no sabemos
b) cabeza rapada su nombre, aunque sí sabemos que el coordinador de la
c) telefonía móvil obra es Luis Ribot). No hay narrador, ya que no se trata
de un texto literario.
d) físico-químico
e) peruano-ecuatoriano c) La intenció n comunicativa del primero es crear belleza,
deleitar con las palabras. En cambio, la del segundo es
f) histórico-artístico simplemente informativa y acadé mica, se trata de dar a
g) carril bici conocer unos datos.
5. Escribe mayúscula donde sea necesario: d) El có digo en el primer caso es algo má s elaborado, intenta
producir extrañ eza en el lector mediante el desvío de lo
Corrige habitual, se centra en la funció n estética. En el segundo,
en cambio, pese a ser un có digo culto, no se pretende lla-
stendhal (1783-1842) es uno de los iniciadores del realis- mar la atenció n sobre có mo está escrito el mensaje, sino
mo francés, aunque su obra convive cronoló gicamente simplemente sobre lo que dice.
con algunas obras romá nticas, como es el caso del
drama hernani, de victor hugo, que se estrenó en 1830. Ese 2. El canal mediante el cual se transmite una obra literaria
mismo añ o se publicó una de las grandes obras de influye en gran medida en ella. Investiga cuáles son las
stendhal, rojo y negro, cró nica social de la francia de la principales diferencias entre la literatura oral y la escrita,
restauració n borbó nica, en la que el aná lisis psicoló gico y prepara una presentación para exponer el tema a tus
de los perso- najes es uno de los rasgos má s compañeros.
interesantes. otro título importante de stendhal es la
cartuja de parma (1839). El alumno puede hacer menció n, por ejemplo, a que la litera-
tura oral tiene un cará cter mucho má s popular y tradicional.
Stendhal (1783-1842) es uno de los iniciadores del realismo Al irse transmitiendo de padres a hijos, está sujeta a un ma-
francé s, aunque su obra convive cronoló gicamente con algu- yor nú mero de cambios y variantes, y ademá s posee diversas
nas obras romá nticas, como es el caso del drama Hernani, de fó rmulas que facilitan su memorizació n. El género oral por
Victor Hugo, que se estrenó en 1830. Ese mismo añ o se publicó excelencia es el lírico, pero tambié n el narrativo (épica, ro-
una de las grandes obras de Stendhal, Rojo y negro, cró nica mances…). A menudo se trata de una literatura anó nima, ya
social de la Francia de la Restauració n borbó nica, en la que que pertenece al pueblo.
el aná lisis psicoló gico de los personajes es uno de los rasgos La literatura escrita tiene un cará cter más culto, no está su-
más interesantes. Otro título importante de Stendhal es La jeta a tantas variantes y suele estar firmada por un autor.
cartuja de Parma (1839). El gé nero más abundante en esta literatura es la narrativa
(novelas, cuentos…).
LA FACTORÍA DE TEXTOS 3. Haz una pequeña encuesta entre tus familiares y amigos
1. Lee con atención los siguientes textos y responde a las sobre el último libro que cada persona haya leído.
preguntas que tienes a continuación: Formu- la las siguientes preguntas:
Comieron. El estó mago sentía un pequeñ o bienestar, las bocas a) ¿Cuál es el último libro que has leído?
bostezaban, los ojos se angostaban por una dulce somnolen-
b) ¿Por qué lo has elegido?
cia. El marido se puso a fumar un puro, se desperezó y se
tumbó en el sofá . La esposa se sentó en la cabecera y empezó c) ¿Qué te ha aportado?
a tararear… Eran felices.
Investiga sobre los títulos que más te llamen la atención y
Anton CHEJOV: Siete cuentos (Cá tedra). averigua si son textos literarios o no.
Hasta los añ os centrales del siglo XV siguieron prolongá ndose Respuesta libre.
los efectos de la terrible crisis de mediados del XIV, perfec-
tamente identificada con la Peste Negra. Fueron cien añ os 4. Lee el siguiente poema y responde las cuestiones que
de repetidas crisis epidémicas, de desaparició n de nú cleos tienes a continuación:
rurales, de general ralentizació n de la actividad econó mica. En fin a vuestras manos he venido,
Luis RIBOT (Coord.): Historia del mundo moderno. do sé que he de morir tan apretado
a) ¿Cuál de los dos textos es literario? Justifica tu res- que aun aliviar con quejas mi cuidado
puesta. como remedio m’es ya defendido;
mi vida no sé en qué s’ha sostenido
de la fortaleza de Le Muy, en Provenza, Garcilaso fue tras-
si no es en haber sido yo guardado
ladado a Niza, donde murió .
para que solo en mí fuese probado
cuá nto corta una ’spada en un rendido. Su escasa obra conservada, escrita entre 1526 y 1535, fue
publicada pó stumamente junto con la de Boscá n, bajo el
Mis lá grimas han sido derramadas
título de Las obras de Boscán con algunas de Garcilaso
donde la sequedad y el aspereza
de la Vega (1543), libro que inauguró el Renacimiento
dieron mal fruto dellas, y mi suerte:
literario en las letras hispá nicas.
¡basten las que por vos tengo lloradas;
no os vengué is má s de mí con mi flaqueza; Siempre se ha creído que muchas de sus composiciones
allá os vengad, señ ora, con mi muerte! reflejan la pasió n de Garcilaso por la dama portuguesa
Isabel Freyre, a quien el poeta conoció en la corte en 1526.
Garcilaso DE LA VEGA Sin embargo, recientes estudios ponen en duda esta teoría
a) ¿Quién es el emisor del texto? ¿Y los receptores? y defienden que la amada secreta de Garcilaso no era Isa-
bel, sino Beatriz de Sá , la esposa de su hermano.
b) ¿A quién se dirige el poeta cuando emplea la forma
vos? ¿Piensas que se trata de una persona en concre- Los cuarenta sonetos y las tres é glogas que escribió se
mueven dentro del dilema entre la pasió n y la razó n que
to? ¿Quién?
caracteriza la poesía petrarquista. Escribió tambié n cinco
c) ¿Cuál es la intención comunicativa de Garcilaso al es- canciones, dos elegías, una elegía a Boscá n y tres odas
cribir este poema? ¿Y la del yo poético que se expresa latinas, inspiradas en la poesía horaciana y virgiliana.
en él? ¿Coinciden ambas? f) El Renacimiento fue un movimiento cultural que se produ-
d) Fíjate bien en el lenguaje con el que está escrito el jo en Europa durante los siglos XV y XVI. Fue un período de
poema, es decir, su código. ¿Qué te llama la atención transició n entre la Edad Media y el mundo moderno. Sus
sobre él? Señala algún recurso poético empleado por principales exponentes se hallan en el campo de las artes,
Garcilaso y explica su finalidad. aunque también se produjo una renovació n en las ciencias.
La ciudad de Florencia, en Italia, fue el lugar de
e) Investiga sobre la biografía de Garcilaso de la Vega y nacimiento y desarrollo de este movimiento.
sobre cómo pudo influir en su obra poética. El Renacimiento fue fruto de la difusió n de las ideas del
f) Busca información sobre la época en la que vivió Gar- humanismo, que determinaron una nueva concepció n
cilaso, el Renacimiento, y explica cuáles son las carac- del hombre y del mundo. El nombre «Renacimiento» se uti-
terísticas de este movimiento y cómo nos ayudan a lizó porque este movimiento retomaba ciertos elementos
entender el poema. de la cultura clá sica, griega y romana. Esta nueva etapa
planteó una nueva forma de ver el mundo y al ser humano,
g) ¿De qué tipo de composición poética se trata? Explica con nuevos enfoques en los campos de las artes, la política,
cómo está formada. la filosofía y las ciencias, sustituyendo el teocentrismo
a) El emisor del texto es Garcilaso de la Vega, y los recep- medieval por cierto antropocentrismo.
tores son, en primer lugar, la mujer a la que se dirige, su Esta informació n nos ayuda a comprender mejor el poema
amada; y después el resto del lectores del poema. ya que observamos como ahora la temá tica es el hombre
b) Con ese vos se está dirigiendo a la mujer que ama y que y sus sentimientos (frente a la Edad Media donde todo
tanto le está haciendo sufrir. giraba en torno a Dios y a la religió n).
c) La intenció n de Garcilaso es sobre todo esté tica: quiere g) Se trata de un soneto. Está formado por catorce versos
crear belleza con sus palabras. La intenció n del yo poé ti- endecasílabos, con rima consonante. Consta de dos cuar-
co es expresar sus sentimientos amorosos, concretamente tetos (rima ABBA) y dos tercetos (CDE, CDE).
quejarse a su amada del sufrimiento que le ha
ocasionado y que le está llevando a la muerte. Como ACTIVIDADES FINALES
siempre se ha con- siderado que la poesía de Garcilaso
tiene una gran carga autobiográ fica, esta intenció n Repasa lo que has aprendido
coincidiría tanto en su autor como en el yo poético. 1. Lee atentamente el siguiente diálogo y responde a las
d) Se trata de un có digo muy cuidado y elaborado, en el que cuestiones que se te plantean a continuación:
predomina la funció n estética del lenguaje. Llama la aten- —Creo que tenemos pendiente una conversació n, ¿no lo crees?
ció n el uso del lenguaje figurado, con recursos como las —Sí, sí lo creo. Y creo que fue desde que te casaste con mi
hipérboles (sé que he de morir, con mi muerte) o metá - novio.
foras (cuánto corta una ’spada en un rendido). Lo que
pretende el poeta es llamar la atenció n sobre el lenguaje y —Está bien, si lo quieres, empecemos por ahí. Tú tuviste un
sorprender al lector. novio indebidamente. No te correspondía tenerlo.
—¿Segú n quié n? ¿Segú n mamá o segú n tú ?
e) Garcilaso de la Vega, (Toledo, 1501?-Niza, 1536) fue un
poeta renacentista españ ol. Perteneciente a una noble —Segú n la tradició n de la familia, que tú rompiste.
familia castellana, participó ya desde muy joven en las —Y que voy a romper cuantas veces sea necesario, mien-
intrigas políticas de Castilla. En 1510 ingresó en la corte tras esa maldita tradició n no me tome en cuenta. Yo tenía el
del emperador Carlos I y tomó parte en numerosas batallas mismo derecho a casarme que tú , y tú eras la que no tenía
militares y políticas. En 1530 Garcilaso se desplazó con derecho a meterse en medio de dos personas que se querían
Carlos I a Bolonia, donde este fue coronado. Permaneció profundamente.
allí un añ o, hasta que, debido a una cuestió n personal man- —Pues ni tan profundamente. Ya ves có mo Pedro te cambió
tenida en secreto, fue desterrado a la isla de Schut, en el por mí a la menor oportunidad. Yo me casé con él, porque é l
Danubio, y después a Ná poles, donde residió a partir de así lo quiso. Y si tuvieras tantito orgullo lo deberías de haber
entonces. Herido de muerte en combate, durante el asalto olvidado ya para siempre.
—Pues para tu informació n, se casó contigo solo por estar granizado de limón.
cerca de mí. No te quería y tú lo sabías muy bien.
—Mira, mejor ya no hablemos del pasado, a mí no me impor-
tan los motivos por los que Pedro se casó conmigo. Se casó y
punto. Y yo no voy a permitir que ustedes dos se burlen de mí,
¡ó yelo bien! No estoy dispuesta a hacerlo.
Laura ESQUIVEL: Como agua para chocolate.
a) ¿Consideras que el registro empleado por ambos per-
sonajes es formal o informal? Busca ejemplos en el
tex- to para argumentar tu respuesta.
b) ¿Observas en el diálogo algún rasgo que te dé una
pista sobre la procedencia de los personajes?
¿Cuál?
c) Continúa tú la conversación entre ambas hermanas.
De- bes mantener el mismo tono y registro. Escribe
unas setenta palabras.
a) Se trata de un registro informal o familiar, ya que emplea
expresiones bastante coloquiales (esa maldita tradición,
pues ni tan profundamente, tantito orgullo, mira, se casó
y punto) e hipocorísticos como mamá.
b) Expresiones como tantito orgullo o ustedes dos, nos llevan
a pensar que se trata de hablantes hispanoamericanos.
c) Respuesta libre.
2. Di si las siguientes afirmaciones son verdaderas o falsas
y, en este último caso, escribe qué sería lo correcto:
a) El astur-leonés y el navarro-aragonés son dos dialec-
tos procedentes del castellano medieval.
b) Los dialectos meridionales de la Península son el an-
daluz, el murciano, el extremeño y el canario.
c) Algunos de los rasgos más característicos del dialecto
andaluz son el seseo, el ceceo, el yeísmo, la pérdida
de
-d- intervocálica, la aspiración de -s final y la confusión
entre r y l.
d) Las variedades sociales se denominan también varie-
dades diafásicas.
e) Las variedades de situación son las que emplea un
mis- mo hablante dependiendo del contexto
comunicativo en el que se encuentre.
f) Las frases hechas y los refranes son propios de un
registro informal de la lengua.
g) Una jerga es la forma de hablar característica de un
grupo marginal.
a) Falso. En realidad proceden del latín medieval.
b) Verdadero.
c) Verdadero.
d) Falso. Se denominan variedades diastrá ticas.
e) Verdadero.
f) Verdadero.
g) Falso, tambié n puede ser la de un grupo profesional, de
jó venes…
3. Explica si los siguientes enunciados corresponden al sig-
nificado denotativo o connotativo de la palabra definida
y razona tu respuesta:
a) El verano: ese tiempo de relax, de levantarse tarde,
de no saber muy bien en qué emplear el día, de
intermi- nables siestas y de noches al fresco con un
b) El verano: época más calurosa del año, que en el estribillo inventado por é l, que hasta los mismos cuervos le
hemis- ferio septentrional comprende los meses de desaprobaban.
junio, julio y agosto. En el hemisferio austral Wenceslao FERNÁ NDEZ FLÓ REZ: El bosque animado.
corresponde a los meses de diciembre, enero y
febrero.
a) Se trata del significado connotativo, ya que no se trata
de algo objetivo y comú n a todos los hablantes, sino de
un significado muy personal para una persona en
concreto, que no tiene por qué ser compartido por
otras.
b) Se trata de un significado denotativo, es el que recoge el
diccionario y es comú n para todos los hablantes.
4. ¿Cuántos refranes conoces que hagan alusión a los me-
ses del año? Escribe en tu cuaderno todos los que
sepas. A continuación, pregunta a algún familiar si
conoce alguno más, y cópialo también. Por último,
busca otros en la red y escríbelos. ¿Cuántos has logrado
reunir en total? ¿Has conseguido al menos uno para
cada mes del año?
Respuesta libre.
5. Escribe en tu cuaderno cinco palabras compuestas
donde sea necesario el uso de guion.
Ejemplos: astur-leonés, navarro-aragonés, teórico-
práctico, franco-alemán, histórico-crítico, lingüístico-
literario…
6. Explica el uso que se hace de la raya en el diálogo de la
acti- vidad 1.
La raya aquí se emplea para introducir las intervenciones
de los dos personajes que está n dialogando.
7. Recuerda cuáles son los principales usos de las
mayúsculas, escríbelos en tu cuaderno y añade un
ejemplo para cada uno de ellos.
Según la puntuación: al comienzo de un escrito, detrá s de
punto, tras puntos suspensivos, exclamaciones e interrogacio-
nes que cierren un enunciado, tras dos puntos en el encabeza-
miento de una carta o comienzo de una cita textual.
Sin tener en cuenta la puntuación: los nombres propios de
persona, lugar, animal o divinidad; los apellidos y sobrenom-
bres; los nombres de calles y vías pú blicas; los nombres de
entidades o instituciones; la letra inicial de cualquier obra
de creació n artística; con los periodos histó ricos.
8. Explica en qué consiste un texto literario y cuáles son
sus elementos desde el punto de vista de la
comunicación. Ejemplifícalo con una obra literaria que
conozcas.
Un texto literario es el que pretende crear belleza a
través de las palabras, es decir, tiene una finalidad estética.
Sus elementos desde el punto de vista comunicativo son el
emi- sor (el autor), el receptor (los lectores), el mensaje (la
propia obra), el canal (oral o escrito), el có digo (lenguaje
con el que está escrito) y la situació n (é poca, contexto
en el que se escribió ).
9. Lee el siguiente texto y a continuación responde a las
cuestiones que se te plantean:
Los goterones que caían de las altas hojas hacían un ruido
como de pisadas. Todo el bosque parecía estar lleno de
gente en marcha. Pero el Hombre aú n tardó en llegar. Era
menos alto que una aguijada y su traje roto y manchado de
barro mos- traba por má s de un sitio la tierna carne
infantil; agujereada en el centro, la boina dejaba asomar los
cabellos; al tropezar con los guijarros, las zuecas sonoras
rimaban su caminata, y silbaba tan hó rridamente un
a) ¿Crees que se trata de un texto literario? Justifica tu jardín del sol, piedra… El agujero que tenía en medio,
respuesta con ejemplos. se calentaba mucho al sol, estaba muy lisa (refiriéndose
b) Observa el lenguaje con el que está escrito. ¿Te llama a la piedra).
la atención? ¿Por qué? c) Encontramos conectores aditivos o copulativos (y, tam-
c) Busca en el texto ejemplos de personificación. bién, lo mismo que), causales (de tanto como, porque),
¿Qué efecto consigue con este recurso el autor? temporales (hasta que, cuando), consecutivos (por eso)
y locativos (alrededor de).
a) Sí, se trata de un texto literario, ya que hace un uso espe-
cial del lenguaje (pretende provocar extrañ eza y llamar 11. Escribe en tu cuaderno una página de un diario
la atenció n sobre el mismo). En él predomina la funció n personal (no tiene por qué ser real, puedes contar
estética (ruido como de pisadas, las zuecas sonoras hechos ficticios), intentando que sea lo más literaria
rimaban su caminata, hasta los mismos cuervos le posible, es decir, que lo que escribas pueda producir un
desaprobaban…). placer estético en quien lo lea.
b) Sí, ya que difiere del lenguaje ordinario. Se emplean com- Respuesta libre.
paraciones (ruido como de pisadas, menos alto que una
aguijada) y personificaciones, que contribuyen a ese ex- MIRA A TU ALREDEDOR Y…
trañ amiento del lector ante el lenguaje.
c) Las personificaciones (su traje mostraba…, la boina
… ve más allá
de- jaba asomar…, las zuecas sonoras rimaban su 1.Para entender la evolución del castellano y de los
camina- ta, hasta los mismos cuervos le desaprobaban) dialectos peninsulares es fundamental conocer los
también consiguen dicho efecto de extrañ amiento, de acontecimientos históricos que tuvieron lugar en el
salirse de lo habitual. pasado, especialmente durante la Reconquista. En los
siguientes mapas puedes observar cómo cambió
Recuerda lo que ya sabías políticamente la Península durante la Reconquista. Fíjate
10. Lee el siguiente texto y resuelve las cuestiones plantea- en el retroceso de al-Ándalus y en el avance de los
das a continuación: reinos cristianos hacia el sur. ¿Qué conse- cuencias crees
El jardín del sol tenía un almendro, junto a la valla, que echaba que tuvo para las lenguas romances de la Península la
sus ramas hacia la calle. Los veranos se ponía una cigarra en invasión árabe?
la corteza de este almendro y cantaba durante toda la siesta.
Respuesta libre.
El aire se aplastaba sobre aquel canto y nadie podía ya
mover- se hasta que la cigarra no se callaba, de tanto como 2. Busca en Internet información sobre los siguientes acon-
pesaban todas las cosas. Era la cigarra de los bochornos tecimientos y prepara una exposición oral para explicarla
plomizos, cuando se envenenan las sandías. en clase:
Tambié n había en el jardín del sol una rueda de molino, a) La invasión musulmana en el año 711 d. C.
incli- nada y hundida en la tierra. Alrededor de esta piedra
nacía la hierba más fuerte, lo mismo que por el agujero que
b) Los primeros núcleos de resistencia en el norte de la
tenía en medio. Se calentaba mucho al sol y por eso gustaba Península.
a las lagartijas. En aquella piedra afilaba el maestro los c) Cómo se organizó la Reconquista hacia el sur.
cuchillos y las herramientas, porque estaba muy lisa. a) En el añ o 711, las tropas á rabes y bereberes del noroeste
Rafael SÁ NCHEZ FERLOSIO: Industrias de Á frica cruzaron el estrecho de Gibraltar, derrotaron
y andanzas de al ejé rcito visigodo en la batalla de Guadalete y, en poco
Alfanhuí. tiempo, se hicieron con el control de casi toda la penín-
a) Observa las estructuras sintácticas que forman el tex- sula ibé rica. El rá pido avance de las tropas musulmanas
fue posible por la debilidad del Reino visigodo y la escasa
to. Busca en él ejemplos de oraciones coordinadas
resistencia de su ejército, aunque tambié n contó con la
copulativas, oraciones subordinadas adjetivas, y ora-
ayuda de las poblaciones descontentas con dos siglos de
ciones subordinadas adverbiales de tiempo y causa. autoridad visigoda.
b) Fíjate ahora en la cohesión del texto. Señala todos b) En el norte de la Península siguen subsistiendo pueblos
los elementos de deixis que encuentres. que opondrá n resistencia al nuevo invasor. Estos pequeñ os
c) Observa los conectores. Escríbelos en tu cuaderno nú cleos resultará n cada vez má s fortalecidos, ganando
indicando de qué tipo son, y qué clase de relaciones te- rreno a costa de la Españ a musulmana.
establecen entre las partes del texto. c) La situació n cambió a partir del siglo XI, cuando los reinos
a) Coordinadas copulativas: los veranos se ponía una ciga- cristianos comenzaron a ganar terreno, en un largo proce-
rra en la corteza de este almendro y cantaba durante toda so conocido como Reconquista. Durante los cinco largos
la siesta; el aire se aplastaba sobre aquel canto y nadie siglos que duró este periodo, se alteraron é pocas de
podía ya moverse; se calentaba mucho al sol y por eso lucha y paz, de avance y retroceso.
gustaba a las lagartijas. 3. Lee el siguiente texto y responde a las preguntas que se
Subordinadas adjetivas: que echaba sus ramas hacia la te plantean a continuación:
calle; inclinada y hundida en la tierra. La invasió n musulmana del añ o 711 trastornó violentamente
Subordinadas adverbiales de tiempo: hasta que la cigarra el desarrollo lingü ístico de los romances hispá nicos. La Penín-
no se callaba; cuando se envenenan las sandías. sula quedó inundada por la ú ltima gran oleada expansiva del
Subordinadas adverbiales de causa: porque estaba muy islam. El latín se vio agobiado bajo el peso de otra lengua de
lisa. cultura, el á rabe, ó rgano de una civilizació n completamente
extrañ a, que durante muchos siglos se mostró superior en mú l-
b) Que echaba (se refiere al almendro); aquel canto (el de
tiples aspectos a la civilizació n de Occidente. La lengua
la cigarra); repetició n de las palabras almendro, cigarra,
popu- lar romá nica, en la época de su má s temprana
evolució n, se
vio, en gran parte de la Península, obligada a convivir vulgar Con veintiuna de las veinticinco letras en forma de cruz
y diariamente con la extrañ a lengua oficial á rabe. En esta se obtiene dos veces «Padre Nuestro».
con- vivencia, el á rabe, como lengua dominadora, llegó a Utilizando las cuatro letras restantes ( A, A, O, O) se llega a
debilitar o a extinguir los dialectos romá nicos, y cuando «alfa y omega, el principio y el fin».
progresivamente el á rabe fue vencido, y los dialectos
mozá rabes fueron susti- tuyéndose por los dialectos de los
reconquistadores, esta sus- titució n suscita problemas muy
varios, uno de los cuales, el principal y más debatido, es el que
se refiere al solar de las len- guas catalana, aragonesa,
castellana, leonesa y portuguesa.
Ramó n MENÉ NDEZ PIDAL, en Enciclopedia
Lingüística Hispánica.

a) Según Menéndez Pidal, ¿qué fue lo que motivó la pro-


funda transformación de las lenguas romances hispá-
nicas?
b) ¿Cómo influyó la lengua árabe en los incipientes dia-
lectos románicos de la Península?
c) Una vez vencidos los árabes, ¿qué sucedió con los
dia- lectos empleados por los reconquistadores?
d) ¿Estás de acuerdo con Menéndez Pidal cuando afirma
que «la fragmentación lingüística actual de la
península ibérica es, en lo fundamental y decisivo,
resultado de la Reconquista»? Argumenta tu
respuesta.
a) La invasió n musulmana, que trajo consigo la entrada de
otra lengua de cultura, el á rabe.
b) Llegó a debilitarlos o a extinguirlos.
c) Que se empezaron a suscitar problemas, como el de la
len- gua catalana, aragonesa, castellana, leonesa y
portuguesa.
d) Respuesta libre.
4. El hecho de que el español de América comparta muchos
de los rasgos propios de los dialectos meridionales de
nuestro país obedece también a una razón histórica, rela-
cionada con el origen de los colonizadores de las nuevas
tierras descubiertas. Investiga la causa de este fenómeno
y copia en tu cuaderno la información más relevante que
halles sobre el tema.
Esto es debido a que una gran parte de los colonizadores de
las tierras recién descubiertas eran de origen andaluz y extre-
meñ o. Por ello, llevaron consigo su particular variedad
dialec- tal, y fue esta la que se extendió por las tierras
colonizadas.

… encuentra la clave
1. En la unidad anterior estudiamos lo que era un
palíndromo. Lee la siguiente información y realiza las
actividades que tienes a continuación:
El cuadrado mágico SATOR AREPO
Es un misterioso palíndromo cristiano escrito en latín. Se en-
contró por primera vez inscrito en una columna de las ruinas
de Pompeya y posteriormente ha sido hallado en numerosas
tumbas y templos cristianos.
Está formado por cinco palabras de cinco letras que, en hori-
zontal, de arriba abajo y de abajo arriba, forman un
palíndro- mo. Su significado no se conoce con exactitud. Una
traducció n libre sería: «El sembrador dirige las ruedas del
trabajo».
Con las veinticinco letras se forma el anagrama « ORO TE, PATER,
ORO TE, PATER, SANAS» (Te ruego Señ or, te ruego Señ or, cú rame).
Una de las interpretaciones que se le ha dado a este cuadrado
mágico defiende que era en realidad una especie de anuncio
colocado en las casas de cristianos que ofrecían refugio a otros
cristianos, quienes serían los ú nicos que podrían orde- nar las
letras para obtener el significado real del cuadrado.
La gran cantidad de lugares donde se ha encontrado el cua- drado
es coherente con la interpretació n cristiana; pero el hallazgo de la
inscripció n grabada en una columna del gran gimnasio de
Pompeya plantea dudas sobre este origen, pues, aunque existían
comunidades cristianas en las ciudades ve- subianas antes del añ o
79, el Apocalipsis de san Juan, al que harían referencia la «alfa» y la
«omega» (la a y la o griegas) colocadas a los lados de la cruz, no
se difunde en Italia hasta casi cien añ os después.
www.almacendeclasicas.blogspot.com.es
a) ¿Por qué se califica a este palíndromo como «mágico» y
«misterioso»?
b) ¿Por qué crees que los cristianos necesitarían colocar en
sus casas un símbolo que únicamente entendieran otros
cristianos?
c) ¿Qué es lo que puede poner en duda la interpretación
cristiana del símbolo?
d) Busca información en Internet acerca de otros lugares
donde se haya encontrado esta inscripción.
e) Investiga otras posibles interpretaciones que se hayan
barajado sobre el significado de este misterioso palín-
dromo, copia en tu cuaderno la información más rele-
vante y explícasela oralmente a tus compañeros. Pue-
des elaborar una presentación en Prezi o en PowerPoint
que te servirá de apoyo para la exposición oral.
a) Se le califica de mágico y misterioso porque no se ha podi- do
llegar a una interpretació n clara de lo que significa. Es muy
curioso que haya aparecido en tantos lugares diferen- tes y
que con sus letras se puedan formar esos mensajes, por lo
que está rodeado de un halo de misterio.
b) Porque en esa é poca los cristianos eran perseguidos y ne-
cesitaban mantener su religió n en secreto.
c) El haberse hallado esta inscripció n en una columna de
Pompeya, datando de una fecha muy anterior a la difusió n
del apocalipsis de san Juan en Italia (las letras alfa y la
omega de la cruz hacen referencia a dicho apocalipsis).
d) Se han encontrado en ruinas romanas de Cirencester (la
antigua Corinium), en Inglaterra. En Italia en la regió n
Abruzzo en la iglesia de San Pietro ad Oratorium, entre Bussi
y Capestrano; en el castillo de Rochemaure, en Oppè- de; en
Siena (pared de la catedral); en la abadía de Colle- pardo; en
Santiago de Compostela, etc.
e) Existen otras interpretaciones relacionadas con el esote- rismo
y, en particular, con la alquimia. Cabe mencionar en este
sentido la relació n con los constructores de catedrales
mencionada por Louis Charpentier y, en el mismo sentido, su
vínculo con la teoría quiliá smica o milenarismo, hipó - tesis que
analiza el cuadrado sustituyendo las letras por nú meros para
conformar una matriz de cuya resolució n podría derivar en una
suerte de calendario antiguo. Para Pedro Guirao, el cuadrado
esconde el secreto hermético de la cuadratura del círculo. De
igual forma, existen diversos desarrollos cabalísticos y
matemá ticos presentados por el profesor Rafael de Có zar de la
Universidad de Sevilla que, partiendo de la configuració n
formal de un texto poético y en razó n de sus exigencias
métricas, desarrolla geome- trías herméticas con las que
concluye que las razones de su conformació n parecen má s
estéticas que contener un fundamento filosó fico.
AMAR, LUCHAR, REZAR… 7
APERTURA DE UNIDAD 4. Según Patronio, ¿hay alguna razón por la que merezca la
1. ¿Qué dos personajes dialogan en este poema? ¿Coincide pena arriesgar la propia vida? ¿Cuál es?
lo que cuenta la muchacha acerca de dónde ha estado Sí, para defender su honra o por alguna cosa en la que esté
con lo que cree la madre? obligado.
Dialogan una madre y una hija. La hija miente a la madre di- 5. El propio autor de El conde Lucanor aparece en este y
ciendo que viene de la fuente y ha tardado porque los
en todos los ejemplos del libro. ¿Dónde? ¿Qué es lo que
ciervos revolvían el agua, pero la madre sabe que en
realidad viene de estar con su amado.
hace?
Aparece al final. Viendo don Juan que este cuento era bueno,
2. Fíjate en la forma en la que está escrito. ¿Te llama la
lo hizo poner este libro y escribió unos versos que dicen así.
atención? ¿Por qué? Señala todas las repeticiones que
encuentres. Vocabulario
Al alumno le puede llamar la atenció n que la versió n origi-
nal está escrita en lengua galaico-portuguesa, así como que 6. Busca en el diccionario el significado de los verbos
está estructurado en forma de diá logo, y por supuesto, las preciar y preciarse. Escribe una frase con cada uno donde
abundantes repeticiones (dime hija, hija mía; ¿por qué se ob- serve claramente su significado.
tar- daste…?; tardé, madre mía; porque los ciervos del • preciar: apreciar. Los que han estado en paro precian mu-
monte…; mientes, hija mía; nunca vi un ciervo que…) así cho su trabajo.
como el estribillo (amores tengo).
• preciarse: gloriarse, jactarse y hacer vanidad de algo. Se
3. Estudiaremos que en la lírica popular o tradicional es precia constantemente de lo bien que hace las cosas.
muy frecuente el empleo de símbolos, que veladamente
7. Escribe dos sinónimos de cada una de las siguientes pa-
aluden a otros significados. Busca información sobre qué
labras: beneficio, riqueza, codicia, frivolidad, estimación.
significa- ban los ciervos y las fuentes en dicha literatura.
• beneficio: provecho, ganancia.
Las fuentes en la lírica popular simbolizaban el lugar de en-
cuentro de los amantes, y los ciervos que bebían en el agua • riqueza: fortuna, opulencia.
de la fuente o del río eran un símbolo del encuentro sexual • codicia: avaricia, egoísmo.
entre los amantes.
• frivolidad: trivialidad, banalidad.
4. La lírica galaico-portuguesa nos ha dejado poemas bellísi-
mos. Escucha una cantiga de amor portuguesa tecleando • estimación: aprecio, estima.
en YouTube «O que vos nunca cuidei a dizer». Pertenece 8. En el texto, Patronio menciona la honra. Busca el signifi-
al rey de Portugal don Dionís, que también fue un impor- cado de este término y cópialo en tu cuaderno. A conti-
tante trovador. ¿Qué sensaciones te evoca? Coméntalas nuación, investiga qué importancia tenía la honra para
en voz alta. los hombres y mujeres medievales.
Respuesta libre. La honra es la estima y respeto de la dignidad propia; buena
opinió n y fama, adquirida por la virtud y el mé rito.
VIVE LA LECTURA
En la Edad Media la honra tenía muchísima importancia,
Comprensión lectora por- que perderla equivalía a estar muerto para la sociedad.
Las mujeres podían quedar deshonradas si mantenían
1. ¿Cuál es el problema que dice tener el conde? ¿Qué le
relaciones sexuales fuera del matrimonio, y en este caso
aconseja Patronio ante tal situación? también eran deshonrados los varones de su familia.
El conde tiene miedo de que, si se queda en cierto sitio donde
puede ganar mucho dinero, ponga en riesgo su vida. Patronio 9. ¿Crees que ha cambiado el concepto de honra en la ac-
le aconseja que nunca arriesgue su vida por enriquecerse; se tualidad? Razona tu respuesta.
lo ejemplifica mediante un cuento. Respuesta libre.
2. Explica en qué partes se divide el texto, qué dos historias 10. Fíjate en la moraleja del cuento. ¿Conoces algún refrán
hay y cómo una historia se enmarca dentro de la otra. po- pular que signifique lo mismo? Escríbelo en tu
El texto se divide en las siguientes partes: cuaderno.
• El conde expone su problema a su consejero. La avaricia rompe el saco.
• Este le responde mediante un cuento o ejemplo. 11. Busca en el diccionario el origen de la palabra
• A continuació n le aconseja directamente lo que debe hacer. majadero.
• Se explica que el conde sigue el consejo y le va bien, y ¿Crees que el verbo del que procede puede tener algo
có mo don Juan Manuel mandó escribirlo en su libro. que ver con su significado? Explica por qué.
• Moraleja final en verso. La palabra majadero proviene del verbo majar (que significa
machacar, y coloquialmente, molestar, importunar).
Hay dos historias, la del conde que le pide consejo a Patronio,
y el cuento que le narra este sobre el hombre que se ahogó Investigación y redacción
en el río por no querer desprenderse de un saco de piedras
preciosas. Esta ú ltima está dentro de la primera, que le sirve 12. Busca información sobre otras tres importantes obras
de marco. literarias escritas por don Juan Manuel. ¿De qué trata
cada una de ellas? ¿Tienen algo en común?
3. ¿Te parece que Patronio le da un buen consejo a
Lucanor? El alumno puede hablar, por ejemplo, de las siguientes obras:
¿Y la forma en que lo hace? Argumenta tus respuestas. • El Libro del caballero y el escudero; narra la historia de
un escudero mancebo aspirante a cortesano que ha de ir
Respuesta libre.

Lengua castellana y Literatura. 3.º ESO. Solucionario 59


7 AMAR, LUCHAR, REZAR…
a unas cortes convocadas por el rey y que recibe las má s
diversas enseñ anzas por parte de un ermitañ o que ha
sido

60 Lengua castellana y Literatura. 3.º ESO. Solucionario


caballero. Asiste a unas justas y vuelve a la ermita para
• lenguas vernáculas: son las lenguas propias del país o lu-
recibir nuevas enseñ anzas. El anciano excaballero muere
gar de nacimiento de uno.
y su joven discípulo le sepulta.
• escolástica: perteneciente o relativo a las escuelas medie-
• El Libro de los estados ofrece una visió n de có mo debe
vales o a quienes estudiaban en ellas.
ser una sociedad ideal en el siglo XIV, aunque es asimismo
una narració n de fin didáctico (educació n de un príncipe) 3. Vuelve a leer la cantiga «Pela rebeira do rio». ¿Cuál es su
inspirada en la leyenda de Barlaam y Josafat, forma cris- temática? Señala en él todas las características de la lírica
tianizada en que fue transmitida a Occidente la leyenda de galaico-portuguesa que encuentres. Analiza también su
Buda. estructura.
• El Libro de la caza, tratado de cetrería que encierra dos Esta cantiga nos presenta a una muchacha que va cantando
aspectos novedosos: la inclusió n de anécdotas personales y gozosa a la orilla del río, por el que se aproximan las barcas.
un capítulo con una descripció n geográ fica de los mejores
En la lírica galaico-portuguesa, todo lo que tiene relació n con
lugares para practicar la cetrería en el reino de Castilla.
el mar, el agua, las barcas… guarda una estrecha conexió n
Todos tienen en comú n que son libros didá cticos, está n pensa- con los sentimientos amorosos.
dos para enseñ ar algo a los príncipes de su época.
Como es habitual en este tipo de lírica, la protagonista es una
13. Lee la siguiente moraleja de otro de los cuentos perte- doncella enamorada, feliz posiblemente porque va a encon-
necientes a El conde Lucanor. A partir de ella piensa una trarse con su amado, en armonía con la naturaleza.
situación que hubiera podido motivar dicho consejo y Formalmente destacan en el poema las repeticiones y el pa-
escribe el cuento en tu cuaderno, respetando la estruc- ralelismo (pela ribeira, cantando ia la dona), así como ese
tura: introducción donde alguien cuenta un problema estribillo que se repite detrá s de cada estrofa (venhan nas
que tiene y para el que pide consejo, respuesta en forma barcas polo rio a sabor).
de cuento, conclusión y esta moraleja final. Deberá 4. Lee ahora la jarcha y el villancico. ¿Cuál es el tema de
constar, al menos, de cien palabras. cada uno? Señala en cada uno de ellos todos los aspectos
Al que antes tu enemigo solía ser que te llamen la atención por lo que se refiere a su forma
ni en nada ni nunca le debes creer. y estructura.
Respuesta libre. En la jarcha nos encontramos una protagonista femenina, que
14. Compara el cuento que has escrito con el de un compa- se lamenta a su amado (habib) por su sufrimiento amoroso.
ñero y comentad oralmente las diferencias y semejanzas. Está escrito en lengua mozá rabe y es un poema muy breve,
donde se observan repeticiones (tant’ amare).
Respuesta libre.
En cuanto al villancico, es una composició n tradicional en
LITERATURA castellano. En este caso su protagonista tambié n es una mu-
chacha, que se dirige a su madre. El significado del poema
1. Busca datos sobre de la Reconquista de la península es bastante enigmá tico, ya que tiene un contenido simbó lico.
ibéri- ca: quiénes la iniciaron, qué figuras históricas El vergel donde la muchacha va a coger rosas posiblemente
fueron más relevantes en ella, qué periodo de tiempo simbolice el encuentro sexual. Aquí las palabras morir y
abarcó… Expón tus averiguaciones oralmente a la clase. ma- tarme aluden al goce sexual.
La Reconquista abarcó un periodo de tiempo de siete siglos Formalmente tambié n es un poema breve basado en repeti-
(del VIII al XV d. C.), en el cual los reinos cristianos del norte de ciones y paralelismos.
la Península que habían resistido la invasió n á rabe del añ o 711 5. Lee el siguiente fragmento (en versión modernizada) del
trataron de recuperar el territorio perdido y liberar Españ a de
la dominació n islá mica.
Cantar de Mio Cid y responde a las preguntas:
Alargó entonces las manos el de la barba florida
Uno de los personajes má s cruciales del inicio de la misma
fue un noble asturiano, don Pelayo, que lideró la batalla de y a las niñ as, sus dos hijas, en los brazos las cogía;
Covadonga en el 722, considerada el inicio de la al corazó n acercolas porque mucho las quería.
Reconquista.
Con lá grimas en los ojos muy fuertemente suspira:
2. Busca el significado de los siguientes términos y explica
—Oídme, doñ a Jimena, tan entera mujer mía;
qué importancia pudieron tener en el desarrollo de la
cultura medieval: mozárabe, mudéjar, románico, gótico, como yo quiero a mi alma otro tanto a vos quería.
lenguas vernáculas, escolástica. Ya lo veis, nada má s cabe que separarnos en vida.
• mozárabe: se dice del individuo de la població n hispá nica a) ¿Qué rasgos de la personalidad de héroe se observan
que, consentida por el derecho islá mico como tributaria, en él?
vivió en la Españ a musulmana hasta fines del siglo XI con-
servando su religió n cristiana e incluso su organizació n
b) Busca algún epíteto épico que se use para referirse
eclesiá stica y judicial. al Cid.
• mudéjar: se dice del musulmá n a quien se permitía seguir c) ¿A cuál de los tres cantares crees que pertenece este
viviendo entre los vencedores cristianos sin mudar de reli- fragmento? Razona tu respuesta.
gió n, a cambio de un tributo. d) Analiza la métrica de este fragmento. ¿Qué tipo de
• románico: se dice de las lenguas derivadas del latín y de rima aparece en él?
sus correspondientes manifestaciones literarias y cultura- a) Vemos aquí al Cid como un personaje muy humano,
les. En arquitectura, se dice del estilo arquitectó nico que capaz de emocionarse y llorar al despedirse de su mujer
dominó en Europa durante los siglos XI, XII y parte del XIII. y sus hijas.
• gótico: en arquitectura, se dice del arte que se desarrolla b) El de la barba florida.
en Europa desde el siglo XII hasta el Renacimiento.
c) Al primero, ya que muestra la despedida del Cid cuando le
obligan a marchar de Castilla. Análisis de la forma y el contenido. El poema está escrito
siguiendo la estrofa típica del mester de clerecía, la cuaderna
d) En cuanto a la mé trica, se trata de versos de diecisé is sí-
vía.
labas, separados por una cesura central. La rima es aso-
nante (i-a). 6. Cuenta los versos y señala cuál es su medida.
6. El Sendebar y el Calila e Dimna son dos colecciones de Son estrofas de cuatro versos, cada uno de ellos de catorce
cuentos de la época, que fueron traducidas del árabe sílabas.
al castellano. Investiga cuál era la historia que servía de 7. Observa la rima. ¿Es asonante o consonante? ¿Cómo
marco a los cuentos en ambas obras y qué tienen en riman entre sí los versos?
común con El conde Lucanor. Los versos riman en consonante dentro de cada estrofa
En el Sendebar, el pretexto narrativo que enmarca los cuentos (AAAA) (BBBB), etc.
es la leyenda del hijo ú nico de Alcos, rey de Judea, que
rehú sa los ofrecimientos amorosos de una de las mujeres del
8. En el texto abundan los sustantivos. Señálalos e indica a
haré n de su progenitor. Este rechazo provoca que la qué campos semánticos pertenecen.
cortesana lo acuse falsamente de intentar violarla. El joven Términos relacionados con el hombre y sus necesidades car-
príncipe es sentenciado a muerte y se ve obligado a guardar nales y físicas: hombre, sustentamiento, unión, hembra, hu-
silencio por espacio de siete días. Para entretener la mano, pecador, mujeres, amor.
espera, los sabios de la corte le narran cuentos que tienen Relacionados con el principio de autoridad en el que basa sus
cará cter misó gino; entre estos, su madrastra cuenta otros razonamientos: filósofo, sabio, razonar, verdad.
que tienen por objeto condenar al infante. El desenlace,
sentenciado por el rey, es la condena de su madrastra a Relacionados con los seres vivos y la naturaleza: hombres,
morir en un «caldero seco» al fuego, mientras que el aves, bestias, animal, cueva, natura, criaturas, naturaleza.
príncipe se salva. Relacionados con las consecuencias del amor: seso, mesura,
En Calila e Dimna, el marco son las conversaciones entre un locura, mal, fuego, cenizas, bien.
rey y un filó sofo, que dan paso a cuentos o exempla protagoni- 9. Aunque no aparecen en el texto demasiados recursos
zados y narrados por animales, entre ellos dos lobos llamados poéticos, sí que se utiliza un símbolo muy llamativo para
Calila e Dimna. referirse al pecado carnal. ¿Cuál es?
Ambos libros tienen en comú n con El conde Lucanor que son Se identifica el pecado carnal con el fuego.
ejemplarizantes, que tenían como objetivo la educació n de los
príncipes. Conclusiones. Este fragmento es un buen ejemplo de la am-
bigüedad que caracteriza a la obra.
7. Lee un cuento perteneciente a El conde Lucanor (búscalo
en la biblioteca o en Internet). Resume en tu cuaderno su 10.¿Pretende Juan Ruiz advertirnos de los peligros del amor
argumento. Después, cuéntaselo oralmente a tus carnal o, por el contrario, invitarnos a gozar de él o al
compañe- ros. ¿Cuál es su moraleja? ¿Crees que el me- nos a probarlo?
consejo que ofrece Patronio es acertado? El texto es una clara invitació n a probar, al menos, el amor
Respuesta libre. carnal, y luego ya decidir y obrar en consecuencia.
11. ¿En qué versos encontramos la conclusión de todo lo
COMENTARIO DE TEXTO planteado anteriormente?
Localización. Este fragmento del Libro de buen amor se Que probemos las cosas no siempre es lo peor, / el bien y el
ins- cribe dentro de la corriente literaria del mester de mal sabed y escoged lo mejor.
clerecía del siglo xıv. 12. ¿Estás de acuerdo con la visión del amor que
1.¿Qué características de dicha corriente se pueden apre- propone el arcipreste?
ciar en él? Respuesta libre.
Podemos inscribirlo dentro del mester de clerecía del siglo XIV
por el uso de la cuaderna vía. EL JARDÍN DE LA LITERATURA
Tema y estructura. En el fragmento, el arcipreste nos habla 1. Lee estos fragmentos de poesía medieval. A
acerca del amor carnal. continuación, resuelve las tareas.
2. ¿Cuál es su postura ante este tipo de amor? ¿Te parece I
que lo defiende o que lo cuestiona? Decidme, ay hermanitas,
El arcipreste nos invita a gozar del amor carnal. ¿có mo contener mi
mal? Sin el amado no
3. ¿En qué figura de la Antigüedad se apoya para defender
viviré :
sus argumentos? ¿Cómo se denomina dicho recurso?
¿adó nde iré a buscarlo?
Defiende esta postura basá ndose en el argumento de autori- Mi corazó n se me va de mí.
dad (citando a Aristó teles). Oh Dios, ¿acaso se me tornará ?
4. ¿Cómo se presenta a sí mismo el arcipreste con respecto ¡Tan fuerte es mi dolor por el
a las cuestiones amorosas? amado! Enfermo está , ¿cuá ndo
sanará ?
Explica que él mismo es pecador y ha amado a muchas mu-
¿Qué haré , madre?
jeres.
Mi amado está a la puerta.
5. ¿El poeta va de lo general a lo particular, o viceversa?
II
Va de lo general (los hombres y su necesidad de estar con mu-
jeres) a lo particular (su propia experiencia amorosa). Olas del mar de Vigo,
¿visteis a mi amigo?
¡Ay Dios! ¿Vendrá pronto?
Olas del mar agitado,
¿visteis a mi amado? a) ¿Qué características del mester de clerecía se resaltan
¡Ay Dios! ¿Vendrá pronto? en el texto? ¿A qué se contrapone?
¿Visteis a mi amigo, b) Comprueba que, efectivamente, la estrofa anterior
aquel por quien yo suspiro? está escrita en cuaderna vía. Mide los versos y señala
¡Ay Dios! ¿Vendrá pronto? su rima.
¿Visteis a mi amado, a) Que es un mester hermoso, sin pecado, ya que es de
por quien tengo gran cuidado? clere- cía. Utiliza la cuaderna vía lo cual denota una gran
¡Ay Dios! ¿Vendrá pronto? maes- tría. Se contrapone al mester de juglaría.

III b) Son cuatro versos de catorce sílabas, que riman entre sí


en consonante (AAAA).
No me habléis, conde,
d’amor en la calle: 3. Fíjate en el estilo en el que está escrito el siguiente frag-
cata que os dirá mal, mento y en su sencillez.
conde, la mi madre. a) ¿Por qué crees tú que el autor prefirió usar una forma
Mañ ana iré , conde, de expresión tan clara y sencilla?
a lavar al río: b) ¿Qué propósito podía tener la obra?
allá me tené is, conde,
V
a vuestro servicio.
ESCENA II
Cata que os dirá mal
MELCHOR
conde, la mi madre.
Señ ores, ¿a qué tierra queréis andar?
No me habléis, conde
d’amor en la calle. ¿Queréis ir conmigo al Criador honrar?
¿Lo habé is visto? Yo lo voy a adorar.
a) ¿Qué tienen en común los tres poemas? ¿En qué se GASPAR
diferencian? Veamos por ventura si le podremos hallar.
Vayamos tras la estrella, veremos el lugar.
b) Señala quién es el yo poético en cada uno de los poe-
MELCHOR
mas y a quién se dirige en cada caso.
¿Có mo comprobaremos si es hombre
c) ¿Sabrías decir qué composición poética es cada uno mortal, si es rey de la tierra, o si es Rey
de ellos? Argumenta tu respuesta. celestial?
a) Los tres son composiciones poé ticas breves, pertenecien- BALTASAR
tes a la lírica tradicional y popular. En los tres hay una ¿Queréis saber bien có mo lo
muchacha que habla con diferentes interlocutores, y su sabremos? Oro, mirra e incienso a é l
temá tica es el amor. Se diferencian en que los dos prime- ofreceremos. Si fuere rey de tierra, el
ros transmiten un sentimiento más desasosegado, de su- oro lo querrá ,
frimiento amoroso por la pé rdida o ausencia del amado, si fuere hombre mortal, la mirra tomará ,
mientras que el tercero es má s sereno, la muchacha está y si Rey celestial, estos dos dejará ,
concertando un encuentro amoroso con su amado. tomará el incienso que le pertenecerá .
GASPAR Y MELCHOR
b) En los tres poemas el yo poé tico es una muchacha, una
Andemos, pues, y así lo haremos,
joven doncella. Los interlocutores difieren: en el primero
nuestros dones le ofreceremos.
se dirige a sus hermanas y a su madre, en el segundo a un
elemento de la naturaleza (las olas del mar de Vigo) y en Auto de los Reyes Magos
el tercero a su amigo, el conde. a) Su expresió n es clara y sencilla para que sea má s fá cil-
c) El primero de ellos es una jarcha (muchacha enamora- mente comprendido por el pueblo. Recordemos que en
da que hace confidencias de amor a su madre y herma- esta época la mayoría de la població n era analfabeta, y
nas, sencillez y brevedad, abundantes interrogaciones y los autos teatrales servían para que el pueblo conociera
exclamaciones). los Evangelios y la vida de Cristo.
El segundo es una cantiga, como vemos en la conexió n de b) La obra tenía un fin didá ctico y moralizante, enseñ ar la
la muchacha con los elementos de la naturaleza, a los que vida de Cristo y el modo de comportamiento cristiano.
hace partícipes de sus sentimientos. Formalmente se basa
en las repeticiones y los paralelismos. LA FACTORÍA DE TEXTOS
El ú ltimo de ellos es un villancico, su tono es má s sereno 1. Vuelve a leer el cuento perteneciente a El conde Lucanor
y reposado. Tambié n aparece el sentimiento amoroso y del apartado «Vive la lectura».
la figura de la madre, esta vez no como confidente sino
a) Analiza ahora en él todos los elementos de la
como ajena a los amores de la muchacha. Aparecen
asimismo repeticiones y paralelismos. narración: tipo de narrador, personajes, tiempo y
espacio.
2. Ahora lee este fragmento del Libro de Alexandre, una
obra en verso de principios del siglo xııı, que narra la vida b) Observa cómo una historia se inserta dentro de otra.
de Alejandro Magno. Distingue entre la historia marco y el cuento que sirve
como ejemplo.
IV
a) y b) Vamos a distinguir entre la historia que sirve como
Mester os traigo hermoso, no es de juglaría; marco al cuento de Patronio, y dicho cuento.
mester es sin pecado, que es de clerecía;
hablar curso rimado por la cuaderna vía En la historia-marco, nos encontramos con un narrador om-
a sílabas contadas, que es gran maestría. nisciente en tercera persona. Los personajes son el conde
Lucanor, su consejero Patronio (personajes principales) y el
propio autor don Juan Manuel al que se menciona al final
(per- sonaje secundario). El espacio no está definido, y
tampoco el tiempo (Un día…).
En cuanto al ejemplo que le cuenta Patronio a Lucanor para
con moraleja es intemporal). El tiempo del discurso son
aconsejarle, el narrador es el propio Patronio, que es un na-
unos días.
rrador omnisciente en tercera persona. El personaje principal
es el hombre que quiere cruzar el río con la bolsa de piedras d) El planteamiento sería cuando se nos presenta al ladró n,
preciosas, y el secundario el que le grita desde la orilla que la con sus vicios (robar) y sus virtudes (es devoto de la Vir-
suelte o se ahogará . El espacio es un río, y el tiempo indefini- gen). Serían las dos primeras estrofas.
do, ya que es un relato atemporal que vale para adoctrinar El nudo sería cuando el ladró n finalmente es apresado y
en cualquier é poca y situació n. condenado a la horca, có mo la Virgen María lo socorre
y evita que muera, sujetá ndole por los pies con sus ma-
2. Tras leer el siguiente fragmento perteneciente a los nos, y có mo llegan sus parientes y descubren el milagro.
Mila- gros de Nuestra Señora, de Gonzalo de Berceo, (De la tercera a la sexta estrofa).
responde a las preguntas que se te plantean. Y finalmente el desenlace sería có mo le dejan vivir en
Había un ladró n malo que prefería hurtar paz, é l cambia de vida y se reconduce y al fin muere
a ir a las iglesias o a puentes levantar; cuando le llega su hora. También se incluiría aquí la
solía con lo hurtado su casa gobernar; moraleja final de alabanza a la Virgen y sus virtudes.
tomó costumbre mala que no podía dejar. (Estrofas sé ptima y octava).
Entre todo lo malo tenía una bondad 3. Lee el cuento XXXIV de El conde Lucanor: «Lo que suce-
que al final le valió y le dio salvedad: dió a un ciego que llevaba a otro» (puedes encontrarlo
creía en la Gloriosa de toda voluntad,
en una biblioteca o en la biblioteca virtual Miguel de
y siempre saludaba hacia su majestad.
Cervan- tes), y a continuación responde a las siguientes
Como aquel que mal anda en mal ha de caer, preguntas:
una vez con el hurto lo hubieron de
prender; como ningú n consejo lo pudo a) ¿Quién es el autor de este texto? ¿Y el narrador del
defender juzgaron que en la horca lo debían texto completo?
poner. b) ¿Y el narrador del cuento dentro del cuento? ¿Hay un
[…] La Madre gloriosa, tan ducha en acorrer, narratario? ¿Quién es?
la que suele a sus siervos en las cuitas valer, c) Explica cuál es el marco de ambas narraciones.
a este condenado quísolo proteger,
recordose el servicio que solía hacer. d) ¿Qué personajes aparecen en la historia? ¿Cómo son?
Puso bajo sus pies, donde estaba colgado, a) El autor del texto es don Juan Manuel. El narrador del
sus manos preciosísimas; tú volo levantado: texto completo es un narrador omnisciente en tercera
no se sintió por cosa ninguna embarazado, persona.
ni estuvo má s vicioso nunca, ni má s pagado. b) El narrador del cuento dentro del cuento sería Patronio, el
Al fin al tercer día vinieron los parientes, consejero. Sí hay un narratario, que sería el propio conde
vinieron los amigos y vecinos clementes; Lucanor, que escucha el relato de Patronio.
venían por descolgarlo rascados y dolientes, c) El tiempo y el espacio en la narració n-marco son bas-
pero estaba mejor de lo que creían las gentes. tante indeterminados. En cuanto al relato que le cuenta
[…] Lo dejaron en paz que siguiese su vía, Patronio a Lucanor, tiene lugar en una ciudad
porque no querían ir contra Santa María; (indetermi- nada), por caminos y un barranco. El tiempo
su vida mejoró , se apartó de folía, tambié n es indeterminado.
cuando cumplió su curso muriose de su día. d) En la historia-marco aparecen como principales Lucanor
A madre tan piadosa, de tal benignidad, y su consejero Patronio, y como secundario don Juan Ma-
que en buenos como en malos ejerce su nuel, al que se menciona al final.
piedad, debemos bendecirla de toda voluntad: En el cuento dentro del relato aparecen como protago-
aquel que la bendijo ganó gran heredad. nistas dos ciegos temerarios que acaban cayendo por un
barranco.
a) Este milagro se titula «El ladrón devoto». ¿Crees que
está bien elegido el título? ACTIVIDADES FINALES
b) Explica qué características se le atribuyen al ladrón y
cuáles a la Virgen. ¿Por qué esta intercede por aquel? Repasa lo que has aprendido
c) Analiza en este fragmento todos los elementos de la 1. Lee el siguiente fragmento y después realiza las activida-
narración: narrador, personajes, espacio y tiempo. des que se te plantean:
d) Divídelo en planteamiento, nudo y desenlace. De cómo el Amor visitó al Arcipreste
y de la disputa que ambos sostuvieron:
a) Respuesta libre.
Una noche sostuve combate peregrino:
b) El ladró n era malo, prefería robar a trabajar. Pero tenía
pensaba yo en mi suerte, furioso (y no de vino)
una virtud: que era un gran devoto de la Virgen María.
cuando un hombre algo hermoso, corté smente a mí vino.
La Virgen aparece como piadosa, misericordiosa. Ejerce Le pregunté quié n era; dijo: —Amor, tu vecino.
su piedad para con aquellos que la siguen y creen en ella.
Con enojo muy grande le empecé a denostar;
Por esto intercede por el ladró n devoto.
le dije: —Si amor eres, no puedes aquí estar,
c) El narrador es un narrador omnisciente en tercera perso- eres falso, embustero y ducho en engañ ar;
na. Los personajes principales son el ladró n y la Virgen salvar no puedes uno, puedes cien mil matar.
María. Como secundarios aparecen los parientes, vecinos
Con engañ os, lisonjas y sutiles mentiras
y amigos del ladró n. En cuanto al espacio, es indetermina-
emponzoñ as las lenguas, envenenas tus viras,
do, no sabemos dó nde se desarrolla la acció n. El tiempo de
hiere a quien más te sirve tu flecha cuando tiras;
la historia también es indeterminado (al ser una historia
separas de las damas a los hombres, por iras.
Enloquecidos trae a muchos tu saber;
Como diferencias podemos señ alar, entre otras, que las jar-
les estorbas el sueñ o, el comer y el beber,
chas son má s breves y está n escritas en lengua mozá rabe.
haces a muchos hombres a tanto se atrever
Las cantigas está n escritas en lengua galaico-portuguesa y
por ti, que cuerpo y alma llegará n a perder.
se basan en las repeticiones y en el paralelismo, así como en
No tienes regla fija, ni te portas con tiento, el procedimiento del leixa-pren. Ademá s en ellas es muy im-
a voces arrebatas con ímpetu violento, portante la naturaleza y está n dotadas de un gran simbolismo.
a veces, poco a poco, con maestrías ciento; Los villancicos, por su parte, está n escritos en castellano y
en cuanto yo te digo tú sabes que no miento. poseen una mayor variedad temá tica, aparte del amor apa-
Eres tan enconado que al que hieres de golpe recen otro tipo de asuntos. Está n formados por una glosa y
no sana medicina, emplasto ni jarope, un estribillo.
no hay hombre recio y fuerte que contigo se tope 3. Di si las siguientes afirmaciones son verdaderas o falsas.
que por diestro que sea no se haga blando y torpe. En caso de que sean falsas, explica qué sería lo correcto:
Arcipreste DE HITA: Libro de buen amor
a) La sociedad medieval se basaba en las relaciones
(versió n modernizada de María Brey Mariñ o, Castalia).
feudales.
a) Busca en el diccionario el significado de estas b) En la lírica tradicional o popular un hombre se
palabras: peregrino, denostar, ducho, lisonja, lamenta- ba de la ausencia de su amada.
emponzoñar, viras, tiento, enconado, emplasto y
c) Las jarchas estaban escritas en lengua mozárabe.
jarope.
d) En los cantares épicos se narraban en prosa las haza-
b) ¿Qué sabes acerca del Libro de buen amor, obra a la
ñas de algún héroe.
que pertenece este fragmento?
e) El autor del Cantar de Mio Cid fue Per Abbat.
c) Explica qué tipo de narrador es el que cuenta la
historia. f) Trovador y juglar significaba lo mismo en la Edad
Media.
d) ¿Qué le reprocha el protagonista al amor?
g) El Libro de buen amor es una autobiografía amorosa
e) ¿Cuáles son los personajes que aparecen? Señala ficticia.
las características de cada uno de ellos.
a) Verdadero.
f) Analiza la métrica de los primeros ocho versos del
b) Falso. Es una muchacha la que expresa sus sentimientos
fragmento. ¿De qué tipo de estrofa se trata? amorosos.
a) • peregrino: extrañ o, especial, raro, pocas veces visto. c) Verdadero.
• denostar: injuriar gravemente, infamar de palabra. d) Falso. Se narraban dichas hazañ as, pero en verso.
• ducho: experimentado, diestro. e) Falso. Es un Cantar anó nimo. Per Abbat fue tan solo el
• lisonja: alabanza afectada para ganar la voluntad de copista.
alguien. f) Falso. Los trovadores eran también autores de sus poemas,
• emponzoñar: envenenar. y pertenecían al mundo de la corte. Los juglares general-
• viras (del verbo virar): cambiar de rumbo. mente eran retransmisores, no autores, y viajaban por
pueblos y villas recitando y cantando los poemas.
• tiento: cuidado, prudencia.
g) Verdadero.
• enconado: encarnizado, violento y muy porfiado.
4. Investiga quién fue Rodrigo Díaz de Vivar, y escribe en
• emplasto: preparado farmacéutico de uso tó pico, só lido,
moldeable y adhesivo.
tu cuaderno su biografía. Cuéntasela después a tus com-
pañeros.
• jarope: jarabe.
El alumno puede recoger, por ejemplo, la siguiente informació n:
b) Respuesta libre.
Rodrigo Díaz de Vivar, llamado El Cid Campeador (Vivar,
c) Se trata de un narrador en primera persona. Es el propio actual Españ a, h. 1043-Valencia, 1099). Caballero castellano,
arcipreste que cuenta có mo una noche le visitó el Amor y fue educado junto al infante Sancho, hijo del rey Fernando I de
la conversació n que sostuvieron. Castilla y Leó n, quien, al acceder al trono castellano, lo nom-
d) Que es falso y embustero, engañ a a los hombres y los en- bró alférez real (1065). Como jefe de las tropas reales, Rodrigo
venena, hiere a sus servidores y los vuelve locos, hacié n- participó en la guerra que enfrentó a Sancho II de Castilla
doles perder el hambre y el sueñ o. Por su culpa todos los con su hermano Alfonso VI de Leó n. El destino, sin embargo,
hombres pierden cuerpo y alma. quiso que Sancho II muriera en 1072, con lo que Alfonso VI se
e) Los personajes son el propio arcipreste (enojado y airado convirtió en soberano de Castilla y Leó n.
en contra del Amor) y el Amor (aquí aparece personificado El nuevo monarca no solo no manifestó resentimiento hacia el
como un hombre hermoso y cortés). Campeador, sino que, consciente de la valía de sus servicios,
f) Se trata de estrofas en cuaderna vía: cuatro versos lo honró concedié ndole la mano de su sobrina, doñ a Jimena,
alejan- drinos con rima consonante AAAA, BBBB. con quien se casó en julio de 1074. No obstante, unos añ os
después, en 1081, una inoportuna expedició n a tierras toleda-
2. Explica cuáles son las diferencias y las semejanzas princi- nas sin el permiso real, provocó su destierro de Castilla y la
pales entre las jarchas, las cantigas de amigo y los villan- confiscació n de todas sus posesiones.
cicos. Acompañ ado de su mesnada, el Campeador ofreció sus ser-
Las principales semejanzas son que las tres son composicio- vicios primero a los condes Ramó n Berenguer II y Berenguer
nes poé ticas pertenecientes a la lírica tradicional y popular, Ramó n II de Barcelona, pero, al ser rechazado, decidió
de cará cter oral. Son breves y en ellas el sujeto lírico suele ayudar a al-Muqtadir, rey de Zaragoza, en la lucha que
ser una muchacha que le hace confesiones amorosas a algú n mantenía con su hermano al-Mundir, rey de Lé rida, Tortosa
confidente (generalmente su madre, hermanas o amigas). y Denia.
Al servicio de al-Muqtadir, venció en Almenar a Berenguer desdichada!
Ramó n II (1082) y cerca de Morella a al-Mundir y al
soberano aragoné s (1084). Durante este período fue cuando
recibió el sobrenombre de Cid, derivado del vocablo á rabe
sid, que sig- nifica señor. (Fuente: biografiasyvidas.com).
5. Investiga cómo se trabajaba en la Escuela de Traductores
de Toledo y expón la información que has recabado ante
la clase.
Los métodos de traducció n evolucionaron con el tiempo. En
un primer momento, un judío o cristiano conocedor del á rabe
traducía la obra original al romance oralmente ante un ex-
perto conocedor del latín que, a continuació n, iba redactando
en esta lengua lo que escuchaba. Más tarde, en la época de
Alfonso X, los libros fueron traducidos por un ú nico traduc-
tor conocedor de varias lenguas, cuyo trabajo era revisado al
final por un enmendador.
(Fuente: www.uclm.es/escueladetraductores).
6. Lee los siguientes poemas pertenecientes a la lírica po-
pular y a continuación realiza las actividades propuestas:
I
¡Oh madre, mi amigo
se va y no vuelve!
Dime qué haré , madre,
si mi pena no afloja.
II
Si la noche hace escura
y tan corto es el camino,
¿có mo no venís, amigo?
La media noche es pasada
y el que me pena no
viene:
mi desdicha lo detiene,
¡qué nascí tan desdichada!
Há ceme vivir penada
y mué straseme enemigo:
¿có mo no venís, amigo?
III
Al río me salgo
y en su ribera,
escribo en el agua
toda mi pena.
a) ¿Quién es el yo poético en cada una de las tres com-
posiciones? ¿A quién se dirige?
b) ¿Qué sentimiento se muestra en todas ellas?
c) En la lírica tradicional son muy frecuentes las oracio-
nes exclamativas e interrogativas. Busca algún ejem-
plo en estos textos. ¿Qué efecto producen?
d) Observa el tipo de métrica empleada (medida de los
versos, rima). ¿Qué conclusiones puedes sacar?
e) ¿Hay en estos poemas algún recurso literario? ¿Cuál?
f) ¿Cuál de las tres composiciones crees que transmite
mejor la desazón que siente la mujer? Argumenta tu
opinión.
a) En todas ellas el yo poé tico es una muchacha enamorada.
En la primera se dirige a su madre, en la segunda a su
amigo y en la tercera no hay un interlocutor determinado.
b) En todas el sentimiento expresado es de pena por la au-
sencia del amado.
c) I ¡Oh madre, mi amigo se va y no vuelve! / Dime qué
haré, madre…
II ¿Cómo no venís, amigo? / ¡Qué nascí tan
Consiguen crear un efecto de mayor expresividad y
desa- sosiego, nos transmite mejor los sentimientos
apenados de la joven.
d) La primera de ellas es una composició n muy breve, de
ver- sos de arte menor (hexasílabos) y sin rima.
En la segunda, observamos que los versos son
tambié n de arte menor, en este caso octosílabos, y que sí
aparece rima asonante (-aabccbbaa).
La ú ltima composició n también es muy breve, son
versos de arte menor (de 5 y 6 sílabas), con rima
asonante en los pares.
Este tipo de mé trica es propia de la poesía tradicional
y popular (brevedad, versos de arte menor, ausencia de
rima o rima asonante).
e) Observamos alguna personificació n (mi desdicha lo
de- tiene), hipérboles (qué nascí tan desdichada,
escribo en el agua toda mi pena).
f) Respuesta libre.

Recuerda lo que ya sabías


7. Lee el siguiente texto literario y realiza las actividades
que se te plantean después.
Antes de partir hacia un largo viaje, el comerciante fue a
des- pedirse de su mujer.
—Nunca me has dado un regalo que esté a mi altura —
dijo ella.
—Mujer ingrata, todo lo que te he dado me costó añ os de
tra- bajo —respondió el hombre—. ¿Qué má s te podría dar?
—Algo que sea tan bello como yo.
Durante dos añ os, la mujer esperó su regalo. Finalmente
el comerciante regresó .
—Conseguí encontrar algo tan bello como tú —dijo él—.
Lloré ante tu ingratitud, pero decidí cumplir tu deseo. He
pasado todo este tiempo pensando qué regalo sería tan
bello como tú , y acabé encontrá ndolo.
Y le tendió a su mujer un pequeñ o espejo.
Paulo COELHO: Maktub (Planeta
DeAgostini).
a) Analiza el relato desde el punto de vista de la
comuni- cación (emisor y receptor, mensaje,
código, canal, situa- ción e intención comunicativa).
b) Indica cuál es la estructura de esta narración:
plantea- miento, nudo y desenlace.
c) Señala todos los adverbios que aparezcan en el
texto e indica de qué tipo son. Haz lo mismo con
determinantes y pronombres.
d) Analiza todos los verbos que aparezcan en este
relato.
e) Copia en tu cuaderno todos los sintagmas
nominales y analízalos.
f) Analiza morfológica y sintácticamente las
siguientes oraciones:
• Durante dos años, la mujer esperó su regalo.
• Y le tendió a su mujer un pequeño espejo.
a) El emisor es el narrador. El receptor el lector del
cuento. El mensaje es la propia historia que se narra
del comer- ciante y su esposa, el có digo es el
castellano, el canal es escrito, la situació n es que
pertenece a un libro de pequeñ os relatos de Paulo
Coelho y la intenció n comuni- cativa es entretener,
enseñ ar y crear belleza a través de las palabras.
b) El nudo sería cuando la mujer le dice a su marido que nun- • dos: determinante numeral.
ca le ha regalado nada tan bello como ella. El planteamien-
to cuando él se marcha de viaje y tras dos añ os buscando
ese regalo vuelve a casa. Y el desenlace cuando le da a su
esposa un regalo a su altura: un espejo.
c) • Adverbios: antes (de tiempo), nunca (de tiempo),
más
(de cantidad), finalmente (de tiempo).
• Determinantes: un (indefinido), el (artículo), su (po-
sesivo), mi (posesivo), dos (numeral), la (artículo), tu
(posesivo), todo (indefinido), este (demostrativo),
qué (interrogativo).
• Pronombres: me (personal), que (relativo), ella (perso-
nal), todo (indefinido), te (personal), qué (interrogati-
vo), algo (indefinido), yo (personal), tú (personal), él
(personal), lo (personal), le (personal).
d) Partir (infinitivo), fue (preté rito perfecto simple de indi-
cativo, tercera persona de singular), despedirse (infiniti-
vo), has dado (preté rito perfecto de indicativo, segunda
persona de singular), esté (presente de subjuntivo, terce-
ra persona de singular), dijo (pretérito perfecto simple de
indicativo, tercera persona de singular), he dado (pretérito
perfecto de indicativo, primera persona de singular), costó
(preté rito perfecto simple de indicativo, tercera persona
de singular), respondió (pretérito perfecto simple de indi-
cativo, tercera persona de singular), podría (condicional
simple de indicativo, primera persona de singular), sea
(presente de subjuntivo, tercera persona de singular),
es- peró (pretérito perfecto simple de indicativo, tercera
per- sona de singular), regresó (pretérito perfecto simple
de indicativo, tercera persona de singular), conseguí
(pre- té rito perfecto simple de indicativo, primera
persona de singular), encontrar (infinitivo), dijo
(preté rito perfecto simple de indicativo, tercera persona
de singular), lloré (preté rito perfecto simple de
indicativo, primera persona de singular), decidí (pretérito
perfecto simple de indicati- vo, primera persona de
singular), cumplir (infinitivo), he pasado (pretérito
perfecto de indicativo, primera perso- na de singular),
pensando (gerundio), sería (condicional simple de
indicativo, tercera persona de singular), acabé (pretérito
perfecto simple de indicativo, primera persona de
singular), encontrándolo (gerundio), tendió (pretérito
perfecto simple de indicativo, tercera persona de singular).
e) • un largo viaje (Det + Adj + Nú cleo)
• el comerciante (Det + Nú cleo)
• su mujer (Det + Nú cleo)
• un regalo (Det + Nú cleo)
• mi altura (Det + Nú cleo)
• mujer ingrata (Nú cleo + Adj)
• años de trabajo (Nú cleo + CN)
• el hombre (Det + Nú cleo)
• dos años (Det + Nú cleo)
• la mujer (Det + Nú cleo)
• su regalo (Det + Nú cleo)
• tu ingratitud (Det + Nú cleo)
• tu deseo (Det + Nú cleo)
• todo este tiempo (Det + Det + Nú cleo)
• qué regalo (Det + Nú cleo)
• un pequeño espejo (Det + Adj + Nú cleo)
f) Aná lisis morfoló gico:
• durante: preposició n.
• años: sustantivo. • Laú d: instrumento de cuerda con clavijero hacia atrá s y
• la: determinante artículo. caja de resonancia en forma abombada. Se tocaba con
los dedos. Es de origen á rabe.
• mujer: sustantivo.
• esperó: verbo.
• su: determinante posesivo.
• regalo: sustantivo.
• y: conjunció n.
• le: pronombre personal.
• tendió: verbo.
• a: preposició n.
• su: determinante posesivo.
• mujer: sustantivo.
• un: determinante indefinido o numeral.
• pequeño: adjetivo.
• espejo: sustantivo.
Aná lisis sintá ctico:
• Durante dos años la mujer
SPrep-CC SN-Suj
esperó su
regalo.
NV SN-CD
Sintagma nominal sujeto: la mujer (Det + Nú cleo)
Sintagma verbal predicado: durante dos años esperó su
regalo.
• Y le tendió a su mujer
CI NV Sprep-CI
un pequeño
espejo.
SN-CD
Sujeto omitido: él (tercera persona del singular).
Sintagma verbal predicado: y le tendió a su mujer un
pe- queño espejo. Y (conjunció n), le (complemento
indirecto); un pequeño espejo (CD).

MIRA A TU ALREDEDOR Y…
… ve más allá
1. Es imposible desligar la literatura medieval de la
música, ya que casi siempre la recitación de poemas
iba acom- pañada de instrumentos musicales. Si
buscas «Old Music Middle Ages» en YouTube, podrás
disfrutar de unos minu- tos de música medieval,
acompañada además de imáge- nes de cuadros y
miniaturas de la época.
a) ¿Qué te sugiere la música que has escuchado?
b) Investiga cuáles son los principales instrumentos
musi- cales de la Edad Media. ¿Se siguen tocando
en la actua- lidad?
c) Escucha ahora al trovador occitano Peire Raimon
de Tolosa (busca su nombre en YouTube), mientras
canta y toca instrumentos musicales. ¿Crees que
música y poe- sía forman una buena combinación?
¿Qué te transmite?
a) Respuesta libre.
b) En la Edad Media encontramos gran cantidad de
instru- mentos, algunos provendrá n de la Antigü edad
y otros vendrá n de Oriente, introducidos a raíz de las
cruzadas. Vamos a citar algunos de los má s
importantes.
Cordófonos
• Vihuela de arco o fídula: instrumento de arco. Será el a) Investiga por qué se llamaban miniaturas. ¿De dónde
prototipo del violín moderno. procede el término?
• Organistrum o zanfoñ a: es un instrumento de tres cuer-
das que se toca girando una manivela.
• Arpa: de forma triangular y pequeñ o tamañ o. Se toca
con las dos manos, una pulsa las cuerdas y la otra
acor- ta su longitud.
Viento
• Platerspiel: es una especie de gaita con forma de cuer-
no. Está hecho de madera y tiene seis agujeros.
• Chirimía: especie de oboe de mayores dimensiones muy
usado en los desfiles y actos relevantes.
• Cornamusa: parecido a las gaitas actuales.
• Flauta.
• Corneta.
Percusión
• Címbalos: compuesto por un arco de madera y una o
dos membranas tensas. Se percutía con la mano.
• Tejoletas: rudimentarias castañ uelas hechas con trozos
de teja que se entrechocan.
• Darbuka: tambor con forma de copa y una membrana
tensa en la parte superior. Normalmente está hecho
de cerá mica y se golpea con las manos.
• Carrilló n.
(Fuente: musicaedadmedia.com).
Algunos de ellos se siguen empleando en la actualidad en
sus formas evolucionadas (arpa, flauta, corneta, carri-
lló n…). Otros solo se tocan en conciertos que recrean la
mú sica medieval.
c) Respuesta libre.
2. La figura del Cid Campeador ha despertado gran inte-
rés a lo largo de la historia, no solo en la literatura, sino
también en el cine. En 1961 se produjo una gran película,
dirigida por Anthony Mann y con Charlton Heston en el
papel del Cid.
Busca en YouTube el fragmento en el cual Rodrigo Díaz
de Vivar se despide de su esposa, doña Jimena: teclea
«El Cid 1961 (español) 12/18». ¿Te imaginabas así la
ambientación y a los personajes? ¿Por qué deben
separarse?
a) Después de despedirse, el Cid comparece ante el rey.
¿Cómo se llama el monarca? ¿Sobre qué discuten?
b) A continuación, el Campeador se dirige a un
monas- terio. ¿Con quién se reencuentra allí?
¿Cómo se lla- maban las hijas del Cid en el Cantar?
¿Y en realidad?
a) El monarca es Alfonso VI (aunque no se menciona su nom-
bre) y discuten sobre la necesidad de tomar Valencia antes
de enfrentarse a Ben Yusuf. Finalmente, se obliga a don
Rodrigo combatir en Sagrajas si quiere ser indultado.
b) En el monasterio el Cid se reencuentra con su mujer y
sus hijas. Las hijas en el Cantar se llaman doñ a Elvira y
doñ a Sol, pero en realidad eran María y Cristina.
3. Los códices y manuscritos medievales eran
frecuentemente decorados en sus márgenes con
miniaturas: pequeños dibujos ornamentales de gran
colorido, que representa- ban escenas propias de la
época. Aquí tienes como ejemplo una miniatura de Le
roman de la rose, un libro didáctico del siglo xııı, y la letra
capitular V de un cantoral gregoriano.
b) Un tipo muy especial de miniaturas son los beatos. • Los temas más representados fueron las escenas
Bus- ca información sobre ellos. ¿Por qué se llaman de lucha entre hombres y animales, y las faenas
así? ¿De qué siglo datan? ¿Qué representaban? coti- dianas.
Aquí tienes algunas imágenes de uno de los más • El arte medieval servía para simbolizar la lucha en-
im- portantes, el Beato de Liébana. tre el bien y el mal, y para disuadir a los hombres
a) Se llamaban miniaturas porque se elaboraban con de pecar.
minio, ó xido de plomo de color rojo que se utilizaba
como com- ponente fundamental para fabricar la
tinta.
b) Se trata de có dices manuscritos, compuestos entre los
siglos X al XII. Se llaman así porque los má s importantes
fueron creados por Beato de Liébana, un monje del
monas- terio de san Martín de Turieno (actualmente
Toribio), en el valle de Liébana. Contienen comentarios
al Apocalipsis de san Juan, ilustrados con hermosas
miniaturas.

… encuentra la clave
1.Aparte de la literatura, el arte también fue un
instrumento clave para adoctrinar al pueblo durante la
Edad Media. Lee el siguiente texto y resuelve las
actividades que se te plan- tean a continuación:
Instruir a los fieles era el principal propó sito de las
imá ge- nes. Pintura y escultura, normalmente coloreadas,
fueron fó r- mulas primordiales para difundir el mensaje
divino; aquello que la literatura daba a conocer con
palabras, la decoració n del templo debía mostrarlo
mediante imá genes que permitie- ran visualizar los
acontecimientos bíblicos, ú nica forma, junto con la
predicació n oral, de acceso al conocimiento que tenía la
inmensa mayoría de los devotos.
El arte debía enseñ ar y emocionar mediante la sencillez,
la claridad y la expresividad de sus imá genes, las cuales
hacían visible al ser humano aquello que era invisible.
[…]
Los motivos má s representados fueron los que narraban
histo- rias del Antiguo Testamento, los Evangelios o el
Apocalipsis.
Les siguen los que no pretenden contar, sino hacer visible
un universo simbó lico: el que se quiere transmitir a
travé s de las luchas entre hombres y animales, la
representació n de anima- les rampantes enfrentados, o
imá genes del bestiario —arpías, sirenas, basiliscos, grifos,
quimeras—. Se trata de figuras car- gadas de un fuerte
simbolismo que apunta a la dicotomía entre el bien y el mal,
al horror del pecado, a las visiones infernales y
demoniacas, etc.
[…] También se reproduce el mundo cotidiano, amable y
cerca- no de las faenas agrícolas o los oficios, los rostros
familiares y las costumbres, sucesos lú dicos, la fauna local
o los animales domésticos.
VV. AA.: Historia del arte de la antigua Edad
Media
(Editorial Universitaria Ramó n
Areces).
a) Resume el texto que acabas de leer.
b) Di si las siguientes afirmaciones son verdaderas o
falsas:
• La única forma que tenían los fieles de acceder al
mensaje divino era mediante la pintura y la escultura.
• Las imágenes únicamente servían para narrar, en-
señar y transmitir cultura.
• Para cumplir con su propósito era importante
que
las imágenes fueran claras, sencillas y expresivas.
c) Investiga acerca de los principales animales fantásti- • basilisco: animal fabuloso, al cual se atribuía la propie-
cos del bestiario medieval: arpías, basiliscos, grifos y dad de matar con la vista.
quimeras. ¿Cómo eran y qué simbolizaban?
• grifo: animal fabuloso, con la mitad superior del cuerpo
d) Observa estas imágenes y di si se corresponden con de un á guila, y la mitad inferior de un leó n.
historias de la Biblia, del bestiario medieval o de fae-
nas agrícolas y cotidianas. ¿Qué es lo que representa • quimera: monstruo imaginario que, segú n la fá bula, vo-
cada una? mitaba llamas y tenía cabeza de leó n, vientre de cabra
y cola de dragó n.
a) Respuesta libre.
b) • Falso, otra forma era la predicació n oral. Todos ellos eran seres fantá sticos y monstruosos que sim-
• Falso, tambié n debían emocionar. bolizaban el mal y el pecado.
• Verdadero. d) La primera representa una faena cotidiana, agrícola y
• Falso, los motivos má s representados fueron los ganadera.
bíblicos. La segunda pertenece al bestiario medieval, en ella obser-
• Verdadero. vamos lo que parecen quimeras con cabeza de leó n.
c) • arpía: ave fabulosa, con rostro de mujer y cuerpo de
En la tercera aparece una imagen bíblica, de la vida de
ave de rapiñ a. Jesucristo, concretamente es la huida a Egipto.
MELIBEO SOY… 8
APERTURA DE UNIDAD considerada una especie de divinidad, a la que el
enamorado debía
1. En el poema dialogan dos personajes. ¿Quiénes son?
¿Crees que se trata de una composición perteneciente al
género lírico o al narrativo? Razona tu respuesta.
Dialogan un hombre y una mujer que han tenido relaciones
amorosas. Pertenece al gé nero narrativo ya que a través del
diá logo de los personajes conocemos su historia, y lo que les
ha ocurrido a ambos.
2. ¿Quién es rosa fresca y por qué rechaza a su amigo?
¿Cuál dirías entonces que es el tema del poema?
Rosa fresca es la mujer que rechaza a su amigo porque se ha
enterado de que este está casado y tiene hijos en tierras de
Leó n. El tema del poema sería, pues, el desengañ o amoroso
de esta mujer al enterarse de la verdad sobre su amante.
3. Escucha al compositor y cantautor español Amancio Pra-
da cantando este romance: teclea en YouTube «Amancio
Prada» «Romance de la rosa fresca». A continuación, es-
cribe en tu cuaderno qué sensaciones te ha generado la
audición. ¿Qué crees que le aporta la música al
romance?
¿Piensas que en el siglo xv los romances se cantaban
de este modo?
Respuesta libre.
4. Los romances son composiciones poéticas típicas del si-
glo xv. Uno de sus temas más importantes es el amor,
como en el que acabas de leer. Pero también son muy
frecuentes los romances que cuentan acontecimientos
re- lacionados con la Reconquista y las luchas entre
cristianos y árabes. Escucha ahora el Romance de la
mora cautiva, tecleando en YouTube «Amancio Prada»
«Romance de la hermana cautiva». ¿Cuál es su tema?
¿Qué personajes aparecen en él? Explica oralmente su
argumento.
En este romance aparecen un caballero y una mora cautiva.
El caballero se lleva a la mora de vuelta a tierras cristianas, y
cuando van llegando, debido a los comentarios hace la joven
acerca de los lugares por los que pasa, se da cuenta de que
en realidad es una hermana suya que se llevaron los moros
cuando era niñ a.

VIVE LA LECTURA
Comprensión lectora
1. Esta escena recoge el momento en el que Calisto y Meli-
bea se ven por vez primera. ¿Con qué compara Calisto el
sentimiento que tiene al contemplar a Melibea?
Lo compara con contemplar una visió n divina.
2. Melibea, al principio, parece darle esperanzas a Calisto.
¿Dónde se manifiesta esto?
Cuando le dice que aú n mayor galardó n le dará ella, si per-
severa.
3. Sin embargo, después, se muestra enojada con él. ¿Qué
le dice? ¿A qué crees tú que se debe este cambio de ac-
titud?
Se enfada con él y le dice que la paga será tan fiera como
merece su atrevimiento, se escandaliza de que quiera hacerla
perder su virtud y lo echa de su lado sin miramientos. Posi-
blemente se enoje tanto porque Calisto está siendo demasiado
osado en sus declaraciones, no está respetando las reglas del
amor corté s.
4. Quizá te ayude conocer cuáles eran las convenciones en
las que se basaba el amor cortés: la mujer era

Lengua castellana y Literatura. 3.º ESO. Solucionario 69


8 MELIBEO SOY…
rendir vasallaje durante un largo tiempo antes de recibir
de ella un galardón que mostrara su aceptación.
¿Respeta Calisto estas convenciones?
No, Calisto es un enamorado impaciente que quiere recibir su
recompensa demasiado rá pido; esto enoja a Melibea.
5. Los sentimientos hacia Melibea que Calisto expresa a su
criado Sempronio son muy exagerados. ¿Qué es lo que
desea? ¿Por qué esto se opone a ser un buen cristiano?
Desea la muerte, y estos sentimientos son contrarios a la re-
ligió n cristiana.

Vocabulario
6. La palabra acatamiento procede de catar («mirar»).
¿Qué significado tiene el verbo acatar en la actualidad?
Escribe una frase donde se ejemplifique.
Acatar en la actualidad significa obedecer, aceptar con sumi-
sión una autoridad o normas. Ejemplo: Los cristianos deben
acatar los mandamientos de la Iglesia.
7. La palabra galardón es un germanismo que significa «re-
compensa a los servicios prestados». El galardón tenía
una gran importancia en el amor cortés. Investiga por
qué y en qué consistía.
El galardó n en el amor cortés era una prenda (por ejemplo,
un pañ uelo de seda o una cinta) que la mujer entregaba a su
enamorado para mostrarle que aceptaba su vasallaje y lo
re- conocía como siervo (en el sentido amoroso) suyo.
8. Las palabras inmérito, incomparablemente,
indignamente, desventuradas, ilícito y desdichado
tienen en común el estar formadas con un prefijo que
les aporta un deter- minado significado. ¿Cuál es?
Construye una frase con cada una de ellas.
En todas ellas el prefijo significa negació n, sin (sin mérito,
sin comparación, sin dignidad, sin ventura, que no es lícito,
sin dicha).
Frases: respuesta libre.
9. En el diálogo que mantiene Calisto con Melibea, este
emplea un gran número de términos relacionados
con la religión y lo divino. Señálalos. ¿Por qué piensas tú
que se vale de este tipo de palabras? Piensa de nuevo
en la relación de la obra con el amor cortés.
Emplea palabras y expresiones relacionadas con la religió n
como la grandeza de Dios, obras pías, tengo a Dios ofreci-
do, cuerpo glorificado, los gloriosos santos, visión divina, si
Dios me diese en el cielo la silla sobre sus santos.
Recordemos que en el amor corté s la amada era como una
especie de dios, al que el enamorado debía rendir culto y
devoció n.
10. En la actualidad, decimos de alguien que es una
celestina cuando se dedica a intentar unir a dos
enamorados. Este procedimiento, que consiste en
poner el nombre apelati- vo por el propio (o viceversa),
se denomina antonomasia. Así, decimos, por ejemplo,
que una persona que lucha por sus ideales sin tener en
cuenta la realidad es un quijote. Busca otros cinco
ejemplos del mismo fenómeno. No tie- nen por qué ser
únicamente ejemplos literarios, también pueden ser
inventores cuyo nombre sirve para designar el invento
creado por ellos.
Por ejemplo, ser un nerón (despiadado y cruel), hacer de
cice- rone (de guía o maestro), ser un hércules (hombre muy
fuerte), ser un atila (destruir todo lo que se te pone por
delante), ser un judas (un traidor).

70 Lengua castellana y Literatura. 3.º ESO. Solucionario


Investigación y redacción la religió n de sus sú bditos, por lo cual en 1492 se procedió a
11. Muchos de los nombres de los personajes de La expulsar a los judíos. Ya en 1478 se había creado la Inquisi-
Celes- tina no están escogidos al azar, sino que tienen ció n para perseguir a los cristianos nuevos que volvían a sus
algo que ver con las características del personaje al que antiguas creencias. En 1504 fallecía la reina Isabel y aunque
dejaba como regente de la heredera al trono, Juana I, a su
designan. Investiga qué significado tienen los nombres
marido Fernando el Cató lico, la nobleza castellana no lo apo-
de los ena- morados Calisto y Melibea, del criado yó por lo que este marchó a sus estados de Aragó n. De este
Sempronio y del padre de Melibea, Pleberio. modo quedaba encargado del gobierno de Castilla Felipe de
Calisto significa el más bello, Melibea quiere decir dulce como Austria, el Hermoso, esposo de la reina Juana I de Castilla,
la miel, Sempronio viene de siempre (lo cual es iró nico porque la Loca. Pero la muerte de Felipe en 1506 obligó a restituir
se trata del criado que traiciona a su señ or) y Pleberio hace a Fernando, llamado por el Cardenal Cisneros a Castilla en
referencia a la plebe (tambié n iró nicamente, ya que Pleberio 1507. Los ú ltimos añ os de su reinado se caracterizaron por los
pertenece a la nobleza). enfrentamientos con Francia en terreno italiano. A la muerte
de Fernando el Cató lico heredó el trono su nieto Carlos I de
12. En 1996 Gerardo Vera llevó la obra al cine. El resultado
Españ a.
fue una película muy bien ambientada, que refleja con
bastante fidelidad la vida de la época. Ved en clase un (Fuente: www.artehistoria.com).
resumen, al que podéis acceder tecleando «mi película 2. Ve este fragmento de la serie televisiva Isabel, tecleando
Celestina» en un buscador. Después, anota en tu en YouTube «Ya es reina Isabel», donde asistirás al
cuaderno qué cosas te han llamado la atención en momen- to de su coronación como reina de Castilla.
cuanto a la caracterización de los personajes, el vestuario, Fíjate bien en cómo está recreada la época histórica.
la ambientación, etc. Isabel cumplió un importantísimo papel para la historia
Respuesta libre. de Castilla. ¿Crees que era lo normal en la época,
tratándose de una mujer?
LITERATURA Respuesta libre.
3. Investiga y escribe en tu cuaderno cuál es el argumento
1. Prepara una presentación en Prezi o PowerPoint con al-
de la novela sentimental Cárcel de amor.
gunos de los datos más relevantes del reinado de los Re-
El autor, que aparece como un personaje en la obra, se ex-
yes Católicos. Recoge información sobre sus problemas
travía en Sierra Morena, y se encuentra con un joven enca-
para acceder al trono, las luchas contra los musulmanes, denado; este joven es Leriano, hijo del duque Guersio, que
su apoyo a la empresa de Cristóbal Colón, su política de es llevado a la prisió n del Amor por un monstruo llamado
alianzas matrimoniales a través de sus hijos o qué ocurrió Deseo. Siguié ndoles, el autor entra en un castillo en la mon-
con la sucesión al trono cuando ellos murieron. tañ a, donde Leriano le confiesa su pasió n por Laureola, hija
El alumno podrá recoger en su presentació n, entre otros, los del rey Gaulo de Macedonia, y le ruega que actú e como inter-
siguientes datos: mediario. El autor va en busca de Laureola y, tras hablarle
Isabel, hija de Juan II de Castilla y de Isabel de Portugal, y de Leriano y del dolor que padece, logra ablandar su corazó n
Fernando, hijo de Juan II de Aragó n y de Juana Enríquez, y consigue que la muchacha le escriba una carta. Con la
contrajeron matrimonio en Valladolid el 19 de octubre de ayuda de una serie de personajes alegó ricos (la Esperanza,
1469, entre fuertes oposiciones al mismo. Isabel heredaría la Satisfacció n, la Tranquilidad, la Alegría…) libera de su
el trono de Castilla en 1474 despué s de la muerte de su her- cautiverio a Leriano, que se dirige a Macedonia para ver a
mano Enrique IV, autoproclamá ndose reina, ya que había un Laureola, quien le recibe con agrado. Pero otro personaje
conflicto sucesorio entre ella y Juana, hija de Enrique IV, de que tambié n pretende a Laureola, el malvado Persio, hijo
la que se decía era hija de Beltrá n de la Cueva y no del rey. del señ or de Gavia, difunde rumores que ponen en peligro
Mientras tanto Fernando era nombrado heredero a la muerte el honor de la doncella, de modo que Leriano lo desafía, ven-
de su hermano Carlos. En 1468 recibió el trono de Sicilia y ciéndole en dos ocasiones. La victoria de Leriano sobre Per-
a la muerte de su padre en 1479, el de la corona de Aragó n. sio no soluciona la situació n, porque el rey Gaulo, padre de
Participó en las luchas a favor de su esposa Isabel y a par- Laureola, ha dado crédito a los falsos testimonios y conde-
tir de esta fecha se produjo la unió n diná stica de Aragó n y na a muerte a su hija. Leriano y sus partidarios asaltan la
Castilla y el comienzo del reinado conjunto. Este matrimonio prisió n donde Laureola aguarda la ejecució n y la liberan,
ha sido considerado como el punto de partida de la unidad enfrentá ndose a todo el ejé rcito real en una cruel batalla. En
y de la grandeza de Españ a. El primer objetivo de los nue- la lucha, capturan a uno de los calumniadores, que admite
vos monarcas fue el de restablecer la autoridad real, para su culpa, con lo que el rey perdona a su hija. Laureola, preo-
lo cual se sirvieron de una poderosa organizació n: la Santa cupada por defender su honor y no levantar má s sospechas,
Hermandad. Tambié n constituyeron el Consejo Real que sus- rechaza definitivamente a Leriano. Este, echa en una copa
tituía a las Cortes y nombraron corregidores para controlar las cartas de Laureola, que ha roto, se bebe su contenido
las ciudades y vincularon la direcció n de la Mesta al Consejo y se deja morir de inanició n, mientras su madre entona un
Real. De este modo quedaba controlada la política del reino, desesperado planto.
aunque estas medidas pesaron má s sobre el reino de Castilla 4. Lee el siguiente fragmento de las Danzas de la Muerte y
que sobre el de Aragó n. La siguiente misió n era concluir la realiza las actividades que se te plantean:
Reconquista en el reino nazarí de Granada, lo que consiguie-
MUERTE.— ¡Oh, cuá n sin acuerdo de mí, y sin temor
ron en 1492. La paz interior y la buena organizació n del reino
yaces en vicios terrenos jatando1,
permitieron que las arcas reales se llenaran y con ellas se
la gloria pasible de acá procurando,
acometieran nuevas empresas como el apoyo al almirante
soberbia mostrando por ser gran señ or;
genové s Cristó bal Coló n, que descubriría Amé rica en 1492,
en quien la humildad, segú n que a Pastor,
aportando riquezas para el reino y un fuerte expansionismo
había de ser grande ejemplo al ganado!
exterior. El éxito de la guerra antimusulmana y la presió n
Y, pues fue al revé s, irá s muy priado2
de los confesores de la reina indujeron a los Reyes a unificar
conmigo a do cuenta dará s de tu error.
PAPA.— ¡Oh, muerte!, no vengas con tanto
furor; aplaca tu ira; ten má s sufrimiento: 7. Relee el Romance de la rosa fresca (página 147). Señala
mira que es grande mi merescimiento en él las características típicas del género.
de muy alta estima mi estado y valor; En el Romance de la rosa fresca aparecen también caracte-
no muestres conmigo tan grande rigor, rísticas como la concisió n y el fragmentarismo (se nos pre-
que tengo en la tierra muy gran señ orío. senta la historia en su mitad y no sabemos có mo concluye),
MUERTE.— Muy poco te excusa tan gran la expresividad, la sencillez, el uso de diá logos para dar
desvarío el golpe mortal de mi pasador. mayor dramatismo al texto, así como algunas repeticiones,
Sin má s resistencia sabrá s, sin mentir, aná foras y paralelismos.
aunque tu estado a todos hoy sobre, 8. Lee la siguiente serranilla del marqués de Santillana y
muy breve será s igual con el pobre, realiza las actividades:
en solo este paso que llaman morir.
La vaquera de Morana
Juan DE PEDRAZA
En toda la su montana
de Trasmoz a Berató n
1 2
non vi tan gentil serrana.
jatando: jactando, presumiendo. priado: presto, rápido. Partiendo de Conejares,
allá suso en la montañ a,
cerca de la Travesañ a,
a) ¿De qué acusa la Muerte al Papa? ¿Qué le responde
camino de Trasobares,
este? ¿Le sirven de algo sus protestas? encontré moza lozana
b) En cervantesvirtual.com podrás leer esta danza de poco má s acá de Añ ó n,
la Muerte completa. ¿Qué moraleja se desprende del riberas d’una fontana.
poema? […]
Dije: «Dios te salve, hermana;
a) Le acusa de haberse dejado llevar por los placeres terre-
aunque vengas d’Aragó n,
nales, así como de soberbia. El Papa le responde que su
d’esta será s castellana».
estado y valor son grandes, así como su señ orío, pero no
Respondiome: «Caballero,
le sirve de nada porque la Muerte no se apiada de él.
non penséis que me tenedes,
b) Que todos somos iguales ante la muerte, ricos y pobres, ca primero probaredes
poderosos y humildes, todos habremos de morir algú n día. este mi dardo pedrero;
ca despué s d’esta semana
5. Lee el siguiente romance e indica qué características pro-
fago bodas con Antó n,
pias del género observas en él. Luego, señala cuál es su vaquerizo de Morana».
tema y realiza el análisis métrico.
Biblioteca Virtual MIGUEL DE CERVANTES
Que por mayo era por mayo,
cuando hace la calor, a) Busca en el diccionario el significado de las palabras
cuando los trigos encañ an lozana, fontana y guisa.
y está n los campos en flor, b) Investiga dónde se encuentran los lugares menciona-
cuando canta la calandria dos en el poema: Trasmoz, Beratón, Conejares, Traso-
y responde el ruiseñ or, bares, Añón y Morana.
cuando los enamorados
van a servir al amor, c) ¿Qué es lo que pretende hacer el hombre con la
sino yo, triste, cuitado, serra- na? ¿Qué le responde esta?
que vivo en esta prisió n, a) • lozana: fresca, garrida.
que ni sé cuá ndo es de día • fontana: fuente.
ni cuá ndo las noches son,
• guisa: modo, manera o semejanza de algo.
sino por una avecilla
que me cantaba al albor: b) • Trasmoz: provincia de Zaragoza.
mató mela un ballestero; • Berató n: provincia de Soria.
¡dele Dios mal galardó n! • Conejares: provincia de Soria.
Observamos en el romance características propias de su • Trasobares: provincia de Zaragoza.
género, como la concisió n y la fragmentació n (no sabemos • Añ ó n: provincia de Zaragoza.
nada del antes ni del despué s del prisionero), ademá s es
muy expresivo (a ello contribuye el uso de la primera • Morana: provincia de Pontevedra.
persona), y abundan en é l las repeticiones y los c) El hombre pretende llevá rsela a Castilla. Ella le responde
paralelismos, que favore- cen su memorizació n. que antes probará uno de sus dardos, ya que está compro-
Su tema sería el contraste entre la primavera que llega al resto metida con un vaquerizo de Morana.
del mundo, y el estado del prisionero, que vive encerrado y ha 9. Lee atentamente las anteriores coplas y responde:
perdido su ú nica conexió n con el exterior: un pajarillo que le a) ¿Cuál es el tema de cada una de ellas?
cantaba cada mañ ana y ha muerto en manos de un
ballestero. b) Analiza métricamente la primera copla. ¿Cuáles son
Formalmente, está compuesto por versos octosílabos, que ri-
los pies quebrados?
man en asonante en los pares, quedando sueltos los impares. c) La copla III se basa en una metáfora. ¿Cuál es el tér-
6. Teclea en YouTube «Paco Ibáñez» «Romance del mino real y cuál es el término imaginario?
prisione- ro» y escucha una versión cantada. d) Explica cómo se manifiesta el tópico del ubi sunt? en
Audició n. la copla XVII.
a) La copla I nos habla de la rapidez con que se pasa la vida de la metá fora, donde el término real es la vida y el imaginario
y nos llega la muerte; la III de có mo todos somos iguales dicho camino que nos lleva a la muerte.
ante la muerte; y la XVII desarrolla el tó pico del ubi sunt?,
el autor se pregunta qué fue de las damas, los trovadores
y la vida de la corte.
b) La copla está formada por dos sextillas, con la siguien-
te disposició n: versos octosílabos y tetrasílabos, con
estructura 8a8b4c, 8a8b4c, 8d8e4f, 8d8e4f. La rima es
consonante.
Los pies quebrados son los versos tetrasílabos, que son
má s cortos que el resto.
c) Está identificando nuestras vidas (té rmino real) con ríos
(término imaginario), que van a dar a la mar (término ima-
ginario), que es el morir (té rmino real).
d) El autor se va preguntando qué se hizo de las damas de la
corte, de sus amadores, de su trovar y danzar…, para que
reflexionemos có mo todos ellos han muerto y ese mundo
de la corte ha desaparecido para siempre.
10. Investiga acerca del festival celestinesco que se
celebra cada año en La Puebla de Montalbán y expón
dicha infor- mación ante tus compañeros.
El festival de La Celestina de La Puebla de Montalbá n se em-
pezó a celebrar en el verano de 1999, con motivo del V centena-
rio de la publicació n de la Comedia. Se celebra todos los añ os
en el mes de agosto, los dos ú ltimos fines de semana de
dicho mes. En é l se realizan diversas actividades
relacionadas con La Celestina y la vida medieval: mercado
de época, actuacio- nes teatrales en las cuevas y, sobre
todo, la representació n de la obra que tiene lugar en la plaza
mayor de la localidad, y cuya escena final del suicidio de
Melibea se realiza en la torre de san Miguel. Todo el pueblo
participa y se implica muchísi- mo en esta celebració n.
Ademá s los visitantes pueden acudir al museo celestinesco de
La Puebla, donde está n expuestos los cuadros con los que el
pueblano Teo Puebla ilustró la edició n conmemorativa del
centenario.

COMENTARIO DE TEXTO
Localización. Estas tres coplas pertenecen a diferentes blo-
ques temáticos.
1.¿A qué bloque pertenece cada una? Justifica tu
respuesta. Tanto la copla nú mero V como la VIII pertenecen
a la primera parte de la obra: reflexió n general sobre la vida
y la fugacidad
de las cosas terrenales. La copla XXV, en cambio, pertenece
a la tercera y ú ltima parte de la obra, en la que Manrique se
centra en la figura de su padre y lo elogia como ideal de
caba- llero y ejemplo del saber morir cristianamente.
2. ¿En qué expresiones se refleja claramente la mentalidad
medieval?
Se observa dicha mentalidad medieval en la religiosidad de
la aceptació n de la muerte, al considerar, por ejemplo, que
este mundo es el camino para el otro, el de la vida eterna, o
que cuando morimos descansamos. Así mismo lo
observamos en el desprecio que hace de las cosas terrenales
(la belleza, la ju- ventud, la fuerza). Todo eso no importa
porque es perecedero, lo importante es preparar el alma para
la vida eterna.
Tema y estructura. Las coplas V y VIII comparten el mismo
tema.
3. ¿Cuál es el tema de la copla V? ¿En qué recurso literario
se sustenta?
El tema de la copla V sería có mo nuestra vida es un camino, y
al llegar a nuestra meta, morimos. Está basado en el recurso
4. Respecto a la copla VIII, ¿mantiene el mismo tono que la
anterior? ¿En qué se diferencia?
La copla VIII pretende hacernos reflexionar sobre có mo la be-
lleza y la fuerza de la juventud desaparecen al llegar la
vejez, y lo hace mediante preguntas retó ricas.
5. La copa XXV trata otro tema. ¿Qué cualidades del padre
se destacan?
En la copla XXV Manrique elogia las virtudes de su padre,
don Rodrigo, destacando especialmente có mo este era amado
por todos, virtuoso, valiente, famoso…
Análisis de la forma y el contenido. El poema está escrito en
coplas de pie quebrado o manriqueñas.
6. Cuenta los versos y señala cuál es su medida.
Se trata de coplas agrupadas en dos sextillas: conjunto de
seis versos con la siguiente estructura 8a8b4c, 8a8b4c.
7. Observa la rima. ¿Es asonante o consonante? ¿Cómo
riman los versos?
Se trata de rima consonante abc, abc.
8. En el texto abundan los sustantivos. Señálalos e indica a
qué campos semánticos pertenecen. Señala también los
adjetivos que los complementan.
Sustantivos que hacen referencia a la vida como camino: mun-
do, camino, morada, tino, jornada.
Aquellos que se refieren a las características de la juventud:
hermosura, frescura, tez, cara, color, blancura, mañas, li-
gereza, fuerza, juventud.
Sustantivos relativos a la vejez: vejez, graveza, arrabal,
senectud.
Sustantivos que hacen referencia a su padre: abrigo, gente,
maestre, Rodrigo Manrique, hechos, mundo.
No abundan los adjetivos, pero sí aparecen algunos acom-
pañ ando a ciertos sustantivos: buen tino, gentil frescura,
fuerza corporal, hechos grandes e claros.
9. Anota los recursos literarios que aparezcan y explica el
por- qué de su uso.
No abundan tampoco los recursos literarios, aunque sí ob-
servamos una metá fora en la copla V, que identifica
nuestras vidas con un camino, la duració n de la vida con
una jornada y el fin del camino con la muerte.
En la copla VIII aparecen preguntas retó ricas para
hacer- nos reflexionar acerca de có mo desaparece la
juventud y la belleza.
Conclusiones. Estas coplas nos muestran claramente la fina-
lidad de la obra.
10. ¿Cuál es esa finalidad?
La finalidad de la obra es doble: por un lado hacernos reflexio-
nar sobre có mo se pasa de rá pido la vida y de la necesidad de
despreciar las cosas terrenales y prepararnos para la vida que
de verdad importa: la vida eterna. Por otro lado pretende re-
saltar las virtudes del padre muerto y presentarlo como
ideal de caballero cristiano, así como un ejemplo en su
aceptació n de la muerte.
11. ¿Qué tipo de mentalidad se refleja en las coplas?
La mentalidad es claramente medieval, y se observa por
ejem- plo en dicho desprecio por las cosas terrenales, en la
importan- cia que se le concede a la religió n y a la
preparació n para la vida eterna, en la defensa de ciertos
valores y virtudes cristianas y caballerescas… Sin embargo,
tiene un punto de modernidad al presentar una tercera vida
(junto con la terrenal y la eterna), la vida de la fama, el hecho
de que nuestra fama perdurará tras nuestra muerte si
nuestros actos han sido dignos de ello.
EL JARDÍN DE LA LITERATURA Dice que es labrandera, perfumera, maestra de hacer aceites
y virgos, alcahueta y hechicera.
1. Lee el siguiente fragmento del acto I de La Celestina,
en el que Calisto describe a Melibea. Copia en tu
cuaderno los rasgos físicos con los que la describe y
busca informa- ción sobre cómo era el canon de belleza
femenina de la época. ¿Responde a la descripción que
hace Calisto de Melibea? ¿Hay en el texto algún
recurso poético?
CALISTO.— Comienzo por los cabellos. ¿Vees tú las madejas del
oro delgado que hilan en Arabia? Má s lindos son, y no
res- plandecen menos. Su longura hasta el postrero
asiento de sus pies, después, crinados y atados con la
delgada cuerda, como ella se los pone, no ha má s
menester para convertir los hombres en piedras. […]
Los ojos verdes, rasgados; las pestañ as, luengas; las cejas,
delgadas y alzadas; la nariz, mediana; la boca, pequeñ a;
los dientes, menudos y blancos; los labios, colorados y gro-
sezuelos; el torno del rostro, poco más luengo que redondo;
el pecho, alto; la redondeza y forma de las pequeñ as
tetas,
¿quié n te la podrá figurar? ¡Que se despereza el hombre
cuando las mira! La tez lisa, lustrosa; el cuero suyo escure-
ce la nieve; la color, mezclada, cual ella la escogió para sí.
Editorial ANAYA
Cabellos como el oro, lindos y resplandecientes, largos y ata-
dos en una coleta. Ojos verdes y rasgados, pestañas largas,
cejas delgadas y altas, nariz mediana, boca pequeña,
dientes pequeños y blancos, labios gruesos y colorados,
rostro alar- gado, pecho redondo, tez lustrosa y blanca.
Esta descripció n sí corresponde con el ideal de belleza de la
época, en el que las mujeres eran rubias, de largos cabellos,
bocas sonrosadas y piel muy clara, así como bien proporcio-
nadas de cuerpo.
Aparecen algunos recursos poé ticos, por ejemplo un símil
cuando compara sus cabellos con las madejas de oro que hilan
en Arabia, o cuando dice que su piel oscurece la nieve. Son,
ademá s, sendas exageraciones o hipé rboles.
2. Lee ahora la descripción que hace Pármeno de Celestina
y sus oficios. Este la conoce bien porque su madre fue
com- pañera suya y él mismo trabajó para ella de niño.
Teniendo en cuenta cómo la describe, ¿crees que le
tiene aprecio?
¿Por qué?
PÁ RMENO.— Si entre cien mujeres va y alguno dice: «¡Puta vie-
ja!», sin ningú n empacho luego vuelve la cabeza y
respon- de con alegre cara. En los convites, en las fiestas,
en las bodas, en las cofradías, en los mortuorios, en todos
los ayuntamientos de gentes, con ella pasan tiempo. Si
pasa por los perros, aquello suena su ladrido; si está
cerca las aves, otra cosa no cantan; si cerca los
ganados, balando lo pregonan; si cerca las bestias,
rebuznando dicen «¡Puta vieja!». […]
Ella tenía seis oficios, conviene a saber: labrandera, per-
fumera, maestra de hacer afeites y de hacer virgos, alca-
hueta y un poquito hechicera. […] ¿Quién te podrá decir lo
que esta vieja hacía? Y todo era burla y mentira.
Editorial ANAYA
No parece demostrarle mucho aprecio, ya que insiste en cali-
ficarla de puta, diciendo ademá s que esto a ella no le importa.
Cuando da cuenta de todos sus oficios termina diciendo y todo
era burla y mentira. Con todo esto entendemos que Pá rmeno
no se fía en absoluto de ella.
3. Copia en tu cuaderno todos los oficios de Celestina.
4. En el acto X, Celestina consigue que finalmente Me- sucede a la mora Moraima después de abrir las
libea confiese su amor por Calisto, gracias a sus grandes puertas?
dotes dialécticas. Léelo y señala cómo va e) Indica qué elementos típicos de los romances se dan
convenciendo a Melibea hasta que esta lo reconoce. en este. Analiza también su métrica.
CELESTINA.— Tu llaga es grande, tiene necesidad de á spera
cura. […] No concibas odio ni desamor, ni consientas a
tu lengua decir mal de persona tan virtuosa como
Calisto, que si conocido fuese…
MELIBEA.— ¡Oh por Dios, que me matas! ¿Y no te tengo dicho
que no me alabes ese hombre ni le nombres en bueno ni
malo?
CELESTINA.— Señ ora, este es otro y segundo punto: si tú
con tu mal sofrimiento no consientes, poco aprovechará
mi venida; y si, como prometiste, lo sufres, tú quedará s
sana y sin deuda, y Calisto sin queja y pagado. […]
MELIBEA.— Má s agradable me sería que rasgases mis
carnes y sacases mi corazó n, que no traer esas palabras
aquí.
CELESTINA.— Sin te romper las vestiduras se lanzó en tu
pecho el amor; no rasgaré yo tus carnes para le curar.
MELIBEA.— ¿Có mo dices que llaman a este mi dolor, que así
se ha enseñ oreado en lo mejor de mi cuerpo?
CELESTINA.— Amor dulce.
MELIBEA.— Eso me declara qué es, que en solo oírlo me alegro.
CELESTINA.— Es un fuego escondido, una agradable llaga, un
sabroso veneno, una dulce amargura, una delectable do-
lencia, un alegre tormento, una dulce y fiera herida, una
blanda muerte.
Editorial ANAYA
Primero le dice que su llaga necesita cura, y alaba a Calisto.
Cuando Melibea se enfada al oír nombrar a Calisto, Celesti-
na le asegura que para curarse necesita algo de
sufrimien- to. A continuació n da nombre a lo que siente
Melibea: amor, para terminar describié ndolo con una serie
de metá foras y contradicciones.
5. ¿De qué recurso poético se vale Celestina para describir
el amor?
Se vale de dos recursos, por un lado la metá fora, ya que va
identificando el amor con diferentes cosas: fuego
escondido, agradable llaga, sabroso veneno, etc. Por otro se
basa en las antítesis o contradicciones: la llaga es agradable,
el veneno sabroso, la amargura dulce, la dolencia delectable,
el tormento alegre, la herida es dulce y fiera y la muerte es
blanda.

LA FACTORÍA DE TEXTOS
1.Lee los primeros versos de este romance, busca su
conti- nuación en Internet y responde a las cuestiones
siguientes:
Yo me era mora
Moraima, morilla de un
bel catar; cristiano vino a
mi puerta, cuitada, por
me engañ ar.
a) Busca en el diccionario las palabras bel, catar,
cuitada, algarabía, almejía, brial.
b) Copia en tu cuaderno los verbos que aparecen en el
poema. ¿Qué tiempos verbales son los más
abundan- tes? ¿Predominan los verbos de acción?
Señálalos.
c) Analiza los elementos de la narración presentes en
este romance: narrador, personajes, tiempo y
espacio.
d) Señala su estructura: planteamiento, nudo y desen-
lace. ¿Hay un final cerrado? ¿Qué crees que le
a) • bel: bello (adjetivo en desuso).
b) ¡Oh muerte, cruel enemiga, que ni perdonas los culpados ni
• catar: mirar. absuelves los inocentes! Tan traidora eres, que nadie para
• cuitada: afligida, desventurada. contigo tiene defensa. Amenazas para la vejez y llevas en la
mocedad. A unos matas por malicia y a otros por envidia.
• algarabía: lengua á rabe.
Aunque tardas, nunca olvidas. Sin ley y sin orden te riges.
• almejía: tú nica o manto á rabe que usaban también los Má s razó n había para que conservases los veinte añ os
cristianos. del hijo mozo que para que dejases los sesenta de la vieja
• brial: vestido de seda o tela rica que usaban las mujeres. madre. ¿Por qué volviste el derecho al revés? Yo estaba
b) Era, vino, engañar, hablome, sabe, ábrasme, guarde, harta de ser viva y é l en edad de vivir. Perdó name porque
abriré, sé, serás, soy, dejo, viene, abres, verás, matar, así te trato, que no eres mala del todo, porque si con tus
oí, comenceme, levantar, vistiérame, hallando, fuérame, obras causas los dolores, con ellas mismas los consuelas
abrila. llevando a quien dejas con quien llevas, lo que si conmigo
haces, mucho te seré obligada.
Predomina el pretérito perfecto simple de indicativo alter-
Diego DE SAN PEDRO: Cárcel de amor.
nado con el presente simple de indicativo.
Hay abundancia de verbos de acció n, como vino, ábrasme, • ¿A qué subgénero pertenece cada texto? Resume
abriré, viene, abres, matar, comenceme, levantar, vistié- brevemente el tema de cada fragmento.
rame, fuérame y abrila. • ¿Quiénes protagonizan el primer texto? ¿Por qué
c) El narrador es la propia mora Moraima, la protagonista del luchan? Averigua el nombre de la amada de
romance, que cuenta lo que le sucedió en primera persona. Amadís.
Los personajes son dicha mora, y el cristiano que pretende • En el segundo texto, ¿quién se lamenta por una
hacerse pasar por su tío, el moro Mazote. El tiempo no pér- dida? ¿A quién crees que ha perdido?
aparece señ alado en el romance, pero podemos situarlo
en la é poca medieval de la Reconquista y las luchas entre • Ambos son fragmentos de una novela. En el caso del tex-
á rabes y cristianos. En cuanto al espacio, se desarrolla en to a), se trata de una novela de caballerías, y su tema
la puerta de la casa de la mora Moraima. sería la lucha entre dos caballeros. El texto b) pertenece a
una no- vela sentimental, y en este fragmento una mujer
d) El planteamiento sería cuando un supuesto á rabe llama a increpa a la muerte lamentá ndose porque se ha llevado a un
las puertas de Moraima pidié ndole que le deje entrar. ser querido.
El nudo, cuando Moraima duda y el hombre la convence • El primer texto está protagonizado por el caballero Ama-
diciéndole que es su tío y que ha matado a un cristiano y dís de Gaula, y su contrincante, el caballero Angriote.
por eso está siendo perseguido. Ambos está n luchando para que el otro reconozca que su
El desenlace ocurre cuando Moraima es engañ ada, se vis- amada es má s bella que la de su rival. La amada de Amadís
te y acude a abrir la puerta. se llama Oriana.
El final queda totalmente abierto a la imaginació n del lec- • La que se lamenta es la madre de Leriano, el protagonista
tor. Podemos suponer que el cristiano se llevará cautiva de la novela Cárcel de amor. Ha perdido a su hijo, ya que
a la mora Moraima. este se ha suicidado por el desdé n de su amada.
e) Aparecen las características propias de los romances,
como el fragmentarismo y el final abierto, la concisió n, ACTIVIDADES FINALES
el dramatismo y la expresividad gracias a los diá logos, la Repasa lo que has aprendido
capacidad sugestiva…
1. ¿Qué importantes acontecimientos históricos tuvieron
En cuanto a su métrica, vemos que está formado por ver-
lugar en la península ibérica durante el siglo xv?
sos octosílabos y que riman en asonante los pares, que-
dando sueltos los impares. Entre otros, el alumno podrá hacer alusió n al reinado de los
Reyes Cató licos, a la creació n del Tribunal de la Inquisició n,
2. Lee los siguientes fragmentos y responde: a la conquista de Granada, que supondrá el final de la Recon-
a) Amadís se lo otorgó , pues que le placía, y luego se fueron quista, a la expulsió n de los judíos o al descubrimiento de
ambos y tomaron sendas lanzas, las que má s les conten- Amé rica en el añ o 1492.
taron, y alongá ndose uno de otro se dejaron venir contra
sí e hirié ronse de las lanzas muy gravemente; y Angriote
2. Completa en tu cuaderno:
fue en tierra y el caballo sobre é l, y Amadís, que pasaba, a) Los autores más importantes de la poesía de Cancio-
tropezó en el caballo de Angriote y fue caer con él de nero fueron…
la otra parte, y un trozo de la lanza que por el escudo b) Los temas principales de las Coplas a la muerte de
le había entrado —con la fuerza de la caída— entrole su padre, de Jorge Manrique, son…
por el arné s y por la carne, mas no mucho; é l se levantó
muy ligero […] y poniendo mano a la espada, se dejó ir c) Los dos subgéneros narrativos más importantes del
contra Angriote, que le vio con su espada ir en la mano, siglo xv fueron…
y Angriote le dijo: d) Las danzas de la Muerte eran poemas escenificados
—Caballero, yo os tengo por buen mancebo y ruégoos que, en los que…
antes que má s mal recibá is, otorguéis ser má s hermosa mi e) La autoría de La Celestina ha sido controvertida
amiga que la vuestra. porque…
—Callad —dijo Amadís—, que tal mentira nunca será por f) El argumento de La Celestina es…
mi boca otorgada.
a) … el marqués de Santillana, Juan de Mena y Jorge Manrique.
Garci RODRÍGUEZ DE MONTALVO: Amadís de Gaula
b) … la brevedad de la vida, el desprecio de los bienes terre-
(tomado de Aventuras de los libros de caballerías,
nales, el poder igualitario de la muerte, la importancia de
Editorial Akal).
la vida eterna, la existencia de una tercera vida de fama,
el elogio a su padre don Rodrigo y la aceptació n cristiana
de la muerte.
c) … las novelas de caballerías y las novelas sentimentales.
Muy de prisa se calzaba,
d) … la Muerte se personificaba para ir sacando a bailar a má s de prisa se vestía;
todos los miembros de la sociedad, representando así ya se va para la calle
que todos somos iguales ante ella y todos vamos a morir. en donde su amor vivía.
e) … porque el propio Fernando de Rojas nos dice en la Carta «¡Á breme la puerta, blanca,
del autor a un su amigo, que é l no fue el autor del acto I, á breme la puerta, niñ a!».
sino que se lo encontró y decidió continuarlo. «¿Có mo te podré yo abrir
si la ocasió n no es venida?
f) … que Calisto se enamora de Melibea y para conseguir
Mi padre no fue al palacio,
su amor pide ayuda a una vieja alcahueta, Celestina. Di-
mi madre no está dormida».
chos amores llevará n a un fin desgraciado tanto a los jó -
«Si no me abres esta noche,
venes amantes, como a sus criados y a la propia
ya no me abrirá s, querida;
Celestina.
la Muerte me está buscando:
3. Explica en tu cuaderno los siguientes conceptos: Cancio- junto a ti vida sería».
nero, petrarquismo, amor cortés, mito, epopeya y fábula. «Vete bajo la ventana
• Cancionero: colecciones de poemas en estilo cortesano, donde labraba y cosía,
compuestos para ser cantados, de tema alegó rico, satírico te echaré cordó n de seda
y sobre todo amoroso. Destacan el de Palacio, el de para que subas arriba,
Stú ñ iga o el de Baena. y si el cordó n no alcanzare
mis trenza añ adiría».
• petrarquismo: corriente italiana encabezada por Frances-
La fina seda se rompe;
co Petrarca, en la que destaca el culto a la belleza
la Muerte que allí venía:
femenina, junto a una naturaleza bucó lica y el predominio
«Vamos, el enamorado,
del amor cortés.
que la hora ya es cumplida».
• amor cortés: convenció n literaria de la época medieval que
presenta a la amada como un ser superior, divino, a la a) ¿Cuál es el argumento del romance? ¿Qué significado
que el enamorado deber rendir vasallaje y sumisió n. tiene?
• mito: historia protagonizada por dioses, héroes o mons- b) ¿Qué características típicas de su género se observan
truos que da explicació n a ciertos fenó menos de la en él?
natura- leza o del origen del mundo. Destaca la mitología
griega y romana. c) Explica qué función cumplen los diálogos. ¿Y las repe-
• epopeya: narració n muy extensa en verso que relata las
ticiones?
hazañ as de un hé roe. d) En el poema hay una cierta identificación entre amor
• fábula: narració n en verso o en prosa, cuyos protagonis- y muerte. ¿Dónde se observa?
tas generalmente son animales, y que sirve para enseñ ar e) Recuerda la clasificación de los romances en cuanto a
mediante una moraleja final.
su temática. ¿Dentro de qué tipo de romance podría-
4. Resume mediante un esquema cuáles fueron los poetas mos encuadrar este? Justifica tu respuesta.
cultos más importantes del siglo xv y señala sus obras
f) Analiza la métrica del romance.
prin- cipales.
En el siglo XV destacan los siguientes autores cultos: g) Escucha una versión del romance tecleando en You-
Tube «Amancio Prada» «Romance del enamorado y
• Íñ igo Ló pez de Mendoza, marqués de Santillana, reconoci-
do por su poesía de cancionero, sus serranillas y sus sone-
la muerte».
tos fechos al itálico modo. a) Un hombre sueñ a que la muerte va a ir a buscarle, y para
• Juan de Mena, que destacó especialmente por su obra La- escapar de ella huye a casa de su amada. Pero precisamen-
berinto de Fortuna. te cuando intenta subir con una cuerda a su ventana, esta
se rompe, y el hombre cae y muere.
• Jorge Manrique, que ademá s de poesía amorosa de
Su significado es que no podemos escapar de la muerte
cancio- nero compuso su obra cumbre Coplas a la
cuando nos llega nuestra hora.
muerte de su padre.
b) El fragmentarismo (no sabemos nada del antes de los
5. Lee el siguiente romance y responde: personajes), el dramatismo (gracias a los diá logos y a la
Un sueñ o soñ aba anoche primera persona), las repeticiones y los paralelismos, su
soñ ito del alma mía, capacidad de sugestió n…
soñ aba con mis amores c) Los diá logos contribuyen a dotar al poema de mayor ex-
que en mis brazos los tenía. presividad y dramatismo. Las repeticiones tienen la fun-
Vi entrar señ ora tan blanca ció n de favorecer la memorizació n del poema.
muy má s que la nieve fría.
«¿Por dó nde has entrado, amor? d) El joven está soñ ando con sus amores cuando ve entrar a
¿Có mo has entrado, mi vida? una señ ora blanca y fría, a la que confunde con su amada.
Las puertas está n cerradas, Un poco má s adelante a su enamorada también la califica
ventanas y celosías». de blanca.
«No soy el amor, amante: e) Estaría dentro de los romances novelescos y líricos, ya que
la Muerte que Dios te envía». su temá tica es amorosa, de misterio…
«¡Ay, Muerte tan rigurosa, f) Está formado por versos octosílabos que riman en aso-
dé jame vivir un día!». nancia en los pares (í-a), quedando sueltos los impares.
«Un día no puede ser, g) Audició n.
una hora tienes de vida».
6. En el último acto de La Celestina, Pleberio, el padre de desierto espantable, una morada de fieras, un juego de
Melibea, se lamenta amargamente por la trágica pérdida hombres que andan en corro, etc. Todas ellas son imá genes del
de su hija. Lee este fragmento y resuelve las tareas: caos y del desorden má s absoluto.
¡Oh mi hija y mi bien todo! Crueldad sería que viva yo so-
bre ti; má s dignos eran mis sesenta añ os de la sepultura
que tus veinte. […] ¡Oh duro corazó n de padre! ¿Có mo no
te quiebras de dolor, que ya quedas sin tu amada herede-
ra? ¿Para quién edifiqué torres? ¿Para quién adquirí honras?
¿Para quié n planté á rboles? ¿Para quié n fabriqué navíos?
[…]
¡Oh vida de congojas llena, de miserias acompañ ada! ¡Oh
mundo, mundo! […] Yo pensaba en mi má s tierna edad que
eran tus hechos regidos por alguna orden; agora, visto el pro
y la contra de tus bienandanzas, me pareces un laberinto de
errores, un desierto espantable, una morada de fieras, juego
de hombres que andan en corro, laguna llena de cieno,
regió n llena de espinas, monte alto, campo pedregoso, prado
lleno de serpientes, huerto florido y sin fruto, fuente de
cuidados, río de lágrimas, mar de miserias, trabajo sin
provecho, dulce ponzoñ a, vana esperanza, falsa alegría,
verdadero dolor. […]
¡Oh amor, amor! ¡Que no pensé que tenías fuerza ni poder
de matar a tus sujetos! […] ¿En qué pararon tus sirvientes
y ministros? […] ¡Oh mi hija despedazada! ¿Por qué no que-
siste que estorvase tu muerte? ¿Por qué no hobiste lástima
de tu querida y amada madre? ¿Por qué te mostraste tan
cruel con tu viejo padre? ¿Por qué me dejaste cuando yo
te había de dejar? ¿Por qué me dejaste triste y solo in hac
lachrimarum valle?
Biblioteca Didá ctica ANAYA

a) Compara la manera de afrontar la muerte de Plebe-


rio con la de don Rodrigo en las Coplas de Manrique.
¿Cuál es más cristiana? Razona tu respuesta.
b) Pleberio se queja contra el mundo y contra el amor.
¿Crees que esa rebeldía es ya muestra de la nueva
sen- sibilidad renacentista? ¿Por qué?
c) ¿Cómo se describe el mundo? ¿De qué recursos
lite- rarios se vale Rojas?
d) Como recuerda Pleberio, el amor loco ha hecho que
casi todos los personajes de la obra tengan un fin
trágico.
¿Crees que aquí se encuentra el propósito
moralizador de la obra? Justifica tu respuesta.
e) Compara este planto con el de la madre de Leriano,
en el fragmento que leíste de Cárcel de amor. ¿Qué
tienen en común? ¿Cuál te parece más emotivo?
a) Es mucho má s cristiana la visió n de la muerte que nos
presenta Manrique en las Coplas, ya que en ellas hay una
aceptació n total de la muerte como algo natural y necesa-
rio. Sin embargo en el planto de Pleberio no hay tal acep-
tació n, el padre se rebela contra Dios y contra la muerte
por haberse llevado a su hija antes de tiempo.
b) Sí, ya que en la mentalidad medieval no cabía tal
rebeldía, la muerte se aceptaba como un designio de la
voluntad divina. Pleberio, en cambio, increpa a la vida, al
mundo y al amor como culpables de su desdicha.
Considera que en el mundo no hay orden sino que está
regido por el capricho y la falta de ló gica. Esto es más
renacentista que medieval, ya que en la Edad Media Dios
era el centro de todo y había que aceptar su voluntad sin
quejas.
c) Rojas describe el mundo basá ndose en las metá foras. Así,
lo identifica, por ejemplo, con un laberinto de errores, un
d) Posiblemente, ya que todos los personajes que se han muy breves, normalmente sin rima o con rima asonante, don-
de- jado llevar por las pasiones y el loco amor han de el yo poético generalmente era una muchacha que expresa-
terminado desastradamente. Calisto, Melibea, ba sus sentimientos amorosos hacia su amigo. En numerosas
Celestina, Pá rmeno y Sempronio, todos han muerto por
dejarse llevar o bien por el amor o bien por la codicia.
e) Tienen varios elementos en comú n, ya que la madre
de Leriano tambié n se queja contra la muerte y la
increpa amargamente, diciendo que se rige sin ley y
sin orden. Se asemejan mucho en la parte en la que la
madre de Leriano dice más razón había para que
conservases los veinte años del hijo mozo que para
que dejases los sesenta de la vieja madre, y Pleberio
a su vez se queja diciendo: más dignos eran mis
sesenta años de la sepultura que tus veinte. Ambos
se lamentan de seguir vivos tras las muer- tes de sus
hijos, aunque quizá el planto de Pleberio sea más
emotivo en sus increpaciones al mundo y al amor.

Recuerda lo que ya sabías


7. Explica cuál sería el registro lingüístico empleado por
Ple- berio en el planto que has leído en la actividad 6 y
usa ex- presiones que aparezcan en el texto a modo
de ejemplo.
Se trataría de un registro lingü ístico culto y formal,
propio de su condició n de noble. Pese a su dolor, Pleberio
emplea expresiones bien elaboradas, un léxico rico y
cuidado y una sintaxis compleja, por ejemplo cuando se
lamenta diciendo:
¡oh duro corazón de padre! ¿Cómo no te quiebras de
dolor, que ya quedas sin tu amada heredera? ¡Oh vida de
congojas llena, de miserias acompañada! ¿Por qué me
dejaste triste y solo in hac lachrimarum valle? (aquí
incluso emplea una expresió n en latín).
8. Indica en qué tiempo están las formas verbales del
roman- ce de la actividad 5.
Aparecen sobre todo verbos en pasado: pretérito
imperfecto de indicativo (soñaba, tenía, se calzaba, se
vestía, vivía, la- braba, cosía, venía), preté rito perfecto
simple de indicativo (vi, fue) y preté rito perfecto
compuesto de indicativo (has entrado).
Pero también abundan los verbos en presente,
especialmente en los diá logos: están soy, envía, puede,
tienes, se va, es, abres, está, rompe, es.
Aparecen ademá s algunos imperativos (déjame, ábreme,
vete, vamos), futuros (abrirás, echaré) y condicionales (se-
ría, añadiría).
9. Analiza los elementos de la narración de dicho
romance, y divídelo en planteamiento, nudo y
desenlace.
El narrador es un narrador en primera persona, ya que es
el propio enamorado el que cuenta lo que le sucede. Los
perso- najes son el enamorado, su amada y la muerte. El
espacio es la casa del enamorado y despué s la de su
amada, y el tiempo es la noche anterior a la narració n.
En cuanto a la estructura de la narració n, el
planteamiento sería cuando el enamorado está soñ ando y
se le aparece la muerte, que le concede tan solo una hora
má s de vida. El nudo, cuando el enamorado, intentando
escapar de la muerte, se va a casa de su amada y esta le
propone que trepe por un cordel hasta su ventana, y el
desenlace, cuando finalmente el cordel se rompe y la
muerte llega a buscar al enamorado.
10. Explica las semejanzas y diferencias que encuentres
entre la lírica tradicional medieval (jarchas, cantigas y
villancicos) y el Romancero del siglo xv.
La lírica tradicional medieval estaba formada por poemas
ocasiones se dirigía a algú n confidente, generalmente su ma- • Muy escaso en esta é poca. La ú nica obra conservada es el
dre, sus hermanas o amigas… Era una poesía sencilla, fá cil Auto de los Reyes Magos, anó nima, de estilo sencillo y tono
de memorizar, y muy expresiva. Estaba escrita en las dife- ingenuo.
rentes lenguas romances de la Península (mozá rabe, gallego-
portugués o castellano). Su temá tica, por tanto, era sobre todo
el amor, acompañ ado en muchas ocasiones de alusiones a la
naturaleza. Formalmente destacan en ellos las repeticiones,
anáforas y paralelismos.
En cuanto a los romances, son poemas má s extensos, que si-
guen una mé trica fija: versos octosílabos con rima asonante
en los pares, quedando sueltos los impares. Si bien sigue
apa- reciendo en ellos la temá tica amorosa, ahora predominan
otro tipo de asuntos: romances histó ricos, fronterizos,
moriscos, novelescos… Se caracterizan por tener tambié n
una gran ex- presividad, gracias a los diá logos, las llamadas
de atenció n al oyente, etc. Ademá s son muy concisos y
fragmentarios, con una gran capacidad sugestiva. Al igual
que en la lírica tradi- cional, abundan en ellos las
repeticiones para favorecer su memorizació n. Está n escritos
en lengua castellana.
11. Recuerda lo que era un juglar y lo que era un
trovador. ¿En el siglo xv continúan existiendo dichos
oficios? Justifica tu respuesta.
Los juglares eran los encargados de difundir la literatura
oral por los pueblos y las villas. Llevaban, por tanto, una vida
am- bulante, y la mayoría de las veces no eran autores, sino
simple- mente transmisores de dicha literatura, cuyo
recitado solían acompañ ar de mú sica.
Los trovadores, en cambio, sí eran los creadores de sus poe-
mas. Su lírica se acompañ aba tambié n de mú sica y el
entorno en el que desarrollaban su arte era el mundo de la
corte. Su clase social era, pues, má s elevada que la de los
juglares.
En el siglo XV ambas prá cticas comienzan a decaer. Con el fin
de la Edad Media va desapareciendo el oficio de juglaría, al
igual que sucede con los trovadores, cuya poesía será susti-
tuida por la poesía de Cancionero.
12. Haz un esquema donde recojas las obras más
importantes de la lírica, el teatro y la narrativa de la Edad
Media, inclu- yendo también el siglo xv. Puedes
organizarlo por siglos y señalar en él características
principales, autores y obras.
Siglos XI-XIV
Lírica
• Lírica tradicional y popular: jarchas, cantigas y villanci-
cos. De autor anó nimo, se caracterizan por su sencillez y
brevedad. Su temá tica suele ser el amor en boca de una
muchacha enamorada. Se basan en repeticiones, parale-
lismos y aná foras. Escritos en los principales dialectos de
la Península.
• Lírica culta: lírica trovadoresca, en la línea del amor cortés.
Narrativa
• Poemas é picos y caballerescos. Cuentan en verso las haza-
ñ as de un héroe. Destaca el Cantar de Mio Cid, anó nimo,
donde se narran las heroicidades de Rodrigo Díaz de Vivar,
el Cid Campeador.
• Mester de clerecía: autores cultos, normalmente clérigos,
que escribían una poesía narrativa, de temá tica religiosa,
en cuaderna vía. Destacan Gonzalo de Berceo (Milagros de
Nuestra Señora) y el arcipreste de Hita (Libro de buen
amor).
• Prosa de autores cultos, como don Juan Manuel (El conde
Lucanor).
Teatro
Siglo XV
Lírica
• Por un lado tenemos la lírica popular, cuyo mayor
repre- sentante es el romance. De temá tica variada
(romances histó ricos, fronterizos, moriscos,
novelescos), se trata de poemas no estró ficos de versos
octosílabos con rima aso- nante en los pares. Se basan
en la expresividad, la conci- sió n, el fragmentarismo, las
repeticiones, etc.
• Lírica culta y cortesana: compuesta en la corte por
autores cultos, se basa en los presupuestos del amor
cortés. Desta- can autores como Juan de Mena
(Laberinto de Fortuna), el marqué s de Santillana
(serranillas, Sonetos fechos al itálico modo) y Jorge
Manrique (Coplas a la muerte de su padre).
Narrativa
Destacan dos subgéneros:
• La novela sentimental, como Cárcel de amor, de Diego
de San Pedro.
• La novela de caballerías, como el Amadís de Gaula,
de Garci Rodríguez de Montalvo.
Teatro
• Continuació n de los autos medievales.
• Aparece un nuevo subgénero: las danzas de la Muerte, don-
de la muerte iba sacando a bailar a diferentes personajes
de todas las clases sociales.
• La obra clave del teatro del siglo XV es La Celestina, de
Fernando de Rojas, que relata los amores
desgraciados de Calisto y Melibea bajo la intercesió n de
la alcahueta Celestina.

MIRA A TU ALREDEDOR Y…
… ve más allá
1.Las danzas de la Muerte se relacionan muy
estrechamente con el arte de la danza. En algunos
pueblos, como Verges, en Girona, se celebra cada
Jueves Santo un ritual de dan- zas de la Muerte.
a) Investiga desde cuándo se celebra, en qué consiste y
qué simbología emplea.
b) Teclea en YouTube «danzas de la Muerte» y ve
algún vídeo sobre la fiesta de Verges. ¿Qué
sensaciones te produce?
a) Esta danza se conserva desde la Edad Media, cuando se
asociaba a las epidemias de peste negra. La danza tiene
lugar de noche y forma parte de una procesió n, en la
que diez esqueletos recorren las calles al ritmo de un
timbal, iluminando la escena con antorchas. Los
esqueletos van danzando y saltando, y portan elementos
simbó licos como guadañ as, banderas, cuencos con
cenizas, o un reloj sin agujas que nos recuerda que la
muerte puede llegar en cualquier momento.
b) Respuesta libre.
2. La relación entre literatura y cine siempre es muy inte-
resante, en el caso de La Celestina muy especialmente.
Lee este fragmento de la escena amorosa entre Calisto
y Melibea en el huerto, justo antes de su muerte, y
realiza las actividades:
MELIBEA.— ¡Oh sabrosa traició n! ¡Oh dulce sobresalto! ¿Es
mi señ or de mi alma? ¿Es é l? No lo puedo creer. ¿Dó nde
esta- bas, luciente sol? ¿Dó nde me tenías tu claridad
escondida?
¿Hacía rato que escuchabas? […]
CALISTO.— Pues, señ ora y gloria mía, si mi vida quieres, no y cristianos (ya sean nuevos o viejos) es perjudicial para
cese tu suave canto. […] la fe
MELIBEA.— ¿Qué quieres que cante, amor mío? […] Y pues tú ,
señ or, eres el dechado de cortesía y buena crianza,
¿có mo mandas a mi lengua hablar y no a tus manos que
estén que- das? […] Deja estar estas mis ropas en su lugar, y
si quieres ver si es el há bito de encima de seda o de pañ o,
¿para qué me tocas en la camisa? Pues cierto que es de
lienzo. Hol- guemos y burlemos de otros mil modos que yo
te mostraré; no me destroces ni maltrates como sueles.
¿Qué provecho te trae dañ ar mis vestiduras?
CALISTO.— El que quiere comer el ave quita primero las plumas.
[…]
MELIBEA.— ¿Señ or mío, quieres que mande a Lucrecia traer
alguna colació n?
CALISTO.— No hay otra colació n para mí sino tener tu cuerpo
y belleza en mi poder. Comer y beber, donde quiera se da
por dinero, en cada tiempo se puede haber, y cualquiera lo
puede alcanzar; pero lo no vendible, lo que en toda la tierra
no hay igual que en este huerto, ¿có mo mandas que se
me pase ningú n momento que no goce?
LUCRECIA.— (Ya me duele a mí la cabeza de escuchar, y no a
ellos de hablar, ni los brazos de retozar, ni las bocas de be-
sar. ¡Andar! Ya callan; a tres me parece que va la vencida).
CALISTO.— Jamá s querría, señ ora, que amaneciese, segú n la
gloria y descanso que mi sentido recibe de la noble con-
versació n de tus delicados miembros.
MELIBEA.— Señ or, yo soy la que gozo, yo la que gano; tú , señ or,
el que me haces con tu visitació n incomparable merced.
Biblioteca Didá ctica ANAYA

a) ¿Qué opinas del comportamiento de Calisto? ¿Se está


conduciendo como un verdadero enamorado dentro
de las convenciones del amor cortés?
b) Melibea ha perdido algo muy importante para una
mujer de su época: la honra. ¿Crees que eso puede
explicar su suicidio tras la muerte de Calisto?
c) Ved este fragmento de la adaptación al cine,
tecleando en YouTube «La Celestina escena huerto».
¿Qué te parece la adaptación? ¿Reflejan Calisto y
Melibea el mismo amor que en el texto? Razona tus
respuestas.
a) El alumno puede señ alar que Calisto no está respetan-
do las convenciones del amor corté s, ya que este era un
amor de tipo plató nico, en cambio Calisto tiene mucha pri-
sa por consumar carnalmente su amor con Melibea.
b) Es una posible explicació n, ya que al haber quedado des-
honrada (y siendo dicha deshonra conocida por varios ha-
bitantes de la ciudad), pierde su honor y el de su familia,
y no le sería ya posible encontrar marido; seguramente su
ú nica salida fuera ingresar en un convento.
c) Respuestas libres.

… encuentra la clave
1. Durante el reinado de los Reyes Católicos se produjo uno
de los acontecimientos decisivos de la historia de
España: la expulsión de los judíos. Lee el siguiente texto
sobre el tema y responde a las cuestiones que se
plantean:
La expulsión de los judíos de España
El reinado de los Reyes Cató licos fue decisivo tanto para los
judíos como para los conversos. Sabedores de su importancia
econó mica para su monarquía, el matrimonio real mostró ac-
titudes protectoras hacia la minoría hebrea. Sin embargo, los
monarcas tendrá n la opinió n de que el contacto entre judíos
cató lica, por lo que aceptan en las Cortes de Toledo de 1480 el
confinamiento de los judíos en barrios apartados. El siguiente paso
fue su definitiva expulsió n.
El 31 de marzo de 1492 Fernando e Isabel publicaron por me- dio
de las Chancillerías de Aragó n y Castilla dos edictos muy
similares por los que obligaban a sus sú bditos de religió n judía a
abandonar sus reinos si antes del 31 de julio de ese añ o no se
habían bautizado. Asimismo se informaba a los cristianos de
que si ayudaban a algú n hebreo a desobedecer las ó rdenes
serían castigados con la pérdida de sus bienes. A los semitas que
decidieran no convertirse al cristianismo el edicto les per- mitía
sacar de la península sus bienes muebles, salvo caballos y mulas,
metales preciosos y moneda españ ola. La decisió n de la
expulsió n de los judíos no debe atribuirse ú nicamente a los
monarcas sino que la influencia e incluso presió n de la
Inquisició n jugaron un papel decisivo. Esta medida fue presen-
tada como necesaria pues era de vital importancia separar a los
judeoconversos de sus antiguos correligionarios para que su
cristianizació n fuera verdadera y efectiva. Con el cumpli- miento
de este edicto concluyó la desintegració n de la que fue durante
varios siglos la mayor comunidad judía de Europa. Parece que
la cifra final de los hebreos que se negaron a cam- biar de
religió n y que, por consiguiente, se vieron obligados a
abandonar la península, fue de entre 50 000 y 70 000 perso- nas.
Durante el verano de 1492 se produjo dentro de los reinos de
Fernando e Isabel un importante movimiento forzoso de
població n.
El destino de los expulsados fue diverso. Algunos optaron
por no abandonar la península y establecerse en los vecinos
reinos de Portugal (donde el 3 % de la població n era de religió n
judía) y Navarra, en los que su presencia era tolerada, aunque
por poco tiempo, pues en ambos lugares se seguirá n los pasos
dados por Isabel y Fernando. Otros destinos fueron Italia, el
Imperio Otomano o el norte de Á frica.
Laura PÁ RAMO DE VEGA: La historia
de las tres culturas.

a) ¿Por qué comienzan a tomarse medidas represoras


contra los judíos? ¿Cuál fue la primera de ellas?
b) ¿Cuál era la condición imprescindible para permane-
cer en España a partir de 1492?
c) ¿Quiénes eran los semitas? Investiga en la red por qué
recibían este nombre.
d) ¿Podían llevarse los judíos sus posesiones cuando se
marcharan de España?
e) ¿Cuál fue la principal razón alegada para llevar a cabo la
expulsión? ¿Quién la defendía?
f) ¿Cuál fue el destino de los judíos que se marcharon?
g) Investiga la relación entre este acontecimiento y la ex-
pansión y pervivencia de la lengua sefardí en diversos
lugares del mundo.
h) En la actualidad, ¿en qué países del mundo se dan
problemas similares de intolerancia religiosa o choque
entre diferentes creencias? Investígalo y exponlo ante
tus compañeros.
a) Porque los Reyes Cató licos opinan que el contacto con los
judíos es perjudicial para la fe cató lica. La primera medi- da
fue tomada en 1480 cuando se decidió confinarlos en
barrios apartados.
b) Bautizarse.
c) Los semitas son los judíos. Se les llamaba así porque eran
los descendientes de Sem, primer hijo de Noé .
d) Podían llevarse sus muebles, pero no sus caballos ni mulas
el castellano que se hablaba aquí en dicha época. Esta
o metales preciosos y moneda españ ola.
lengua se ha conservado durante siglos sin sufrir apenas
e) Decían que era necesario separar a los conversos de los modificaciones, por lo que hoy en día podemos encontrar
judíos, para que su cristianizació n fuera auté ntica y defi- comunidades judías diseminadas por todo el mundo que
nitiva. Defendió la expulsió n el tribunal de la Inquisició n. aú n hablan el castellano sefardí del siglo XV.
f) Algunos se quedaron un tiempo en Portugal y Navarra,
h) Respuesta libre. Los alumnos pueden hablar, por ejem-
otros marcharon a Italia, al norte de Á frica o al Imperio
plo, del problema israelí-palestino, o de la persecució n de
otomano.
cristianos por parte de radicales musulmanes en algunos
g) El hecho de que los judíos se vieran obligados a abando- países á rabes.
nar la Península hizo que se llevaran consigo su lengua,
9 COGED DE VUESTRA ALEGRE PRIMAVERA…

APERTURA DE UNIDAD Calisto en La Celestina. El amante entiende que la amada es


el objeto de su vida y en ella reside todo lo que anhela. Las
1. Rafael Alberti (1902-1999) es uno de nuestros escritores cualidades de la amada son tales que incluso está n por encima
contemporáneos más sobresalientes. En la composición de su comprensió n, pero el poeta se declara incondicional de
anterior, muestra su admiración hacia el poeta renacen- su fe (su amor).
tista Garcilaso de la Vega, resaltando sobre todo dos as-
pectos: ¿cuáles son?
Su condició n de caballero sobre todo. Pero tambié n habla de
su poesía (¡Qué dulce oírle!).
2. De Garcilaso dice Alberti que «buen caballero era»… Pero
¿sabes en qué consistía ser un caballero en el
Renacimien- to? Investiga sobre este particular y anota
las cualidades que, según el código de conducta de la
época, debía tener un hombre para ser considerado un
buen caballero.
Patriotismo y lealtad hacia su rey. Valentía y cortesía. Debía
dominar el ejercicio de la guerra pero también debía tener una
formació n pulida en las doctrinas humanísticas.
3. En el Centro Virtual Cervantes (cvc.cervantes.es)
puedes encontrar este poema junto a otros destinados a
homena- jear a Garcilaso. Teclea en su buscador
«Homenaje poético a Garcilaso». Visita la página, escoge
uno de los poemas que encontrarás ahí, cópialo en tu
cuaderno y recítalo en clase para todos tus
compañeros.
Respuesta libre.
4. Realiza una línea del tiempo con todos los autores que
has encontrado en la página de la actividad anterior y
sitúa en ella a todos los poetas que aparecen en el
homenaje.
Respuesta libre.

VIVE LA LECTURA
Comprensión lectora
1.En el primer cuarteto, el poeta dice que su amada es la
verdadera autora de sus versos, puesto que ella los ins-
pira. Explica esta afirmación teniendo en cuenta el con-
tenido de dicha estrofa.
El poeta entiende que escribir es leer los versos que ella le
inspira. No obstante, esa tarea la realiza en absoluta soledad
y ocultamiento.
2. El yo poético manifiesta su absoluta incondicionalidad
ha- cia la amada. Localiza las frases o expresiones del
poema en las que se manifieste dicho sentimiento.
Especialmente en los dos tercetos. Yo no nací sino para que-
reros […] por vos nací, por vos tengo la vida, por vos he de
morir, y por vos muero.
3. Además, el afecto está expresado como si se tratase de
una religión: la «religión del amor», esa es la fe que
profe- sa el amante. Justifica esta afirmación con citas
del texto.
Aunque no cabe en mí cuando en vos veo, de tanto bien
lo que no entiendo creo.
4. El motivo temático de la «religión del amor» estaba ya
pre- sente en la centuria anterior. ¿Recuerdas la
declaración
«Melibeo soy, en Melibea creo y a Melibea adoro»?
¿Qué personaje hacía esta manifestación? ¿En qué obra?
¿Cómo relacionas esas palabras con los versos de
Garcilaso?
80 Lengua castellana y Literatura. 3.º ESO. Solucionario
COGED DE VUESTRA ALEGRE PRIMAVERA… 9
5. En la tradición del amor cortés, el yo poético se ve a sí Re- forma protestante de Martín Lutero?
mismo como un vasallo de la amada, mujer idealizada y
elevada, a la que el enamorado somete su voluntad.
¿Está presente esta tradición en el soneto de Garcilaso
que aca- bamos de leer? ¿Por qué?
Sí. El poeta le dedica toda su vida, la convierte en fin de su
existencia.

Vocabulario
6. Observa las siguientes palabras y expresiones del tex-
to: gesto (verso 1), me guardo (verso 4), puesto (verso
5).
¿Qué sentido tienen en el soneto? Compara sus signifi-
cados con los que les damos en la actualidad.
Gesto: rostro; en la actualidad se utiliza con el sentido de
mueca o ademá n. Me guardo: me escondo; en la actualidad
el verbo guardar no suele utilizarse con sentido reflexivo y
casi siempre se emplea con el sentido de recoger. Puesto:
dedicado; en la actualidad se emplea a veces con un sentido
semejante en pe- rífrasis verbal (me he puesto a trabajar)
pero no es sinó nimo; fuera de perífrasis, el verbo poner se
utiliza casi siempre para referirnos a colocar o dejar algo en
una posició n determinada.
7. ¿Con qué sentido se utiliza la palabra hábito en el verso
11? Se emplea con doble sentido: costumbre y ropa de una
orden religiosa.
8. En el último verso, el yo poético dice «por vos muero».
¿Qué sentido tiene aquí el verbo morir?
Amar desesperadamente algo inalcanzable y sufrir por ello.
Es el sentido habitual en el amor cortés.

Investigación y redacción
9. Como has visto en la biografía de Garcilaso de la Vega,
este poeta estuvo muy enamorado de una mujer
llamada Beatriz de Sá. Investiga sobre esta historia y
escribe un resumen.
Respuesta libre.
10.Garcilaso fue un gran admirador del poeta italiano Fran-
cesco Petrarca, quien abrió una corriente (el
petrarquismo) que seguirían muchos escritores
europeos a partir del siglo
xv. Busca la información necesaria sobre Petrarca para
con- testar a las siguientes preguntas:
a) ¿En qué época vivió y desarrolló su obra?
b) ¿Cuál es la temática del Cancionero?
c) ¿Quién era Laura?
a) Nació en 1304 y murió en 1374. Sería uno de los
iniciadores del Renacimiento italiano, que se desarrolló
antes que el españ ol.
b) En el Cancionero Petrarca reflexiona sobre el sentimiento
amoroso, y presenta a la amada como un ser idealizado
aunque distante, que desdeñ a al poeta amante.
c) Laura es la amada idealizada de los poemas de Petrarca.

LITERATURA
1. Investiga y responde:
a) ¿En qué año se expulsó a los judíos? ¿Qué otros
he- chos ocurrieron ese año?
b) ¿Qué corriente cultural asentó con su obra Erasmo
de Róterdam? ¿En qué consistía?
c) ¿Qué novedades introducía en el cristianismo la

Lengua castellana y Literatura. 3.º ESO. Solucionario 81


d) ¿Durante qué años se extiende el reinado de Carlos I?
4. Investiga sobre la película El club de los poetas muertos,
¿Y el de Felipe II? dirigida en 1989 por Tom Schulman y protagonizada por
e) ¿Cuándo se inició la Contrarreforma? ¿Con qué acon- Robin Williams. Escribe un resumen de su argumento en
tecimiento histórico? tu cuaderno y, a continuación, señala qué tópico literario
f) El Renacimiento italiano se desarrolló antes que el importantísimo se trata en ella.
español. ¿En qué época fue? ¿Qué figuras literarias La película narra la historia de un profesor de literatura que
y artísticas destacaron en ese periodo? ejerce en un colegio masculino de estrictas normas. Los mé-
todos poco ortodoxos del profesor no encajan en las
g) Averigua qué era el Índice de libros prohibidos y si al- tradicio- nes de la institució n pero ayudan a sus alumnos a
guna de las obras del periodo que ahora estudias figu- descubrir su propio potencial creador y la importancia de
raban en él. exprimir lo mejor de la vida. El tó pico literario que aborda
a) En 1492. Unificació n territorial por parte de los Reyes esta película es el Carpe diem.
Cató licos. Descubrimiento de Amé rica. Publicació n de la 5. Investiga sobre el mito de Orfeo y Eurídice y resume su
Gramática de Nebrija. contenido. A la luz del mito, ¿cómo interpretas el verso 8
b) Humanismo. Corriente de pensamiento que considera al del soneto que acabas de leer?
ser humano como medida de todas las cosas (antropocen-
La ninfa Eurídice era la esposa del mú sico Orfeo. Eurídice
trismo) y capaz de desarrollar todo su potencial, tanto el
murió por una mordedura de serpiente y Orfeo, desconsolado,
físico como el espiritual, por encima de sus limitaciones,
va a buscarla al Hades. Su canto es tan bello y conmovedor
incluso por encima de su propia condició n social.
que convence a Caronte para que le deje pasar y a Perséfone
c) Buscaban la renovació n de ciertos há bitos desarrollados y a Hades para que le permitan llevarse a su amada. Ellos
por la Iglesia cató lica, con la finalidad de devolver el cris- se lo conceden con la ú nica condició n de que no se vuelva a
tianismo a sus valores primitivos. Cuestionaban el domi- mirarla hasta que salgan nuevamente a la luz del sol. Orfeo
nio del Papa sobre toda la cristiandad. cumple esta condició n hasta que está n a punto de abando-
d) Carlos I: 1516-1556. Felipe II: 1556-1598. nar el Hades, pero en ese momento tiene la sospecha de que
Perséfone puede haberle engañ ado, así que mira hacia atrá s
e) Con el Concilio de Trento, que se desarrolló entre 1545 para ver si es realmente Eurídice quien le sigue. Así la pierde
y 1563. definitivamente. En el verso 8 del soneto la cita de los reinos
f) Desde finales del XIV y durante el siglo XV. Como inicia- del espanto es una referencia al Hades.
dor literario podemos citar a Petrarca (1304-1374). Es 6. ¿Qué elementos característicos de la temática
muy importante la figura del filó sofo político Maquiavelo
renacentis- ta podrías comentar en este poema? Señala
(1469-1527). En el arte destacan Miguel Á ngel (1475-1564)
y Leonardo da Vinci (1452-1519). si encuentras epítetos y explica su valor.
El tratamiento del amor desde la perspectiva petrarquista (la
g) Era una lista publicada por la Inquisició n españ ola que
amada esquiva) y la alusió n a motivos de la mitología clá si-
incluía aquellos libros cuya lectura estaba prohibida en el
ca. El mito clásico sirve al poeta de ejemplo para tratar el
territorio nacional. Su primera edició n es de 1551, aunque
asunto del poema. Como epítetos pueden citarse fieros, fríos,
luego se fue corrigiendo en ediciones sucesivas. En 1564,
y trabajosa.
el Papa promulgaría otro Índice (Index librorum prohi-
bitorum) que afectaría a toda la cristiandad. De las obras 7. Haz un resumen del contenido del soneto XIII y, aten-
estudiadas en esta unidad, pueden citarse como lecturas diendo a este, diferencia las partes en que podrías dividir
prohibidas la obra de Erasmo de Ró terdam y el Lazarillo este texto.
de Tormes. Los brazos de Dafne se transformaban en ramas, sus cabellos
2. Fray Luis de León fue un gran defensor de la lengua cas- en hojas, sus piernas en corteza y sus pies en raíces. Apolo
tellana. Fíjate en esta afirmación suya y explícala llora y riega con sus lá grimas el á rbol, de este modo su pro-
después: pio llanto alimenta la razó n de su dolor. Estructuració n en
partes: los dos cuartetos describen el momento de la trans-
Assí que no piensen porque veen romance, que es poca estima formació n; el primer terceto se refiere al llanto de Apolo; el
lo que se dize; mas al revés, viendo lo que se dize, juzguen que segundo terceto contiene la reflexió n final del yo poé tico,
puede ser de mucha estima lo que se escrive en romance, y que aclara la relació n que existe entre el sentimiento
no desprecien por la lengua las cosas, sino por ellas estimen expresado y el mito que le ha servido de ejemplo para
la lengua. expresarlo.
En el siglo XVI aú n existía el prejuicio de que los temas impor-
8. ¿De qué se lamenta el poeta al final del soneto XIII? ¿Hay
tantes y elevados se trataban en latín (por escrito) y que el
romance era solo para los asuntos menores. Fray Luis desea
algún hecho paradójico en el poema? Explícalo.
desmontar este prejuicio y demostrar que la lengua romance Se lamenta, en efecto, de una gran contradicció n: el llanto
es vehículo perfecto para abordar toda clase de temas. aumenta el motivo que lo provoca, es decir, mientras mayor
es el llanto, mayores son las razones para llorar.
3. Investiga sobre las figuras de Juan de Valdés y Juan Bos-
cán. Explica qué importancia tuvieron para la lengua y la 9. Redacta una descripción de la mujer a la que se dirige el
literatura españolas del Renacimiento. yo poético en el soneto XXIII.
Juan de Valdé s fue uno de los má s importantes prosistas del Es una mujer joven, de largos cabellos rubios, de piel blanca
Renacimiento españ ol. Destaca por su obra Diálogo de la len- y mejillas sonrosadas.
gua, en la que reflexiona sobre nuestro idioma, depura sus 10. Localiza y explica las metáforas utilizadas para describir
normas y presenta un ideal de estilo sobrio y cuidado. Juan a la dama de dicho soneto.
Boscá n fue un poeta renacentista, amigo de Garcilaso de la
Los colores de su rostro se identifican con la rosa y la azuce-
Vega, que tuvo una importancia decisiva en la introducció n de
na. La hermosa cumbre es la cabeza de la dama, que quedará
las formas métricas italianas en la poesía españ ola de la época.
cubierta de nieve, es decir, de canas.
11. Siguiendo con el mismo texto, ¿qué simboliza la San Juan de la Cruz (1542-1591). Nació en Fontiveros
primave- ra? Justo después, se alude al invierno, que (Á vila). Ingresó en los carmelitas y emprendió junto a santa
también tiene un tratamiento simbólico. ¿Cuál es? ¿En Teresa de Jesú s la reforma de esa orden. Esta tarea le supuso
qué elementos del texto podrías justificar dicha alusión varios problemas: estuvo encarcelado durante ocho meses en
al invierno? Toledo y ademá s fue perseguido por varios de sus propios
La juventud. El invierno simboliza la vejez. Se alude al invier- compañ e- ros, sin embargo llegó a ser prior de varios
no cuando se habla de la cumbre cubierta de nieve o del conventos. Las obras que le dan su importancia en el á mbito
viento helado que marchitará la rosa. de la literatura son sus poemas Cántico espiritual, Llama
de amor viva y Noche oscura del alma.
12. Investiga sobre los orígenes del tópico carpe diem. ¿Qué
poetas lo habían utilizado en épocas anteriores? 18. ¿Cuál es la aspiración del poeta en el fragmento de la
Oda a la vida retirada? Cita los versos en que se ponga
Esta expresió n procede del poeta Horacio que la había utiliza- de manifiesto.
do en una oda. Cuatro siglos despué s el poeta Ausonio acude
al mismo concepto mediante el tó pico collige, virgo, rosas. Liberarse de las cosas mundanas (la gloria y el dinero) que
arrebatan la verdadera felicidad. Un no rompido sueño […]
13. Haz el análisis métrico de la estancia en la que Nemoroso no quiero ver el ceño / vanamente severo / de a quien la
llora la muerte de su amada Elisa. san- gre ensalza o el dinero.
11A, 11B, 11C, 11B, 11A, 11C, 7c, 7d, 7d, 11E, 11E, 11F, 7e, 11F. 19. Analiza la métrica de las estrofas de la Noche oscura del
14. Busca en un diccionario la palabra nemoroso y relaciona alma que acabas de leer. ¿Qué tipo de composición se
su significado con el hecho de que el pastor se llame así. utiliza?
Perteneciente al bosque. Cubierto de bosques. El nombre del La lira.
personaje se relaciona con el espacio en que se ambienta la 20. La poesía religiosa utiliza temas propios del
égloga. Renacimiento para hablar de asuntos religiosos. Explica
15. Investiga y elabora un breve resumen con el argumento esta afirmación basándote en los dos fragmentos que
de las églogas de Garcilaso. acabas de leer.
En la É gloga I nos encontramos con dos pastores que lloran El fragmento de la Oda a la vida retirada de Fray Luis nos
penas de amor: Salicio lamenta el distanciamiento de presenta un espacio idílico de la naturaleza donde el poeta
Galatea. Nemoroso llora la muerte de Elisa. pueda alejarse de las cosas mundanas que le impiden ser feliz.
Este poema puede interpretarse como una bú squeda de la paz
En la É gloga II se dan cita dos temas: el sufrimiento por
espiritual. El fragmento de la Noche oscura del alma presenta
amor y la alabanza de la vida heroica y militar. Albanio se
el motivo amoroso, el encuentro de los amantes en medio de la
lamenta de los desdenes de su amada Camila, quien llega a
noche es la máxima aspiració n de la amada. La interpretació n
hacerle enloquecer de dolor con su rechazo. Presa de la
religiosa de este poema nos remite a la bú squeda del alma, que
neurosis, Sali- cio y Nemoroso se ven obligados a reducirle.
ansía encontrar a Dios y unirse a él.
Nemoroso narra entonces una historia épica sobre la carrera
del duque de Alba. La dureza de la vida militar se presenta 21. ¿Qué rasgos de la novela picaresca puedes comentar en
como el mejor antídoto contra las locuras del amor herido. el fragmento?
En la É gloga III, cuatro ninfas del Tajo bordan tapices que El relato en primera persona, con narrador personaje (pro-
representan escenas de tragedias amorosas. Tres de ellas se tagonista). El realismo. La baja extracció n social de los
refieren a historias de la mitología clá sica, pero la ú ltima es personajes. El aprendizaje del protagonista: la lecció n de la
nuevamente la historia de Elisa y Nemoroso. desconfianza para saber salir adelante en un mundo hostil.

16. Una de las canciones de Garcilaso es la Oda a la flor de 22. ¿Cuál es la enseñanza que Lázaro aprende con ese epi-
Gnido. En ella, el poeta utilizó una composición métrica sodio?
que acabó conociéndose como lira, debido a que en la A no confiar en nadie, pues está solo en el mundo.
primera estrofa Garcilaso hablaba de ese instrumento 23. ¿Qué rasgos diferencian al escudero de los dos amos que
musical. Haz el análisis métrico: Lázaro había tenido anteriormente? ¿Qué comparación
Si de mi baja lira hace el protagonista entre ellos?
tanto pudiese el son, que en un Es bondadoso y educado, y trata bien a Lá zaro. No le da de
momento aplacase la ira comer porque no tiene nada que darle, pero Lá zaro ve la di-
del animoso viento ferencia que hay entre é l y los dos amos anteriores, quienes
y la furia del mar y el movimiento… sí tenían, pero no le daban nada por avaricia y mezquindad.
7a, 11B, 7a, 7b, 11B. 24. ¿Cuál es el único defecto que Lázaro encuentra en el
17.Investiga sobre las figuras de fray Luis de León y san Juan escudero? ¿Qué opinión le merece este personaje al pro-
de la Cruz y resume sus biografías. En el caso de fray tagonista?
Luis, haz una breve relación de las obras que escribió y el Su presunció n y el orgullo con que se comportaba, pese a es-
género al que pertenecen. tar muerto de hambre. Pese a ello, Lá zaro le tiene lá stima y
aprecio y afirma que le gustaría servir a alguien como él antes
Fray Luis de Leó n (1527-1591) nació en Belmonte (Cuenca).
que a cualquier otro.
Pertenecía a una familia con antepasados judíos. Ingresó en
los agustinos. Estudió en Salamanca. Durante cinco añ os 25. Un motivo temático muy importante en la novela
estuvo preso por la Inquisició n por su defensa de la versió n picares- ca es el contraste entre la apariencia y la
original de la Biblia. También fue acusado de traducir el Can- realidad. Obser- va cómo se trata este tema en los
tar de los cantares al castellano. Obra poética: Oda a la vida episodios del escudero y del buldero. ¿Qué intenta
retirada, Noche serena, A Francisco de Salinas, Profecía del aparentar cada uno de ellos?
Tajo. Obras en prosa: La perfecta casada, De los nombres de ¿Por qué?
Cristo, Exposición del libro de Job.
El escudero pretende aparentar riqueza y esconder la
nece- sidad que está pasando por puro orgullo y
presunció n. El
buldero era un estafador que intentaba agradar a sus víctimas
Al descubrir la llave que Lá zaro tenía en la boca y ver que
para luego poder engañ arlas.
era la del arcó n.
26. ¿De qué recursos se valía el buldero para engañar a sus
4. Atendiendo al desarrollo del episodio, ¿qué partes dife-
víctimas?
renciarías en el texto?
Hacía regalos a los clé rigos para ponerlos a su favor y logran
En el primer pá rrafo se narra la mala fortuna de Lá zaro, al
que convenciesen a los feligreses a tomar la bula. También era
co- locá rsele la llave en la boca y provocar el sonido que
muy observador y así veía con quié nes podía fingir cultura
llamaría la atenció n del clérigo.
y formació n, y con quié nes debía ser prudente para que no
sospechasen su falta de conocimientos. En los pá rrafos segundo y tercero se narra có mo el clé rigo
cree haber dado con la culebra y se dispone a matarla, para
27. En el fragmento del Tratado VII hay ciertas frases que
descubrir al final que es Lá zaro quien está allí.
aluden a la prosperidad que ha alcanzado Lázaro. Loca-
lízalas y anótalas. En los dos ú ltimos pá rrafos, el clé rigo expulsa a Lá zaro de
su casa.
Tengo en mi señor arcipreste todo favor y ayuda. Y siempre
en el año le da, en veces, al pie de un carga de trigo: por Análisis de la forma y el contenido. Las novelas picarescas
las Pascuas, su carne; y cuando el par de los bodigos, las tienen un narrador autobiográfico. Además, como todo texto
calzas viejas que deja […]. Los domingos y las fiestas casi narrativo, insertan otros tipos de discurso, como el diálogo
todas las comíamos en su casa. o la descripción.
28. Observa la intervención del arcipreste al final del último 5. ¿En qué elementos del texto puede apreciarse el tipo de
texto. La expresión «lo que le toca a una persona» solía narrador?
emplearse en el Siglo de Oro para referirse a la honra, lo En el uso de la primera persona del singular y los verbos en
que más debía importar a alguien. Sin embargo, el arci- pasado.
preste la usa con otro sentido. ¿Qué es, según él, lo que
6. Señala los momentos en que se introducen el diálogo y la
debe importar a Lázaro?
descripción, y comenta sus rasgos.
Su provecho, la comodidad que ahora existe en su vida, la
posició n social que ha alcanzado. Podemos hablar de descripció n en el primer pá rrafo, cuando
el narrador se refiere al sonido de su respiració n al rozar la
29. A partir de los fragmentos del Lazarillo que has leído, llave que estaba en su boca. Obsérvense las estructuras mo-
caracteriza al personaje: ¿Es inteligente? ¿Es compasivo dales: de tal manera y postura que el aire que yo durmiendo
o cruel? ¿Qué diferencias hay entre el Lázaro niño y el echaba salía por lo hueco de la llave; de tal manera que el
adulto? Justifica tus respuestas. sobresaltado de mi amo lo oyó. El diá logo se introduce al final
Es inteligente, pues aprende rá pido y se las arregla para so- del texto, en la intervenció n del clérigo. No obstante, también
brevivir. Es compasivo, como queda bien demostrado en el antes se ha empleado este tipo de discurso para exponer los
episodio del escudero, a quien Lá zaro mantiene, mendiga pensamientos del cura: Debió de decir el cruel cazador: El ra-
para él aunque para ello él mismo deba quedarse con hambre. tón y culebra que me daban guerra y me comían mi
El Lá zaro niñ o era inocente, pero perdió esa inocencia con el hacienda he hallado. Se utiliza el estilo directo. Obsérvense
paso de los añ os y las vivencias. Se convierte en un hombre también los verba dicendi: debió de decir, díjome.
escé ptico que conoce bien cuá les son las diferencias entre la Conclusiones. El pícaro es un personaje que aprende a so-
apariencia y la realidad de la vida, y decide vivir lo má s feliz brevivir con su astucia.
y có modamente que pueda con ese aprendizaje.
7. ¿En qué momentos del texto se alude a ese aprendizaje?
COMENTARIO DE TEXTO Al final, con la alusió n a su pasado como mozo de ciego.
Localización. El segundo amo de Lázaro es un clérigo ava-
ro que esconde bollos de pan en un arca, bajo llave. EL JARDÍN DE LA LITERATURA
Lázaro, hambriento, se hace con una copia de esta. El clérigo 1. La obra de santa Teresa de Jesús (1515-1582) tuvo una gran
cree al principio que los ratones se comen su pan y, después, importancia en la literatura mística de la segunda mitad
que se trata de una culebra. del siglo xvı. Lee este poema suyo y contesta después a
1. ¿Qué nos da a entender la mención al ciego que se hace las preguntas:
al final? Vivo sin vivir en mí,
Que los mozos de ciego (como lo ha sido el propio Lá zaro) y tan alta vida espero,
tienen una vida tan dura que aprenden muchos recursos para que muero porque no muero.
sobrevivir, no siempre lícitos. Vivo ya fuera de mí,
2. Señala las características de la novela picaresca que ob- después que muero de amor;
porque vivo en el Señ or,
serves en este fragmento.
que me quiso para sí:
El narrador personaje (protagonista) que cuenta los hechos cuando el corazó n le di
en primera persona, de manera autobiográ fica. El relato de puso en él este letrero,
los ardides que el personaje se ve obligado a poner en prá ctica que muero porque no muero.
para poder salir adelante, ardides que tienen que ver con su
Esta divina prisió n,
aprendizaje de niñ o hambriento en un medio hostil.
del amor en que yo vivo,
Tema y estructura. Lázaro se las ingenia para saciar su ham- ha hecho a Dios mi cautivo,
bre, pero el clérigo le acaba descubriendo. y libre mi corazó n;
3. ¿Cómo comprende el avaro que es Lázaro quien ha esta- y causa en mí tal pasió n
do comiendo el pan? ver a Dios mi prisionero,
que muero porque no muero.
¡Ay, qué larga es esta vida! lacionadas con la lucha o la guerra, como si la amada
¡Qué duros estos destierros,
esta cá rcel, estos hierros
en que el alma está metida!
Solo esperar la salida
me causa dolor tan fiero,
que muero porque no muero.
¡Ay, qué vida tan amarga
do no se goza el Señ or!
Porque si es dulce el amor,
no lo es la esperanza larga:
quíteme Dios esta carga,
más pesada que el acero,
que muero porque no muero.
Solo con la confianza
vivo de que he de morir,
porque muriendo el vivir
me asegura mi esperanza;
muerte do el vivir se alcanza,
no te tardes, que te espero,
que muero porque no muero.
a) Recuerda lo que has estudiado sobre los poetas as-
céticos y místicos, y delimita el tema central de este
texto.
b) ¿Qué sentido tienen los versos «vivo sin vivir en mí» y
«que muero porque no muero»? ¿Qué recurso estilís-
tico se emplea en ellos?
c) ¿Qué elementos propios del tema amoroso se em-
plean en este poema?
d) ¿Cómo interpretas las alusiones a la prisión que se
dan en todo el texto?
a) El yo poé tico expresa su dolor por no poder alcanzar al
amado (Dios).
b) La paradoja. Con estas expresiones se presenta el dolor por
el amor inalcanzable y el afá n persistente de alcanzarlo.
c) Se emplea el léxico propio del tema amoroso (muero de
amor, el corazón le di, prisión del amor, pasión, no te
tardes, que te espero). El planteamiento es el mismo del
amor cortés: el ser amado es inalcanzable y la espera del
amor provoca dolor en el yo poético.
d) La ausencia del amado hace sentir la vida como un lugar
de dolor y de angustia, de asfixia.
2. También tuvo una gran relevancia en el Renacimiento la
obra del poeta sevillano Fernando de Herrera (1534-
1597). El siguiente soneto es suyo, y aborda el tema
amoroso.
Yo vi unos bellos ojos, que hirieron
con dulce flecha un corazó n cuitado;
y que, para encender nuevo cuidado,
su fuerza toda contra mí pusieron.
Yo vi que muchas veces prometieron
remedio al mal, que sufro no cansado,
y que cuando esperé vello acabado,
poco mis esperanzas me valieron.
Yo veo que s’esconden ya mis
ojos y crece mi dolor, y llevo
ausente en el rendido pecho el
golpe fiero.
Yo veo ya perderse los despojos
y la membrana de mi bien presente
y en ciego engañ o de esperanza muero.
a) En la tradición petrarquista y del amor cortés, el sen-
timiento amoroso solía expresarse con metáforas re-
fuese un enemigo poderoso y bello, que somete al psicológicamente.
amante. Localiza en el texto dos momentos en que Respuesta libre.
se aprecia ese enfoque.
b) También es habitual en estas corrientes poéticas el
tópico de los ojos de la amada, que enamoran
inmedia- tamente al poeta. Explica la presencia de
este tópico en este soneto.
c) El yo poético habla de una «dulce flecha», del mal
«que sufro no cansado» y afirma que «en ciego
engaño de es- peranza muero». ¿Cómo interpretas
estas expresiones?
a) Versos 1 y 2: unos bellos ojos que hirieron con dulce
fle- cha un corazón cuitado. Verso 12: Yo veo ya
perderse los despojos.
b) Se presenta ya desde el principio: el amante dice
haber caído rendido y herido con la mera
contemplació n de esos ojos que lo hirieron (primer
cuarteto). Después, esos ojos está n ya presentes en
todo el texto, como promesas que encienden el deseo
del amante pero que nunca llegan a cumplirse.
c) La dulce f lecha nos remite al tó pico del dios que
lan- za flechas de amor provocando heridas
definitivas en los amantes. Es un arma que hiere,
pero es dulce, por- que es de amor. Que sufro no
cansado es una expresió n que alude a la obstinació n
del amante por aquello que le provoca dolor. Es un
tó pico en la literatura que tambié n encontramos al
hablar de la dulce flecha. En el mismo sentido
podemos interpretar la frase en ciego engaño de
esperanza muero, es decir, se presenta una vez má s
la contradicció n del sentimiento amoroso. En este
caso, la obstinació n del amante se centra en una
esperanza a la que se aferra aun sabié ndola vana.

LA FACTORÍA DE TEXTOS
1.¿Cómo es el espacio que describe fray Luis? Reescribe
su descripción empleando sinónimos, expresiones de
nuestro idioma en la actualidad y, en definitiva, tus
propias palabras. Eso sí: mantén el orden de los
elementos tal y cómo ha or- ganizado el poeta la
información sobre el espacio descrito.
Espacio idílico de la naturaleza. Locus amoenus.
2. ¿Cuáles son las categorías gramaticales más presentes
en los textos descriptivos? Justifica tu respuesta
basándote en el fragmento de la Oda a la vida
retirada.
Los adjetivos calificativos. Bella, cierto, codiciosa,
airosa, pura, sosegada, diversas, manso.
3. Piensa en cuál sería para ti un lugar idílico, donde te
en- contrarías sereno y feliz. Visualiza ese espacio,
dibújalo y, a continuación, descríbelo en un texto de
entre diez y quince líneas.
Respuesta libre.
4. Lee atentamente el fragmento del Lazarillo de
Tormes y, a continuación, divídelo en partes. ¿Qué
contenido tiene cada una?
En el primer pá rrafo se habla de su capacidad para fingir
de- voció n y humildad. En el segundo, de sus habilidades
cura- tivas para sacar el dinero a la gente. En el ú ltimo
pá rrafo se alude a su condició n mezquina y avarienta.
5. Piensa ahora en alguna persona con la que hayas
tenido experiencias desagradables en el pasado, como
le ocurrió a Lázaro con el ciego. Descríbela física y
ACTIVIDADES FINALES b) Atendiendo al contenido, diferencia los elementos
Repasa lo que has aprendido propios del amor cortés y los que se refieren a la na-
1. Define los siguientes conceptos y pon ejemplos: carpe turaleza idílica.
diem, égloga, estancia, poesía mística, novela picaresca, c) El yo poético hace una invitación o propuesta a su
texto descriptivo. ama- da: ¿cuál es? ¿qué argumentos le da para que
Carpe diem: tó pico literario heredado de la poesía clásica que acepte?
consiste en la invitació n al goce de la belleza y la juventud a) 11A, 11B, 11C, 11B, 11A, 11C, 7c, 7d, 7d, 11E, 11E, 11F, 7e, 11F.
presentes. Ejemplo: soneto XXIII de Garcilaso. b) El amor cortés se refleja en la actitud esquiva de la
É gloga: composició n poética de tema amoroso, ambientada en amada, que huye del amante desesperado y dolorido, y
un espacio idílico de la naturaleza y protagonizada por ninfas en la pro- pia expresió n del sufrimiento de é l. La
y pastores. Ejemplo, las tres que escribió Garcilaso. naturaleza idílica está presente en la descripció n del lugar
Poesía mística: corriente poética de tema religioso que aborda donde antañ o los amantes se encontraron y que ahora se
el tema del ansia de unió n con Dios, utilizando para ello los llena de recuer- dos tristes para el amante, por la
tó picos y motivos temá ticos de la lírica amorosa. Ejemplo: ausencia y esquividad de ella. Es un espacio que
Noche oscura del alma, de san Juan de la Cruz. corresponde al tó pico del lo- cus amoenus (un prado
lleno de verdura, un´espesura, un´agua clara…).
Novela picaresca: subgénero narrativo que se desarrolló en
Españ a a partir de la publicació n del Lazarillo de Tormes. Es c) La propuesta es que ella vuelva a ese lugar que una vez le
un relato en forma autobiográ fica y epistolar por parte de un fue tan querido, y le asegura que si él es la razó n por la que
personaje de baja extracció n social, que tiene diversas expe- se niega a venir, é l se alejará .
riencias, amos y oficios, y va aprendiendo a desenvolverse 4. Como sabes, los poetas religiosos del Renacimiento
en un medio hostil, con recursos no siempre lícitos ni utiliza- ban los tópicos amorosos para expresar su sentir
honestos. Este personaje es un pícaro.
religioso. Explica esta afirmación a partir del siguiente
Texto descriptivo: tipo de discurso que expone ordenadamen- fragmento del Cántico espiritual de san Juan de la Cruz:
te la informació n relativa a los rasgos y cualidades de
Allí me dio su pecho;
objetos, personas, espacios, etc. Su finalidad es pintarlos con
allí me enseñ ó ciencia muy sabrosa,
palabras para que puedan ser visualizados o comprendidos
y yo le di de hecho
por el lec- tor. Ejemplo: descripció n del locus amoenus en la
a mí, sin dejar cosa;
Oda a la vida retirada de Fray Luis de Leó n.
allí le prometí de ser su esposa.
2. En esta unidad has leído un famoso soneto de Garcilaso Mi alma se ha empleado,
que comenzaba así: «En tanto que de rosa y azucena…». y todo mi caudal, en su servicio;
¿Qué elementos propios de la descripción encuentras en ya no guardo ganado,
ese texto? ni ya tengo otro oficio,
Presencia de sustantivos que nombran el objeto descrito (co- que ya solo en amar es mi ejercicio.
lor, gesto, mirar, cabello, cuello, cumbre, rosa, etc.); abun- Pues ya si en el ejido1
dancia de adjetivos calificativos (ardiente, honesto, clara, de hoy más no fuere vista ni hallada,
presto, alegre, dulce, airado, hermosa, helado, ligera) o, en diréis que me he perdido;
su lugar, de elementos que complementan al nombre (del oro); que andando enamorada,
presencia de verbos que expresan acciones (mueve, esparce, me hice perdidiza, y fui ganada.
desordena, cubra, etc.) Comparaciones y metá foras (de rosa De flores y esmeraldas,
y azucena se muestra la color, por «rosa y blanco»; vena del en las frescas mañ anas escogidas,
oro, por «rubio»; primavera por «juventud»; cubra de nieve la haremos las guirnaldas,
hermosa cumbre, por «llene canas la cabeza»; marchitará la en tu amor florecidas
rosa el viento helado por «se ajará la belleza con la llegada y en un cabello mío entretejidas.
de la vejez», etcétera).
3. En el siguiente fragmento de la Égloga I de Garcilaso de 1
ejido: pasto.
la Vega, el pastor Salicio lamenta el desprecio de su
amada. Léelo atentamente y realiza después las
La unió n del alma con Dios se expresa como si se tratase
actividades:
de un encuentro amoroso entre amantes: le prometí de ser
Mas ya que a socorrerme aquí no vienes, su esposa (verso 5); que ya solo en amar es mi ejercicio
no dejes el lugar que tanto amaste, (verso 10); que andando enamorada (verso 14); haremos las
que bien podrá s venir de mí segura. guirnaldas, en tu amor florecidas y en un cabello mío en-
Yo dejaré el lugar do me dejaste; tretejidas (versos 10-20). La idea de esa unió n es una idea
ven si por solo aquesto te detienes. feliz, llena de promesas e ilusiones, como las del sentimiento
Ves aquí un prado lleno de verdura, amoroso.
ves aquí un’ espesura,
5. ¿Encuentras alguna relación entre el contenido del frag-
ves aquí un’ agua clara,
en otro tiempo cara,
mento anterior y el soneto de Garcilaso que has leído en
a quien de ti con lá grimas me quejo; esta unidad, «Escrito está en mi alma vuestro gesto»?
quizá aquí hallará s, pues yo m’alejo, ¿En qué partes?
al que todo mi bien quitar me La declaració n de incondicionalidad y la devoció n que se di-
puede, que pues el bien le dejo, rige al amado está n presentes en ambas composiciones. En
no es mucho que’l lugar también le quede. el fragmento de san Juan, obsé rvese especialmente la segun-
da lira (mi alma se ha empleado, y todo mi caudal, en su
a) Haz el análisis métrico del fragmento.
servicio...).
6. Haz el análisis métrico del fragmento del Cántico espiri- b) Las diferencias se refieren a las que existen entre aquellos
tual que acabas de leer. ¿Qué tipo de estrofa se emplea? que heredaron sus bienes y los que han tenido que salir
Todas las estrofas son liras. Esquema métrico: 7a11B7a7b11B. adelante en la vida por sus propios medios. Lá zaro está
en el segundo grupo.
7. Investiga en alguna enciclopedia y averigua cuál es el
argumento de la Diana, de Jorge de Montemayor. A con- c) En las ú ltimas líneas: Cuánto más hicieron los que, sién-
tinuación, haz un breve resumen de la trama en tu cua- doles contraria [la Fortuna] con fuerza y maña remando
derno y explica qué rasgos propios de la novela pastoril salieron a buen puerto.
puedes señalar.
Recuerda lo que ya sabías
Los siete libros de la Diana, de Jorge de Montemayor, es
una obra que se articula en torno a un argumento lleno de 10. ¿Qué representativo autor español había escrito sonetos
conflic- tos sentimentales y triá ngulos amorosos: la pastora antes que Garcilaso, en el siglo xv?
Diana es famosa por su hermosura. Es amada por los El marqué s de Santillana.
pastores Sireno y Sylvano, pero ella solo corresponde al
primero. Sin embar- go este tuvo que ausentarse durante 11. ¿Cuál es la estructura métrica del soneto?
largo tiempo del reino y ella se casó con otro, Delio, lo cual Catorce versos endecasílabos agrupados en dos cuartetos y
desencadena el conflicto en la trama, que ademá s se dos tercetos.
complica con la introducció n de otros personajes. Elemento
12.La novela picaresca, por el hecho de tener un antihéroe
propio de la literatura pastoril es el espacio bucó lico, pues la
novela está ambientada junto al río Esla y protagonizada por
como protagonista, se opone a otros géneros narrativos
pastores idealizados cuyas historias son amorosas. que se habían desarrollado durante el siglo xv. ¿Cuáles?
¿En qué consistían esos relatos?
8. Elabora un esquema con la vida de Lázaro de Tormes en
Los géneros que se habían desarrollado eran sobre todo los
el que recojas su origen, los amos que tuvo y la situación
relatos sentimentales y los caballerescos. Los sentimenta-
final que consiguió alcanzar y desde la que escribe. les narraban historias amorosas basadas a menudo en conflic-
Nacimiento de Lá zaro junto al río Tormes, en familia de baja tos y triá ngulos, y sus personajes estaban siempre tipificados
extracció n social. e idealizados. Los caballerescos narraban las aventuras de
Amos (Tratados I-V): ciego, clé rigo avaro, escudero pobre, caballeros andantes, heroicos, idealizados e intachables, y
fraile de la Merced y buldero estafador. protagonistas de historias igualmente idealizadas y, en oca-
siones, con elementos fantá sticos.
Oficios (Tratados VI y VII): repartidor de agua, ayudante de
alguacil y pregonero de vinos. 13. En el siguiente fragmento, localiza los adjetivos califica-
Situació n final: marido de la criada de un arcipreste, tivos y, cuando proceda, clasifícalos en especificativos y
posició n social buena. explicativos:
9. El siguiente fragmento pertenece al prólogo del Lazarillo Corrientes aguas, puras, cristalinas,
de Tormes. El protagonista se dirige al caballero a quien á rboles que os está is mirando en ellas,
verde prado de frescas sombras lleno,
se dispone a contar su historia. Léelo y contesta después
aves que aquí sembrá is vuestras querellas,
a las preguntas: hiedra que por los á rboles caminas,
Suplico a Vuestra Merced reciba el pobre servicio de mano torciendo el paso por su verde seno:
de quien lo hiciera má s rico si su poder y deseos se yo me vi tan ajeno
conformaran. Y pues Vuestra Merced escribe se les escriba y del grave mal que siento,
relate el caso muy por extenso, paresciome no tomalle por el que de puro contento
medio, sino del principio, porque se tenga entera noticia de con vuestra soledad me recreaba,
mi persona; y también porque consideren los que heredaron donde con dulce sueñ o reposaba,
nobles estados cuá n poco se les debe, pues Fortuna fue con o con el pensamiento discurría
ellos parcial, y cuá nto má s hicieron los que, siéndoles por donde no hallaba
contraria, con fuerza y mañ a remando salieron a buen sino memorias llenas de alegría.
puerto.
Garcilaso DE LA VEGA: fígloga I.
a) Repasa lo que has estudiado sobre el argumento del Corrientes, puras, cristalinas, verde (verso 3), frescas, lleno,
Lazarillo y explica el contenido de este fragmento. verde (verso 6), grave, puro, dulce. Especificativos: puras,
b) Lázaro justifica su trayectoria por la existencia de cristalinas, lleno. Explicativos: corrientes, verde (en los dos
dife- rencias sociales. ¿Cuáles son esas diferencias, casos), frescas, grave, puro, dulce.
según sus palabras? ¿En qué grupo se sitúa él? 14. ¿Recuerdas lo que es un epíteto? ¿Hay ejemplos en el
c) La novela picaresca narra el modo en que un persona- texto anterior?
je sale adelante pese a las dificultades de su entorno. Adjetivo calificativo explicativo, con valor retó rico y expre-
¿En qué parte del fragmento se alude a esta idea? sivo. En ese texto pueden citarse como ejemplos de epítetos
a) Lá zaro atiende la demanda que le ha hecho cierto caballe- los dos usos de verde, así como grave o puro.
ro (cuyo nombre no se desvela) para que explique el 15. ¿A qué sustantivos complementaban los adjetivos que
caso que motiva la narració n. Este caso no se desvela de has señalado en el fragmento de la actividad 13? ¿Qué
mo- mento, pero al final de la novela sabremos a qué se otros complementos del nombre localizas en ese texto?
refiere: hay rumores sobre la reputació n de Lá zaro, pues al
parecer su mujer tiene relaciones con el arcipreste que los Corrientes, puras y cristalinas se refieren a agua. Verde (ver-
protege a ambos. Lá zaro dice que para atender esa so 3) y lleno, a prado. Frescas, a sombras. Verde (verso 6),
demanda, le es necesario contar toda su historia, pues a seno. Grave se refiere a mal. Puro se refiere a contento.
solo así será comprendido. Dulce se refiere a sueño.
16. Señala ahora los verbos de ese fragmento y di qué com-
b) Redacta una descripción de Venus. ¿Responde al
plementos los acompañan.
canon de belleza de la época?
Estáis mirando: está complementado por el circunstancial
de lugar en ellas. c) Explica el parecido físico que hay entre la Venus de
Botticelli y la dama a la que se refería Garcilaso en el
Sembráis: está complementado por el objeto directo vuestras
querellas y por el circunstancial aquí.
poema «En tanto que de rosa y azucena».
Caminas: está complementado por el circunstancial por los d) En el cuadro La primavera, aparecen varios persona-
árboles y por la subordinada adverbial construida con gerun- jes de la mitología clásica: Mercurio, las tres Gracias,
dio torciendo el paso por su verde seno. Venus, Cupido, Flora y Céfiro. ¿Quiénes son? Explí-
Torciendo: se refiere a la oració n anterior (hiedra que calo por escrito.
cami- nas torciendo el paso). Está complementado por el a) Venus nació de la espuma del mar, que a su vez había
objeto di- recto el paso y por el circunstancial por su verde surgido de los ó rganos arrancados a Urano. Emerge en
seno. el interior de una concha y es empujada por los vientos
Vi: está complementado por el objeto directo (pronombre cé firos hasta la playa, donde la aguardan las Estaciones
re- flexivo) me y por el complemento predicativo tan ajeno. (las Horas), quienes la visten, la adornan, la cuidan y la
conducen al Olimpo. En el cuadro de Botticelli vemos a Ve-
Siento: está complementado por el pronombre relativo en fun- nus emergiendo del mar en la concha. A un lado los vientos
ció n de objeto directo, que (referido al grave mal). cé firos soplan sobre ella para impulsarla hacia la playa.
Recreaba: está complementado por el circunstancial con vues- Allí la aguarda la primavera dispuesta para cubrirla con
tra soledad. su manto.
Reposaba: está complementado por el circunstancial con dulce b) Sí responde a dicho canon. Es rubia, de piel blanca y son-
sueño. rosada, y ojos claros. Responde al tó pico de la donna
Discurría: está complementado por el circunstancial con el angelicata.
pensamiento y por la subordinada adverbial por donde no c) Ambas responden al tó pico de la donna angelicata, por
hallaba sino memorias llenas de alegría. la blancura sonrosada de su piel y el color dorado de sus
Hallaba: está complementado por el objeto directo memorias cabellos.
llenas de alegría y por el circunstancial no. d) Mercurio era el mensajero de los dioses. Las tres Gracias
17.Di qué modalidades oracionales se dan en este poema: representan el placer, el amor y la belleza, respectivamen-
te. Venus, diosa del amor, la belleza y la fertilidad, presi-
Ojos claros, serenos, de la escena y sobre ella vuela su hijo, Cupido. Céfiro (el
si de un dulce mirar sois alabados, viento) había raptado por amor a Clori, y de su unió n había
¿por qué, si me mirá is, mirá is airados? nacido Flora, que simboliza la primavera.
Si cuanto má s piadosos,
más bellos parecé is a aquel que os mira, 2. Otro artista italiano que trabajó motivos mitológicos fue
no me miréis con ira, el escultor Benvenuto Cellini. Observa esta escultura
porque no parezcá is menos hermosos. suya, titulada Perseo, que se alza en la plaza de la
¡Ay, tormentos rabiosos! Signoria de Flo- rencia. Busca información sobre Perseo y
Ojos claros, serenos, explica la escena que se reproduce en dicha escultura.
ya que así me mirá is, miradme al menos.
Gutierre DE CETINA Se reproduce la escena en que Perseo ha vencido a Medusa,
Modalidad apelativa interrogativa: ¿Por qué, si me miráis, cortá ndole la cabeza. Recordemos que Perseo logró derrotarla
miráis airados?. sin mirarla directamente, pues su mirada era letal. Luchó con
ella orientá ndose por su reflejo en el espejo de su escudo.
Modalidad apelativa exhortativa: no me miréis con ira; mi-
radme al menos. 3. El tema del amor ha recorrido siglos de poesía, a
Modalidad expresiva exclamativa: ¡Ay, tormentos rabiosos! través de la literatura y la música. El siguiente fragmento
corres- ponde a una bella canción del álbum La danza de
18. «Ya que así me miráis, miradme al menos»: ¿cuál es el la ciudad, del dúo Cómplices.
sujeto de la oración? ¿Es una oración compuesta? ¿Por
coordina- ción o por subordinación? ¿De qué tipo? Lee la letra y relaciona su contenido con el soneto de
Garcilaso «Escrito está en mi alma vuestro gesto».
El sujeto es gramatical, omitido, segunda persona del plural,
y se refiere a los ojos, que aparecen como vocativo. La Es por ti que veo ríos
oració n es compuesta por subordinació n, pues la estructura ya donde solo hay asfalto
que así me miráis es una subordinada adverbial causal. Es por ti que hay océ anos
donde solo había charcos
MIRA A TU ALREDEDOR Y… Es por ti que soy un
duende có mplice del viento
… ve más allá que se escapa de madrugada
1. El Renacimiento fue una corriente que floreció en el arte para colarse por tu ventana
y decirte:
y en la escultura, especialmente en Italia. Contempla
Tus labios son de seda,
estos cuadros del pintor Sandro Botticelli, y contesta a las
tus dientes del color de la luna llena,
pre- guntas: tu risa la sangre que corre por mis venas,
a) El nacimiento de Venus representa un motivo de la tus besos la tinta de mis versos,
mitología clásica. Investiga: ¿cómo nació esta diosa que siempre te cuentan.
según el mito? Explica cómo se recoge esta historia La amada justifica el modo de vivir del amante. Obsé rvese
en el cuadro de Botticelli. có mo se reproduce el paralelismo sintá ctico que recogía la
incondicionalidad del enamorado en el soneto de Garcilaso:
arzobispo de Sevilla. Fue presidente de la chancillería de
por vos nací, por vos tengo la vida, por vos he de morir, por
Valladolid e inquisidor general a partir de 1547. Se empeñ ó
vos muero, y aquí se dice: es por ti que veo ríos […] es por ti
en la confecció n del Índice de libros prohibidos y en la
que hay océanos […] es por ti que soy un duende. Por otra represió n de los focos protestantes de Valladolid y Sevilla.
parte, en el mencionado soneto de Garcilaso se presentaba a
la amada como inspiradora de los versos del poeta (cuanto yo Carlos V: Carlos I de Españ a y V de Alemania, reinó entre
escribir de vos deseo, vos sola lo escribisteis, yo lo leo) y este 1516 y 1556.
mismo motivo temá tico aparece en la canció n de Có mplices: Felipe II: hijo de Carlos I, reinó tras la abdicació n de su
tus besos la tinta de mis versos, que siempre te cuentan. padre, entre 1556 y 1598.
Don Carlos de Seso, corregidor de la ciudad de Toro y
… encuentra la clave fun- dador del grupo protestante de Valladolid.
1.El escritor español Miguel Delibes (1920-2010) es el b) En el añ o 1558, Carlos I escribe una carta a su hija Juana.
autor de una de las novelas históricas más documentadas En este momento el rey es ya Felipe II, y Juana gobier-
e inte- resantes de la narrativa española contemporánea: na como regente en su ausencia. En dicha carta, Carlos I
El hereje, ambientada en el siglo xvı, durante el reinado recomienda rigor y represió n inflexible contra el protes-
de Felipe II, en la época de los conflictos religiosos. En el tantismo para evitar que ocurriese lo mismo que había
siguiente fragmento se recogen los temores de un grupo sucedido en Alemania. Esto se relaciona con el proceso
de la Contrarreforma, iniciado en la segunda mitad del
de pre- sos luteranos que han sido descubiertos por la
siglo XVI por la Iglesia cató lica para frenar el avance de
Inquisición y son conducidos ante el tribunal del Santo los luteranos.
Oficio. Léelo atentamente y contesta después a las
c) Respuesta libre.
preguntas:
Don Carlos de Seso conocía la carta del inquisidor Valdés a 2. Hay ciertos temas que recorren los siglos sin perder su
Carlos V, retirado en Yuste, en la que rogaba que se atajase interés y su vigencia. Cambian solo las formas de
tan gran mal y que los culpados fueran punidos y castigados expre- sión. Compruébalo en el siguiente poema del
con el mayor rigor sin excepció n de ninguna clase. Seso poeta Fran- cisco Brines, y responde a las preguntas
inter- pretaba esto en el sentido de que se preparaba un después de leerlo:
escarmiento ejemplar, sin precedentes en Españ a. El corregidor Collige, virgo, rosas
de Toro dis- ponía de una gran habilidad para hacer amigos y
Está s ya con quien quieres. Ríete y goza. Ama;
hablaba con unos y otros sin distinció n, tanto con los oficiales
Y encié ndete en la noche que ahora empieza,
como con los soldados y, si se terciaba, con los familiares de
y entre tantos amigos (y conmigo)
la Inquisició n. Estaba al día de todo. Sabía todo. Temía tanto
abre los grandes ojos a la vida
a Felipe II como a Carlos V, y tenía el convencimiento de que
con la avidez preciosa de tus añ os.
antes de 1558 los castigos habrían sido má s leves, pero hoy
Pablo IV no ceja- ba, decía. En los descansos de la tarde les La noche, larga, ha de acabar al alba,
informaba de estos asuntos, de la carta del inquisidor Valdés al y vendrá n escuadrones de espías con la luz;
Emperador, de las de este a su hija, la gobernadora en se borrará n los astros, y también el recuerdo;
ausencia de su hermano, y a Felipe II, pidiendo prisa, rigor y y la alegría acabará en su nada.
recio castigo. Muchos no saldremos de esta, decía y llegó a Mas aunque así suceda, encié ndete en la noche,
tramar un plan para fugarse pero no encontró ocasió n de pues detrá s del olvido puede que ella renazca,
llevarlo a cabo. y la recobres pura, y aumentada en belleza,
si en ella, por azar, que ya será elecció n,
a) ¿Qué personajes históricos aparecen en el fragmento? sellas la vida en lo mejor que tuvo,
¿Qué sabes sobre ellos? cuando la noche humana se acabe ya del todo,
b) En el texto se habla acerca del año 1558. Investiga y venga esa otra luz, rencorosa y extrañ a,
sobre esa fecha e indica por qué el corregidor de Toro que antes que tú conozcas, yo ya habré conocido.
piensa que antes de ese año los castigos habrían sido a) ¿A qué tema estudiado en esta unidad alude el título
más leves. ¿Cómo relacionas esto con lo que has estu- del poema? ¿Qué significa?
diado al principio de la unidad sobre la sociedad y la
cultura españolas del siglo xvı? b) Escribe un texto literario con el siguiente tema: las
cosas más bellas de la vida son efímeras. Puedes esco-
c) Vivimos hoy en una sociedad en la que existe la liber- ger entre el verso o la prosa poética.
tad religiosa. Desde esta perspectiva del siglo xxı,
a) El título significa «coge, doncella, las rosas». Se alude al
¿cómo entiendes las luchas y persecuciones religio-
tema del disfrute de la juventud, la belleza y el momento
sas del siglo xvı? Redacta un escrito argumentado presente, representado habitualmente con el tó pico del
sobre estas ideas de unas quince líneas de extensión. Carpe diem.
a) Inquisidor Valdés: Fernando de Valdés (1487-1568), pre- b) Respuesta libre.
lado contrarreformista, obispo de Oviedo y Sigü enza y
AQUELLOS LOCOS TAN INGENIOSOS 10
APERTURA DE UNIDAD apellidos que acrediten la longevidad de un linaje. Por otro,

1. ¿A qué tipo de texto corresponde este fragmento?


Jus- tifica tu respuesta.
Corresponde a un texto descriptivo. El narrador se interesa
en perfilar la apariencia externa de Monipodio y representa
su aspecto físico y su indumentaria con gran abundancia de
adjetivos.
2. Al poco de llegar a Sevilla, los muchachos Rincón y Cor-
tado se integran en la cofradía de maleantes cuyo jefe
es Monipodio. ¿Qué impresión te causan los rasgos con
que es caracterizado dicho personaje? Por su parte, ¿qué
sienten hacia él todos los miembros de aquella «virtuosa
compañía»?
La apariencia de Monipodio es, sin duda, aterradora e
inspira un profundo respeto. No obstante, sus cofrades
sienten hacia él una extrañ a «devoció n», pues parecen verlo
con buenos ojos y se le muestran sumisos al recibirlo con
reverencia.
3. ¿Te parece que el retrato que ofrece Cervantes del señor
Monipodio es verosímil o es demasiado exagerado?
¿Qué rasgos horrendos destaca el narrador en él?
¿Cómo lo hace?
Al recurrir a la hipérbole, el narrador transforma a Monipodio
en una criatura tan desmesurada como poco verosímil. Su
pecho y sus manos está n cubiertas por el bello; su estatura
es tan descomunal como la talla de unos pies llenos de
juanetes. Para exageran, se ha echado mano, especialmente,
de la adje- tivació n en grado superlativo: muy negra (barba),
el hombre más basto y el más feo que se haya visto jamás
en el mundo.
4. Desde la más remota antigüedad, los textos literarios
siempre han identificado a los personajes más perversos
con una fealdad física, como dando a entender que son
incompatibles la hermosura y la maldad moral.
Haz un poco de memoria y enumera los personajes del
mundo de la literatura, el cine o el cómic que cumplan
con la tendencia aludida.
A continuación, intenta citar algún personaje que incum-
pla dicha costumbre.
Respuesta libre.

VIVE LA LECTURA
Comprensión lectora
1. ¿Cuál es el rasgo más sobresaliente del caballero Agilul-
fo? ¿Dirías que es un personaje fantástico o una
invención absurda? ¿Por qué?
El rasgo má s sobresaliente de Agilulfo es que carece de exis-
tencia corporal. Con tan increíble insuficiencia, má s que un
personaje fantá stico, se antoja una invenció n absurda
forjada así con un propó sito entre alegó rico y burlesco.
2. A pesar de su curiosa carencia, ¿qué le permite identifi-
carse con un caballero real? ¿Cuáles son sus objetivos?
La fuerza de voluntad es la que identifica a Agilulfo, al igual
que le ocurría a don Quijote con un caballero real. Sus objeti-
vos son los de servir con las armas la causa de la fe cristiana,
de ahí lo de santa causa.
3. ¿Qué intención crees que tenía el autor al bautizar a su
protagonista con un nombre con tantos apellidos?
El empleo de numerosos apellidos tiene una doble finalidad
paró dica. Por un lado, podría considerarse como una burla
hacia la ancestral costumbre aristocrá tica de enlazar má s

Lengua castellana y Literatura. 3.º ESO. Solucionario 89


10 AQUELLOS LOCOS TAN INGENIOSOS
resulta chocante que alguien que no existe posea tantos
ape- llidos. De algú n modo, la palabra sustituye en é l a la
materia.
4. ¿Qué actitud manifiesta el emperador Carlomagno hacia
sus caballeros? Justifica tu respuesta con algún ejemplo
del texto.
El emperador no parece guardarles el debido respeto a sus
caballeros. Las palabras del narrador que complementan la
dificultad del personaje a la hora de pronunciar el
larguísimo nombre de Agilulfo: Si tuviera que acordarme
del nombre de todos ¡estaría avisado!, evidencian el
distanciamiento, casi indiferencia, de Carlomagno hacia los
hombres que pelean por é l.
5. ¿Qué significa el pasaje: «cuanto más duraba la guerra
me- nos respeto por la limpieza veía en los paladines»?
El pasaje en cuestió n puede resultar altamente polisé mico.
Conforme avanza la guerra, los paladines, cansados y
heridos, se preocupan menos por su limpieza. Pero,
ademá s, tambié n puede pensar que la guerra embrutece a
los contendientes moralmente, de forma que el narrador
estaría aludiendo a una limpieza de espíritu.

Vocabulario
6. En este fragmento se mencionan varias piezas de la
arma- dura del caballero medieval. Con la ayuda del
diccionario, describe lo que es el quijote.
El quijote es una pieza de la armadura que cubría el muslo
del caballero.
7. Aparte de ser un objeto usado por los caballeros para
proteger la cabeza, ¿qué otra cosa puede ser una
celada? Busca la palabra en el diccionario.
Una celada puede ser también una «pieza de la ballesta», aun-
que, en general, se emplea el sustantivo como sinó nimo
de
«trampa o engañ o», y también con el significado de
«embos- cada de gente armada en un lugar oculto para
asaltar por sorpresa al adversario».
8. ¿Por qué Carlomagno emplea la segunda persona del
plural (alzáis, mostráis, etc.) para dirigirse al caballero
Agilulfo?
En el texto se remeda el estilo arcaico de las cró nicas me-
dievales, donde se usaba el vos, como fó rmula de respeto,
en lugar del tú.
9. ¿Mediante qué verbos representa el narrador el efecto
acústico de las palabras del protagonista?
La voz de Agilulfo llegaba metálica, es decir, «vibraba» y «re-
tumbaba» dentro del yelmo cerrado.
10. ¿Con qué palabras podrías reemplazar en el texto el
adje- tivo férrea y el sustantivo escalofrío? Enumera
cinco térmi- nos intercambiables para cada uno de ellos.
férrea: metá lica, dura, fuerte, resistente, só lida; escalofrío:
estremecimiento, sacudida, espasmo, temblor, impresió n.

Investigación y redacción
11. Averigua qué otras dos novelas completan la trilogía
de Calvino Nuestros antepasados.
a) ¿En qué consiste la originalidad de sus respectivos
argumentos y en qué época sitúan su acción?
b) ¿Existe, pues, alguna relación entre el título genérico
de la trilogía y la localización temporal de las novelas
que la integran?
a) Junto con El caballero inexistente, completan la trilogía:
El vizconde demediado y El barón rampante. El primer
relato
90 Lengua castellana y Literatura. 3.º ESO. Solucionario
transcurre en el siglo XVII. El vizconde Medardo de Terralba versal. Escribió comedias (El sueño de una noche de
es partido en dos por una bala de cañ ó n mientras participa verano)
en una batalla contra los turcos. Los mé dicos consiguen
salvarle la vida, pero el personaje se escinde en dos: una
mitad es buena y la otra es perversa. La segunda novela
se sitú a en el siglo XVIII y relata có mo Cosme Piovasco,
baró n de Ombrosa, se subió a un á rbol, huyendo de la orden
que le imponían sus familiares de comer un plato de
caracoles, y ante la amenaza de recibir un castigo cuando
descendiese del á rbol, decidió permanecer allí arriba
durante añ os.
b) De acuerdo con el título gené rico de la trilogía, Calvino
hace tres calas en épocas distintas, y pretéritas, de nuestra
historia para reflexionar sobre la excepcionalidad de la
condició n humana.
12. Para Agilulfo, la fuerza de voluntad y la fe son virtudes
suficientes para desempeñar una misión que, en su caso,
es imposible: el ejercicio militar. ¿También tú consideras
que tales impulsos son suficientes para conseguir alcan-
zar cualquier meta? ¿Por qué?
Respuesta libre.
13. Cuéntales a tus compañeros alguna situación comprome-
tida de la que tú mismo hayas logrado salir con la ayuda
de la fuerza de voluntad. Luego, convierte dicha historia
en un breve relato, de unas veinte líneas. Por último, ex-
plica qué diferencias sustanciales existen entre tu versión
oral y tu relato escrito.
Respuesta libre.

LITERATURA
1. En la unidad anterior has investigado sobre el Índice de
libros prohibidos, publicado por la Inquisición en 1551.
Explica por qué podía ser peligroso leer los libros en él
incluidos.
Los libros que figuraban en tal Índice resultaban peligrosos
porque desde el punto de vista de la Iglesia cató lica podían
difundir ideas heré ticas. Téngase en cuenta que, con el
avance de la Reforma protestante en el siglo XVI, la Iglesia y
tambié n la monarquía intentaron controlar lo que se
imprimía en las prensas, porque con su invenció n la
imprenta se transformó en un cauce idó neo para difundir
todo tipo de ideas.
2. Felipe II ordenó la construcción del Monasterio de San
Lorenzo de El Escorial, donde tuvo su residencia. Ave-
rigua qué motivos animaron al monarca a levantar tan
singular edificio.
El enorme edificio (compuesto por un palacio, una basílica,
un panteó n para dar sepultura a los monarcas, un monasterio
y una biblioteca), fue mandado construir, sobre todo, porque
el rey Felipe II deseaba conmemorar la victoria de sus tropas
contra los franceses en la batalla de San Quintín, que tuvo
lugar el día de San Lorenzo, el 10 de agosto de 1557. Aparte
de darle a su familia un lugar donde ser enterrada, tambié n
quería darle gracias a Dios, creando un centro que encarnase
el nuevo espíritu religioso surgido tras el Concilio de Trento.
3. Más arriba se ha relacionado la figura de Cervantes con
la de escritores contemporáneos como Shakespeare, Tas-
so y Camões. ¿En qué géneros literarios brillaron dichos
autores? Indica los títulos más representativos de la pro-
ducción de cada uno de ellos con una breve referencia
al asunto tratado.
William Shakespeare (1564-1616) fue, sin duda, la figura cum-
bre del teatro isabelino inglés e incluso de la dramaturgia uni-
y dramas histó ricos (Julio César, Ricardo III), aunque son
más famosas sus tragedias (Romeo y Julieta, Otelo, Macbeth,
El rey Lear y Hamlet).
El italiano Torquato Tasso (1544-1595) escribió el libro Diá-
logos, siguiendo la moda de los diá logos plató nicos, así como
dos mil poemas líricos (Rime) y los poemas Mondo creato
(basado en la Biblia) y Aminta, oscilando entre lo dramá ti-
co y lo pastoril. No obstante, su obra más reconocida fue el
poema épico la Jerusalén liberada (Jerusalén conquistada,
en su versió n definitiva), donde relataba las hazañ as de los
cristianos durante los ú ltimos tiempos de la Primera Cruzada
hasta liberar el Santo Sepulcro.
Finalmente, en Portugal nos hallamos con Luis Vaz de Ca-
mõ es (¿1524?-1580). Escribió poemas líricos, adaptá ndose a
la moda petrarquista. Pero fue Os Lusiadas su obra más ce-
lebrada. Se trataba de un poema é pico en diez cantos, cuyo
modelo era la Eneida de Virgilio y donde hacía un repaso de
los episodios má s gloriosos de la historia de su país, a partir
del motivo central del viaje de Vasco de Gama hasta las
Indias Orientales.
4. En los capítulos XXXVII y XXXVIII de la primera parte del
Quijote, el protagonista enlaza su famoso discurso sobre
las armas y las letras, donde dice cosas como estas: «Quí-
tenseme delante los que dijeren que las letras hacen ven-
taja a las armas, que les diré […] que no saben lo que
dicen» o «No hay ninguno más pobre [que el soldado] en
la misma pobreza, porque está atenido a la miseria de su
paga, que viene o tarde o nunca». ¿Qué relación adviertes
entre estas afirmaciones y algún aspecto de la biografía
cervantina?
Al decir que No hay ninguno más pobre [que el soldado],
Cervantes estaba reflejando su propia experiencia personal.
Si bien siempre se sintió orgulloso de su participació n en la
batalla de Lepanto, estaba muy dolido por el hecho de que su
sacrificio no fuera recompensado por sus superiores, de forma
que, siendo soldado, quedó manco, sufrió cautiverio en Argel
y nunca pudo escapar a la miseria.
5. ¿Qué visión nos ofrece Cervantes de los poetas en estos
tercetos de su Viaje al Parnaso? Justifica tu respuesta con
ejemplos.
Pasa, raro inventor, pasa adelante
con tu sutil designio, y presta ayuda
a Apolo, que la tuya es importante,
antes que el escuadró n vulgar acuda
de má s de veinte mil sietemesinos
poetas que de serlo está n en duda.
Llenas van ya las sendas y caminos
desta canalla inú til contra el monte,
que aun de estar a su sombra no son dignos.
Aunque diversos estudiosos señ alan que la visió n que Cer-
vantes expresa de los poetas contemporá neos es convencio-
nal, en sus versos se advierte cierto rechazo hacia ellos, que
pudo deberse a su resentimiento por no ser valorada su propia
obra por los demá s. Así, en su alegó rico viaje, los poetas se
presentan como escuadrón vulgar […] de más de veinte mil
sietemesinos y como canalla inútil e indigna.
6. En el entremés El viejo celoso, Cervantes teatralizó un
motivo ya desarrollado en varios cuentos del Decamerón
de Boccaccio. ¿De qué tema se trata? Investiga e intenta
descubrir en qué novela ejemplar el autor se ocupó del
mismo asunto y qué diferencias hay entre el argumento
de una obra y otra.
El asunto tratado en el entremés es el del marido viejo y ce-
loso que se casa con una mujer joven y hace todo lo posible
para que la esposa no conozca otro varó n. A partir de aquí,
verdugos que nos tienen lá stima y las mozas que nos socorren
siempre aparece un galá n joven que usará de su picardía
cuando acabamos en la trena 2. ¡Qué sería de nosotros sin nues-
para acceder al lugar donde está la esposa y engañ ar al ma-
tros bienhechores…! Son tantos los favores que les debemos
rido. Astucia e infidelidad son motivos que Cervantes vol-
que, cada añ o, celebramos su adversario3 con la mayor popa4
vió a manejar en la novela ejemplar El celoso extremeño,
y soledad5.
cuyo argumento debe contrastar el alumno con el del citado
entremé s. 1
guros: alguaciles. 2 trena: cárcel. 3 adversario: deformación de
7. En El licenciado Vidriera, tras visitar diversas ciudades «aniversario». 4 popa: deformación de «pompa». 5 soledad: defor-
europeas y haber completado sus estudios en mación de «solemnidad».
Salamanca, Tomás Rodaja es pretendido por una dama
que le engaña de este modo para conseguir su amor: a) ¿Qué aspectos de este pasaje permiten situar el re-
Comió en tal mal punto Tomá s el membrillo, que al lato en el grupo de las novelas realistas? Justifica tu
momento comenzó a herir de pie y de mano como si tuviera respuesta con ejemplos extraídos del texto.
alferecía1, y sin volver en sí estuvo muchas horas, al cabo de b) Explica cómo queda caracterizado el personaje de
las cuales volvió como atontado, y dijo con lengua turbada y
Monipodio a través de su forma de hablar.
tartamuda que un membrillo que había comido le había
muerto, y declaró quié n se le había dado. La justicia, que tuvo c) ¿Te resulta cómico el fragmento? Explica por qué.
noticia del caso, fue a buscar la malhechora; pero ya ella, a) En este pasaje, así como en el conjunto del relato, se ignora
viendo el mal suceso, se había puesto en cobro, y no pareció por completo la temá tica amorosa. En cambio, cobra pro-
jamá s. tagonismo la crítica social a partir de la complicidad que
Seis meses estuvo en la cama Tomá s, en los cuales se secó y desvela Monipodio entre su cofradía de malhechores y una
se puso, como suele decirse, en los huesos, y mostraba tener justicia que es corrupta y favorece a los delincuentes:
turbados todos los sentidos; y aunque le hicieron los remedios Son tantos los favores que les debemos.
posibles, solo le sanaron la enfermedad del cuerpo, pero no b) Monipodio queda ridiculizado como orador y líder al co-
de lo del entendimiento; porque quedó sano, y loco de la má s meter notables incorrecciones lingü ísticas. Prueba de su
extrañ a locura que entre las locuras hasta entonces se había incultura es que confunde términos como aniversario,
visto. Imaginose el desdichado que era todo hecho de vidrio, pompa y solemnidad.
y con esta imaginació n, cuando alguno se llegaba a é l, daba
c) La complicidad del fragmento deriva de la inversió n de
terribles voces, pidiendo y suplicando con palabras y
papeles (segú n el tó pico del «mundo al revés») entre los de-
razones concertadas que no se le acercasen, porque le
lincuentes y los miembros de la justicia, transformados
quebrarían; que real y verdaderamente é l no era como los
aquí en bienhechores. No es habitual que unos pícaros
otros hombres: que todo era de vidrio, de pies a cabeza.
re- zaran por aquellos que, supuestamente, deberían ser
sus perseguidores.
1
alferecía: enfermedad similar a la epilepsia, que provoca des- 9. Averigua cuál es el final de la segunda parte del Quijo-
mayos y convulsiones. te. ¿Cómo puedes relacionar esto de algún modo con la
aparición de la falsa continuación escrita por Fernández
a) ¿Qué sorprendentes efectos tienen en el personaje de Avellaneda?
las artes perversas de la malhechora? Asimismo, ¿qué Al final de la segunda parte, don Quijote regresa a su casa
explicación verosímil podría dársele a un suceso tan derrotado y, poco después, enferma y recupera el juicio an-
inaudito? tes de morir cristianamente. Si su muerte puede ser interpre-
tada como un suceso que apuntala aú n má s la verosimilitud de
b) Aparte de la extraña enfermedad que aflige al licen- la historia, no es menos cierto que con este desenlace Cervan-
ciado, ¿de qué otro modo deslumbra el personaje a tes cerraba las puertas a la posibilidad de que otros autores
todo el mundo? Busca información al respecto. volvieran a continuar su relato como ya lo hizo el enigmá tico
a) Al ingerir el extrañ o membrillo, Tomá s Rodaja tiene con- Alonso Ferná ndez de Avellaneda.
vulsiones y luego sufre un desmayo. Pese a recuperar la 10. En el capítulo IX de la primera parte del Quijote, el narra-
conciencia, permanece seis meses convaleciente. Pero lo
dor nos refiere el casual descubrimiento de unos papeles
que es peor, creyé ndose ser de cristal y temiendo que su
cuerpo se resquebraje al menor contacto. Si a nivel que le servirán de gran utilidad para su tarea literaria.
ficticio, puede llegar a pensarse en un caso de brujería, ¿En qué persona está narrado el siguiente fragmento?
de modo má s verosímil cabe pensar en una misteriosa ¿Es omnisciente el narrador principal? ¿Por qué?
suerte de envenenamiento. Estando yo un día en el Alcaná de Toledo, llegó un mucha-
b) Asimismo, resultará sorprendente lo que ocurre con el li- cho a vender unos cartapacios y papeles viejos a un sedero;
cenciado Vidriera tan pronto sale a la calle. Su aparente y como yo soy aficionado a leer aunque sean los papeles
locura no le impide responder a las preguntas que le formu- rotos de las calles, llevado de esta mi natural inclinació n
la la gente con singular lucidez, convirtiéndose sus tomé un cartapacio de los que el muchacho vendía y vile con
juicios en un instrumento de crítica social. En su doble caracteres, que conocí ser ará bigos. Y puesto que aunque
condició n de loco-cuerdo, Tomá s Rodaja es un personaje los conocía no los sabía leer, anduve mirando si parecía por
muy similar a don Quijote. allí algú n morisco aljamiado1 que los leyese, y no fue muy
dificultoso hallar inté rprete semejante […]. En fin, la suerte
8. En Rinconete y Cortadillo, apenas ingresan los protago- me deparó uno, que, diciéndole mi deseo y poniéndole el libro
nistas en una cofradía de delincuentes, Monipodio les en las manos, le abrió por medio, y, leyendo un poco en é l,
recomienda lo siguiente: se comenzó a reír. […].
Y os pido asimismo que recéis por los bienhechores de nuestra Le di priesa que leyese el principio, y haciéndolo ansí, vol-
cofradía, que son los guros1 que nos alertan de que la justi- viendo de improviso el ará bigo en castellano, dijo que
cia nos persigue y los jueces que hacen la vista gorda y los
decía: Historia de don Quijote de la Mancha, escrita por Cide
no solo es más prudente, sino que rehú ye a los peligros (co-
Hamete Benengeli, historiador arábigo.
menzó a llorar y bien podemos torcer el camino y
desviarnos del peligro). Es, por tanto, menos idealista y má s
1
morisco aljamiado: morisco que habla castellano. prá ctico, e incluso cobarde.
13. ¿Qué tema de los mencionados reconoces en el siguiente
El fragmento está narrado en primera persona: Estando yo. pasaje (Parte I, cap. XXI)? ¿En qué discrepan la opinión
No se trata, sin embargo, de un narrador-personaje, sino de un de don Quijote y la de su escudero?
narrador que habla de su propia tarea creativa y cuenta De allí a poco, descubrió don Quijote un hombre a caballo, que
có mo descubrió unos papeles en á rabe que mandó traducir traía en la cabeza una cosa que relumbraba como si fuera de
al cas- tellano. De ese modo, al dar con una nueva fuente, oro, y aú n é l apenas le hubo visto, cuando se volvió a Sancho
aparte de conferirle a su relato un mayor verismo y le dijo:
historiográ fico, de- muestra que no es un narrador
omnisciente, ya que contrasta fuentes ajenas. —[…] Si no me engañ o, hacia nosotros viene uno que trae en
su cabeza puesto el yelmo de Mambrino […].
11. A partir del motivo del manuscrito encontrado,
—Mire vuestra merced bien lo que dice, y mejor lo que hace
Cervantes deja ver su tendencia a jugar con los tópicos —dijo Sancho— […]. Lo que yo veo […] no es sino un hombre
heredados. En el relato de una de las aventuras más sobre un asno pardo, como el mío, que trae sobre sobre la
cómicas de la pri- mera parte, se nos desvela la insólita cabeza una cosa que relumbra.
identidad del arriero que pretende mantener una
En el pasaje cobra vida el tema del perspectivismo, del con-
relación amorosa con Mari- tornes, la fea criada de una flicto entre los sentidos y las apariencias. Una misma realidad
venta. De él se dice: puede ser interpretada de formas distintas. Mientras, a lo le-
Era uno de los ricos arrieros de Arévalo, segú n lo dice el jos, Sancho solo divisa un hombre sobre un asno llevando en
autor desta historia [Cide Hamete], que deste arriero hace la cabeza una cosa que relumbra, la imaginació n libresca de
particular menció n porque le conocía muy bien, y aun don Quijote dicta que aquel individuo va montado sobre un
quieren decir que era algo pariente suyo. caballo y lleva sobre la cabeza el yelmo de Mambrino.
a) Teniendo en cuenta este pasaje, ¿cómo podemos pre- 14. Gustavo Martín Garzo empieza el relato de Dulcinea y el
sumir que conoció el historiador arábigo los sucesos Caballero Dormido del modo siguiente:
ocurridos en la venta que él relatará sin haber estado Ya nadie recuerda el caballero. Hace añ os sus aventuras es-
presente en dicho lugar? taban en boca de todos, y eran motivo de regocijo en ventas
b) ¿Qué consigue Cervantes con esta estrategia? y mercados, pero hoy nadie pregunta por é l […]. Ahora que
soy vieja, cuando veo a los niñ os del pueblo correr y jugar
a) A partir de este pasaje, puede presumirse que el historia- por las calles, me pregunto có mo podrá n enfrentarse a la
dor á rabe conoció unos sucesos en los que no había estado tristeza del mundo sin la ayuda de alguien como él. También
presente a partir de la versió n de los mismos que pudo pienso en lo afortunada que fui, pues entre todas las mujeres
facilitarle su pariente el arriero. del mundo me eligió a mí para transformarme en su dama.
b) Con tal estrategia, Cervantes no solo juega con la comple- Dulcinea del Toboso, ¿hay un nombre má s gracioso y
jidad de las voces narrativas, sino que le otorga a una de delicado que ese? Suele decirse que Dulcinea no era de este
sus fuentes principales una corporeidad física. De algú n mundo sino una figu- ra fantá stica que el caballero engendró
modo venía a decirnos que el tal Cide Hamete podía ser en su entendimiento, pintá ndola con todas las gracias y
tan real, y ademá s contemporá neo, como el arriero de la perfecciones que acertó a imaginar, pero esto no es cierto y
venta. basta para desmentirlo el que mi nombre y mi linaje
aparezcan escritos sin veladura en el libro que recorrería el
12. ¿Con qué rasgos describirías a los personajes que in- mundo llevando noticias de su ú nica y maravillosa vida.
tervienen en este diálogo del capítulo XX de la primera
parte? a) ¿Quién es el narrador en el fragmento citado? ¿Qué
intenta demostrar sobre sí mismo?
—Sancho amigo, has de saber que yo nací por querer del
cielo en esta nuestra edad de hierro para resucitar en ella la b) ¿Cuál es su actitud hacia don Quijote? ¿Con qué tér-
de oro, o la dorada, como suele llamarse. Yo soy aquel para minos la expresa?
quien está n guardados los peligros, las grandes hazañ as, los a) El narrador es en este fragmento el personaje de la mis-
vale- rosos hechos […]. Yo soy, digo otra vez, quien ha […] de ma Dulcinea del Toboso. Intenta demostrar que no es
poner en olvido los Platires, los Tablantes, Olivantes y una figura fantá stica.
Tirantes, los Febos y Belianises […].
b) Dulcinea evoca con nostalgia al caballero, hacia el que
Cuando Sancho oyó las palabras de su amo, comenzó a llorar se siente agradecida (lo afortunada que fui) por haberla
con la mayor ternura del mundo y a decille: convertido en su dama. Ademá s, lamenta tanto la desa-
—Señ or, yo no sé por qué quiere vuestra merced acometer parició n de don Quijote como que nadie se acuerde de él.
esta tan temerosa aventura. Ahora es de noche, aquí no nos ve Para ella, el caballero fue una criatura ú nica, capaz de
nadie: bien podemos torcer el camino y desviarnos del peligro, enfrentarse en solitario a la tristeza del mundo.
aunque no bebamos en tres días. 15. Localiza y lee la pieza teatral «Sancho Panza en la ínsu-
Don Quijote se nos aparece como un ser idealista y soñ ador, la», perteneciente al Retablo jovial, de Alejandro Casona.
al sentirse destinado a resucitar la edad de oro. Es valien- Resúmela indicando cuáles son los valores que el autor
te y no teme las posibles adversidades que puedan asaltarle destaca del famoso escudero.
en su camino: Yo soy aquel para quien están guardados los
peligros. En su vasto conocimiento de los má s famosos ca- El resumen deberá dejar constancia de la reivindicació n del
balleros literarios, demuestra ser, asimismo, un gran lector sentido comú n de Sancho para impartir, de forma instintiva,
de este tipo de literatura. Por su parte, su escudero Sancho justicia.
COMENTARIO DE TEXTO La existencia de esa dorada edad a la que se refiere don Qui-
Localización. Este fragmento está extraído del discurso de jote no solo es indemostrable, sino que parece un imposible.
la Edad de Oro que enlaza don Quijote, al ser acogido por ¿Có mo, por ejemplo, podían saciar su hambre los antiguos con
los frutos que les daba la naturaleza y sin pelearse? ¿Quizá
unos cabreros, en el capítulo XI de la primera parte. A través
entonces el hombre carecía de cualquier vicio o defecto? La
de su parlamento, el personaje rescata un tópico literario de creencia del personaje en tal utopía viene a destacar su exa-
procedencia clásica. cerbado idealismo.
1. ¿Qué relación existe entre el tópico de la Edad de Oro
y el mito de las Edades del Hombre? EL JARDÍN DE LA LITERATURA
Para el escritor griego Hesíodo, había cinco edades del hom- 1. En este fragmento (Parte I, capítulo II), el narrador del
bre. La Edad de Oro fue la primera de tales edades. En la Quijote reproduce las primeras palabras del protagonista
época imperial romana, autores como Virgilio redujeron a dos de la novela al abandonar su casa para convertirse en
las edades del mundo: la de Oro y la de Hierro.
caballero andante:
Tema y estructura. Según lo dicho por los antiguos: Yendo, pues, caminando nuestro flamante aventurero, iba ha-
2. ¿Qué dos épocas se contrastan? ¿Cuáles son sus respec- blando consigo mismo y diciendo:
tivas características? ¿Cómo valorarías la evolución del —¿Quién duda sino que en los venideros tiempos, cuando sal-
mundo? ga a luz la verdadera historia de mis famosos hechos, que el
El discurso de don Quijote se ajusta a lo dicho por Virgilio sabio que los escribiere no ponga, cuando llegue a contar
y distingue dos épocas: una primera, dorada, remota y ex- esta mi primera salida tan de mañ ana, desta manera?:
cepcional; y otra, presente y detestable. En aquella, como to- «Apenas había el rubicundo Apolo tendido por la faz de la
dos los bienes terrenales eran comunes, a nadie le faltaba de ancha y es- paciosa tierra las doradas hebras de sus
nada y no había conflictos de interés. En esta, el desarrollo hermosos cabellos, y apenas los pequeñ os y pintados
de la malicia, del amor por el oro y de otros vicios ha preci- pajarillos con sus harpadas lenguas habían saludado con dulce
sado de la institució n de la caballería andante para imponer y meliflua armonía la veni- da de la rosada aurora, que, dejando
la justicia en el mundo. Del contraste entre ambas etapas se la blanda cama del celoso marido, por las puertas y balcones
desprende que el mundo ha experimentado una evolució n ne- del manchego horizonte a los mortales se mostraba, cuando
gativa y descendente. el famoso caballero don Quijote de la Mancha, dejando las
Análisis de la forma y el contenido. La evocación de tiem- ociosas plumas, subió so- bre su famoso caballo Rocinante y
comenzó a caminar por el antiguo y conocido campo de
pos pretéritos se concreta mediante un armonioso ritmo
Montiel».
sintáctico.
3. Selecciona dos periodos enumerativos y analiza su a) Según don Quijote, ¿quién recogerá por escrito sus
estruc- tura. ¿Qué se repite en ellos? Asimismo, aventuras? En cambio, ¿por qué crees que es el per-
¿encuentras sin- tagmas en los que aparezcan dos sonaje quien se preocupa por aquello que debe decir
palabras de una misma categoría gramatical? en su libro?
La modulació n sintá ctica se evidencia mediante periodos b) Posiblemente, la forma de hablar de don Quijote te
paralelísticos que poseen, ademá s, una funció n enfá tica. resultará un poco extraña, pues imita el estilo de los
Advié rtase la insistencia en la partícula negativa y los tres libros de caballerías. Con ayuda de Internet o de
complementos directos enumerados en No había la fraude, alguna enciclopedia, explica brevemente cómo era el
el engaño ni la malicia. Algo similar ocurre con la reitera- estilo de dichas obras.
ció n, mediante coordinació n copulativa, de dos subordinadas a) Don Quijote dice que sus aventuras las plasmará en un
sustantivas de complemento directo: no había qué juzgar libro un sabio o mago, tal y como ocurría en los libros de
ni quién fuese juzgado. Asimismo, el personaje se muestra caballerías. Se preocupa tanto por lo que debe decir
re- tó rico al duplicar los adjetivos calificativos en sintagmas dicha obra porque está obsesionado con la idea de
del tipo dulce y sazonado fruto y fértil y espacioso seno. superar a las figuras literarias que pretende imitar y
4. ¿Qué referencia del texto a un episodio de la mitología desea transformar su existencia en una auté ntica obra de
clásica ilustra la importancia de esta materia en el Rena- arte.
cimiento? b) El estilo de los libros de caballerías era, en ocasiones, anti-
El texto se refiere al mito del laberinto de Creta, construido cuado. Otras veces, era enrevesado y complejo, con abun-
por Dédalo y donde estaba encerrado el Minotauro al que lo- dancia de oraciones subordinadas. Ademá s, los autores
gró derrotar Teseo. crearon un lenguaje bastante tipificado en cuanto al
léxico y las estructuras sintá cticas empleadas. De esto
5. ¿Consideras la expresión «nuestra primera madre» una
ú ltimo es reflejo el tó pico del amanecer mitoló gico al que
metáfora? ¿Por qué? Localiza alguna hipérbole.
recurre don Quijote, reproduciendo su misma retó rica.
La expresió n nuestra primera madre es el término imagina-
rio con que se identifica el término real «la tierra». Puesto que 2. En un magnífico estudio sobre el Quijote, el escritor
ambos términos se equiparan podemos hablar de metá fora. mexi- cano Carlos Fuentes destacaba que el protagonista
Por otra parte, en su intento de destacar las bondades de la halla- ba en la lectura un asidero ante el desconcierto y la
mítica edad de oro, el caballero exagera aquel mundo paradi- duda que asaltaban al hombre de la época:
síaco: Todo era paz entonces, todo amistad, todo concordia. La sinonimia de la lectura, la locura, la verdad y la vida en
Conclusiones don Quijote son de una evidencia llamativa cuando pide a los
mercaderes que se encuentra en el camino que confiesen la
6. ¿Con qué argumentos afirmarías que el discurso de don
belleza de Dulcinea sin haberla visto nunca, pues «lo impor-
Quijote tiene un valor utópico? ¿Qué rasgo del personaje tante es que sin haberla visto lo creyeres, confesares, jurares
destaca? y defendieres». Ese lo es un acto de fe. Las fabulosas aventu-
ras de don Quijote son impulsadas por un propó sito avasa-
llante: lo leído y lo vivido deben coincidir de nuevo, sin
las
dudas y oscilaciones entre la fe y la razó n introducidas en el viernes, algú n palomino4 de añ adidura los domingos,
Renacimiento. consumían las tres partes de su hacienda. El resto de ella
Cervantes o la crítica de la lectura concluían sayo de
a) ¿También a ti te merecen los libros la misma
confianza que depositaba en ellos don Quijote? ¿Por
qué?
b) ¿Qué podría ocurrir si el mundo de tus lecturas no se
ajustase a la realidad?
a) y b) Respuesta libre.
3. En La Galatea, Cervantes reproduce los tópicos de la
literatura pastoril, incorporando con frecuencia poesías
y parlamentos con gran lirismo:
El lastimado pastor, creyendo que ninguno le oía, soltó la voz
a semejantes razones:
—¡Amor, […] ya que tanto bien me hiciste, no quieras mos-
trarte agora, hacié ndome el mal en que me amenazas, que es
más mudable tu condició n que la de la variable Fortuna. Mira,
señ or, cuá n obediente he estado a tus leyes, cuá n pronto a
seguir tus mandamientos, y cuá n sujeta he tenido mi voluntad
a la tuya.
¡Oh verdes prados, que con su vista os alegrá bades! ¡Oh
flores olorosas, que de sus pies tocadas, de mayor fragancia
é rades llenas! ¡Oh plantas, oh á rboles desta deleitosa selva!,
haced todos, en la mejor forma que pudiéredes, aunque a
vuestra naturaleza no se conceda, algú n gé nero de
sentimiento que mueva al cielo a concederme lo que le
suplico!
a) ¿A qué famosas composiciones líricas de Garcilaso de
la Vega te recuerdan los lamentos del pastor?
b) ¿A quiénes les traslada el pastor sus penas en este
caso? ¿Encuentras alguna personificación?
a) Los lamentos del pastor cervantino recuerdan aquellos
otros de Salicio y Nemoroso en las églogas garcilasianas.
b) El personaje, despué s de declararse vasallo del Amor y
quejarse, le confía sus penas a los prados, las flores, plan-
tas y á rboles, elementos todos de una naturaleza bucó li-
ca, para que se conmuevan de su desdicha. Aparte de la
tó pica personificació n del Amor, tambié n los elementos
naturales está n personificados, en tanto que se transfor-
man en los confidentes del pastor.

LA FACTORÍA DE TEXTOS
1. Escribe en diez líneas un retrato completo de tu mejor
compañero de clase.
Respuesta libre.
2. Aplica los rasgos característicos de la descripción científi-
co-técnica para informar a tus compañeros de las
virtudes de un instrumento imaginario que acabas de
inventar.
Respuesta libre.

ACTIVIDADES FINALES
Repasa lo que has aprendido
1. A continuación, te invitamos a leer el pasaje con que co-
mienza la primera parte del Quijote. Luego, resuelve las
cuestiones que se te plantean.
En un lugar de la Mancha, de cuyo nombre no quiero
acordar- me, no ha mucho tiempo que vivía un hidalgo de los
de lanza en astillero1, adarga2 antigua, rocín flaco y galgo
corredor. Una olla de algo má s vaca que carnero, salpicó n las
más no- ches, duelos y quebrantos3 los sá bados, lentejas los
velarte5, calzas de velludo6 para las fiestas con sus pantu- flos7
de lo mismo, los días de entre semana se honraba con su
vellorí8 de lo má s fino. Tenía en su casa una ama que pasaba de
los cuarenta, y una sobrina que no llegaba a los veinte, y un
mozo de campo y plaza 9, que así ensillaba el rocín como tomaba
la podadera. Frisaba 10 la edad de nuestro hidalgo con los
cincuenta añ os, era de complexió n recia, seco de carnes,
enjuto11 de rostro; gran madrugador y amigo de la caza. Quie-
ren decir que tenía el sobrenombre de Quijada o Quesada (que en
esto hay alguna diferencia en los autores que de este caso
escriben), aunque por conjeturas verosímiles se deja enten- der
que se llama Quijana; pero esto importa poco a nuestro cuento;
basta que en la narració n de él no se salga un punto de la
verdad.
Quijote, Parte I, capítulo I.

1
astillero: estante para dejar la lanza. 2 adarga: escudo de cuero. 3
duelos y quebrantos: huevos con tocino. 4 palomino: pichón. 5 sayo
de velarte: traje de paño. 6 velludo: terciope- lo. 7 pantuflos:
prenda que cubría los zapatos. 8 vellorí: tela de paño entrefino. 9 de
campo y plaza: que servía para todo. 10 fri- saba: se aproximaba a. 11
enjuto: delgado.

a) ¿Qué tipo de descripción nos ofrece el narrador? Jus-


tifica tu respuesta.
b) Según la imagen que se nos proporciona del protago-
nista de la historia, ¿en qué se podría distinguir inme-
diatamente de los caballeros andantes medievales o de
cualquier héroe del cine de aventuras actual?
c) ¿Cuál dirías que es su situación económica? Justifica tu
respuesta con ejemplos.
d) Por su parte, ¿es acaso el narrador el primero que
escribe sobre el hidalgo? En todo caso, ¿cómo espera
que sea su relato?
e) Como en la mayoría de las descripciones, el narrador
recurre a palabras o sintagmas con valor especificati- vo.
Anota en tu cuaderno al menos cinco adjetivos del texto,
así como otros recursos que emplee el narrador para
proporcionar detalles sobre la realidad descrita.
f) Los ejemplos que acabas de enumerar, ¿poseen un
carácter más objetivo o más subjetivo?
a) El narrador nos ofrece un retrato del hidalgo manche-
go. Se da este tipo de descripció n, porque, junto a los rasgos
físicos del personaje (prosopografía), se alude también a
sus costumbres y aficiones, por ejemplo, la caza (etopeya).
b) La imagen del personaje resulta mucho menos atractiva de
la que se puede esperar en un hé roe. Su complexión recia,
seco de carnes, enjuto de rostro y su edad madu- ra,
ademá s de otorgarle una apariencia poco agraciada, ponen
en duda su capacidad para soportar el peso de las armas y
para acometer empresas donde es fundamental el uso de la
fuerza.
c) Como hidalgo que es, el personaje pertenece al escaló n más
bajo de la nobleza. Por eso puede sobrellevar una existencia
modesta. De ahí que el narrador insista en la rutina de su
menú semanal o en la poca variedad de su armario.
d) Supuestamente, el narrador no es el primero que ha escrito
sobre las peripecias del hidalgo, pues menciona otros au-
tores que de este caso escriben. En todo caso, espera que su
relato sea verosímil y, sobre todo, se ciñ a a la verdad.
e) En el fragmento aparecen adjetivos como antigua, flaco, cuchilladas de don Quijote, le saltó al rostro y le asió de
corredor, recia, seco, enjuto. Además, abundan los sin- las narices con
tagmas preposicionales en funció n de complemento del
nombre: de los de lanza en astillero, de velarte, de vellu-
do; pero también sintagmas en funció n de complemento
del adjetivo: de carnes, de rostro. Asimismo, se reconocen
proposiciones subordinadas adjetivas especificativas: que
pasaba de los cuarenta, que no llegaba a los veinte, que
predominan sobre las adjetivas explicativas como de cuyo
nombre no quiero acordarme.
f) En general, los ejemplos enumerados poseen un cará cter
má s objetivo.
2. De entre los temas del Quijote señalados en la unidad,
identifica el que aparece en el siguiente pasaje. Justifica
tu respuesta.
Viva la memoria de Amadís, y sea imitado de Don Quijote de la
Mancha en todo lo que pudiere; del cual se dirá lo que del otro
se dijo, que si no acabó grandes cosas murió por
acometerlas; y si yo no soy desechado, ni desdeñ ado de mi
Dulcinea, bá sta- me, como ya he dicho, estar ausente della.
Ea pues, manos a la obra, venid a mi memoria, cosas de
Amadís, y enseñ adme por donde tengo de comenzar a
imitaros.
Quijote, Parte I, capítulo XXVI.

El fragmento desarrolla el motivo de la imitació n, estrategia


fundamental para el personaje, a partir de la cual cree que
será reconocido por la posteridad.
3. Elabora una descripción técnica de un molino de viento
y una descripción literaria en la que lo compares con un
gigante.
Respuesta libre.

Recuerda lo que ya sabías


4. En la segunda parte del Quijote son varios los persona-
jes que intentan alimentar la locura del protagonista para
burlarse de él. Así ocurre con la siguiente invención de
los duques:
Aquí llegaba don Quijote de su canto, a quien estaban escu-
chando el duque y la duquesa, Altisidora y casi toda la gente
del castillo, cuando de improviso, desde encima de un corre-
dor que sobre la reja de don Quijote a plomo 1 caía, descolgaron
un cordel donde venían má s de cien cencerros asidos, y
luego tras ellos derramaron un gran saco de gatos, que
asimismo traían cencerros menores atados a las colas. Fue
tan grande el ruido de los cencerros y el mayar 2 de los gatos,
que aunque los duques habían sido inventores de la burla,
todavía les so- bresaltó , y, temeroso, don Quijote quedó
pasmado. Y quiso la suerte que dos o tres gatos se entraron
por la reja de su es- tancia, y dando de una parte a otra
parecía que una regió n de diablos andaba en ella: apagaron
las velas que en el aposento ardían y andaban buscando
por do escaparse. El descolgar y subir del cordel de los
grandes cencerros no cesaba; la ma- yor parte de la gente del
castillo, que no sabía la verdad del caso, estaba suspensa y
admirada.
Levantose don Quijote en pie y, poniendo mano a la espada,
comenzó a tirar estocadas por la reja y a decir a grandes voces:
—¡Afuera, malignos encantadores! ¡Afuera, canalla hechice-
resca, que yo soy don Quijote de la Mancha, contra quien no
valen ni tienen fuerza vuestras malas intenciones!
Y volvié ndose a los gatos que andaban por el aposento les
tiró muchas cuchilladas. Ellos acudieron a la reja y por allí se
salieron, aunque uno, viéndose tan acosado de las
las uñ as y los dientes, por cuyo dolor don Quijote comenzó a dar
los mayores gritos que pudo.
Quijote, Parte II, capítulo XLVI.

1 2
a plomo: verticalmente. mayar: maullar.

a) ¿Cómo reacciona don Quijote ante la gatuna inven-


ción? ¿En qué sentido se reafirma su locura?
b) Por su parte, ¿te parece cruel la burla de los duques?
¿Por qué?
c) ¿A qué obligaciones debían atender los nobles en la
literatura medieval y en los libros de caballerías para
alcanzar la fama? ¿Por qué la conducta de los duques
resulta indigna?
d) Analiza los siguientes sintagmas nominales:
• casi toda la gente del castillo
• un gran saco de gatos
• los mayores gritos que pudo
e) ¿Qué tiempo verbal predomina en las narraciones?
¿Con qué otros tiempos alterna en este texto? Fíjate,
por ejemplo, en la distinta función de las formas fue,
ardían o soy.
f) Según su modalidad, ¿qué tipo de oraciones predomi-
nan en el texto? ¿Hay alguna frase imperativa o exhor-
tativa? ¿Cuál?
g) Indica la función sintáctica de los términos marcados en
negrita:
• don Quijote quedó pasmado
• fue tan grande el ruido
• la gente del castillo estaba suspensa
• les tiró muchas cuchilladas
• le asió de las narices con las uñas
h) Localiza en el texto al menos dos oraciones compues-
tas e indica de qué tipo son.
a) En primera instancia, don Quijote queda pasmado e in-
cluso atemorizado. Pero, de inmediato, se acerca a la reja de
su habitació n. Su locura se ve reafirmada porque con- sidera
que el episodio es una invenció n de los malvados
encantadores que le persiguen.
b) Respuesta libre.
c) En la literatura señ alada, los nobles se definían como be-
llatores. Esto es, tenían a su cargo el mantenimiento de la
justicia, la integridad de sus señ ores y la defensa de la fe
cristiana. Encargados de preservar el orden establecido,
luchaban contra cualquier agresió n con las armas, puesto que
eran los ú nicos que podían permitirse su posesió n y, ademá s,
estaban entrenados para su manejo. Por su parte, los duques se
comportan de forma indigna porque se apro- vechan de su
poder para divertirse a costa de don Quijote.
d) • casi toda la gente del castillo: Det + Det + Det + N + CN
(enlace + término-SN [Det + N]).
• un gran saco de gatos: Det + Ady + N + CN (enlace +
término-SN [N]).
• los mayores gritos que pudo: Det + Ady + N + Sub. adj.
especificativa-Ady.
e) En las narraciones predominan los verbos en pretérito per-
fecto simple de indicativo. En el texto, alterna con el pre-
té rito imperfecto de indicativo (ardían) y el presente de
indicativo (soy). 2. Durante sus aventuras, don Quijote recorrió diversos
escenarios de la geografía manchega. Así, en la obra se
f) La modalidad oracional predominante es la enunciativa.
Cuando irrumpe el estilo directo, sin embargo, observamos
mencionan poblaciones como El Toboso, Argamasilla,
la presencia de la modalidad imperativa en ¡Afuera, Puerto Lápice o Tembleque, y lugares como la comarca
canalla hechiceresca, que yo soy don Quijote…! de Campo de Montiel, la Cueva de Montesinos o las
g) • pasmado: predicativo. Lagunas de Ruidera. Elabora un mapa donde aparezcan
situados dichos espacios.
• el ruido: sujeto.
Respuesta libre.
• suspensa: atributo.
• les: complemento indirecto. 3. Don Quijote imaginaba que, a través de sus hazañas,
• con las uñas: complemento circunstancial de instru-
podría alcanzar el amor de la «sin par Dulcinea del
mento. Toboso», moza labradora por la que, en un tiempo,
estuvo interesado y cuya fantasía convertirá en una
h) • Subordinada adverbial de tiempo: cuando de improviso
[…] descolgaron un cordel... especie de diosa. Busca en el capítulo XXV de la primera
parte la descripción que rea- liza de ella Sancho Panza y
• Subordinada sustantiva de complemento directo: que
dos o tres gatos se entraron por la reja de su resúmela, distinguiendo entre sus rasgos físicos y
estancia… morales. ¿En qué sentido te parece acertada la imagen
• Subordinada adjetiva especificativa: que andaban
de Dulcinea de la Figura 10.11?
por el aposento… Segú n la opinió n de Sancho, Dulcinea es una mujer fuerte,
robusta y con una voz que se oye a lo lejos. Si sus rasgos
MIRA A TU ALREDEDOR Y… físicos le dan un aspecto hombruno, sus cualidades morales
(etopeya) se destacan con acento equívoco. Su fortaleza de
… ve más allá cará cter es capaz de sacar de cualquier aprieto a un
1. En el capítulo I de la primera parte del Quijote, se nos caballero, pero su actitud poco melindrosa y su tendencia a
dice que, llevado por su deseo de ser caballero andante, burlar y a mostrarse donosa con el otro sexo hacen pensar
el hidalgo Alonso Quijano limpió: también en una predisposició n innata a mantener relaciones
sexuales. Se trata de una descripció n totalmente opuesta a la
Unas armas, que habían sido de sus bisabuelos, que tomadas
que sublimó , por ejemplo, la lírica garcilasista de mujer de
de orín1 y llenas de moho, luengos 2 siglos había que estaban
cabellos dora- dos, cutis muy blanco y labios rojos como el
puestas y olvidadas en un rincó n.
clavel. En todo caso, coincide con la imagen reproducida en su
1 2
apariencia rú s- tica y en su desvío de las líneas estilizadas de
tomadas de orín: oxidadas. luengos: largos. la fémina ideal.
4. Indudablemente, don Quijote es uno de los personajes
Para cumplir con su misión, don Alonso necesitaba unos
literarios cuyo recuerdo ha perdurado más en el tiempo.
instrumentos militares. Pero su relación con las armas ya
Por ejemplo, Rubén Darío le dedicó su «Letanía de nues-
está planteada en la elección de su nombre caballeresco.
tro señor don Quijote», mientras que León Felipe lo
¿Qué pieza de la armadura medieval era un quijote?
eligió como protagonista del poema «Vencidos», del que
¿Para qué servía? Enumera otras cuatro piezas de una
pue- des encontrar en YouTube una versión musical
armadura y explica su función.
realizada por Joan Manuel Serrat. Lee ambos poemas y
El quijote era una pieza de la armadura que servía para pro-
escucha la canción. ¿Qué sentimientos despierta en los
teger el muslo de los caballeros. Para otros elementos de la
armadura, vé ase la siguiente ilustració n: respectivos emisores el caballero manchego? ¿Qué
Celada
aspectos se des- tacan de su figura?
Respuesta libre.
Visera 5. Asimismo, famosos pintores han hallado motivo de ins-
Babera piración en el personaje cervantino, e incluso alguno de
Gola
ellos llegó a ilustrar con sus dibujos y pinturas ediciones
Hombrera
cuidadas de las dos partes de la historia. En el portal digi-
tal Quijote Banco de Imágenes (www.qbi2005.com),
Ristre
Peto
Guardabrazo Escarpe
Codal

Avanbrazo

Quijote

Rodillera

Greba
Espuela
descub s d, elabora un listado de versiones cinematográficas
rirás un so (distinguiéndolas según el público al que van dirigidas),
rico br a partir de las cuales podrás demostrar su proyección
tesoro e internacional.
iconogr las Respuesta libre.
áfico im
con ág
ilustrac en
io- nes es
y ele
grabad gid
os as.
sobre Res
los pues
person ta
ajes y libre
las .
aventur 6. Lo
as de la s
obra. A lib
partir ro
de la s
consult ce
a de rv
esta an
página tin
te os
propon ta
e- mos m
dos bié
activida n
des ha
alterna n
tivas. hal
Tras la lad
selecció o
n de las su
imágen lug
es ar
oportu en
nas: el
a) Elabo cin
ra un e.
Ap
mural
oy
temá
án
tico. do
b) Real te
iza en
un la
brev inf
e or
cóm m
ic en aci
sop ón
orte su
digit mi
al, nis
inco tra
rpo- da
rand po
o r
diál la
ogo re
… encuentra la clave c) Según afirma Mario, de Numancia «debe ser eterna la
1. Lee los siguientes tercetos, extraídos de la jornada IV de memoria». La fama se antoja un motivo recurrente en
La Numancia. En ellos, el soldado romano Mario elogia la vida y obra de Cervantes. ¿Cómo pretendió conse-
a los habitantes de la ciudad sitiada por el modo en que guirla don Miguel? ¿Y don Quijote? Pese a los infor-
han puesto fin a su tenaz resistencia contra las tropas tunios con que tuvieron que bregar el uno y el otro,
invasoras: ¿por qué dirías que alcanzaron su objetivo?
En balde, ilustre general prudente, d) En la actualidad, ¿perdura la aspiración del heroísmo?
han sido nuestras fuerzas ocupadas, ¿A través de qué actividades, oficios o empresas pue-
en balde te has mostrado diligente1,
des contribuir a mejorar la sociedad que te rodea?
pues en humo y en viento son tornadas Escribe una redacción de unas cien palabras en la
las ciertas esperanzas de victoria,
que, después de señalar cuáles son los principales
de tu industria2 contino aseguradas.
proble- mas que azotan al mundo, aportes posibles
Del lamentable fin y triste historia soluciones.
de la ciudad invicta de Numancia
a) Respuesta libre.
merece ser eterna la memoria:
b) Tambié n los saguntinos optaron por el suicidio colectivo
sacado han de su pé rdida ganancia;
despué s de resistir ocho meses de asedio. En su caso, los
quitado te han el triunfo de las manos,
romanos no eran sus enemigos, sino que, teó ricamente,
muriendo con magná nima3 constancia.
eran sus aliados, aunque renunciaron a acudir en su ayuda.
Nuestros disignios han salido vanos, Siendo diferente la nacionalidad de los sitiadores, romanos
pues ha podido má s su honroso en Numancia y cartagineses en Sagunto, quienes ofrecen
intento que toda la potencia de su vida en sacrificio de su reputació n lo hacen impidien-
romanos; do que sus rivales puedan exhibirlos como muestra de su
el fatigado pueblo en fin violento triunfo militar.
acaba la miseria de su vida, c) Cervantes buscó la fama distinguié ndose como soldado
dando triste remate al largo cuento. y como escritor. Mientras tanto, don Quijote quiso obte-
Numancia está en un lago convertida nerla convirtiéndose en caballero andante. Tanto el uno
de roja sangre, y de mil cuerpos llena, como el otro lograron su objetivo, curiosamente, despué s
de quien fue su rigor4 proprio de su muerte. Cervantes se vio eclipsado en vida por la fi-
homicida; gura, por ejemplo, de Lope de Vega. Por su parte, su ficticia
de la pesada y sin igual cadena criatura, que legalmente no podía ser considerada como
dura de esclavitud se han escapado auté ntico caballero porque su investidura fue un comple-
con presta5 audacia, de temor ajena. to escarnio, terminaría siendo percibido como un verda-
dero caballero andante, pese a los fracasos que tuvo que
En medio de la plaza levantado
sortear.
está un ardiente fuego temeroso,
de sus cuerpos y haciendas sustentado. d) Respuesta libre.
Prensas Universitarias de Zaragoza 2. Con la colaboración de tus compañeros de aula, enumera
las posibles deficiencias que haya en tu colegio o en tu
1
barrio. Por ejemplo, piensa en personas que pueden
diligente: resuelto y laborioso. 2 industria: habilidad. 3 mag- tener dificultades para integrarse socialmente u otras
nánima: noble. 4 rigor: severidad. 5 presta: rápida.
que care- cen de los recursos económicos necesarios
para llevar una vida digna.
a) En opinión del personaje, la población numantina ha a) ¿De qué modo podríais actuar como grupo para ayu-
vencido al enemigo optando por un suicidio colectivo. darlas a salir de su situación?
¿Te parece una interpretación lógica? ¿Por qué? b) ¿Qué tipo de recompensa obtendríais en caso de
b) En el siglo ııı a. C., también la próspera ciudad de alcanzar el objetivo propuesto?
Sagunto resistió heroicamente contra las tropas del c) ¿Sería la vuestra una conducta heroica? Justifica tu
general Aníbal. Busca información sobre este episodio respuesta.
histórico e indica qué semejanzas existieron entre
a), b) y c) Respuesta libre.
este asedio y el de Numancia.
11 ENTRE PÍCAROS Y POETAS

APERTURA DE UNIDAD serlo. Quienes no podían hacer ostentació n de esos lujos no


1. Fíjate en las condiciones de todo lo que rodea a la eran nada en esa corte de las apariencias.
comida del protagonista y haz una tabla en la que recojas 5. ¿Por qué le parece asombroso a Carlos I el recibimiento
cada elemento y las características negativas que se le que le hacen?
adjudi- can en el texto. Sin duda llamaría su atenció n el espectá culo de carrozas y no-
bles en procesió n recorriendo las principales calles de Madrid,
Elemento Características negativas como demostració n de los lujos y de las grandezas del imperio
españ ol, en contraste con la rigidez de costumbres que luego le
banquillo cojo
muestran Olivares y los consejeros al impedirle que se acerque
[mantel] barredero de horno a saludar a la infanta.
salero suelo de cá ntaro 6. ¿Cuál es el papel del pueblo en la celebración que se des-
agua tiesto de gallinas
cribe en el texto? ¿Piensas que se nos muestra una
división clasista de la sociedad de la época? ¿Por qué?
hogaza negra
El pueblo tiene el papel de espectador, de mero comparsa en
tortilla emplastro medio de los grandes de Españ a, de los que son «algo», a quie-
nes se limita a mirar y tal vez a admirar. El narrador nos lo
presenta como testigo de las celebraciones fastuosas de la
2. ¿Por qué crees que el narrador-protagonista termina di- Corte.
ciendo que tuvo todo aquello «por buena suerte»?
Razona tu respuesta. 7. ¿Qué personajes de los que se mencionan en el texto son
Respuesta libre.
los que van a contraer matrimonio?
Van a contraer matrimonio el príncipe de Gales, Carlos I, y
3. Recuerda las características de la novela picaresca y ex-
la infanta de Españ a, doñ a María, hermana del rey Felipe IV.
plica cuáles de ellas están presentes en este fragmento.
Respuesta libre. Vocabulario
4. ¿Cómo crees que sería la venta en la que el protagonis- 8. Explica con tus palabras en qué consistía «hacer la rúa».
ta estuvo comiendo? Haz una redacción de unas quince Busca en el diccionario la palabra rúa y asocia su signifi-
líneas en la que imagines esta venta y utilices sobre todo cado a la expresión anterior.
la descripción.
Respuesta libre.
Respuesta libre.
9. Explica el significado de la palabra granado en la expre-
VIVE LA LECTURA sión «lo más granado de la Corte». Puedes utilizar un dic-
cionario e informarte sobre la procedencia de la palabra.
Comprensión lectora
Lo más granado de la Corte hace referencia a las personas
1. ¿Qué actividades hacían las gentes en el Prado de San principales. Granado, en su segunda acepció n segú n el DRAE,
Jerónimo? significa notable, señ alado, principal o ilustre. Viene de gra-
El Prado de San Jeró nimo era lugar de paseo, en el que las nar, que significa producir granos algunas plantas, es decir,
gentes concertaban citas y practicaban el galanteo, se encon- crear su futuro fruto.
traban los enamorados y se reunía la sociedad de la época. 10. Busca en el primer párrafo los nombres que se utilizan
2. ¿Quién es Carlos I? ¿Cuál es su papel en el texto que aca- para hacer referencia a los distintos tipos de vías públi-
bas de leer? cas, como calle, y, con la ayuda del diccionario, copia las
Carlos I era el príncipe de Gales, pretendiente de la infanta definiciones en tu cuaderno.
doñ a María, la hermana de Felipe IV. Llega a Madrid para Nombres que designan vías pú blicas: rúa, carrera, calle, vía.
conocer a su futura esposa, y al rey españ ol le sirve de pre- • rúa: calle de un pueblo o camino para carruajes.
texto su llegada para organizar un desfile y una fiesta para
recibirlo, aunque dentro del má s estricto protocolo de la corte • carrera: camino real o carretera.
de los Austrias. Esto ú ltimo se demuestra, sobre todo, cuando • calle: vía pú blica en una ciudad.
Carlos I pide saludar a su prometida y el narrador nos hace
• vía: camino por donde se transita.
ver que eso era impensable segú n la recta moral de Felipe
IV. 11. ¿Con qué nombre conocemos a las palabras que
3. Según el texto, ¿qué normas regían el comportamiento tienen entre sí una relación como la que se observa en
de la corte española en época de Felipe IV? las de la actividad anterior?
Este tipo de palabras son sinó nimos, pues existe una identidad
Los límites y el decoro del protocolo de la corte españ ola era
o similitud en sus significados.
muy rígidos segú n el texto, ya que todas las actividades que
debía realizar la familia real estaban establecidas de antema- 12. En el texto se nos dice que «el pueblo ofició como testi-
no, de acuerdo con unos há bitos que regían cada uno de sus go de aquella exhibición caballeresca»; ¿qué significa la
movimientos. palabra ofició?
4. ¿Qué crees que quiere decir el narrador con «todo aquel Oficiar es una palabra que está relacionada con las celebra-
que era algo en la Corte»? Razona tu respuesta. ciones religiosas y significa ayudar en una misa o en otra
Cuando el narrador emplea la expresió n todo aquel que era solemnidad litú rgica. Aquí está usada con sentido figurado y
algo en la Corte se refiere a todas aquellas personas que te- viene a dar a entender que el pueblo contribuyó a esa
nían importancia o relevancia, ya fueran nobles, damas, ca- exhibi- ción caballeresca con su presencia y su admiració n
balleros o cualquier otro personaje principal o que aparentara ante las grandezas de la Corte.

98 Lengua castellana y Literatura. 3.º ESO. Solucionario


13. La palabra corte aparece en el texto escrita tanto con
4. Lee la siguiente octava real de la Fábula de Polifemo y
mayúscula como con minúscula inicial. ¿Por qué crees
Galatea y contesta a las preguntas:
que sucede esto? Infórmate sobre los distintos
significados de esta palabra y construye luego una Un monte era de miembros eminente
este (que de Neptuno hijo fiero,
oración con cada uno de ellos.
de un ojo ilustra el orbe de su frente,
Como otros nombres que designan entidades o instituciones é mulo casi del mayor lucero)
del estado, la palabra corte se escribe con mayú scula cuando cíclope, a quien pino más valiente,
hace referencia a esa institució n y no lleva ningú n complemen- bastó n le obedecía tan ligero,
to: la Corte, frente a la corte española. Puede tener un valor y al grave peso junco tan delgado,
distintivo, para diferenciar la institució n de otros significados que un día era bastó n y otro cayado.
que se asocian a la misma palabra. Algunos de esos otros
significados son: filo de un cuchillo, acció n de cortar, herida,
a) ¿Qué rasgos destaca Góngora en la descripción del
corral donde se recoge el ganado, etc. gigante mitológico? ¿Cuántos ojos tiene?
Respuesta libre. b) ¿En qué distingues su estilo culterano? Localiza e
iden- tifica tres recursos literarios usados por el autor.
Investigación y redacción c) Reescribe tú el fragmento de la forma más natural po-
sible.
14. Infórmate sobre los espacios reales que se citan en el
texto y que se sitúan en Madrid: d) Investiga sobre el ser mitológico Polifemo e indica
cuá- les de sus características aparecen en el
a) Busca información sobre ellos y descubre si siguen fragmento.
exis- tiendo, han cambiado de nombre o han
a) Entre los rasgos sobresalientes en la descripció n del gi-
desaparecido.
gante, vemos que el autor lo identifica con un monte y
b) Localiza estos espacios en un plano de Madrid e dice que utiliza como bastó n un fuerte pino. De é l nos
inten- ta trazar el itinerario que se cita en el texto. dice que solo tiene un ojo.
a) y b) Respuesta libre. b) El culteranismo lo vamos a identificar en el uso de deter-
minados recursos literarios como los siguientes:
15. Infórmate sobre quiénes fueron Lerma y Olivares y so-
• hipérbaton (cambio del orden ló gico de la frase): un
bre cuál fue su papel en la España del siglo xvıı. ¿Crees
monte era de miembros eminente, en lugar de «era
que fue beneficioso o perjudicial para el país? Razona tu un eminente monte de miembros».
respuesta.
• metá fora: en la misma identificació n del gigante con
Respuesta libre. un monte.
• uso de una sintaxis compleja, como en cíclope, a quien
LITERATURA pino más valiente, / bastón le obedecía tan ligero,
donde, ademá s del desorden de los términos (hipérba-
1. Busca información y define los siguientes tópicos litera- ton), vemos la ausencia de palabras; tal vez faltaría un
rios, acompañándolos de una imagen o de un texto ilus- como comparativo entre los dos versos, que facilitaría
trativo: «pañales y mortaja», vanitas vanitatum y «el mun- la comprensió n del símil que se produce entre el pino y
el bastó n.
do al revés».
c) Respuesta libre.
• «Pañ ales y mortaja» hace referencia a la brevedad de la
vida, a la fugacidad del tiempo, que junta casi en un d) Respuesta libre.
instan- te el momento del comienzo de la vida (los 5. Escucha el soneto de Quevedo que comienza con el verso
pañ ales) con la muerte (la mortaja, que es la ropa con la «Cómo de entre mis manos te resbalas» (lo puedes
que se viste a un difunto antes de enterrarlo). encon- trar en YouTube). ¿En qué bloque temático de
• Vanitas vanitatum: vanidad de vanidades; se define así, la poesía quevedesca incluirías este soneto y por qué?
en el Barroco, a la existencia humana, como algo vacío y ¿A quién o quiénes se dirige el poeta en tono de
sin sentido, como consecuencia del pesimismo existencial reproche?
que domina a esta é poca: nada parece tener sentido ante
El soneto se incluye dentro de la poesía filosó fica y moral de
la certeza de la muerte.
Quevedo, ya que el autor medita sobre la brevedad y la fragi-
• «El mundo al revés» es otro tó pico barroco que se centra en lidad de la existencia humana.
la idea del caos, del desorden, de que nada es lo que El reproche del poeta va dirigido a la muerte, que se acerca
parece y todo es cambiante. a traició n con sus mudos pasos, y a la vida, que pasa rá pi-
Imá genes y textos: respuesta libre. damente, y se resbala entre las manos de poeta, segú n é l
mismo dice.
2. Comenta la posible relación del cuadro de Bartolomé
Murillo (Figura 11.2) con las circunstancias históricas de 6. Las disputas entre Quevedo y Góngora dieron lugar a
la época. poe- mas satíricos intercambiados entre ambos. Así
comienza un soneto de Quevedo: «¿Qué captas,
Respuesta libre.
noturnal, en tus can- ciones, / Góngora bobo, con
3. Buena prueba del interés barroco hacia la literatura es la crepusculallas, / si cuando anhelas más garcivolallas, / las
existencia en el Madrid de la época de los mentideros. reptilizas más y subterpo- nes?». ¿De qué manera se
Busca información sobre ellos y trata de localizar los más burla Quevedo en estos versos?
importantes que existieron en dicha ciudad. ¿Crees que parodia la poesía de Góngora? ¿Por qué?
Respuesta libre. La burla de Quevedo hacia Gó ngora, en estos versos, está
basada en la imitació n del lenguaje recargado y comple-
jo del poeta cordobés, al que se dirige citá ndolo y tachá ndolo
de bobo.
11 ENTRE PÍCAROS Y POETAS

Se trata de una evidente parodia, ya que vemos có mo imita


burlescamente el hipérbaton (las reptilizas más y subter- Autor Título Protagonista
pones), utiliza neologismos de su invenció n que recuerdan el Lazarillo de Tormes
lenguaje excesivamente culto de Gó ngora (garcivolallas, sub- Anó nimo Masculino
(1554)
terpones, crepusculallas...) y hace referencia a la oscuridad
Segunda parte de
de los versos de su rival, difíciles de entender, con palabras
Anó nimo Lazarillo de Tormes Masculino
que sugieren la noche: noturnal, crepusculallas. (1555)
7. Uno de los males del amor es la ausencia, la separación Guzmán de Alfarache
de los enamorados. Fíjate en los versos iniciales de un Mateo Alemá n Masculino
(1599 y 1604)
soneto de Lope de Vega: «Ir y quedarse y con quedar Mateo Lujá n de Guzmán de Alfarache
partirse, Masculino
Sayavedra (apó crifo, 1602)
/ partir sin alma y ir con alma ajena». Analiza la confusión
Gregorio Gonzá lez El Guitón Onofre (1604) Masculino
que producen las antítesis y relaciónala con el estado de
ánimo de los enamorados que se han tenido que separar Francisco de Ú beda La pícara Justina (1605) Femenino
por un tiempo. Francisco de
El Buscón (c. 1606) Masculino
Quevedo
La antítesis provoca en estas palabras de Lope de Vega una
confusió n que contradice lo que realmente se quiere decir. Es Alonso Jeró nimo La ingeniosa Elena
Femenino
un juego de contrarios que, mezclados, parecen hacer imposi- de Salas Barbadillo (1612-1614)
ble aquello que pretenden comunicar. De esta forma se refle- Vida del escudero
jan las incertidumbres del amor distante, de la separació n de Vicente Espinel Masculino
Marcos de Obregón
los enamorados que, aunque se van, realmente se quedan: se (1618)
van físicamente, pero se quedan en espíritu junto a la persona Segunda parte de
amada, de ahí lo de partir sin alma y ir con alma ajena, pues Juan de Luna Lazarillo de Tormes Masculino
se produce un intercambio segú n el cual el que se va deja su (1620)
alma con la amada, mientras esta envía la suya con el amado.
Juan Cortés de Lazarillo de Manzanares
En definitiva, un ser y no ser, un estar y no estar: la tristeza Masculino
Tolosa (1620)
y la melancolía de los enamorados que se tienen que separar
contra su voluntad. Jeró nimo de Alcalá Alonso, mozo de muchos
Masculino
Yá ñ ez amos (1624)
8. El siguiente fragmento del Buscón nos muestra las activi-
La niña de los embustes,
dades delictivas del padre del protagonista: Alonso de Castillo
Teresa de Manzanares Femenino
Soló rzano
a) ¿Cuál era la profesión del padre del protagonista? (1632)
¿Qué otra ejercía al mismo tiempo? Alonso de Castillo Aventuras del bachiller
Masculino
Soló rzano Trapaza (1637)
b) ¿Cómo crees que puede influir la actividad del padre
en el comportamiento posterior del protagonista? Ra- Antonio Henríquez Vida de don
Masculino
Gó mez Gregorio Guadaña
zona tu respuesta. (1644)
[Mi madre] Padeció grandes trabajos recié n casada, y aun Vida y hechos de
despué s, porque malas lenguas daban en decir que mi padre Anó nimo Estebanillo Gonzá lez Masculino
metía el dos de bastos para sacar el as de oros1. Probó sele (1646)
que a todos los que hacía la barba a navaja2, mientras les
daba con agua, levantá ndoles la cara para el lavatorio, un mi 10. Con la ayuda de una enciclopedia, descubre el argumen-
herma- nico de siete añ os les sacaba muy a salvo los to de El Criticón y explica en qué consiste su carácter
tué tanos de las faldriqueras3. Murió el angelico de unos alegórico.
azotes que le dieron en la cá rcel. Sintiolo mucho mi padre,
por ser tal que robaba a todos las voluntades4. Respuesta libre.

Francisco DE QUEVEDO: Vida del Buscón.


11. En las obras de María de Zayas, la mujer muestra una
libertad de acción impropia de su época. Señala qué
com- portamientos y actuaciones crees que le estarían
1
metía el dos de bastos para sacar el as de oros: quiere decir vetados a la mujer en aquella época.
que con dos dedos (el dos de bastos) le sacaba las monedas de los Respuesta libre.
bolsillos (el as de oros). 2 hacía la barba a navaja: afeitaba. 3 los
tuétanos de las faldriqueras: lo más escondido de los bolsillos,
hasta la última moneda. 4 robaba a todos las voluntades: juega
COMENTARIO DE TEXTO
con el doble sentido de que todos se encariñaban de él y de que
Localización. Este poema pertenece a los sonetos de juven-
él les quitaba lo que no querían dar voluntariamente.
tud (hacia 1582) de Luis de Góngora. En él sigue de cerca la
tradición petrarquista que tanta importancia tuvo en la lírica
a) El padre del protagonista era barbero, pero al mismo tiem- del Renacimiento.
po ejercía el oficio de ladró n, ya que aprovechaba los 1. ¿Qué imagen ideal de la mujer se ofrecía en dicha tra-
des- cuidos de sus clientes para robarles el dinero.
dición?
b) Respuesta libre. La figura femenina es retratada, en la tradició n petrarquis-
9. Además de las novelas picarescas citadas, se escribie- ta, como un ser inalcanzable e idealizado, cú mulo de todas
ron varias obras más de este género. Elabora una rela- las perfecciones y de toda la belleza. También se nos presen-
ción completa de títulos y autores distinguiendo si tienen ta como una mujer desdeñ osa y distante, que hace sufrir al
enamorado con su comportamiento arbitrario. El poeta se
un protagonista masculino o femenino.
convierte en un admirador de la dama y siente su amor tan
100 Lengua castellana y Literatura. 3.º ESO. Solucionario
profundamente que se conformará con ser odiado por ella con
En las primeras estrofas, los adjetivos está n dispuestos for-
tal de estar en sus pensamientos.
mando un quiasmo, ya que en cada verso aparecen, al princi-
2. Realiza un análisis métrico del poema y justifica después pio, delante del sustantivo, y al final, detrá s, dando una clara
por qué es un soneto. sensació n de orden y armonía.
Es un soneto porque está compuesto por catorce versos en- Precisamente esta ordenació n de los adjetivos contribuye a
decasílabos de rima consonante, divididos en dos cuartetos la creació n de algunos paralelismos, como por ejemplo entre
y dos tercetos, segú n la siguiente estructura mé trica: ABBA los versos 5 y 6: pequeña puerta de coral preciado / claras
ABBA CDE CDE. lumbreras de mirar seguro (adjetivo + nombre + de + nombre
Tema y estructura. Como Góngora emplea un esquema me- + adjetivo).
tafórico sin mencionar el elemento real que compara con un En la expresió n ornan de luz, coronan de belleza podemos
término figurado, quizá te resulte difícil descubrir la apreciar un asíndeton, provocado por la ausencia del nexo
identidad del personaje superior del que nos habla. Primero, copulativo (y). Tambié n hay otro paralelismo.
debes ave- riguar qué elementos describe: Conclusiones. Góngora fue el máximo exponente del culte-
3. ¿Qué versos abarca la descripción? ¿Qué partes del cuer- ranismo.
po humano enumera y en qué orden? ¿Con qué elemen- 7. ¿Qué aspectos de este poema te permiten reconocer
tos imaginarios los identifica? dicho estilo literario?
La descripció n abarca casi todo el poema, concretamente los El culteranismo se reconoce en este soneto principalmente
once primeros versos (los dos cuartetos y el primer terceto). por el uso continuado de metá foras puras que, con un sentido
Las partes del cuerpo que enumera, por orden, y la imagen alegó rico, identifican el cuerpo de una mujer hermosa con un
con que las identifica son las siguientes: templo (por ejemplo, pequeña puerta de coral preciado, en
lugar de boca, por el color rojo del coral asimilado a los labios,
• el cuerpo y las piernas: un templo (gentil muro y bello
que se abren como una puerta). También llama la atenció n el
cimiento).
empleo del hipérbaton (De pura honestidad templo sagrado,
• la boca: pequeña puerta de coral preciado. donde se coloca el complemento del nombre delante del nom-
• los ojos, verdes y luminosos, identificados con las claras bre al que complementa).
lumbreras.
8. ¿Qué crees que le aporta al texto la identificación de la
• la cabeza y los cabellos rubios: (soberbio techo y cimbrias mujer con un templo? ¿Piensas que tiene algo que ver
de oro). con la idea de la mujer que ofrece el petrarquismo?
4. En cambio, ¿qué sentimientos le despierta al poeta su Razona tus respuestas.
«ídolo bello»? ¿Dónde los expone? Respuesta libre.
Los sentimientos que despierta en el poeta ese ídolo bello apa-
recen reflejados sobre todo en el ú ltimo terceto (versos 12-14). EL JARDÍN DE LA LITERATURA
Para él, esta mujer es como una divinidad a la que hay que
adorar y rezar, a la que hay que suplicar, como si de un dios 1. Te mostramos a continuación un soneto satírico de Que-
se tratara, para que tenga piedad de uno. vedo:
É rase un hombre a una nariz pegado,
Análisis de la forma y el contenido. La correspondencia en-
é rase una nariz superlativa,
tre el contenido y la forma del poema es total. é rase una alquitara medio viva,
5. ¿Qué significado poseen los adjetivos usados por el poe- é rase un peje espada mal barbado;
ta en su descripción? Por su parte, ¿qué pretende decirle era un reloj de sol mal encarado,
a su destinatario? Fíjate en el sentido de los verbos é rase un elefante bocarriba,
adoro, suspira, canta y reza. é rase una nariz sayó n y escriba,
La mayor parte de los adjetivos que emplea Gó ngora en la des- un Ovidio Nasó n mal narigado.
cripció n son epítetos, ya que expresan cualidades que ya está n É rase el espoló n de una galera,
presentes en el sustantivo al que acompañ an: pura honesti- é rase una pirá mide de Egipto,
dad, templo sagrado, blanco nácar, alabastro duro, claras las doce tribus de narices era;
lumbreras, claro sol... Con ellos se refuerzan los encantos y é rase un naricísimo infinito,
maravillas de todas esas realidades con las que identifica la frisó n archinariz, caratulera,
belleza física de la mujer a la que se dirige. sabañ ó n garrafal, morado y frito.
Lo que pretende decirle a su destinatario es que admira pro-
a) Haz una relación de todas las realidades con las que
fundamente su belleza, que todo su cuerpo es un templo en
Quevedo identifica a la nariz. ¿Sabes qué figura retó-
el que rezar a la divinidad que ella representa y disfrutar de sus
divinos encantos. De ahí el uso de los verbos adoro, suspira, rica emplea?
canta y reza, con los que demuestra muy claramente la devo- b) Busca en el diccionario las palabras que no entiendas.
ció n que siente hacia esa mujer perfecta: todos estos verbos
c) Quevedo utiliza el recurso de la hipérbole (exagera-
se pueden relacionar con el culto a Dios y contribuyen, por
tanto, a la divinizació n de la figura femenina.
ción). ¿Cómo crees que contribuye a la burla?
6. La distribución de las palabras reproduce la misma ar- d) ¿Te parece que esta descripción es una caricatura?
monía y simetría de un templo. ¿Dónde aparecen los Razona tu respuesta.
adjetivos en las dos primeras estrofas: antes o después a) Palabras con las que se identifica la nariz: alquitara,
del término modificado? ¿Encuentras algún paralelismo? peje espada, reloj de sol, elefante, espolón de una galera,
pirá- mide de Egipto. En todos estos casos, Quevedo está
¿Qué recurso literario aparece en «ornan de luz, coronan
uti- lizando la metá fora, ya que se produce una
de belleza»? identificació n de la nariz con una serie de realidades que,
en su opinió n,
se asemejan a ella. Tambié n utiliza la hipé rbole, figura b) Respuesta libre.
retó rica basada en la exageració n, porque es evidente
que los objetos o seres con los que identifica la nariz
tienen un tamañ o desmesurado.
b), c) y d) Respuesta libre.
2. Hemos hablado mucho de los sonetos. Ahora podrás leer
uno en el que Lope de Vega nos da la receta de cómo es-
cribirlos:
Un soneto me manda hacer Violante,
que en mi vida me he visto en tanto aprieto;
catorce versos dicen que es soneto:
burla burlando van los tres delante.
Yo pensé que no hallara consonante
y estoy a la mitad de otro cuarteto,
mas si me veo en el primer terceto,
no hay cosa en los cuartetos que me espante.
Por el primer terceto voy entrando,
y parece que entré con pie derecho,
pues fin con este verso le voy dando.
Ya estoy en el segundo, y aun sospecho
que voy los trece versos acabando:
contad si son catorce, y está hecho.
a) Según nos dice Lope, ¿cuál es la estructura de un
soneto?
b) ¿Cómo tiene que ser la rima de un soneto, asonante o
consonante? Copia la parte en que el poeta nos
informa de esto.
a) Lope dice que un soneto son catorce versos y nos hace
ver que se compone de dos cuartetos y de dos tercetos.
b) La rima de un soneto tiene que ser consonante, y así lo
expresa Lope de Vega cuando dice al principio del segundo
cuarteto: Yo pensé que no hallara consonante.
3. Lee ahora este fragmento de El Criticón y contesta luego
las preguntas:
—¿Có mo es esto? Viviendo entre las fieras, no me previniste
de algú n riesgo, ¿y ahora con tanta exageració n me cautelas?
¿No era mayor el peligro entre los tigres, y no temíamos, y
ahora de los hombres tiemblas?
—Sí —respondió con un gran suspiro Critilo—, que si los hom-
bres no son fieras es porque son má s fieros, que de su crueldad
aprendieron muchas veces ellas. Nunca mayor peligro
hemos tenido que ahora que estamos entre ellos. Y es tanta
verdad esta, que hubo rey que temió y resguardó un
favorecido suyo de sus cortesanos (¡qué hiciera de los
villanos!) más que de los hambrientos leones de un lago; y así,
selló con su real anillo la leonera para asegurarle de los
hombres cuando le dejaba entre las hambrientas fieras. ¡Mira tú
cuá les será n estos! Verlos has, experimentarlos has, y
dirá smelo algú n día.
Baltasar GRACIÁ N: El Criticón (texto modernizado).
a) ¿Qué concepción del ser humano nos ofrece
Gracián por boca de Critilo? ¿Qué opinas tú de lo
que dice?
b) Con lo que has estudiado sobre esta época, razona
por qué el fragmento que acabas de leer se puede
insertar en el Barroco.
a) El ser humano se nos presenta como una fiera, como un
ser despiadado al que hay que temer mucho má s que los
auté nticos animales. Nos hace ver que es má s fiero un
hombre que un leó n.
Respuesta libre.
LA FACTORÍA DE TEXTOS de Quevedo. Léelo y contesta a las preguntas:
1. Lee el siguiente texto de carácter expositivo y realiza Es cosa averiguada, así lo siente Metrodoro Chío y otros mu-
chos, que no se sabe nada y que todos son ignorantes. Y aun
lue- go las actividades que te proponemos:
Durante siglos, la danza tuvo por encima de todo una
signi- ficació n ritual y religiosa. La adoració n y
propiciació n de los dioses, las rogativas de fertilidad,
buen tiempo, etc., se expre- saban con frecuencia mediante
formas tradicionales o impro- visadas de danza y de
melodía. En la é poca de los griegos y romanos, la danza
conoció una evolució n lenta, convirtiéndo- se de rito en
arte consciente. Sin embargo, ya sea sagrada o profana,
arte o no arte, hay algo fundamentalmente eró tico en la
danza que no parecía agradar a la Iglesia. Durante la Edad
Media se desaprobaba cualquier tipo de mú sica de danza
e incluso la danza en general. A pesar de ello, la danza
conservó su popularidad, hasta llegar finalmente a su
renacimiento en las Cortes europeas del siglo XVI.
Ottó KÁ ROLYI: Introducción a la
música
(Alianza Editorial).
a) Explica de forma razonada cómo están presentes
en el texto las tres cualidades que debe tener una
ex- posición (claridad, concisión y organización de
ideas).
b) Analiza la estructura del texto en función de la
orga- nización interna de las ideas, ¿de qué
manera está estructurada la información?
c) Di razonadamente cómo se emplea en el texto la
fun- ción representativa y por qué es un texto
objetivo.
¿Crees que, además, puede tener algún elemento
subjetivo? ¿Por qué?
a) El texto se ajusta a las tres características
señ aladas. A la claridad, porque no utiliza expresiones
ni ideas enre- vesadas, y expone de manera sencilla
aquello que quiere comunicar, sin dobles sentidos,
como cuando dice: En la época de los griegos y
romanos, la danza conoció una evolución lenta,
convirtiéndose de rito en arte consciente. A la
concisió n, puesto que no hay digresiones de ningú n
tipo y el autor se limita a exponer y explicar aquello
que se había propuesto, sin salirse del tema. A la
organizació n de ideas, porque va exponiendo
ordenadamente los pasos que se han producido en el
nacimiento y desarrollo de la danza, cronoló gicamente,
desde los tiempos prehistó ricos en que tenía un
cará cter ritual, hasta el siglo XVI en que ya se ha
convertido en arte, pasando por el mundo clá sico y
por la Edad Media.
b) La estructura del texto está basada, como acabamos
de señ alar, en la exposició n cronoló gica del desarrollo
de la danza: los ritos que la hicieron posible en tiempos
remotos, su conversió n progresiva en arte durante el
periodo gre- corromano, su desaprobació n por parte de
la Iglesia en los siglos medievales y su asimilació n a las
cortes europeas durante el siglo XVI, como arte y como
actividad para el disfrute de los sentidos.
c) Respuesta libre.
2. Redacta un texto expositivo sobre las principales
formas del baile en el siglo xx (tango, rock and roll, hip
hop, etc.), en el que tengas en cuenta lo que has
estudiado en esta página. Documéntate sobre el tema
antes de hacer el texto.
Respuesta libre.
3. Este texto argumentativo está extraído de los Sueños
esto no se sabe de cierto: que, a saberse, ya se supiera algo;
sospéchase. Dícelo así el doctísimo Francisco Sá nchez, médi- a) Investiga sobre el rapto de Europa por Júpiter. ¿En
co y filó sofo, en su libro cuyo título es Nihil scitur: No se sabe qué animal se transforma el dios para engañarla?
nada. En el mundo hay algunos que no saben nada y estudian b) Una constelación de estrellas tiene el nombre de este
para saber, y estos tienen buenos deseos y vano ejercicio: por- animal. Busca información sobre ella y sobre el
que, al cabo, solo les sirve el estudio de conocer có mo toda la tiempo en que el sol la ilumina, y relaciona lo que
verdad la quedan ignorando. Otros hay que no saben nada y
averigües con los versos.
no estudian, porque piensan que lo saben todo. Son de estos
muchos irremediables. A estos se les ha de envidiar el ocio y la c) Con los resultados de tus investigaciones y teniendo
satisfacció n y llorarles el seso. Otros hay que no saben nada, en cuenta todo el fragmento, explica el significado de
y dicen que no saben nada porque piensan que saben algo de la metáfora pura que contiene el último verso.
verdad, pues lo es que no saben nada, y a estos se les había
de castigar la hipocresía con creerles la confesió n. Otros hay, a) , b) y c) Respuesta libre.
y en estos, que son los peores, entro yo, que no saben nada 2. En los siguientes sonetos se dan definiciones del amor.
ni quieren saber nada ni creen que se sepa nada, y dicen de Léelos y resuelve las actividades que se plantean a con-
todos que no saben nada y todos dicen de ellos lo mismo y tinuación:
nadie miente.
Soneto amoroso definiendo el amor
Francisco DE QUEVEDO: «El mundo por de dentro»,
en Sueños y discursos. Es hielo abrasador, es fuego helado,
es herida que duele y no se siente,
a) Analiza la estructura del texto y razona a cuál de los es un soñ ado bien, un mal presente,
tres tipos expuestos arriba pertenece. es un breve descanso muy cansado.
b) ¿Qué predomina en el texto, la objetividad o la subje- Es un descuido que nos da cuidado,
tividad? ¿Crees que el autor intenta implicar al lector? un cobarde con nombre de valiente,
Razona tu respuesta. un andar solitario entre la gente,
c) Busca en el fragmento las principales características un amar solamente ser amado.
de los textos argumentativos y haz luego una tabla Es una libertad encarcelada
donde recojas cada uno de ellos con algún ejemplo que dura hasta el postrero parasismo1,
al lado. enfermedad que crece si es curada.
a) La estructura del texto es deductiva, ya que parte de una Este es el niñ o amor, este es su abismo:
tesis principal (Es cosa averiguada [...] que no se sabe ¡mirad cuá l amistad tendrá con nada
nada y que todos son ignorantes), para luego desarrollar el que en todo es contrario de sí mismo!
esta idea por medio de argumentos varios que, en este
Francisco DE QUEVEDO
caso, expresan las diferentes maneras de no saber nada
que considera el autor. 1
parasismo: paroxismo, momento extremo de una enfermedad,
b) Respuesta libre. agonía.
c) No hay un marcado uso de verbos de opinió n, pero sí apa-
rece a menudo el verbo ser, empleado para señ alar verda- Soneto 126
des que parecen indiscutibles en casi todas las ocasiones
Desmayarse, atreverse, estar furioso,
(es cosa averiguada, son los peores, etc.)
á spero, tierno, liberal, esquivo,
alentado, mortal, difunto, vivo,
Uso de verbos de opinió n dícelo, dicen
leal, traidor, cobarde y animoso;
Argumentos de autoridad «así lo siente Metrodoro Chío»
no hallar fuera del bien centro y reposo,
Obras científicas Nihil scitur, de Francisco mostrarse alegre, triste, humilde, altivo,
Sá nchez enojado, valiente, fugitivo,
satisfecho, ofendido, receloso;
4. Elabora tú un texto argumentativo en el que hagas una huir el rostro al claro desengañ o,
reflexión sobre la introducción de palabras extranjeras beber veneno por licor suave,
en nuestra lengua. Valora de manera razonada si esta olvidar el provecho, amar el dañ o:
inclu- sión de vocablos te parece positiva o negativa. creer que el cielo en un infierno cabe,
Respuesta libre. dar la vida y el alma a un desengañ o;
esto es amor, quien lo probó lo sabe.
ACTIVIDADES FINALES
Lope DE VEGA
Repasa lo que has aprendido a) Compara los dos poemas y explica qué características
1. Los siguientes versos son el comienzo de la «Soledad pri- comunes tienen.
mera» de Luis de Góngora. En ellos se sitúa el tiempo de
b) En los dos sonetos se emplea abundantemente la an-
la acción que se va a contar: la primavera.
títesis. Haz una relación de todas las que veas en cada
Era del añ o la estació n florida uno de ellos y explica qué sensaciones producen en
en que el mentido robador de Europa
el lector.
(media luna las armas de su frente,
y el Sol todos los rayos de su pelo), c) Los dos autores manejan la enumeración, pero de
luciente honor del cielo, forma diferente. Analiza este fenómeno en los dos
en campos de zafiro pace estrellas. poemas y di qué aporta al contenido de estos.
d) Relaciona las sensaciones contradictorias que mane- y má s, si pudiese, que todos. No sé si salí con ello, pero yo
jan los dos autores con la mentalidad barroca. ¿Te pa- aseguro a v. m. que hice todas las diligencias2 posibles.
rece que definen bien la época? ¿Por qué? Lo primero, yo puse pena de la vida a todos los cochinos que
a) Una característica comú n a los dos sonetos es la enumera- se entrasen en casa y a los pollos de la ama que del corral pa-
ció n, que en el caso de Quevedo se basa en una sucesió n sasen a mi aposento. Sucedió que un día entraron dos puercos
de metá foras y en el de Lope en un despliegue de del mejor garbo3 que vi en mi vida. Yo estaba jugando con
sensaciones contradictorias que se sustentan en la los otros criados, y oílos gruñ ir, y dije al uno:
antítesis. Tambié n en Quevedo se percibe esta ú ltima —Vaya y vea quién gruñ e en nuestra casa.
figura, asociada a las imá genes metafó ricas que contiene
el poema. Fue, y dijo que dos marranos. Yo que lo oí, me enojé tanto que
salí allá diciendo que era mucha bellaquería y atrevimiento
Otro rasgo comú n muy importante es el de postergar has- venir a gruñ ir a casa ajena. Y diciendo esto, envá sole4 a cada
ta el ú ltimo terceto el nombre de aquello de lo que se nos uno a puerta cerrada la espada por los pechos, y luego los
habla: el amor. En Quevedo vemos que lo desvela en el acogotamos5. Porque no se oyese el ruido que hacían, todos a
verso 12: este es el niño amor; en Lope de Vega, en el 14: la par dá bamos grandísimos gritos como que cantá bamos y
esto es amor. así expiraron6 en nuestras manos.
b) Antítesis en el poema de Quevedo:
hielo abrasador, fuego helado; herida que duele y no se 1
bellaco: perverso. 2 diligencias: trámites. 3 garbo: elegancia
siente; un soñado bien, un mal presente; un breve des-
en sus movimientos. 4 envásole: aquí, le metí. 5 acogotamos:
canso muy cansado; un descuido que nos da cuidado; un
sujetamos. 6 expiraron: murieron.
cobarde con nombre de valiente; andar solitario entre la
gente; una libertad encarcelada; enfermedad que crece
si es curada.
a) ¿Cómo se pone de manifiesto la desvergüenza del
Antítesis en el poema de Lope de Vega: protagonista?
Desmayarse, atreverse; áspero, tierno; liberal, esquivo;
b) ¿Quién actúa como narrador en el texto? ¿A quién se
alentado, mortal; difunto, vivo; leal, traidor; cobarde y
dirige? ¿Existe, pues, alguna semejanza con el Lazari-
animoso; alegre, triste; humilde, altivo; valiente, fugi-
tivo; satisfecho, ofendido; beber veneno por licor suave;
llo de Tormes? ¿Por qué?
olvidar el provecho, amar el daño; creer que el cielo c) En cambio, ¿qué diferencias sustanciales se ponen de
en un infierno cabe. relieve en la descripción de pícaros como Pablos y en
En ambos casos crean en el lector una sensació n de de- la de los protagonistas de los libros de caballerías tan
samparo y desasosiego, como si toda la vida se sustenta- admirados por don Quijote?
ra en ese sufrimiento placentero que resulta ser el amor d) Analiza los siguientes sintagmas nominales:
segú n nos lo pintan estos dos poetas: algo que nos gusta
y a la vez nos inquieta. • todas las diligencias posibles
c) Como dijimos antes, estas antítesis se basan en la metá - • dos puercos del mejor garbo
fora si hablamos de Quevedo, y en la enumeració n si lo e) ¿Cuál es el tiempo verbal que se suele usar en las
hacemos de Lope. Con las metá foras se nos muestra un uni- narra- ciones? ¿Qué tiempo predomina en este texto?
verso de imá genes que se asocian a la idea del amor
Fíjate, por ejemplo, en las formas puse, sucedió,
(hielo abrasador, fuego helado, libertad encarcelada,
etc.); con la enumeració n se nos presentan multitud de entraron o enojé.
sensaciones y estados de á nimo que tambié n f) La oración Vaya y vea quién gruñe en nuestra casa es
representan con firmeza lo que es estar enamorado. imperativa. Sin embargo, las formas verbales vaya y
d) Respuesta libre. vea no están en modo imperativo. ¿Por qué? ¿Cuál es
el tiempo y el modo de dichas formas?
3. Busca información sobre los reyes del siglo xvıı y elabora
un cuadro cronológico en el que recojas los principales g) Indica la función sintáctica de los términos marcados
acontecimientos históricos del siglo. en negrita:
Respuesta libre • dije al uno
4. Infórmate sobre la obra poética de Lope de Vega y com- • yo puse pena de la vida a todos los cochinos
pleta la información que te hemos ofrecido en esta uni- • envásole la espada por los pechos
dad. Haz luego una tabla en la que clasifiques sus obras
• y luego los acogotamos
en verso según sean líricas o épicas.
Respuesta libre. • no se oyese el ruido que hacían
• yo estaba jugando con los otros criados
Recuerda lo que ya sabías • yo aseguro a v. m.
5. Como es habitual en la novela picaresca, el protagonista a) El protagonista utiliza el engañ o y la falsedad para conse-
de Historia de la vida del Buscón llamado don Pablos de guir sus propó sitos. En este caso, hacerse con los cerdos
Quevedo comete múltiples tropelías. Ahora puedes leer para luego comé rselos é l. Para ello hace ver que está po-
lo que le ocurrió, estando en Alcalá de Henares al niendo orden en la casa en la que vive como hué sped. Por
servicio de don Diego Coronel. Después realiza las tareas otro lado, en lugar de intentar hacer el bien y enderezar
que se te plantean. su vida, decide irse por el camino má s fá cil, que es el de
hacerse bellaco, siguiendo el refrá n «haz como vieres»,
«Haz como vieres» dice el refrá n, y dice bien. De puro consi-
como él mismo nos dice.
derar en é l, vine a resolverme de ser bellaco1 con los bellacos,
b) El narrador es la primera persona, que, en este caso, se
Guarnició n1 tosca de este escollo duro
identifica con el protagonista de la novela, quien se dirige
troncos robustos son, a cuya greñ a2
a una tercera persona ausente, a quien llama vuestra mer-
menos luz debe, menos aire puro
ced (v.m.). En estos dos aspectos, el Buscón sigue de cerca
la caverna profunda, que a la peñ a;
al Lazarillo, que también está escrito en primera persona
caliginoso3 lecho, el seno oscuro
y tambié n se dirige a vuestra merced.
ser de la negra noche nos lo enseñ a
c) La descripció n de pícaros se centra en las características infame turba4 de nocturnas aves,
má s propias de su condició n, cercana a la delincuencia, gimiendo tristes y volando graves.
y, por tanto, habrá un realismo muy marcado en el que se
destacará n estos rasgos: bellaquería, mentiras, trampas, 1
guarnición: adorno. 2 greña: cabellos revueltos, desor-
doble moral, etc. Por el contrario, los caballeros andantes denados. 3 caliginoso: oscuro, tenebroso. 4 turba: multitud
suelen ser descritos siempre con una tendencia a la ideali- desordenada.
zació n, destacando sus cualidades caballerescas: el honor,
la valentía, las finezas como enamorados, la cortesía, etc.
d) todas las diligencias posibles describe la cueva que habita el cíclope Polifemo:
Det Det N Adj
dos puercos del mejor garbo
Det N Prep Adj N
+ Det
e) En las narraciones predominan los tiempos pasados, sobre
todo el preté rito imperfecto de indicativo y el preté rito
perfecto simple. En el texto se da esta sucesió n de
tiempos y se mezclan tambié n otros como el preté rito
imperfecto de subjuntivo o el presente de indicativo,
pero predomina el pretérito perfecto simple, como
demuestran los ejemplos extraídos: puse, sucedió,
entraron, enojé.
f) El imperativo es un modo que se expresa a menudo por
medio del presente de subjuntivo, como es el caso que
aquí se da. En general suele ser la segunda persona del
imperativo la que tiene una forma especial diferenciada
de otras formas verbales (ve tú), mientras que la tercera se
suele expresar, como hemos dicho, por medio del presente
de subjuntivo (vaya él). Como en este caso se emplea el
tratamiento de respeto (vuestra merced), aparece ligado
a la tercera persona, como ocurriría hoy con usted: «Vaya
usted a tal sitio».
g) • dije al uno
Complemento indirecto.
• yo puse pena de la vida a todos los cochinos
Complemento directo.
• envásole la espada por los pechos
Complemento indirecto (le); complemento circunstan-
cial de lugar (por los pechos).
• y luego los acogotamos
Complemento circunstancial de tiempo (luego); com-
plemento directo (los).
• no se oyese el ruido que hacían
Sujeto.
• yo estaba jugando con los otros
criados
Complemento circunstancial de compañ ía.
• yo aseguro a v. m.
Complemento indirecto.

MIRA A TU ALREDEDOR Y…
… ve más allá
1. El arte barroco se caracteriza por el recargamiento y el
retorcimiento de las formas. Un ejemplo puede ser el
altar del Transparente de la catedral de Toledo:
Observa la relación entre la imagen que te ofrecemos
y los siguientes versos de Góngora, en los que el poeta
a) ¿Qué elementos se emplean para dar la sensación de
recargamiento en cada caso?
b) En ambos casos se describe un lugar físico. Haz tú
una redacción en la que describas con tus palabras la
imagen del Transparente.
a) En el altar podemos ver có mo hay una gran cantidad de
adornos y figuras de á ngeles en medio de un recargamien-
to que deja pocos huecos sin decoració n. Las columnas
retorcidas que sustentan la parte alta y que delimitan el
altar (llamadas salomó nicas) contribuyen, con su extra-
ñ a forma, a dar ese aspecto abigarrado al conjunto. Los
má rmoles y los dorados completan el monumento barroco.
Por su parte, los versos de Gó ngora se retuercen también
por medio del hipérbaton: caliginoso lecho, el seno oscuro
/ ser de la negra noche nos lo enseña...; guarnición tosca
de este escollo duro / troncos robustos son. Para recargar
más la forma, el poeta introduce palabras de poco uso
y cultismos, como caliginoso. Tambié n llama la atenció n
el refuerzo de la oscuridad que se presenta en el verso
infame turba de nocturnas aves, donde lo tenebroso se
subraya con los acentos rítmicos del verso, que recaen
sobre la letra u en las sílabas que marcamos en negrita.
b) Respuesta libre.
2. La pintura de Juan de Valdés Leal (1622-1690) refleja de
manera visual el pesimismo y el desengaño barrocos:
a) La inscripción que se lee bajo el ataúd del primer
plano dice: Finis gloriae mundi («fin de las glorias del
mundo»). Fíjate en los elementos que componen el
cuadro y explica según ellos el sentido de la frase.
b) ¿Cómo contribuyen las tonalidades oscuras del cua-
dro a reflejar el pesimismo de la época?
c) La técnica del contraste, en pintura, se llama claros-
curo; infórmate acerca de este concepto y explícalo
en relación con la forma de pensar del Barroco.
d) En los ataúdes hay un obispo y un hombre de aspecto
rústico, tal vez un pastor; ¿qué significado tiene la
pre- sencia de dos personas de estamentos sociales
tan dispares? ¿Qué idea de la muerte nos transmite?
e) La balanza que hay en el centro pesa los atributos de
los dos personajes, sobre la inscripción «Ni más, ni
menos». Relaciona esto con la cuestión anterior.
a) Llama la atenció n, sobre todo, la figura del obispo, en pri-
mer plano, ataviado con todas sus galas terrenales: la mi-
tra, el bá culo, sus vestimentas lujosas… Todo ello para
dar cobertura a un cadá ver putrefacto cuya faz es ya una
calavera descarnada. En esto quedan las glorias del mun-
do, como dice la inscripció n que hay en primer plano,
bajo los ataú des.
b) La oscuridad reinante en el cuadro es la de la sepultura,
belleza. De nuevo la mentalidad pesimista del barroco, el tó -
la de un osario en el que reposan varios muertos. Esta
pico del tempus fugit, del vanitas vanitatum.
negritud hace má s terrible, si cabe, la triste escena que
nos muestra el cuadro. La ausencia de luz remarca el 2. En el siguiente texto se cuenta una de las bromas que
pesi- mismo barroco, que se concentra en las tumbas del solían hacerse durante el carnaval:
fondo: todo es tristeza, todo es pesar, todo es muerte. Dos mujeres que está n en una reja de un cuarto bajo, con un
c) Respuesta libre. instrumento de disparar agua, por las troneras 1 de una celo-
sía2, a un hombre vestido de negro que, descuidado, arrimado
d) El pintor ha elegido a un obispo y a un rú stico pastor a ella pasaba, le dan una rociada por el rostro que le turban 3
para hacernos ver la terrible realidad de la muerte y su los ojos y le desaderezan4 la valona5. El hombre prosigue su
poder igualador, de tan clara procedencia medieval. camino sin volver la cara al lugar de su ofensa. Pasa por la
Nadie es má s que nadie a los ojos de la muerte; para necedad del uso con silencio, no sé yo si con paciencia. […]
ambos se acaban con ella las glorias del mundo, sean Esta agua ofensiva, que se arroja las Carnestolendas 6, ni se
estas cuales sean. mezcla con la prudencia ni con la virtud. Con la prudencia
e) La balanza que una mano misteriosa sostiene sobre los no, porque es error descubierto hacer a otro ni aun el menor
dos muertos, y en la que se pesan los atributos del pastor dañ o. No con la virtud, porque no puede dejar de ser malo
a la izquierda (un carnero, un perro...), y los del obispo a la causar enojos vengativos.
derecha (un libro, un corazó n con el monograma de Cristo Juan DE ZABALETA: El día de fiesta por la tarde.
[JHS], etc.), es una alegoría de esa igualdad de todos ante
la muerte: tanto vale haber sido un obispo o haber sido 1
troneras: abertura pequeña en una muralla para disparar con
un pastor. Por eso se refuerza la imagen con la terrible
los cañones. 2 celosía: enrejado de madera. 3 turban: inquietan,
inscripció n: Ni más, ni menos.
incomodan. 4 desaderezar: descolocar. 5 valona: cuello grande
de una prenda de vestir. 6 Carnestolendas: Carnaval.
… encuentra la clave
1. Lee este fragmento del poema «A la rosa» de Francisco
de Rioja (1583-1659): ¿Qué piensas tú de esta forma de divertirse? ¿Te parece
que se siguen dando este tipo de bromas en la actuali-
Pura, encendida rosa,
dad? Razona tus respuestas.
é mula de la llama
que sale con el día, Respuesta libre.
¿có mo naces tan llena de alegría 3. Lee ahora el comienzo de una letrilla de Luis de Góngora
si sabes que la edad que te da el cielo
en la que se elogia la sencillez y la libertad de vivir según
es apenas un breve y veloz vuelo,
y ni valdrá n las puntas de tu rama
uno quiera, sin hacer caso del qué dirán:
ni tu pú rpura hermosa Ándeme yo caliente
a detener un punto y ríase la gente.
la ejecució n del hado presurosa? Traten otros del gobierno
del mundo y sus monarquías,
En la poesía barroca se convirtieron en un tópico las
mientras gobiernan mis días
com- posiciones dedicadas a determinadas flores. Fue
mantequillas y pan tierno;
una ten- dencia que puede relacionarse con la gran y las mañ anas de invierno
importancia que adquirió en la época la pintura de naranjada y aguardiente,
bodegones como este de Antonio de Pereda (1611- y ríase la gente.
1678).
a) ¿Qué te parece esta forma de pensar? ¿Crees que es
¿Qué relación crees que existe entre los versos de Rioja, una invitación a vivir? ¿Por qué?
el bodegón de Pereda, el tiempo y la belleza?
b) ¿Qué sensaciones provoca la vida que elige el poeta?
Tanto en el poema de Rioja dedicado a una rosa como en el ¿Positivas o negativas? Razónalo.
bodegó n de Pereda, se nos muestran realidades perecederas,
fugaces, ya que las flores se marchitan pronto y las naturale- c) Escribe tú una letrilla satírica sobre la importancia de
zas muertas del bodegó n se pudrirá n tambié n en poco tiem- las apariencias y de la libertad individual en la
po. Todo es bello (la flor, las manzanas, las granadas...), pero actualidad.
el tiempo acabará con ello, marchitá ndolo, ajando su lozana a), b) y c) Respuesta libre.
TODOS A UNA 12
APERTURA DE UNIDAD sería una amenaza para su propio padre, el rey. Segismundo,
cuya
1. ¿Qué es lo que pretende averiguar el juez? ¿A quién in-
terroga y por qué crees que lo hace?
Pretende averiguar quién de los habitantes de Fuente Ovejuna
mató al comendador. Interroga a un niñ o porque espera que,
al tratarse de un ser má s dé bil y al no estar instruido en el
arte de la mentira y el engañ o, su pureza y debilidad hará n
que diga la verdad.
2. Explica cuál es la actitud de los habitantes de Fuente
Ove- juna durante el interrogatorio a que los somete el
juez.
Se trata de un comportamiento solidario y valiente, aunque
un tanto temerario. Se escudan en la má xima de que la unión
hace la fuerza para proteger al verdadero ejecutor del cri-
men. Lo hacen porque, aunque uno ha sido el que ha llevado
a cabo la acció n, todos sentían el mismo odio hacia el Co-
mendador y estaban de acuerdo en cometer el crimen. Para
todo el pueblo de Fuente Ovejuna ha resultado liberadora
esta muerte.
3. Teniendo en cuenta dicha actitud, ¿qué queremos expre-
sar cuando hoy en día usamos la expresión «Todos a una,
como Fuente Ovejuna»? ¿Recuerdas alguna ocasión en
que hayas obrado así junto con tus amigos o
compañeros?
Queremos expresar que hay que permanecer unidos, actuan-
do de forma solidaria en una acció n o proyecto. Esto signifi-
ca que no debe haber discrepancia o actitudes enfrentadas,
sino que todo un grupo de personas, interesadas o afecta-
das por un mismo hecho, deben seguir las mismas directrices.
Normalmente se emplea en contextos en los que varias perso-
nas han sufrido lo que consideran una desgracia o injusticia
y deciden unirse para hacerse fuertes y encontrar apoyos en
dicha unió n.
4. Puedes ver la versión cinematográfica de esta obra de
Lope de Vega tecleando en YouTube «Fuente Ovejuna
pe- lícula» y compararla con la adaptación de Antonio
Gades (versión de ballet flamenco).
Respuesta libre.
5. Debate y comenta con tus compañeros cuál de estas ver-
siones te ha gustado más aportando argumentos claros y
ejemplos concretos que apoyen tu valoración.
Respuesta libre.
6. En el pueblo de Fuente Obejuna (Córdoba) se realiza
anualmente un montaje teatral en torno a esta obra. In-
vestiga sobre él y pon en común con tus compañeros la
información que hayas obtenido.
Respuesta libre.

VIVE LA LECTURA
Comprensión lectora
1.¿Cuál crees que es el tema general de este texto: una
ala- banza, una maldición, una declaración de amor, una
expo- sición de hechos, una queja o una narración?
Justifica tu respuesta.
La queja de Segismundo por su falta de libertad.
Todas las comparaciones a las que recurre el protagonista
inciden en la libertad de que gozan los elementos natrales y la
hace contrastar con su estado de riguroso encarcelamiento.
2. ¿Qué le ocurre a Segismundo? ¿Por qué?
Está preso porque antes de su nacimiento, un adivino
predijo que el niñ o que iba a nacer causaría grandes dañ os y

Lengua castellana y Literatura. 3.º ESO. Solucionario 107


12 TODOS A UNA
madre muere en el parto (signo que parece demostrar los severidad. Oraciones libres.
va- ticinios), es encarcelado nada má s nacer en una torre,
para intentar evitar que se cumplan las predicciones.
3. ¿Con qué elementos se compara Segismundo? ¿De dón-
de proceden tales elementos y qué tienen en
común?
¿Por qué crees que ha elegido el autor esta
comparación?
Se compara con elementos de la naturaleza que inspiran
sensació n de libertad: un ave, un pez y un bruto (las bestias
salvajes).
4. ¿Ha cometido algún delito Segismundo? ¿Cuál es, según
él? ¿Estás de acuerdo en que es un delito? Busca la
palabra en el diccionario y explica si Segismundo ha sido
autor de tal hecho.
No, pero é l se cuestiona si lo ha cometido y cree que sí. El
delito de que se acusa es de su propio nacimiento. Como no
ha tenido libertad nunca para cometer ninguna atrocidad,
llega a la conclusió n de que solo el hecho de existir debe ser
su delito.
El significado literal de la palabra delito es: culpa, crimen,
quebrantamiento de la ley.
Segú n esta definició n, podemos afirmar que Segismundo, al
carecer de libertad, no ha podido ser autor de ningú n delito
en este sentido. Aquí delito se utiliza casi en el sentido de
pe- cado, una especie de pecado original con el que el
personaje ha nacido, pues no ha tenido ocasió n de
adquirirlo por sus propias obras.
5. ¿Te resulta conmovedor este monólogo? ¿Conoces
algún otro personaje de la literatura aquejado por la
falta de libertad?
Respuesta libre.
Otros personajes de la literatura aquejados de falta de libertad:
• Cenicienta, en el cuento popular, carece de libertad para
actuar debido al estado de esclavitud a que la somete su
malvada madrastra.
• Rapunzel, protagonista de un cuento de los hermanos
Grimm, tambié n sufre falta de libertad, pues vive
encerra- da en una torre por culpa de un antojo que su
madre tuvo antes de traerla al mundo. Una bruja la ha
condenado a vivir presa en una torre en medio del
bosque. Esta prisió n también recuerda a la de algunas
damas de la literatura caballeresca, a las que los
caballeros han de rescatar.
• La Sachette de Nuestra Señora de Notre Dame de
París, de Victor Hugo, que sufre un patético e injusto
encarcela- miento parecido o Jen Valjean en Los
Miserables, también de Victor Hugo.
• El reo de muerte en el relato de Espronceda.
• El protagonista de Cárcel de amor de Diego de San
Pedro. En esta obra medieval, la cá rcel es metafó rica,
pues se tra- ta en realidad del sentimiento amoroso que
hace preso al protagonista.
6. Procura ponerte en la piel de este personaje y explica
cómo crees que te sentirías tú en su misma situación.
Respuesta libre.

Vocabulario
7. Segismundo acusa al cielo —al poder divino— de su justicia
y rigor. ¿Qué significa este último término? El
adjetivo riguroso deriva del sustantivo rigor; escribe
dos oraciones con él.
Rigor: excesiva y escrupulosa

108 Lengua castellana y Literatura. 3.º ESO. Solucionario


8. ¿Qué quiere decir el protagonista con el verso «para apu-
Procura vocalizar bien, marcar las pausas y darle a cada
rar mis desvelos»?
oración el sentido y la intención que el autor pretendía
Para ahondar en la causa de su sufrimiento y llegar al fondo que transmitiesen. Si quieres, puedes consultar la repre-
de la cuestió n, aunque sea para su mal. sentación que algún actor o actriz hayan realizado de
9. Busca en el diccionario las siguientes palabras y anota su este fragmento. Una de las representaciones más
significado. Después construye una oración con cada una recientes y novedosas es la de la actriz Blanca Portillo.
de ellas: Puedes acceder a ella a través de YouTube. Teclea
privilegio etéreas docto lamas albedrío «Blanca Portillo» «La vida es sueño» (RDE).
Respuesta libre.
• Privilegio: exenció n de una obligació n o ventaja
exclusiva o especial que goza alguien por concesió n de un
LITERATURA
superior o por determinada circunstancia propia.
• Etéreas: que pertenecen al cielo. (Etéreas salas: es una me- 1. Lee el siguiente texto de El capitán Alatriste, de Arturo
tá fora de los distintos lugares del cielo por donde pasa el Pérez-Reverte, y anota las características del teatro
ave). que aparecen en él. Compara este espectáculo con otros
• Docto: que a fuerza de estudiar ha adquirido má s conoci- ac- tuales de semejante seguimiento e importancia:
mientos que los comunes y ordinarios. Sabio. Desde el monarca hasta el ú ltimo villano, la Españ a del Cuar-
to Felipe amó con locura el teatro. Las comedias tenían tres
• Lamas: cieno blando, suelto y pegajoso, de color oscuro,
jornadas o actos, y eran todas en verso, con diferentes metros
que se halla en algunos lugares del fondo del mar o de los
y rimas. Sus autores consagrados, como hemos visto al re-
ríos, y en el de los recipientes o lugares en donde hay o ha
ferirme a Lope, eran queridos y respetados por la gente; y la
habido agua largo tiempo. Alga u ova de los lamedales o
popularidad de actores y actrices era inmensa. Cada estreno o
charcales.
reposició n de una obra famosa congregaba al pueblo y la corte,
• Albedrío: potestad o derecho de obrar por reflexió n y elec- teniéndolos en suspenso, admirados, las casi tres horas que
ció n. Dícese má s ordinariamente libre albedrío. duraba la representació n; que en aquel tiempo solía desarro-
llarse a la luz del día, por la tarde después de comer, en locales
Investigación y redacción al aire libre conocidos como corrales. Dos había en Madrid: el
10. Indaga sobre dos aspectos debatidos en la época en que del Príncipe, también llamado de La Pacheca, y el de la Cruz.
se escribió esta obra: la predestinación y el libre albedrío. • Socializació n del teatro (se extiende a todas las clases so-
Explica tu opinión sobre si existe o no un destino fijo del ciales): Desde el monarca hasta el último villano, la Es-
que el ser humano no puede huir. paña del Cuarto Felipe amó con locura el teatro.
En el siglo de oro existe una polé mica filosó fica entre el con- • Estructuració n de las obras en tres actos.
cepto de la predestinació n y el libre albedrío. Ante la duda • Escrito en verso, con diversidad de estrofas.
metafísica de si el hombre es libre y decide sobre su propia • Constituía un espectá culo de masas, muy querido por el
vida, los que defienden la idea de la predestinació n opinan pueblo.
que es Dios el que marca los designios del hombre y que este
en realidad no elige su destino, pues le viene dado desde el • Se representaba en corrales de comedias al aire libre y se
veían varias piezas.
momento del nacimiento. Los defensores del libre albedrío
opinan, en cambio, que si Dios creó al hombre, lo creó libre 2. Busca el esquema métrico de las estrofas que se mencio-
para que decidiera los derroteros de su propia vida y la direc- nan en el fragmento de El arte nuevo de Lope y anótalo
ció n de sus acciones. en tu cuaderno.
La vida es sueño recoge y analiza el conflicto entre ambas • Dé cima: estrofa de diez versos octosílabos cuya rima obe-
interpretaciones, por eso, Segismundo comienza la obra pre- dece al siguiente esquema: abbaaccddc.
destinado a la falta de libertad. Se le concede luego el libre • Soneto: poema estró fico de versos endecasílabos y
albedrío pero este es utilizado para obrar el mal y acaba de rima consonante formado por dos cuartetos (11A11B-
nuevo encarcelado. Parece así de nuevo predestinado al mal y 11B11A) y dos tercetos que pueden o no ser encadenados
a la prisió n. Pero la reflexió n final de la obra deja una puerta (11C11D11C-11D11C11D).
abierta a la idea del libre albedrío relacionada con la idea del
bien. El hombre debe obrar bien, tanto si se siente libre para • Romance: poema estró fico compuesto por versos octosíla-
elegir como si no lo hace, porque las buenas acciones nunca bos que riman en asonante los pares (8-8a8-8a).
está n de má s. • Octava: la llamada octava real es una estrofa de ocho ver-
sos endecasílabos y rima consonante cuyo esquema métri-
11. La vida es sueño se divide en tres actos y en cada
co es: ABABABCC.
uno de ellos hay un monólogo importante. ¿Recuerdas
qué es un monólogo? Busca los del segundo y el tercer • Terceto: estrofa de tres versos endecasílabos que riman en
asonante. Puede tener varios esquemas mé tricos, pero en
acto. Encontrarás la obra completa en la biblioteca
el caso de los tercetos encadenados, la rima es: CDC, DCD.
digital Ciu- dad Seva (ciudadseva.com).
• Redondilla: estrofa de arte menor compuesta por versos
El monó logo es un tipo de texto teatral en el que interviene octosílabos, con rima asonante o consonante, y el siguiente
solo un personaje. Sus palabras dan a conocer al pú blico sus esquema: abba.
pensamientos, reflexiones y sentimientos, pues expresa
sus ideas en voz alta. 3. Lee este fragmento de Tuerto, maldito y enamorado, de
Rosa Huertas, y busca información sobre los datos bio-
12. Ahora que has trabajado el vocabulario, vuelve a leer el
gráficos de Lope que menciona.
texto intentando comprenderlo por completo y realiza
una lectura dramatizada para tus compañeros. Le conté todos los amoríos de Lope, su juventud díscola y su
madurez desquiciada; le hablé de sus magníficas obras, de
12 TODOS A UNA

lo que significaron para la escena españ ola, de su fama sin


Los ú ltimos añ os de la vida de Lope estará n llenos de amar-
límites, de su increíble capacidad creadora y de su
guras: el rechazo de los nuevos gobernantes en los que
concepció n novedosa del teatro.
buscó el favor, la locura, ceguera y posterior muerte de
Pero le mentí. Le había dicho que iba a contarle todo lo que Marta de Nevares, la muerte de su hijo Lope Félix y el rapto
sabía y no lo hice. Mi narració n de la biografía del Fé nix de de su hija Antonia Clara.
los ingenios se detuvo en 1616, el añ o en que conoció a El Fénix de los Ingenios, apelativo que se había ganado por lo
Marta de Nevares. prolífico de su obra, murió en Madrid, el 27 de agosto de 1635
—Há blame ahora tú de él —le pedí—. Cué ntame qué recuer- y fue enterrado solemnemente en la iglesia de San Sebastiá n.
das de Lope despué s de lo que acabas de escuchar. Quiero La novela de Rosa Huertas repasa, a través del protagonismo
saber có mo era. […] de un espíritu que sufre la maldició n del propio Lope de Vega,
—Recuerdo sus cabellos grises, su bigote recortado y su la vida del autor. Se repasan los lugares en los que vivió y las
peri- lla. Era obstinado y vanidoso, pero claro y natural, sin mujeres a las que amó . Se recrea su cará cter vital y su
afeites ni rebozos. Se mostró siempre paternal conmigo, su intensa vida amorosa.
afecto era hondo y sincero y sé que me quiso tanto como 4. Reconstruye el argumento de El condenado por descon-
después me aborreció hasta maldecirme cruelmente. No era fiado de Tirso basándote en la intervención del Demonio
una actitud extrañ a en él, que amaba y odiaba con la y la última del protagonista, Paulo.
intensidad de los genios.
DEMONIO.— Diez añ os ha que persigo
Resumimos los datos biográ ficos de Lope de Vega, su intensa a este monje en el desierto […].
vida amorosa y su relació n con Marta de Nevares: Hoy duda de su fe; que es duda
Félix Lope de Vega Carpio nació en Madrid a finales de 1562. de la fe lo que hoy ha hecho,
Hay discusió n acerca de la fecha exacta. porque es la fe en el cristiano
que, sirviendo a Dios y haciendo
En cuanto a su formació n, Lope de Vega estudió primero en
Madrid, en la escuela regentada por Vicente Espinel y poste- buenas obras, ha de ir
riormente en la Compañ ía de Jesú s. Tambié n cursó un añ o a gozar dél en muriendo.
en Alcalá de Henares, aunque sin conseguir titulació n y, solo PAULO.— ¡Malditos mis padres sean
tal vez, en Salamanca. mil veces pues me engendraron!
¡Y yo tambié n sea maldito,
En 1583 zarpó a Lisboa y a su regreso, conoció a la primera de
las muchas mujeres a las que amó : Elena Osorio (a la que llama pues que fui desconfiado!
Filis en su poesía), hija del empresario Jeró nimo Velasquez, a Argumento de El condenado por desconfiado de Tirso: Las
la que acabó destinando una carta ofensiva que le ocasionó vidas del soberbio monje Paulo y la del criminal Enrico que,
el destierro. En 1588 contrajo matrimonio con Isabel de sin embargo, tiene esperanza en su salvació n, se entrecruzan.
Urbina (Belisa) y se alistó en la Armada. El criminal salvará su alma porque es capaz de amar, mientras
que el monje, por desconfiado, acabará condená ndose.
Tras finalizar el destierro, vivió en Toledo junto a su esposa
Isabel, quien murió al dar a luz. 5. Busca información sobre la obra de Tirso titulada Don Gil
En 1595 obtiene el perdó n (acaba el destierro de la corte) y se
de las calzas verdes y explica por qué la protagonista se
traslada a Madrid, en donde conoce a Micaela Lujá n (Camila disfraza de hombre.
Lucinda en sus versos), mujer casada —bella pero inculta— Juana, noble doncella de escasa fortuna, es seducida por
con quien mantendrá una relació n amorosa hasta 1608. Pero Martín, a quien su padre obliga a prometerse a una dama
en 1598 había contraído segundas nupcias con Juana de Guar- que considera mejor partido: Inés.
do, hija de un rico empresario, que apenas encontró menció n Como Juana carece de un caballero de su familia que pueda
en su literatura. vengar su honor perdido, será ella misma, disfrazada de hom-
En 1605 conoce y traba amistad con don Luis Ferná ndez de bre (Don Gil de las calzas verdes) quien se dirija a Madrid,
Có rdoba y de Aragó n, duque de Sessa, con el que mantendrá acompañ ada de su criado fiel (Quintana) con el propó sito de
a lo largo de toda su vida una extrañ a relació n en la que se rescatar su honor.
mezclan los papeles de secretario y confidente. 6. Lee este fragmento de El gran teatro del mundo, en el
En septiembre de 1610 Lope se traslada definitivamente a que, de forma metafórica, el autor del mundo reparte los
Madrid, en donde vivirá el resto de sus días. En 1609 había papeles.
ingresado en la Congregació n de Esclavos del Santísimo a) Explica cómo recibe el pobre el papel que le ha toca-
Sacramento. do en suerte.
Pero no duró mucho esta experiencia plá cida y sin contratiem- b) ¿Crees que está conforme? Explica por qué.
pos. Doñ a Juana sufre frecuentes enfermedades y en 1612 su
¿Por qué tengo de hacer yo
hijo Carlos Félix muere de unas calenturas. En agosto del añ o
el pobre en esta comedia?
siguiente Juana de Guardo muere también, al dar a luz. Lope
¿Para mí ha de ser tragedia,
decide entonces ordenarse de sacerdote. La huella literaria de
y para los otros no?
esta crisis y sus arrepentimientos irá a parar a las Rimas
¿Cuando este papel me dio
sacras, publicadas en 1614,
tu mano, no me dio en él
El recié n ordenado entró enseguida en la carrera de los be- igual alma a la de aquel
neficios eclesiá sticos. Pero duró muy poco la castidad del que hace al rey? ¿Igual sentido?
nuevo sacerdote. Ademá s de la relació n con una comedianta
a) Lo recibe con desagrado porque sabe que la pobreza será el
(«La loca») durante su viaje a Valencia de 1616, Lope tiene el
origen de las desdichas por las que tendrá que pasar en su
ú ltimo gran amor de su vida en otra mujer casada, Marta de
vida. Por eso habla de su vida como tragedia. Si el teatro
Nevares, a la que en los textos literarios llamará Amarilis y
es metá fora de la vida de estos personajes, la tragedia es
Marcia Leonarda.
metáfora de una vida mísera.

Lengua castellana y Literatura. 3.º ESO. Solucionario


1o9
TODOS A UNA 12
b) No está conforme porque en otros aspectos se siente igual
La ambigü edad se desprende del doble significado de soldados
al rey o al rico y no entiende qué criterio se ha usado
(participio del verbo soldar y sustantivo: luchadores a sueldo)
para repartir los papeles, es decir, las diferentes formas de
y celos (sustantivo en su significado de recelo, desconfianza
vida. No entiende que Dios le sitú e en inferioridad
o persona que está a tu cargo).
porque no siente que su alma sea inferior.
Llevar los celos soldados significa que se lleva con él los celos
7. Escribe en verso o en prosa una respuesta para el pobre, o el recelo que le produce saber que Casilda se queda a merced
como si tú hubieses repartido los papeles. de los deseos del Comendador.
Respuesta libre. Llevar a los soldados que son sus celos tiene el significado
8. Busca información sobre el mito grecolatino de literal: va a llevar a Toledo a los soldados a los que tiene a su
Narciso y Eco y lee luego este fragmento de la obra de cargo por orden del Comendador, encargo que este le ha
dado para que no estorbe sus planes.
Calderón titulada Eco y Narciso. Explica a qué
momento corres- ponden estas palabras que dice 2. ¿Qué características de la comedia barroca aprecias en
Narciso mirándose en una fuente. este fragmento?
NARCISO.— Có mo tú , hermoso prodigio, Los personajes. Aparecen tres de los personajes que má s se
solo me miras y callas, repiten en la comedia barroca: la dama, el villano
yo no hago má s que mirarte, (encarnado en la figura de Peribá ñ ez) y el poderoso
y callar; pero esto basta, (encarnado por el Comendador de Ocañ a).
porque como yo te vea, Los juegos verbales como el que se aprecia en las palabras
¿qué má s dicha? […] celos y soldados.
¡Qué divina
O este otro juego con el pronombre vos:
eres, deidad soberana!
Bella me pareció Eco … vengo, y a decir que os dejo
antes que a ti te mirara: a vos de vos misma en guarda,
pero después que te vi porque en vos y con vos quedo;
aú n no es tu sombra. Hay otro juego verbal con el que Peribá ñ ez da a entender a
Eco es una ninfa del bosque que protagonizó varios relatos Casilda que no tiene celos o recelo de ella sino de la causa
mitoló gicos. Era habladora y juguetona y solía entretener a por la que lleva a los soldados a Toledo, es decir, de la
la diosa Hera. Mientras, su esposo Zeus aprovechaba que persona que le aleja de allí con malas artes. Se refiere al
Hera estaba distraída para cometer sus infidelidades. Por Comendador.
ello, la diosa condenó a Eco a no poder hablar por sí misma El tema de la honra y los abusos de poder.
sino a re- petir tan solo la ú ltima palabra que otros dijesen.
Asustada, Eco huyó de los bosques en donde vivía y se
3. ¿En qué grupo de las comedias de Lope la incluirías?
refugió en una cueva, cerca de un riachuelo. En las de tema histó rico, concretamente en el grupo de los
dramas del poder injusto en los que la figura del poderoso
Narciso era un joven de gran belleza al que, al nacer, el adivino
suele abusar de su poder, en perjuicio de algú n villano hon-
Tiresias le había predicho que el ver su propia imagen en un
rado (Peribá ñ ez).
espejo causaría su perdició n. Por eso su madre se había encar-
gado de destruir todos los espejos y de evitar que pudiese con- Tema y estructura. Lope trata en esta obra los temas más
templarse a sí mismo. Así creció , sin conocer su propia belleza, importantes en el siglo xvıı. La obra consta de tres actos con
y se convirtió en un joven introvertido que se distraía dando una veintena de escenas cada uno.
largos paseos. Un día pasó cerca de la cueva en la que habitaba
4. ¿Cuáles de los temas más recurrentes de Lope y de la
Eco, quien se enamoró perdidamente al verlo. La ninfa empezó
a observarlo de lejos, pero un día Narciso la descubrió espian-
comedia barroca aparecen en este fragmento?
do. Cuando el joven le preguntó qué hacía allí, ella no pudo sino El amor (y su variante de los celos) y la honra.
repetir sus ú ltimas palabras. Esto le causó la risa a Narciso y 5. ¿En cuántas partes se divide este fragmento? Justifica
el despecho a Eco, que se fue a su cueva repitiendo las ú ltimas tu respuesta.
palabras de Narciso: Qué tonta…, Qué tonta…
Hay tres partes. Una primera abarcaría la primera interven-
Otra versió n cuenta que Eco, que sí podía hablar, le pidió ció n de Peribá ñ ez y Casilda, en la que se saludan y manifiestan
ayuda a la diosa Afrodita. Esta le concedió un tiempo para su amor.
que intentara conmover a Narciso, pero este estaba más
Las dos siguientes intervenciones de cada uno de ellos consti-
pendiente de mirar su reflejo en el agua. Pasado el tiem-
tuyen la segunda parte, en la que tratan el tema de la partida
po estipulado, la diosa castigó a Narciso a enamorarse de
de Peribá ñ ez. Siempre usando un có digo cifrado que solo los
su propio reflejo y a Eco a repetir las ú ltimas palabras de
enamorados entienden. Hablan del recelo que siente Peribá ñ ez
cuanto oyese.
al partir, no de Casilda, sino del Comendador y sus oscuras
El texto corresponde al momento en el que Narciso ve su re- intenciones.
flejo en el agua y se enamora profundamente de é l.
La tercera y ú ltima parte es la despedida de ambos y comienza
con la intervenció n de Casilda: Tomad… Contiene esta parte
COMENTARIO DE TEXTO el símbolo del listó n negro con el que Casilda intenta preve-
nir a Peribá ñ ez del peligro y que este sabrá interpretar, pues
Localización. Los esposos, Casilda y Peribáñez, sospechan vuelve de improviso a Ocañ a y salva a Casilda.
que el comendador trata de distanciarlos, mandando al espo-
Análisis de la forma y el contenido. Lope, en su Arte nuevo
so a Toledo, porque está enamorado de Casilda.
de hacer comedias, defiende la idoneidad de un tipo de
1. Explica la ambigüedad que se crea entre llevar «los celos estrofa según el tema tratado.
soldados» y «llevar a los soldados que son sus celos».
6. Mide los versos. ¿Qué estrofa se usa en este texto? Ponla
en relación con su contenido.
12 TODOS A UNA
11o Lengua castellana y Literatura. 3.º ESO. Solucionario
TODOS A UNA 12
Se trata del romance, poema estró fico de versos octosílabos Recela del Comendador porque se da cuenta de que le hace
con rima asonante en los versos pares. En este fragmento, salir de Ocañ a con la excusa de llevar soldados a Toledo para
efectivamente, los personajes tienen una relació n amorosa y mantenerlo lejos. Se aprecia en estos versos:
está n dialogando sobre los obstá culos que le han surgido a
la misma, para lo cual, segú n Lope, es adecuado emplear el
romance.
7. Aunque el personaje de Peribáñez se ha caracterizado
por usar un registro lingüístico más sencillo desde el
principio de la obra, quizá por haber sido ordenado
caballero su ex- presión se complica. Busca y explica los
recursos retóricos más significativos en sus parlamentos.
Uno de los recursos en los que mejor se plasma esa
complica- ció n del lenguaje es la paronomasia, que consiste
en jugar con palabras parecidas pero de distinto significado.
Así ocurre con las palabras soldados/soldándose y zelos.
Cuando Peri- bá ñ ez habla de los soldados (sustantivo) que
lleva a Toledo, Casilda, que ha entendido el sentimiento de
celos que Peribá - ñ ez se lleva consigo, usa otras palabras con
igual sonido pero diferente significado:
Si soldados los lleváis,
ya no ternéis pena de
ellos; que nunca el honor
quebró en soldándose los
zelos.
Tambié n encontramos exclamaciones retó ricas con las que
se nos da a conocer el hondo y sincero amor de Casilda y
Peribá ñ ez: ¡Ah, gallardo capitán!, ¡Ah dama, la del balcón!
Otro recurso es la metá fora del honor como un objeto quebra-
dizo: pues nunca el honor quebró, metá fora muy usada en
el teatro barroco.
Se recurre a la aliteració n de la n y la aná fora (ni) para rei-
terar la idea de que Peribá ñ ez está velando por su honor en
los versos:
… ni ellos fueran donde
van ni yo, señora, con
ellos.
Por otro lado, la situació n contradictoria que la intervenció n
del Comendador ha creado entre los enamorados se refleja
en antítesis como: La seguridad que es paz / De la guerra en
que me veo o juegos de palabras como a vos de vos misma
en guarda, porque en vos y con vos quedo.
Conclusiones. Peribáñez manifiesta en este texto que lo que
está en juego es su honor.
8. ¿Cree que va a perderlo o confía en que Casilda sabrá
ser fiel? Busca los versos en los que expresa esta opinión.
Peribá ñ ez es consciente del peligro, pero confía plenamente
en Casilda. Se lo hace saber en los versos:
Que si zelos fueran tales
que yo los llamara
vuestros ni ellos fueran
donde van ni yo, señora,
con ellos.
Con estos versos quiere decir que si creyera que tiene motivos
para estar celoso de ella o desconfiar, no se iría a ningú n
sitio.
Ademá s le dice má s adelante que la deja a cargo de sí misma,
es decir, que sabe que ella sabe cuidarse y que su firmeza es
para é l una garantía de seguridad. Pero Casilda, que le ve
confiado, le da a entender (con el símbolo del listó n negro)
que el peligro no está en ella, pero está , y él debe actuar en
consecuencia.
9. Localiza los versos donde Peribáñez manifiesta su temor,
y explica a quién hacen referencia.

Lengua castellana y Literatura. 3.º ESO. Solucionario 111


12 TODOS A UNA
No los llevo tan soldados, del amor antigua y
que no tengan mucho clásica tierra, y en ella el
miedo, no de vos, mas de la Emperador,
causa con ella y con Francia en guerra,
por quien sabéis que los llevo. díjeme: «¿Dó nde mejor?
Donde hay soldados hay juego,
EL JARDÍN DE LA LITERATURA hay pendencias y amoríos».
1.Tirso de Molina presenta por primera vez en la
literatura española la figura del donjuán: hombre
insensible y conquis- tador. Con él nace un mito que
encontrará su mejor versión, ya en el siglo xıx, en la obra
de José Zorrilla, Don Juan Teno- rio. Lee este monólogo
y contesta a las preguntas:
DON JUAN.— A mí el papel ha llegado
por la estafeta del viento.
Sin duda que es de una dama
que el marqué s me ha
encarecido. Venturoso en esto
he sido.
Sevilla a voces me llama
El Burlador, y el mayor
gusto que en mí puede haber
es burlar una mujer
y dejarla sin honor. […]
Ya está abierto el tal
papel y que es suyo es
cosa llana,
porque aquí firma: —Doñ a Ana—.
Dice así: «Mi padre infiel
en secreto me ha casado
sin poderme resistir;
no sé si podré vivir,
porque la muerte me ha
dado. Si estimas, como es
razó n,
mi amor y mi voluntad,
muéstralo en esta ocasió n.
Porque veas que te estimo
ven esta noche a la puerta,
que estará a las once
abierta, donde tu esperanza,
primo, goces, y el fin de tu
amor. […] Mi amor todo de
ti fío,
y adió s». Desdichado amante.
¿Hay suceso
semejante? Ya de la
burla me río.
Gozarela, vive Dios,
con el engañ o y
cautela
que en Ná poles a Isabela.
a) Explica la actitud del personaje con las mujeres y el
significado de la expresión «burlar a una mujer».
b) Teclea en un buscador los primeros versos de este
parlamento del Don Juan de Zorrilla; lee la
descripción completa de sus hazañas y realiza una
comparación de ambos personajes (el don Juan de
Tirso y el don Juan de Zorrilla).
DON JUAN.— Como gusté is, igual es,
que nunca me hago esperar.
Pues, señ or, yo desde
aquí, buscando mayor
espacio para mis
hazañ as, di
sobre Italia, porque allí
tiene el placer un
palacio. De la guerra y
112 Lengua castellana y Literatura. 3.º ESO. Solucionario
c) ¿Cómo crees que es Doña Ana? Describe su carácter. vigilancia
a) Su actitud es deplorable, pues de ella se desprende una
concepció n machista de la mujer como objeto. Don Juan
utiliza a las mujeres para el propio disfrute sin tener en
cuenta las expectativas amorosas que crea en ellas. No le
importa engañ arlas, utilizarlas y herirlas con su egoísmo.
Burlar a una mujer significa hacer que no oponga re-
sistencia para ser seducida. Por eso añ ade don Juan: es
burlar a una mujer / y dejarla sin honor. Deshonrar a
una mujer burlando su vigilancia y venciendo su firmeza
constituía una gravísima ofensa en esta é poca.
b) Respuesta libre.
c) Es una joven cá ndida e inocente que no se imagina las
intenciones de don Juan. Le abre las puertas de su casa
y de su corazó n porque cree firmemente en el amor que
don Juan le ha prometido, por eso afirma: Mi amor todo
de ti fío.
Es imprudente y apasionada, carne de cañ ó n para el in-
compasible seductor, que ya compara la burla que pla-
nea con otras realizadas en Italia, concretamente sobre
Isabella.
2. El siguiente texto de Calderón pertenece a La dama
duende, graciosa comedia en donde doña Ángela (joven
viuda) es custodiada rígidamente por sus hermanos. Con
mucha astucia, encuentra la manera de escribir e
intentar conquistar al huésped don Manuel entrando y
saliendo por una alacena:
BEATRIZ.— ¿Có mo este hombre
viendo que hay quien lleva y trae
papeles no te ha espïado
y te ha cogido en el lance?
Á NGELA.— No está eso por prevenir
porque tengo a sus umbrales
un hombre yo que me avisa
de quien entra y de quien
sale y así no pasa Isabel
hasta saber que no hay nadie.
a) ¿A qué tipo de obras de las que escribió Calderón
pertenece esta? ¿Por qué lo has sabido?
b) En esta obra Calderón utiliza el recurso de «la
tapada» en el personaje de doña Ángela, es decir,
convierte a esta en un personaje invisible cuya
identidad es desco- nocida. ¿Crees que este recurso
favorece que el hués- ped don Manuel, curioso por
naturaleza, se enamore de la que considera una
«dama duende»?
c) Continúa el fragmento con una intervención de don
Manuel explicando su curiosidad y amor hacia la
dama duende.
a) A las de capa y espada. Porque se ve el enredo, propio de
estas comedias, que lleva a cabo el personaje de Á ngela.
Se aprecia tambié n en la presencia de un objeto (la
alacena) que propicia este enredo.
Es habitual en estas comedias encontrar conflictos amo-
rosos y, en esta, el conflicto está servido por el hecho de
que la protagonista desafía la vigilancia de los hermanos
para jugar al amor con quien no debe.
b) Por supuesto. El hecho de que el personaje femenino
apa- rezca y desaparezca de manera misteriosa en la
habitació n de un curioso por naturaleza hace que Manuel
se sienta doblemente atraído por la dama. Doñ a Á ngela
represen- ta el amor, pero tambié n un apetitoso misterio.
En prin- cipio ella recurre a la alacena para huir de la
de los hermanos, despué s se transformará en un darle gusto. […] Lo trágico y lo cómico
recurso para aumentar el amor de Manuel, al que mezclado,
también ella ama. Y Terencio con Sé neca3, aunque sea
c) Respuesta libre. como otro minotauro de Pasifae4,

LA FACTORÍA DE LOS TEXTOS


1. ¿Cuál de los siguientes actos de comunicación
constituye un diálogo? Explica tu razonamiento.
a) El texto de La vida es sueño con el que se abre
la unidad.
b) El expendedor de gasolina invitándote a servirte
tú mismo.
c) Un interrogatorio de la policía.
d) Una entrevista realizada a un personaje famoso
publi- cada en el periódico.
e) Una conversación telefónica.
f) Una escena teatral en la que intervienen dos o
más personajes.
La c) si el interrogado colabora, la d), la a) y la f).
2. Rellena en tu cuaderno la tira cómica que aparece
en el lateral con un diálogo.
Respuesta libre.
3. En la página www.ivoox.com encontrarás un quiosco
de audios. Entra en ella y teclea en el buscador
«Entrevista a García Márquez» «Crónica de una
muerte anunciada». Escucha la entrevista y explica
luego de dónde nace la inspiración para escribir la
obra de que se habla y a qué género pertenece.
Respuesta libre.
4. Inventa una conversación en la que acuerdas con tus
ami- gos un plan para el fin de semana.
Respuesta libre.
5. Escribe un diálogo poético entre dos elementos
contra- rios (agua/fuego; guerra/paz; amor/odio;
recuerdo/olvi- do). Ilustra tu diálogo con alguna
imagen representativa y ponlo en común con tus
compañeros.
Respuesta libre.
6. Teclea en YouTube: «RTPA. Debate en 30. Jóvenes y
consumo de alcohol». Observa en Internet este
debate y decide si se ha llevado a cabo
correctamente. Señala también con qué ideas estás
de acuerdo y con cuáles no, justificando por qué en
cada caso.
Respuesta libre.

ACTIVIDADES FINALES

Repasa lo que has aprendido


1. Lee este fragmento y contesta a las preguntas:
Y cuando he de escribir una comedia,
encierro los preceptos1 con seis
llaves; saco a Terencio y a Plauto2 de mi
estudio, para que no me den voces; que
suele
dar gritos la verdad en libros
mudos; y escribo por el arte que
inventaron
los que el vulgar aplauso
pretendieron; porque, como las paga el
vulgo, es justo hablarle en necio para
hará n grave una parte, otra ridícula;
Estos preceptos grecolatinos durante el Barroco dejan de
Que aquesta variedad deleita mucho.
ser el modelo literario. La comedia barroca mezcla per-
Lope DE VEGA: El arte nuevo sonajes y gé neros (lo có mico y lo trágico), y desarrolla
de hacer comedias (Planeta). paralelamente varias acciones.

1
2. Lope dice en su Arte nuevo que al «vulgo hay que
preceptos: reglas. 2 Plauto y Terencio: autores más importan- hablarle en necio para darle gusto». Reflexiona y
tes del género teatral que destaca en Roma: la comedia. 3 Sé- responde:
neca: filósofo y escritor romano de tragedias. 4 Según un mito
clásico, Pasifae fue raptada por un minotauro (ser mitológico, a) ¿A qué clase social se refiere con «el vulgo»?
mitad toro, mitad hombre). b) ¿Crees que los espectadores se identificaban con per-
sonajes de las comedias barrocas? ¿Por qué y en
a) ¿Cuál es el tema de este texto? qué aspectos?
b) Explica los versos que están en negrita y conecta su a) Cuando hablamos de un fenó meno de popularizació n del
significado con las claves del éxito teatral de Lope teatro barroco nos referimos a que el pú blico, que en prin-
de Vega. cipio pertenecía a la clase media (funcionarios, soldados,
c) ¿Cuáles son los preceptos que dice Lope encerrar con comerciantes, nobles) empezó a incorporar personas de
seis llaves? Explica en qué consisten. Analiza su origen la clases má s bajas. Este nuevo pú blico estaba formado
principalmente por trabajadores manuales como zapate-
y lo que ocurre con ellos en el Barroco.
ros, artesanos, cocineros, etc. y que recibieron el nombre
a) El tema del texto es la escritura misma. Lope explica de mosqueteros, un pú blico poco culto e irrespetuoso, que
có mo es el proceso de escritura de sus comedias. resultaba especialmente ruidoso. Esa incultura es la
b) Lope dice que encierra los preceptos con seis llaves para que lleva al autor a tildar a este pú blico con el apelativo
referirse a que no sigue las reglas clá sicas ni ninguna de vulgar, aunque son los principales consumidores del
otra regla. En sus comedias busca encontrar argumentos teatro de la época.
que lleguen a un pú blico variopinto, para lo cual escoge b) Seguramente este pú blico se sentía identificado en unos
tam- bié n el lenguaje que todo el mundo puede entender. tipos de comedia má s que en otros, pero era un pú blico afi-
Sabe que su arte no es el arte refinado y sujeto a reglas cionado a las comedias, pues este era el mejor espectá culo
que siguieron los clá sicos porque prefiere que el a su alcance para llenar sus ratos de ocio.
pú blico no lo formen solo personas cultas. A un pú blico
que no es culto no se le puede ofrecer un teatro en el que La obras con las que má s podía identificarse esta clase
prime un registro culto. Por ello dice que sigue el arte social son la de enredo y de costumbres, las de tema his-
que inventa- ron los que el vulgar aplauso pretendieron tó rico (muy del gusto del pú blico), má s concretamente los
(aquellos que no escribían solo para un pú blico culto) y que dramas del poder injusto de Lope, especialmente las que
al vulgo hay que hablarle en necio para darle gusto. cuentan con la figura del villano. Algunos personajes co-
tidianos se prestan más a dicha identificació n, como el
Con la expresió n Lo trágico y lo cómico mezclado se re- villano (labrador honrado, humilde y trabajador) y el ca-
fiere a que, para los clá sicos, el teatro no debe mezclar la ballero (padre, hermano o marido de una dama, que vela
tragedia y la comedia; deben ser gé neros distintos. Pero por ella y que dicta las normas y se opone a los amoríos
Lope no respetó las reglas clá sicas y, precisamente, en entre el galá n y la dama). En este ú ltimo caso, tambié n
ello residió su é xito. Logró temas y tramas que atrajeron contribuye a la empatía con el personaje del caballero la
la atenció n de un pú blico variopinto. Se trataba de obras importancia que el concepto de la honra alcanza en esta
cercanas, en las que, debido a la mezcla de personajes y é poca. Cualquier padre, marido o hermano que asistiese
tonos, todos los espectadores se veían reflejados e implica- a una representació n podía sentirse retratado en el perso-
dos. Por eso habla tambié n de esta variedad: que naje del caballero, así como las mujeres podían sentirse
aquesta variedad deleita mucho. liberadas al contemplar có mo en las comedias de enredo,
c) Se refiere a los preceptos clá sicos, es decir, la regla de las damas logran burlar dicha vigilancia.
las tres unidades de la literatura grecorromana. Los es- Podemos decir que los autores del teatro barroco, especial-
tableció Aristó teles en su Poética. Estas reglas son bá si- mente Lope de Vega, supieron ganarse a los distintos secto-
camente tres: res (de ambos sexos) que componían su variopinto pú blico.
• Unidad de lugar: las obras deben desarrollarse en el 3. Lee el siguiente fragmento de la Poética de Aristóteles
menor nú mero posible de escenarios distintos.
y contesta después a las cuestiones.
• Unidad de tiempo: la acció n debe abarcar el menor tiem-
La perfecció n del estilo es que sea claro y no bajo. El que se
po posible, a poder ser un día.
compone de palabras comunes es sin duda clarísimo, pero
• Unidad de acció n: la obra debe abarcar una ú nica trama bajo. […] Será noble y superior al vulgar el que usa de palabras
principal. No es aconsejable que se desarrollen varias extrañ as. Por extrañ as entiendo el dialecto, la metá fora, la
tramas diferentes para que no se disperse la atenció n prolongació n y cualquiera que no sea ordinaria.
del espectador en varios sucesos.
a) Analiza si se trata de la misma concepción del estilo
Ademá s, se defiende la separació n de gé neros, tonos, re- que la que plantea Lope de Vega en El arte nuevo.
gistros y personajes.
b) Escribe un diálogo donde Lope y Aristóteles discrepen
La tragedia es un gé nero de tono serio, donde se desa-
rrollan sucesos graves y que tiene final desgraciado. Los
sobre algún aspecto teatral, como el uso de las reglas
personajes son de origen noble y el tono elevado. o la mezcla de géneros.
La comedia es un gé nero có mico, donde se desarrollan a) No. Aristó teles defiende aspectos del arte de la escritura
asuntos humorísticos, el tono es desenfadado y los per- radicalmente opuestos a los que defiende Lope. Aristó -
sonajes tienen origen humilde. teles propugna huir del estilo bajo, de la lengua comú n
y de la sencillez. Para Aristó teles, el lenguaje y la comedia
de Lope hubiesen resultado bajos, pues Lope buscaba
ante
todo la sencillez, ser claro. Con su política de hablar ejemplo, me parece un dislate que la ú nica obra que se
clara- mente a gente vulgar, Lope le resultaría a representa de vez
Aristó teles un escritor vulgar, alejado del lenguaje
perfecto y exquisito al que se refiere en su Poética.
El arte que defiende Lope es una literatura para las masas
y el arte que estipula Aristó teles es una literatura para una
minoría selecta, un pú blico culto.
b) Respuesta libre.
4. Investiga sobre las compañías de actores. Por ejemplo,
el bululú era una compañía de un único actor, ñaque es el
nombre de la compañía formada por dos actores que
lleva- ban piezas breves. Define las siguientes: gangarilla,
camba- leo, garnacha, bojiganga, farándula y compañía.
Cada compañ ía recibe un nombre segú n sus integrantes:
• Bululú : un solo actor.
• Ñ aque: dos actores que llevaban piezas muy breves.
• Gangarilla: dos hombres y un muchacho que hace de dama.
• Cambaleo: una mujer que canta y cinco hombres.
• Garnacha: cinco o seis hombres, una mujer y un
muchacho.
• Bojiganga: dos mujeres, un muchacho y seis o siete hombres.
• Fará ndula: siete u ocho hombres, dos mujeres y un
muchacho.
• Compañ ía: unos dieciocho actores y algunos auxiliares.
5. Recoge fragmentos de diálogos espontáneos y planifica-
dos (conversaciones con tu familia o tus amigos, fragmen-
tos de chats, entrevistas de radio o televisión, debates,
etc.) y analiza sus características. Expón oralmente tu
trabajo a tus compañeros.
Respuesta libre.
6. Analiza esta estrofa de El perro del hortelano, de Lope de
Vega. ¿Qué tipo de estrofa es? ¿Para qué se debe
emplear, según Lope?
DIANA.— Hame dicho cierta amiga
que desconfía de sí
que el papel que traigo aquí
le escriba. A hacerlo me obliga
la amistad, aunque yo ignoro,
Teodoro, cosas de amor;
y que le escribas mejor
vengo a decirte, Teodoro.
La estrofa que usa es la redondilla, que Lope recomienda para
las cosas de amor.
7. Este fragmento pertenece a una entrevista realizada a
uno de los directores de las Jornadas de Teatro Clásico
de Al- magro. Léelo y contesta:
PREGUNTA. Si nos fijamos en la cartelera teatral, vemos los
mismos títulos de Lope y Calderó n de toda la vida. Teniendo
en cuenta el gran corpus teatral á ureo, ¿por qué cree que no
se apuesta por ofrecer má s variedad? ¿Es una cuestió n «co-
mercial»? ¿Lo «nuevo» es arriesgado, no vende? Nos gustaría
saber cuá l es su opinió n sobre el repertorio actual del teatro
aurisecular1.
RESPUESTA DE FELIPE B. PEDRAZA. Mi opinió n es que muchas
de las obras que está n en repertorio reú nen mé ritos má s que
sobrados para que fijemos nuestra atenció n en ellas. La
vida es sueño, El alcalde de Zalamea, El perro del hortelano o
Don Gil de las calzas verdes son obras maestras del teatro
univer- sal. Está plenamente justificada la atenció n que les
pueden prestar los có micos y los espectadores. Es verdad
tambié n que una parte del actual repertorio es fruto de
ciertos aza- res del mundo de la crítica y la edició n. Así, por
en cuando de Rojas Zorrilla sea Entre bobos anda el juego y concreta- mente los que se incluyen en El conde Lucanor
no Donde hay agravios no hay celos, que es, en mi son: marco narrativo, el cuento propiamente dicho y una
concepto, una propuesta infinitamente má s perfecta y moraleja que se desprende del cuento.
actual. […] Al margen de esto hay unas decenas de comedias d) En el fragmento poé tico se observa la parte central, la
y tragedias de prime- rísimo orden que rara vez se ven en del cuento. La moraleja que se extrae es que siempre que
los escenarios. Pienso, por ejemplo, en Los cabellos de alguien se queja hay alguien en una situació n peor, pero
Absalón o Los malcasados de Valencia o Eco y Narciso o no está explícita.
El marqués de Mantua o La prueba de las promesas y un
largo etcétera.
Revista Anagnórisis, n.º 2, diciembre de
2010.

1
áureo y aurisecular: del Siglo de Oro.

a) Busca información sobre el autor y el argumento


de las obras teatrales que menciona el
entrevistado.
b) Explica por qué este fragmento es un texto
dialogado.
c) ¿Es un texto dialogado planificado o espontáneo,
oral o escrito?
a) Respuesta libre.
b) Es un texto dialogado porque en él dos personas
intercam- bian informació n respetando el turno de
palabra.
c) Se trata de una entrevista, por lo que es un diá logo
plani- ficado. Aunque se ha recogido por escrito, esta
entrevista se ha realizado seguramente de forma oral.
Es probable que al dejarla por escrito se hayan
abreviado o retocado partes, pues no apreciamos
vacilaciones, repeticiones, mu- letillas, etc., bastante
habituales en la lengua oral.

Recuerda lo que ya sabías


8. Recita en voz alta este fragmento de La vida es
sueño y da respuesta a las cuestiones que se
formulan sobre él:
Cuentan de un sabio que un
día tan pobre y mísero
estaba,
que solo se sustentaba
de unas hierbas que cogía.
¿Habrá otro, entre sí
decía, má s pobre y triste
que yo?; y cuando el
rostro volvió halló la
respuesta, viendo que
otro sabio iba cogiendo
las hierbas que é l arrojó .
a) Realiza su esquema métrico e indica la estrofa que
utiliza.
b) ¿Te parece que el contenido es más propio de la
lírica barroca o de la renacentista? ¿Por qué?
c) La estructura de este fragmento es la misma que la
de los cuentos. Establece la correspondencia entre
los ver- sos y las tres partes de un cuento.
d) Explica qué moraleja se puede extraer del
fragmento.
a) La décima: 8a8b8b8a8a8c8c8d8d8c.
b) De la renacentista, por el pesimismo que encierra. La
vi- sió n de la existencia humana durante el
Renacimiento es má s optimista y no hay alusiones tan
claras a las miserias humanas.
c) Las tres partes propias de un cuento o medieval,
9. El argumento de este fragmento retoma el de un cuento pero mucho antes cayó ,
de El conde Lucanor de Don Juan Manuel, titulado «Los y agora salir querría.
altramuces». Léelo y resuelve las preguntas.
a) Repasa el argumento de ambos textos y establece si-
militudes y diferencias.
b) Recuerda que todos los cuentos de El conde
Lucanor
tienen las mismas partes. Señálalas en «Los
altramuces».
a) Similitudes: la situació n relatada segú n la cual un
hombre come lo que para otro son desechos. Diferencias:
en el cuen- to de los altramuces, el alimento despreciado
son las pieles de los altramuces y los dos hombres
dialogan.
b) Al principio, el conde Lucanor le plantea su problema
a Patronio (que aun teniendo bienes, a veces se siente
acosado por la pobreza); Patronio le cuenta la historia
del hombre que comía la piel de los altramuces de otro
hombre que la despreciaba; luego le dice al conde que
reflexione sobre los motivos que é l puede tener para sen-
tirse desesperado y que piense en personas que está n en
peor situació n. El narrador nos indica que al conde le
sirvió la lecció n. Luego, se alude a que don Juan Manuel
vio que el cuento era interesante y lo mandó escribir.
Por ú ltimo, aparece la moraleja: Por padecer pobreza
nunca os desaniméis porque otros más pobres un día
encontraréis.

MIRA A TU ALREDEDOR Y…
… ve más allá
1. El teatro del Siglo de Oro ha dejado su huella en expre-
siones populares, y algunas expresiones populares se han
inmortalizado gracias a él. Explica estas:
a) Hacer algo todos a una.
b) Ser (como) el perro del hortelano.
c) Ser un donjuán.
a) Actuar de forma uná nime.
b) Impedir que otros disfruten de algo aunque no se esté dis-
puesto a disfrutarlo uno mismo.
c) Se dice de un hombre que es un seductor y tiene é xito
con las mujeres, aunque hoy en día se usa de forma
intras- cendente, sin la carga negativa de la que sí está
dotado el personaje de Tirso.
2. El cine ha sabido recoger las tramas barrocas. Un
ejemplo de ello es la versión cinematográfica dirigida por
Pilar Miró de El perro del hortelano de Lope, parte de
cuyo rodaje se realizó en un escenario de lujo: el castillo
de Sintra (Portugal).
Lee este fragmento y compáralo con la escena cinemato-
gráfica. (Teclea en YouTube «El perro del hortelano ¿Llo-
ras?»). Después, resuelve las cuestiones:
DIANA.— Teodoro.
Tú te partes; yo te adoro.
TEODORO.— Por tus crueldades me voy.
DIANA.— Soy quien sabes, ¿qué he de hacer?
TEODORO.— ¿Lloras?
DIANA.— No, que me ha caído
algo en los ojos.
TEODORO.— ¿Si ha sido
amor?
DIANA.— Sí debe de ser;
TEODORO.— Yo me voy, señ ora mía;
Yo me voy, el alma no. […]
ANARDA.— Perdidos los dos está n.
DOROTEA.— ¡Qué mal se encubre el
amor! ANARDA.— Quedarse fuera mejor.
Manos y prendas se dan.
DOROTEA.— Diana ha venido a ser.
Lope DE VEGA: El perro del hortelano.
a) ¿Es este un texto dialogado? Razona tu respuesta y,
si es afirmativa, indica de qué tipo.
b) Valora qué te ha parecido la adaptación
cinematográ- fica. ¿Se adecúa la representación de
la actriz Emma Suárez al carácter variable de Diana?
c) Escribe unos versos más en los que Diana convenza a
Teodoro para que no se vaya.
a) En realidad hay dos diá logos. El de Teodoro y Diana por
un lado. En él los personajes intercambian informació n,
aunque si bien es cierto, la informació n má s importante
que debieran ofrecerse (que está n enamorados el uno
del otro) no llegan a manifestarla.
Por otro lado, está el diá logo de dos personajes que
espían la escena sin ser vistos: Dorotea y Anarda. Este es
un diá - logo más acertado, pues no se ocultan la
informació n que saben, como sí ocurre en el caso
anterior.
b) Respuesta libre.
c) Respuesta libre.
3. En Inglaterra, el Barroco también es una época de auge
para el teatro. Realiza un mural en el que compares el
teatro barroco en España y en Inglaterra (espacios de
representación, compañías, autores, etc.). Aquí tienes
algunas imágenes de los lugares de representación en
ambos países:
Respuesta libre.
4. El autor más famoso del teatro inglés es William Shakes-
peare y una de sus obras más conocidas es Romeo y
Julieta. Busca información sobre el argumento y haz un
resumen en tu cuaderno.
Romeo y Julieta, dos jó venes de la Verona medieval,
pertene- cen a familias encontradas, cuyo odio es el mayor
impedimen- to que encuentran los muchachos para
disfrutar de su amor. Para poder escapar de esta situació n,
en la que se debaten entre el respeto a sus familias y el
amor que les une, idean un plan que consiste en que
Romeo fingirá su muerte para huir con Julieta. La
casualidad hace que Julieta carezca de la informació n de
que la muerte de Romeo es fingida. Por eso, cuando ve a su
amado aparentemente muerto, saca una daga y se la clava.
Al despertar Romeo de su letargo, contempla a Julieta,
muerta a su lado, y se da muerte también.

… encuentra la clave
1. Te recomendamos que veas la película titulada Shakes-
peare in love, de John Madden. Además de reflejar
cómo funcionaban las compañías teatrales incluye la
represen- tación de fragmentos de Romeo y Julieta.
Observa en ella el papel de la mujer en el teatro barroco
como personaje y como actriz.
Respuesta libre.
2. Lee este fragmento de La dama duende, de Calderón de
la Barca, y valora si la actitud de los hermanos de doña
Ángela te parece la correcta o constituye un caso de dis-
criminación sexista.
Á NGELA.— ¡Vá lgame el cielo, que yo
entre dos paredes muera, 4. Busca en Internet figurines femeninos del Siglo de Oro,
donde apenas el sol sabe conviértelos en personajes femeninos de la época que
quié n soy! Pues la pena mía rei- vindiquen, en prosa o en verso, el derecho a la
en el té rmino del día igualdad.
ni se contiene, ni cabe […] Respuesta libre.
donde, en efecto, encerrada,
sin libertad he vivido, 5. Escucha la canción de Nach e Ismael Serrano titulada
porque enviudé de un marido, Ellas, un bello y poético tributo a las mujeres. En esta
con dos hermanos casada. canción, el autor habla de los sentimientos como si
Y luego delito sea fuesen figuras femeninas. ¿Qué crees que quiere
sin que toque en liviandad, expresar con estas ale- gorías? Relaciona el contenido de
depuesta la autoridad la canción con su estri- billo y su título.
ir donde tapada vea Esta canció n habla de distintos sentimientos personificados
un teatro en quien la fama en distintas mujeres.
para su aplauso inmortal
con acentos de metal Tanto las distintas personificaciones y metá foras de que se
a voces de bronce llama. compone la canció n como el estribillo ensalzan el papel que
¡Suerte injusta! ¡Dura estrella! tiene la mujer en la vida del hombre. Cada una representa un
ISABEL.— Señ ora, no tiene duda sentimiento nuevo y cada uno hace evolucionar al hombre y
de que mirá ndote viuda, todas dejan su huella imborrable:
tan moza, bizarra y bella, Luz (la pureza), Ilusió n (el amor adolescente, la belleza, el
tus hermanos cuidadosos idealismo), Indiferencia (el apartamiento, la intromisió n),
te celen, porque este estado Melancolía (el desencuentro y la tristeza se personifica aquí
es el má s ocasionado en una especie de mujer fatal), Pasió n (el amor carnal) Cons-
a delitos amorosos. tancia (representa el amor constante, la promesa), Envidia
(el fracaso, la envidia misma), Nostalgia (la cobardía),
Respuesta libre.
Soledad (el abandono), Esperanza (la esperanza misma),
y. Debate sobre la consideración de la mujer en el siglo xvıı Libertad (la fidelidad).
y en la actualidad. El trabajo de actor era exclusivo de los La enumeració n de todas ellas habla de la intensa vida senti-
hombres. ¿Hay profesiones en la actualidad que le están mental en la que representan hitos importantes.
vedadas a la mujer? Tomad posición a favor o en contra El estribillo ensalza el valor de cada una de ellas, es decir, de
de que la mujer acceda a cualquier tipo de profesión. cada una de las mujeres que pasaron por la vida del hombre
Nombrad un moderador y entablad un debate. Recordad y el sentimiento que despertaron; la huella que dejaron. Para
que debéis respetar todas las normas estudiadas para conferirles ese valor recurre a una serie de metá foras lumino-
que este diálogo lo sea realmente. sas muy logradas (bálsamo para tu herida, la vida tiritando
Respuesta libre. en una estrella, los restos de un naufragio, el cielo en el que
sueñan los cautivos, brillaban como diamantes). Todo eso y
mucho más son ellas, las mujeres.
ANEXO: LAS TIPOLOGÍAS TEXTUALES EN LA LENGUA ORAL

1. ¿Conoces el canal 24 Horas en directo? Búscalo en www. (El personaje hace una pausa y modifica la expresión del
rtve.es. ¿Cuál es su contenido? ¿Qué tipologías textuales rostro, silencio atento entre los oyentes).
aparecen ahí?
… siento, como siempre, un respeto pavoroso hacia todo
Noticias y temas de actualidad. Las tipologías textuales son aquel que realiza un compromiso como el que Angust y
informativos, entrevistas, tertulias y reportajes. Laura han sellado hoy, yo no sería capaz de hacerlo y creo
2. Busca la página del programa Informe semanal en la web que es mara- villoso que ellos sí hayan sido capaces.
mencionada. Visítala, escoge un reportaje que te interese a) ¿De qué recursos se vale el hablante para llamar la
y realiza una exposición que presentarás oralmente en atención de los oyentes?
clase, con los siguientes puntos: rasgos que lo b) ¿Cuál es la finalidad comunicativa que persigue con
caracterizan como texto oral, rasgos del reportaje, tema su intervención?
tratado, vigencia o interés de este.
c) ¿Hay alusiones a la situación comunicativa? ¿Cuáles?
Respuesta libre.
d) ¿Intenta el emisor conectar emocionalmente con los
3. Tecleando «Carmen Vaquero» «Garcilaso de la Vega» en
oyentes? ¿Cómo?
el buscador de la página march.es, encontrarás una con-
ferencia de la profesora Carmen Vaquero sobre Garcilaso e) ¿En cuál de las tipologías textuales que has estudiado
de la Vega. Escucha los primeros cinco minutos y lo situarías? Justifica tu respuesta.
comenta los rasgos de este tipo de discurso que a) De recursos no verbales, como los toques en las copas de
encuentres. cristal; de expresiones apelativas (vocativos como damas
y caballeros); se pone también de manifiesto el intento de
En ese fragmento pueden comentarse los siguientes rasgos:
conectar con los receptores, primero por medio del hu-
presentació n de la conferenciante, alusió n al contexto co-
mor (la pareja en cuestión aún me habla) y despué s en
municativo, captació n de la atenció n de los oyentes (funció n
el aspecto emocional (siento como siempre un respeto
apelativa del lenguaje), utilizació n de elementos visuales que
pavoroso hacia todo aquel que realiza un compromiso…).
sirven de complemento a la informació n transmitida, funció n
representativa del lenguaje. b) La finalidad es homenajear a los protagonistas de la ce-
lebració n, resaltar un momento especial. El emisor pre-
4. ¿Escuchas la radio? ¿Qué emisoras conoces? Elige una y tende expresar sus ideas y sentimientos hacia ellos, a
busca un programa de tertulia radiofónica. ¿Qué temas modo de dedicatoria.
tratan? ¿Cómo se dirigen unos a otros los tertulianos?
c) Sí, al principio: Siento interrumpirles el delicioso postre.
¿Hay intervenciones de los oyentes? Explica el modo en
que el moderador les da la palabra a estos últimos y d) Sí, al expresar sus ideas y sentimientos, su punto de vista
personal.
cómo se ex- presan cuando intervienen.
Respuesta libre. e) Es un discurso, pues pretende resaltar una ocasió n espe-
cial, y se trata de una exposició n en la que la funció n ex-
5. Aquí tienes un fragmento de una humorística presiva se superpone a la representativa.
intervención del protagonista de la película Cuatro bodas
6. Cread un anuncio publicitario utilizando diversos recur-
y un funeral (Mike Newell, 1994).
sos comunicativos (verbales y no verbales). Vais a lanzar
(La intervención comienza con unos suaves toques del una nueva aplicación para móviles que vosotros mismos
hablan- te sobre las copas de cristal).
inventaréis. Convenced a los compañeros de su utilidad.
Damas y caballeros, siento interrumpirles el delicioso
Respuesta libre.
postre. Ya que soy el padrino me gustaría decirles un par de
cosillas. Esta es la segunda vez que hago de padrino y espero 7. ¿Has asistido alguna vez a una sesión de cuentacuentos?
haberlo hecho bien la primera. La pareja en cuestió n aú n me ¿En qué consiste? ¿Qué recursos emplean los narradores
habla, desgraciadamente en la actualidad no se hablan entre para conectar con el auditorio? El lenguaje verbal pre-
ellos, se divorciaron hace ya un par de meses, pero les domina, pero ¿hay elementos paralingüísticos? ¿Cuáles?
aseguro que yo no tuve nada que ver. […] ¿Qué elementos acústicos o visuales emplea?
(Rostros estupefactos y desaprobadores entre los Respuesta libre. No obstante, el alumno deberá aludir a la
miembros más maduros del auditorio). combinació n de elementos verbales y no verbales, donde
Mi misió n es la de hablar de Angust y é l no tiene ningú n lío la paralingü ística tiene una importancia clave (gestos, ento-
escabroso, o al menos eso creo… nació n, movimientos corporales). Como apoyo el
(Risas de los oyentes). cuentacuen- tos recurre a la mú sica y, en algunos casos, a
la proyecció n de imá genes.
Luego volveré sobre el tema, ahora querría decir que…

Lengua castellana y Literatura. 3.º ESO. Solucionario 117


GUÍAS DE LECTURA
buenas costumbres.

1 Don Juan Manuel. El conde Lucanor.


«De lo que aconteció a una mujer que
le decían doña Truhana»
Localización
1. ¿Recuerdas en qué consiste la «literatura de exempla»?
Busca información sobre otras colecciones de «exempla»
que se difundiesen en la Edad Media y anótalas.
Son colecciones de cuentos de origen folcló rico que un autor
recopila para utilizarlos como ejemplos con objetivos
morali- zantes o didá cticos. Obras muy destacadas son
colecciones: Calila e Dimna y Sendebar.
2. ¿Qué es un apólogo? Busca información sobre este con-
cepto y anota su definición.
Narració n en prosa con finalidad didá ctica o moralizante, y
que suele acompañ arse al final de una moraleja.
3. Investiga qué lugar ocupa el cuento de doña Truhana en-
tre los cincuenta y un cuentos de esta obra.
El nú mero siete.
4. Busca información sobre las otras cuatro partes que,
junto a los cincuenta y un cuentos, constituyen el
conjunto de El conde Lucanor.
Segunda parte: enseñ anza moral a partir de cien aforismos.
Don Juan Manuel declara su intenció n de seguir abordando
los temas que puedan contribuir a la salvació n de las almas
de los hombres a la vez que a conservar su riqueza, su posició n
y su dignidad.
Tercera parte: semejante a la anterior, basada en cincuenta pro-
verbios. Persigue los mismos objetivos que en las dos partes
anteriores pero la comprensió n es má s difícil que en ellas.
Cuarta parte: prevenció n de Patronio sobre la complejidad con
que abordará los temas que le ocupan ahora, y que exigirá
del conde la mayor concentració n.
Quinta parte: Patronio explica los requisitos necesarios para
la salvació n del alma: la fe sin dudas, la prá ctica de bue-
nas obras con la finalidad de merecer el cielo, evitar las
malas acciones.
5. Investiga sobre Calila e Dimna y explica las relaciones
que existen entre este libro y El conde Lucanor.
La estructura es muy semejante, así como el marco
narrativo. En ambos casos se parte del motivo del mentor
que instruye al joven. Ademá s, algunos cuentos de El conde
Lucanor está n tomados del Calila e Dimna, como el de doñ a
Truhana.
6. El argumento del cuento de doña Truhana, sin duda,
debe de resultarte familiar. ¿Con qué otro relato
podemos aso- ciarlo? Justifica tu respuesta.
El cuento de la lechera, que se centra en el tema de la joven
fantasiosa que concibe ilusiones vanas y las pierde antes de
haberlas rozado siquiera.
7. Indica en qué estamentos estaba organizada la sociedad
medieval.
Nobleza, clero (alto y bajo) y pueblo llano (campesinos, villa-
nos, artesanos y burgueses).
8. Señala en manos de quiénes estaban la cultura y el
conoci- miento en la Edad Media. Reflexiona sobre esto y
respon- de: ¿a qué podía deberse el interés de don Juan
Manuel por transmitir enseñanzas a otros?
De los nobles y, sobre todo, del clero. Don Juan Manuel
pretende mantener el orden social establecido pero tambié n
considera que el hecho de ser un hombre formado y culto le
confiere la res- ponsabilidad de instruir a otros sobre las
GUÍAS DE LECTURA
respuesta.
Respuesta libre.
Intención
9. Recuerda la biografía de don Juan Manuel y luego indica 19. ¿Conoces la expresión «vender la piel del oso antes de
dónde se situaba él en la organización social de la Edad cazarlo»? Explica su significado y relaciónalo con la con-
Media. ducta de doña Truhana.
En el estamento de los privilegiados, concretamente en la
nobleza.
10. ¿Crees que la posición social de don Juan Manuel deter-
minaba las conductas morales que transmitió en su
obra?
¿En qué sentido?
Sí. Las conductas que aconseja contribuyen a mantener el
orden social establecido en su época.
11. Señala qué importancia tiene el didactismo en la
literatura medieval. A continuación, indica qué otros
autores y obras de la Edad Media se definen por su
intención didáctica.
Es uno de los rasgos má s llamativos de la literatura de esta
época. Ejemplos de didactismo los encontramos en las
colec- ciones de exempla, pero tambié n en obras del
mester de cle- recía, como las de Gonzalo de Berceo.
Contenido
12. En efecto, muchos de los cuentos de El conde Lucanor
pre- sentan dificultades o conflictos frecuentes entre los
nobles del siglo xıv. ¿Ocurre esto en el cuento de doña
Truhana?
¿Cuál es en este caso la situación que preocupa al conde?
Al conde le han propuesto un negocio que, segú n parece,
podría conllevar consecuencias muy positivas, que irían
des- gajá ndose unas de las otras.
13. Una vez expuesto el conflicto, se produce una elipsis
narra- tiva. Esto es: se dice que el conde explicó los
detalles de la situación que le inquietaba, pero no se
exponen dichos detalles. Localiza el momento en que se
produce dicha elip- sis. ¿Por qué crees que se realiza?
Al comienzo del tercer pá rrafo, cuando el narrador dice:
E contó a Patronio la manera cómmo podría seer. La elipsis
evita detalles innecesarios, aporta sencillez y permite centrar-
se en el asunto que realmente interesa: el apó logo.
14. Antes de narrar su cuento, Patronio anticipa una
opinión al conde sobre el problema que le ha
planteado. ¿Cuál es?
Que lo sensato es atenerse a las cosas ciertas y razonables,
y no a las esperanzas vanas.
15. Redacta un resumen de lo que le ocurrió a doña
Truhana. Debe ocuparte entre cinco y ocho líneas.
Doñ a Truhana se disponía a vender una olla de miel en el
mercado, e iba pensando en có mo invertiría sus ganancias,
de forma que se multiplicaría sucesivamente hasta hacerla
rica. Pero sus ilusiones terminan al rompérsele la olla.
16. Al terminar el relato, Patronio expone claramente su
con- sejo. ¿Cuál es?
Hay que centrarse en las cosas reales y razonables, y no en
los sueñ os vanos.
17. Delimita el tema central de este cuento y responde:
¿cuál es la conducta sobre la que se pretende
moralizar?
El tema central de este cuento es la importancia de las
cosas tangibles y la fragilidad de las ilusiones. La enseñ anza
advier- te sobre las conductas excesivamente fantasiosas.
18. ¿Crees que la enseñanza de este cuento tiene vigencia
en la actualidad y en nuestra época? Justifica tu
La expresió n describe perfectamente la conducta de doñ a
Los personajes son Patronio y el conde, en toda la colecció n,
Truhana. Se refiere a las personas que se dejan llevar por
y los personajes de cada cuento en particular.
fantasías poco razonables sobre proyectos que aú n ni siquiera
han comenzado a poner en prá ctica, vivié ndolos como si ya Discurso propio de la narració n, con elementos espaciotem-
estuviesen cumplidos. porales mínimamente necesarios, verbos en pasado e intro-
ducció n de diá logos y descripciones cuando son requeridos.
20. En el refranero español existen enseñanzas semejantes a
la que se contiene en este cuento. Cita algún ejemplo y 29. ¿Qué palabras y expresiones aluden al tiempo en el
explica su significado. cuen- to? ¿Y al espacio?
Más vale pájaro en mano que ciento volando. Este refrá n Alusiones temporales mediante locuciones como otra vez o
previene sobre el peligro de forjar demasiadas ilusiones sobre un día. Al espacio se alude en el relato de Patronio al hablar
proyectos o deseos que aú n no tienen visos de materializarse. del mercado y del camino.
30. Justifica por qué podemos adscribir el relato de doña
Personajes Truhana al subgénero narrativo del cuento.
21. Patronio es el ayo del conde Lucanor. Investiga y explica Por su brevedad y por la simplicidad de la trama. No interesan
qué era un ayo en la Edad Media. los detalles ni hay descripciones prolongadas, sino el narrador
El ayo era un sirviente que, en las casas de los nobles, se que se centra en lo que se relaciona directamente con el
dedi- caba a instruir y educar a los niñ os y jó venes. tema central. De los personajes no hay una caracterizació n
22. Señala a qué estamento de la Edad Media pertenece el profun- da, sino plana, porque su funció n es, estrictamente,
conde. Ahora reflexiona e indica qué edad crees que tie- presentar el tema que se quiere abordar.
ne, dado que necesita un ayo. Estilo
Pertenece al estamento de la nobleza. Es un hombre joven, que 31. Relaciona la sencillez del estilo con la intención didáctica
está aprendiendo aú n a desempeñ ar las responsabilidades de don Juan Manuel.
que le depara su posició n social.
La sencillez del estilo contribuye a la eficacia comunicativa,
23. ¿Qué datos se nos dan sobre doña Truhana? ¿Son físicos garantiza la comprensió n.
o psicológicos? ¿Hay alguna descripción de ella o
32. Cita frases o partes del cuento que ejemplifiquen dicha
podemos caracterizarla a través de sus conductas?
sencillez.
Se nos dan pocos rasgos y ninguno es físico. Se nos dice que
Señor conde Lucanor, siempre oí decir que era buen seso
era pobre y muy fantasiosa.
atenerse omne a las cosas çiertas e non a las vanas fuças;
24. Escribe tú ahora una caracterización del personaje de E pensando en esto començó a reir con grand placer que
doña Truhana de acuerdo con la información que el avía de la su buena andança, e, en riendo, dio con la mano
narrador proporciona. en su fruente e entonçe cayó la olla de la miel en tierra, e
Respuesta libre. quebrose; Cuidat siempre todas las cosas tales que sean
aguisadas e non fuzas dubdosas e vanas.
25. Teniendo en cuenta dicha información, ¿cuál es el grupo
social de la Edad Media al que pertenece doña Truhana? 33. Localiza los elementos dialogados que existen en el cuen-
Justifica tu respuesta. to de doña Truhana.
Pueblo llano, puesto que se nos dice que es pobre y su medio El diá logo se introduce en la primera parte, cuando el conde
de vida es vender sus productos en el mercado. expone su problema a Patronio; se da por segunda vez cuan-
do el ayo, tras escucharle, expone su opinió n, justo antes de
Estructura hacer su relato.
26. Recuerda qué es un pareado y explícalo. 34. Busca en el cuento ejemplos de estilo directo e indirecto.
Estrofa de dos versos que riman entre sí. Estilo directo: Fablava el conde Lucanor con Patronio en esta
27. Diferencia esas cinco partes en el cuento de doña Truha- guisa: —Patronio, un omne me dixo…; Respondió al conde
na y enuncia las ideas que se recogen en cada una. en esta manera: —Señor conde Lucanor, siempre oí que…
Primera parte, pá rrafo segundo. El conde explica a Patronio Estilo indirecto: E contó a Patronio la manera cómmo podría
la propuesta que le han hecho y que, aparentemente, le seer.
traerá tantos beneficios. 35. Observa los tiempos y modos verbales utilizados por
Segunda parte: narració n del cuento de doñ a Truhana por Patronio en el cuento de doña Truhana. ¿Cuáles emplea
parte de Patronio. para contar el relato? ¿Y para aconsejar al conde?
Tercera parte: Patronio explica la aplicació n prá ctica de su En el relato utiliza el pretérito perfecto simple y el imperfecto
relato (E vos, señor conde, si queredes que lo que vos de indicativo. Cuando aconseja al conde utiliza sobre todo el
dixieren e lo que vos cuidardes sea todo cosa çierta). imperativo.
Cuarta parte: el conde acepta y sigue el consejo (Al conde 36. Ya sabemos que todos los cuentos del libro terminan con
plogo de lo que Patronio le dixo). una moraleja en forma de pareado. ¿Por qué crees que
Quinta parte: pareado final con la moraleja del relato (A las terminan así? ¿Tiene alguna utilidad práctica?
cosas çciertas vos comendat / e las fuizas vanas vos dexat). La versificació n, el ritmo y la rima contribuyen a la memo-
Género rizació n.
28. Localiza en el cuento de doña Truhana todos los elemen- El texto como acto comunicativo
tos característicos de la narración y explícalos. 37. Teniendo en cuenta lo anterior, ¿quiénes son los recep-
El marco tiene un narrador omnisciente que organiza la reco- tores en cada caso?
pilació n de cuentos. En cada uno de ellos, a su vez, hay otro El receptor del texto global es el lector de cualquier época. El
narrador: Patronio. del relato de Patronio es el conde Lucanor.

Lengua castellana y Literatura. 3.º ESO. Solucionario 119


38. ¿Recuerdas la diferencia entre autor y narrador? Recuerde el alma dormida / avive el seso y despierte (copla I);
Explícala a partir de este cuento. Indica si aparece el Ved de cuán poco valor / son las cosas tras que andamos / y
autor en el texto y explica dónde. corremos (copla VIII); Decidme: la hermosura, / la gentil fres-
El autor de la obra es el infante don Juan Manuel. El narra- cura y tez / de la cara, / el color y la blancura, / cuando
dor del texto global es un narrador omnisciente y externo a viene la vejez, / ¿cuál se para? (copla IX); Los estados y
la narració n. El narrador de cada cuento de la colecció n es riqueza / que nos dejan a deshora / ¿quién lo duda? / no les
Patronio. El autor aparece mencionado al final, tras el relato pidamos firmeza (copla XI).
de Patronio. Es el narrador omnisciente quien lo menciona
cuando dice E porque son Johan se pagó deste exiemplo, Contenido
fízo- lo poner en este libro e fizo estos viessos. 5. En la primera copla el poeta presenta el alma humana
39. A su vez, Patronio y el conde son interlocutores de un como un ser dormido que debe despertar. ¿Para qué?
acto comunicativo. Explica sus elementos a partir del ¿De qué debe tomar conciencia?
cuento de doña Truhana. Para que se haga consciente de que su estancia en el mundo
es provisional, un mero trá nsito que pasa rá pido, trayendo la
Patronio y el conde establecen un acto comunicativo que
muerte sin remedio.
con- siste en el problema que expone el conde y el consejo que
le da Patronio, utilizando el cuento como ejemplo. En el 6. ¿Qué quiere decir el poeta en la segunda copla cuando
relato de dicho cuento, Patronio es el emisor, el cuento es el afirma que «si juzgamos sabiamente, daremos lo no veni-
mensaje y el conde es el receptor. do por pasado»?
Se refiere a la fugacidad de las cosas. Todo pasa
Temas para la reflexión
rá pidamente. Igual que el presente se marcha en un instante,
40. Si en la actualidad se escribiese este libro, ¿qué clase el futuro será pasado antes de que nos demos cuenta.
de autor tendría? ¿Quiénes crees que deben transmitir 7. En la Copla III se presenta el río como metáfora de la
enseñanzas a otros en la sociedad actual? vida. Explica cómo esta metáfora le sirve al poeta para
Respuesta libre. hablar del poder igualatorio de la muerte.
41. ¿Sobre qué costumbres crees que podría asentarse en la Igual que todos los ríos van al mar, sea cual sea su caudal,
actualidad una enseñanza para ser un buen ciudadano? todas las trayectorias humanas acaban en la muerte, sea cual
¿Morales, religiosas, cívicas? Redacta un escrito de veinte sea la importancia o condició n de la persona.
líneas sobre el tema. 8. ¿A quién invoca el yo poético en la cuarta copla?
En la actualidad, el didactismo suele orientarse al fomento A Cristo.
de conductas cívicas. 9. En la Copla V, nuevamente se habla de la vida mediante
42. Busca otros ejemplos en la literatura o en el cine que un símbolo. ¿Cuál?
bus- quen enseñar entreteniendo. El del camino.
Respuesta libre. 10. La idea que se recoge en dicha copla es típicamente
medieval. Explícala.
2 Jorge Manrique. Coplas a la muerte La vida solo es un trá nsito hacia la muerte, que nos dará el
merecimiento por nuestros actos. Ese trá nsito debe hacerse
de su padre. Coplas I-XIII con acierto y bondad.
11. En la Copla VI el poeta insiste en la idea anterior. Cita los
Localización
versos que lo demuestran.
1. La poesía culta del siglo xv supone una novedad en el
Este mundo bueno fue / si bien usáramos de él / como debe-
panorama poético porque hasta el momento solo encon- mos / porque, según nuestra fe / es para ganar aquel / que
tramos casos de poesía narrativa y didáctica. Explica las atendemos.
diferencias entre la poesía lírica y la poesía narrativa. 12. Explica la reflexión sobre la belleza espiritual y material
La poesía lírica aborda temas relacionados con los sentimien- de la Copla VII.
tos y su expresió n. La poesía narrativa se conforma de
La belleza espiritual depende de nosotros. La material o físi-
relatos versificados.
ca, no. Si dependiese, nos dedicaríamos solo a ella en vez de
2. En las Coplas de Manrique conviven la visión medieval atender a la del alma.
del mundo y la renacentista. El poeta nos habla del escaso 13. Señala el valor que tienen las cosas mundanas. Resume
valor de las cosas terrenales, y también de la el contenido de la Copla VIII.
preocupación huma- na por dejar memoria nuestra en Las cosas mundanas que nos preocupan tienen escaso valor,
este mundo. ¿Cuál de estas dos ideas es medieval y cuál son frá giles y poco duraderas, se pierden incluso antes de
anticipa el Renacimiento? morir.
El escaso valor de las cosas terrenales es una idea medie- 14. En la Copla IX, ¿cómo se habla del paso de la juventud
val. El deseo de dejar memoria en este mundo tras nuestra a la vejez?
muerte forma parte de una nueva visió n que viene con el
Habla de la transformació n que sufren la belleza y la fuerza
Renacimiento.
de la juventud, que se convierten en gravedad al llegar la
3. ¿Cuál de esos temas se trata en las trece primeras coplas vejez.
de la composición? 15. En la Copla X se alude a la nobleza, cuya elevada
La vanidad de las cosas terrenales. condi- ción no siempre se corresponde con el nivel que
Intención alcanza en esta vida. ¿Cuáles son las dos razones por las
4. Observa el tono exhortativo de las trece coplas que has
que pue- de ocurrir esto, según Manrique?
Unos porque no tienen la valía que corresponde a su elevada
leído: enumera las llamadas de atención y
posició n; otros porque no tienen medios para vivir de acuerdo
recomendacio- nes que hace Manrique.
con esta y deben desempeñ ar oficios impropios de la
nobleza.
16. En la Copla XI aparece la Fortuna como símbolo de lo la fama). Esta idea puede hacer má s llevadero el trance de
precario y lo voluble. Explica esta idea relacionándola la muerte, en el que deberá renunciar al mundo vano y pere-
con la mentalidad medieval. cedero (vida terrenal). La vida de la fama (vida de honor la
Las cosas mundanas son frá giles, pueden ser ganadas o per- llama en la copla XXXV) es mucho má s larga y só lida que la
didas azarosamente. No puede confiarse en su firmeza o su terrenal, pero tambié n acabará extinguié ndose. La celestial
persistencia, hay que asumir la volubilidad de lo material. es la ú nica eterna.
17. La Copla XII sigue hablando de los estados y la riqueza
Personajes
que, según dice, se van a la tumba con su dueño. ¿En qué Copla XXVI Copla XXXIX
momento se hace esta afirmación? Amigo de sus amigos, No dejó grandes thesoros
En los tres primeros versos. ¡qué señ or para criados ni alcanzó grandes riquezas
18. También en la Copla XII se presenta una contraposición y parientes! ni baxillas3,
¡Qué enemigo de enemigos! mas hizo guerra a los moros
entre lo efímero y lo eterno. Localízala y explica el
Qué maestro de anando sus fortalezas
sentido cristiano que tienen estos versos. esforçados1 y valientes! y sus villas;
El placer terrenal es breve; los tormentos de la condenació n, ¡Qué seso para discretos! y en las lides que venció ,
eternos. E los deleites d´acá / son, en que nos deleitamos, / ¡Qué gracia para donosos! muchos moros y cavallos
temporales, / e los tormentos d´allá, / que por ellos esperamos, ¡Qué razó n! se perdieron,
/ eternales. ¡Qué benigno a los subjetos2! y en este oficio ganó
19. ¿Qué conducta humana se describe en la Copla XIII? Y a los bravos y dañ osos, las rentas y los vasallos
¿Tie- ne sentido, según el poeta? ¡un leó n! que le dieron.
La tendencia a caer en la trampa del placer mundano, sin pen-
sar en las consecuencias hasta que es demasiado tarde y ya 1
esforçados: aguerridos. 2
subjetos: vencidos. 3
baxillas: ajuar.
no es posible rectificar.
20. Investiga ahora sobre el contenido de las Coplas XIV-XXIV 22. A partir del contenido de esas estrofas, describe la perso-
y enumera los personajes y hechos a los que Manrique nalidad de este caballero y la importancia de sus hechos.
alude en ellas.
Hombre generoso con sus amigos, buen señ or para sus sir-
Copla XXXIII Y pues de vida y salud vientes, prudente y sensato, magná nimo, valiente y aguerrido.
Después que puso la vida hezistes tan poca cuenta Sus riquezas no fueron muchas, pero sus hazañ as contra los
tantas veces por su ley por la fama, moros fueron notables y le permitieron forjar su patrimonio.
al tablero, esfué rcese la virtud
despué s de tan bien servida para sofrir esta afruenta Estructura
la corona de su rey que os llama. 23. ¿Qué medida tienen los versos utilizados? ¿Cuál es la
verdadero, organización de la rima?
Copla XXXV
después de tanta hazañ a
—No se os haga tan amarga Ocho y cuatro sílabas. En cada copla, la rima duplica el
a que no puede bastar
la batalla temerosa esquema 8a8b4c8a8b4c. La composició n se llama copla de
cuenta cierta
que esperá is, pie quebrado.
en la su villa de Ocañ a,
pues otra vida má s larga 24. Localiza en el conjunto de las trece coplas iniciales estos
vino la muerte a llamar
de fama tan gloriosa contenidos: una exhortación, una invocación a Cristo y
a su puerta,
acá dexá is;
Copla XXXIV una reflexión sobre la vanidad de las cosas terrenales.
aunque esta vida de honor
diciendo: —Buen caballero, tampoco no es eternal Exhortació n, coplas I-III. Invocació n a Cristo, coplas IV-VI.
dexad el mundo engañ oso ni verdadera, Reflexió n sobre la vanidad de las cosas terrenales, coplas
y su halago; mas con todo es muy VII-XIII.
vuestro coraçon de azero mejor que la otra temporal
muestre su esfuerço Género
pereçedera.
famoso en este trago. 25. Explica los elementos del género lírico a partir de las
Copla XIV: los antiguos reyes, papas, emperadores. Copla XV: coplas que has leído.
los hé roes de Troya y los romanos. Copla XVI: el rey Juan II, El uso del verso, la rima y el ritmo. Se puede comentar el
los infantes de Aragó n (hijos del rey Fernando). Copla XVII: las esquema métrico, el arte menor y la rima consonante. Asimis-
damas lujosamente vestidas, los amantes que las cortejaban mo, el empleo del lenguaje figurado mediante recursos como
con mú sica. Copla XVIII: Enrique IV, sucesor del rey don la metá fora (por ejemplo, los ríos y el mar para referirse a la
Juan. Copla XIX: las riquezas de los palacios. Copla XX: don vida y a la muerte respectivamente) o el símbolo (el camino,
Alfonso, hermano de Enrique IV que, aunque fue proclamado que representa la vida). Se puede hablar tambié n de
rey siendo aú n niñ o, no llegó a reinar nunca y murió a los recursos expresivos como la interrogació n retó rica, en las
catorce añ os. Copla XXI: el condestable Á lvaro de Luna, que coplas IX y XI, o el paralelismo sintá ctico, que favorece el
asumió el poder durante el reinado de Juan II y murió ritmo y que se utiliza con mucha frecuencia (copla I, copla
decapitado. Copla XXII: el maestre de Santiago, don Juan III, copla IV, copla XII, etcé tera).
Pacheco, marqué s de Villena y el maestre de Calatrava, don
Pedro Giró n. Copla XXIII: nobles que realizaron notables 26. Podríamos decir que este poema es también una expo-
hazañ as en la guerra. Copla XXIV: los grandes ejé rcitos y las sición de ideas. Argumenta esta afirmación con citas del
fortalezas inexpugnables. texto.
21. Explica cómo se alude a las tres vidas en las Coplas Ejemplos de argumentació n que pueden citarse: copla V, donde
XXXIII, XXXIV y XXXV. presenta la idea de la vida terrenal como un simple trá nsito;
coplas VII-XII, donde habla del escaso valor de las cosas mate-
Don Rodrigo llevó a cabo honrosas y elevadas hazañ as. De riales y de la vida terrenal.
ello quedará memoria para las futuras generaciones (vida de
Estilo
c) Divinización de Melibea por parte de Calisto.
27. Localiza ejemplos de exhortaciones en las coplas que has
d) Planteamiento del tópico del mundo como valle de
leído y relaciónalos con el tono didáctico de la obra.
lágrimas por parte de Pleberio, padre de Melibea, al
¿Qué tiempos y modos verbales predominan?
final de la obra.
Recuerde el alma dormida / avive el seso e despierte…
(copla I); Ved de cuán poco valor / son las cosas tras que a) Medieval.
andamos… (copla VIII); Dezidme: la hermosura, la gentil b) Renacentista.
frescura y tez… (copla IX). Verbos en imperativo. c) Renacentista.
28. Localiza en las Coplas I-XIII ejemplos de metáfora, com- d) Medieval.
paración, paralelismo sintáctico, interrogación retórica.
¿Cómo contribuyen a la intención del poeta? Intención
Metá fora: entre otros ejemplos, se pueden citar los de la Síguese la comedia o tragicomedia de Calisto y Melibea, com-
copla III, cuando se identifica a las vidas con los ríos, y a puesta en reprehensió n de los locos enamorados que, vencidos
la muerte con el mar; copla XIII, cuando se identifican los en su desordenado apetito, a sus amigas llaman y dicen ser
placeres de la vida terrenal con soldados de avanzadilla su Dios. Asimismo hecha en aviso de los engañ os de las
(corredores). Comparació n: en la copla XII se dice que se va alcahue- tas y malos y lisonjeros sirvientes.
la vida apriessa como sueño. Paralelismo sintá ctico: véanse 2. Según esta declaración de Rojas, ¿qué clase de amor se
las coplas I, III, IV o XII. Interrogació n retó rica: hay ejem- condenaría en la obra? ¿Tiene relación con el amor
plos en las coplas IX y XI. Las metá foras y comparaciones cortés?
incrementan la expresividad y, a la vez, afianzan la idea que
El amor loco, el pasional, el carnal. No tiene que ver con el
pretende transmitir al lector, pues presentan una simbología
amor cortés.
propia de la cultura cristiana (por ejemplo, el planteamiento
de la vida como un camino) que resulta perfectamente enten- 3. ¿Por qué se advierte contra la ayuda de las alcahuetas y
dible por el receptor, que comparte con Manrique esos plantea- los sirvientes?
mientos culturales. Los paralelismos sintá cticos contribuyen Porque solo pretenden engañ ar con el fin de obtener beneficio
al ritmo y, a la vez, al didactismo. La finalidad didá ctica tam- propio.
bién queda muy reforzada con las interrogaciones retó ricas.
Por Dios, dejemos enojo y al tiempo el consejo. Hayamos
El texto como acto comunicativo mucho placer. Mientras hoy tuvié remos de comer, no pen-
29. ¿Por qué crees que un poeta que desea hacer una semos en mañ ana. Tambié n se muere el que mucho allega
como el que pobremente vive, y el doctor como el pastor y el
alabanza a la memoria de su padre se detiene tanto en
papa como el sacristá n y el señ or como el siervo y el de alto
esas consi- deraciones previas y generales? linaje como el bajo, y tú con oficio como yo sin ninguno. No
Pretende transmitirnos su visió n de la vida y de la muerte, su habremos de vivir para siempre. Gocemos y holguemos, que
idea sobre el proceder correcto de un buen cristiano, tanto en la vejez pocos la ven, y de los que la ven ninguno murió de
el desarrollo de su vida como en el momento de afrontar la hambre.
muerte. La figura de su padre se convierte en un modelo de
conducta, en ese ideario que ha expuesto. 4. Esta declaración de Elicia alude al poder igualatorio de
la muerte. También Manrique había tratado este asunto
Temas para la reflexión en el siglo xv, aunque con intención distinta. Compara
30. Dos ideas destacan en las trece coplas iniciales: la de que ambos enfoques.
cualquier tiempo pasado fue mejor, y la de que esta vida En ambos casos se afirma que la muerte aguarda a todos, sea
solo es un tránsito para la eterna, que es la que importa. cual sea la condició n o posició n, es lo ú nico que iguala a
Analiza y valora estas dos ideas desde la perspectiva del todos los humanos. Sin embargo, mientras que Manrique
siglo xxı. hablaba de la vida como trá nsito perecedero hacia la eternidad,
Posiblemente la idea de que cualquier tiempo pasado fue mejor que era lo que verdaderamente importaba, aquí se plantea
sea propia de la naturaleza humana, pues en nuestro tiempo que hay que intentar vivir gozando, precisamente por el
sigue ocurriendo que a veces se idealiza el pasado. No ocurre cará cter efímero de la existencia. Es decir, si para Manrique
lo mismo con la idea de que la vida es un trá nsito hacia la eter- ese cará cter pere- cedero era el argumento que restaba valor
nidad, esta idea no está presente ya en el siglo XXI, sino todo lo a la vida, ahora es precisamente lo que se lo confiere.
contrario. En nuestro tiempo y en nuestra sociedad existe el
Contenido
principio de que hay que intentar alcanzar la felicidad porque
esta vida es lo ú nico que tenemos. 5. Resume en tres o cuatro líneas las palabras de Calisto a
Melibea al comienzo de la obra.

3 Fernando de Rojas. La Celestina.


Calisto alaba a Melibea por su perfecta hermosura, a la que
considera el mayor regalo divino. Se mezclan sentimientos
Acto primero: enamoramiento de Calisto carnales con espirituales y la ausencia de Melibea le provo-
cará un gran dolor.
y conversación con Sempronio
6. ¿Cómo reacciona Melibea ante esa declaración? ¿Por
Localización qué crees que reacciona así?
1. Indica si los siguientes elementos de la obra son medie- Con enojo. Considera que Calisto es un atrevido y que pone
vales o renacentistas: en peligro su virtud.

a) Intención moralizante del autor al advertir contra los 7. Al ver el estado en que se encuentra su señor, ¿qué
peligros del amor y el servicio de las alcahuetas. dudas tiene Sempronio?
No sabe si dejarle solo con su dolor o acompañ arle. Si le deja
b) Exaltación de los placeres, del amor, de la juventud y
solo, teme que Calisto se mate. Si le acompañ a, teme que le
de la belleza.
mate a é l.
8. Calisto le pide a Sempronio que cante con un laúd y hace
18. Explica cómo se contrapone el hecho de que Calisto acu-
una intervención que comienza diciendo: «¿Cómo
da a Celestina para conseguir a Melibea con el concepto
templa- rá el destemplado?». ¿Cómo describe el joven su
del amor en la tradición cortés.
estado de ánimo en dicha intervención? Comenta su
En la tradició n cortés, la amada se presentaba como inalcan-
contenido.
zable y así era aceptado por el amante. No podía ser de otro
Está desasosegado. No es capaz de conciliar su voluntad con modo, ese era el có digo de conducta. Ademá s, en la tradició n
su razó n. Tiene diversos sentimientos, todos atormentados. cortés nos encontramos ante un sentimiento idealizado. El de
9. Explica el sentido que tiene en este contexto la aquí es carnal y pasional.
declaración de Calisto: «Melibeo soy y a Melibea adoro y Oh, duro corazó n de padre, ¿có mo no te quiebras de dolor,
en Melibea creo y a Melibea amo». que ya quedas sin tu amada heredera? ¿Para quién edifiqué
Previamente Sempronio le ha dicho: ¿Tú no eres cristiano? torres; para quién adquirí honras; para quién planté á rboles;
Calisto proclama su amor por Melibea como su ú nica fe, su para quién fabriqué navíos? […]
ú nica devoció n. El amor es su religió n. Oh, fortuna variable, ministra y mayordoma de los temporales
10. Cuando Calisto confiesa a Sempronio su estado de bienes, ¿por qué no ejecutaste tu cruel ira, tus mudables ondas,
ánimo, este le hace unas advertencias sobre lo poco en aquello que a ti es sujeto? ¿Por qué no destruiste mi patri-
cristianas que resultan algunas de sus afirmaciones. monio; por qué no quemaste mi morada; por qué no asolaste
mis grandes heredamientos? Dejá rasme aquella florida plan-
¿Qué es exac- tamente lo que le advierte?
ta, en quien tú poder no tenías; diérasme, fortuna fluctuosa,
Que lo que dice es contrario a los fundamentos cristianos, que triste la mocedad con vejez alegre; no pervertieras la orden.
es una especie de herejía.
[…]
11. El asunto de la blasfemia y la herejía continúa ¡Oh, amor, amor, que no pensé que tenías fuerza ni poder de
presente en la conversación. Explícalo. matar a tus sujetos! […] ¿Quién te dio tanto poder? ¿Quién te
Calisto dice que Melibea es su dios, y Sempronio le responde puso nombre que no te conviene? Si amor fueses, amarías a
que eso es una blasfemia y que su pecado es peor que el de tus sirvientes. Si los amases, no les darías pena. Si alegres
Sodoma. Esto ú ltimo, sin embargo, hace reír a Calisto. Sem- viviesen, no se matarían, como agora mi amada hija. ¿En qué
pronio, en su discurso, se mueve entre la advertencia sobre pararon tus sirvientes y tus ministros? La falsa alcahueta
los excesos emocionales de su señ or y la broma o la ironía. Celestina murió a manos de los más fieles compañ eros que
12. Sempronio previene a su señor contra las mujeres en una ella para tu servicio empozoñ ado jamá s halló . Ellos murieron
actitud claramente misógina. Localiza los pasajes en los degollados; Calisto, despeñ ado. Mi triste hija quiso tomar la
que pueda percibirse esta actitud. misma muerte por seguirle. Esto todo causas.
¿Quién te contaría sus mentiras, sus tráfagos, sus cambios, 19. ¿Cuáles son los sentimientos que alberga Pleberio como
su liviandad, sus lagrimillas, sus alteraciones, sus osadías? padre?
[…] ¿Sus disimulaciones, su lengua, su engaño, su olvido, Cualquier desgracia, por grande que hubiese sido, habría sido
su desamor, su ingratitud, su inconstancia, su testimoniar, menor que la que vive ahora. Se ha invertido el orden
su negar, su revolver, su presunción, su vanagloria, su aba- natural, que manda que los padres mueran antes que los hijos.
timiento, su locura, su desdén, su soberbia…?; No tienen Ademá s, su vida ahora no tiene sentido: su hija era la
modo, no razón, no intención […] ¡Oh, qué plaga, oh, qué heredera de todo lo que é l había logrado a lo largo de su
enojo, oh, qué hastío es conferir con ellas más de aquel breve existencia.
tiempo que aparejadas son a deleite! 20. ¿Qué reproches le hace a la fortuna? ¿Y al amor?
13. ¿Tienen efecto en Calisto las palabras de Sempronio? La fortuna ha trastocado el orden debido, pues él tendría que
¿Cómo reacciona a ellas? haber muerto antes que su hija. Al amor le reprocha su tiranía
Calisto persiste en su obsesió n por Melibea, y se considera el con quienes lo albergan, pues los destruye.
más desgraciado de los hombres porque ella le rechaza. 21.¿Qué sentido crees que tiene el hecho de que aluda
14. Calisto hace una descripción de Melibea. ¿Con qué acti- expre- samente al triste final que han tenido todos los
tud la escucha Sempronio? personajes?
Con absoluto desacuerdo ante todo lo que Calisto expo- Un sentido moralizante, como si mostrase las consecuencias
ne. Tambié n experimenta desprecio, aunque esto lo oculta. terribles que tienen los errores que han cometido Calisto y
E incluso, en algunos momentos, podemos ver que se burla Melibea.
de su señ or.
Personajes
15. Finalmente, Sempronio ofrece a Calisto su ayuda. ¿Lo
hace desinteresadamente? Justifica tu respuesta. 22. Melibea aparece al principio tratada como una dama del
amor cortés. Justifica esta afirmación de acuerdo con la
No. Piensa en el beneficio que podría obtener si Calisto con-
sigue sus fines. intervención de ella que has leído.
Se indigna al ver que Calisto se atreve a dirigirse a ella, como
16. Explica cómo se refleja en Calisto esa vivencia del amor
si cuestionase su virtud y el hecho de que es inalcanzable
como enfermedad. para é l.
Está desasosegado, triste y de mal humor. Habla tambié n de
un fuego interior que le consume, como una fiebre intensa.
23. Calisto, conforme a lo elevado de su condición social, se
expresa en términos caballerescos. Sin embargo, en su
17.Al comienzo de la obra, cuando Calisto expresa su admi- fuero interno es egoísta y está tan obsesionado con satis-
ración a Melibea, alude a la mezcla de sentimientos espi- facer su deseo que impone sus aspiraciones a cualquier
rituales y carnales que experimenta. Localiza esa alusión. aspecto moral o ético. Argumenta esta caracterización de
Ellos [los santos] se glorifican sin temor de caer de tal bien- Calisto atendiendo a lo que has leído y a lo que conoces
aventuranza, y yo, mixto, me alegro con recelo del esquivo sobre el resto de la trama.
tormento que tu ausencia me ha de causar.
Su condició n de caballero no le impide mostrar su deseo a
Melibea al comienzo de la obra, provocando las iras de
esta.
Tampoco duda en pedir ayuda a una mujer como Celestina, res y conclusiones. En el segundo, es Calisto el destinatario de
hechicera y alcahueta. En su contexto social, un caballero no sus afirmaciones, pero estamos escuchando al verdadero
debería poner en peligro su honra ni la de su amada, pero él
lo hace sin sentir culpabilidad siquiera. En cuanto al honor,
en este no piensa tampoco. Ademá s, cuando se entera de la
muerte de Pá rmeno y Sempronio el dolor no le dura dema-
siado, pues enseguida se centra nuevamente en su obsesió n.
24. Hay una caracterización paralela de Calisto, que realiza
Sempronio en la conversación que ambos mantienen.
Nos referimos a la intervención del criado que comienza
así:
«Lo primero que eres hombre y de claro ingenio». ¿Crees
que Sempronio está siendo sincero con Calisto en este
momento? ¿Qué intención persigue al exponer todas las
virtudes que le alaba?
Pretende adularle. Sempronio es uno de esos lisonjeros cria-
dos contra los que previene el autor al comienzo de la obra.
25. Sempronio es un personaje sin escrúpulos y lleno de
codi- cia. ¿En qué momentos del fragmento que has
leído crees que se perciben estos rasgos?
En los apartes de Sempronio podemos ver el desprecio que
siente hacia su amo, y el modo en que lo disimula con sus
hipó critas lisonjas. Tambié n en esos partes el criado alude
a los beneficios que espera obtener si apoya a su amo en
sus pretensiones.
26. En la conversación entre Calisto y Sempronio, este pro-
pone a su señor que pida ayuda a Celestina y le habla de
ella. ¿Cómo describe a la alcahueta?
Astuta y hechicera, sagaz, componedora de virgos y capaz de
provocar la lujuria en quien se proponga.
27. Localiza pasajes o citas en los que pueda comentarse la
mencionada dualidad entre lo que los personajes dicen
ser y lo que son en realidad.
En Calisto, la dualidad se manifiesta si comparamos su caba-
lleresco modo de expresarse con Melibea con las pretensio-
nes que manifiesta. En cuanto a Sempronio, pueden citarse
la mayoría de sus apartes, pues en ellos siempre se
contienen ideas y propó sitos opuestos a los que expone ante
Calisto.
Estructura
28. Divide el pasaje que has leído en partes de acuerdo con
su contenido.
Primera parte: intervenció n de Calisto ante Melibea y rechazo
de esta.
Segunda parte: llegada de Calisto a su casa, malhumorado, y
monó logo de Sempronio al ver el estado en que viene su
amo. Tercera parte: conversació n con Sempronio, en la
que a su
vez tenemos que diferenciar apartados; en primer lugar, la
exposició n de Calisto sobre su estado y sus sentimientos; en
segundo lugar, las advertencias de Sempronio sobre las muje-
res; en tercer lugar, la descripció n que hace Calisto de Melibea.
Por ú ltimo, la propuesta de Sempronio de acudir a Celestina.
29. Teniendo presente que el texto es dialogado, diferencia
los pasajes puramente dialógicos de aquellos que pue-
dan considerarse monólogos. ¿Qué función tienen estos
últimos?
La estructura dialó gica se rompe con los monó logos siguien-
tes de Sempronio: el que pronuncia cuando ve llegar a Calis-
to de mal humor (No creo, según pienso, ir conmigo el que
contigo queda…) y la exposició n que hace sobre las muje-
res, atacá ndolas (¿Escociote? Lee los historiales, estudia los
filósofos…). En el primer caso, está a solas con sus propios
pensamientos y el fragmento nos permite conocer sus temo-
Sempronio, no al lisonjero: en esta intervenció n se mucho má s mesurado. Cuando se dirigía a Sempronio, logra-
presentan ideas misó ginas bastante extendidas en la Edad ba la expresividad mediante los coloquialismos, los juegos
Media. de palabras (aojando pájaras) y la gracia de los dichos
Género
30. Localiza y comenta las dificultades para poner en
escena el fragmento que has leído.
Cambios en el espacio (balcó n de Melibea, diversas dependen-
cias de la casa de Calisto). No hay acotaciones.
31. ¿Qué rasgos de la comedia humanística observas en
ese fragmento?
Aparecen registros distintos: Calisto emplea un estilo
galante y caballeresco para dirigirse a Melibea, pero cambia
el tono con Sempronio; este, por su parte, tiene ciertos usos
popula- res. Estilo proverbial: puede apreciarse en el tono
sentencioso de Sempronio, y en el recurso a los refranes y
dichos.
¿Qué, hijo? ¡Una docena de agujetas1 y un torce2 para el
bonete y un arco para andarte de casa en casa tirando a
pá jaros y aojando3 pá jaras a las ventanas! Mochachas digo,
bobo, de las que no saben volar, que bien me entiendes. Que
no hay mejor alcahuete para ellas que un arco, que se
puede entrar cada uno hecho mostrenco4, como dicen: en
achaque de trama5, etc.
¡Má s, ay, Sempronio, de quien tiene que mantener honra y
se va haciendo vieja como yo!

1
agujetas: correas. 2 torce: collar. 3 aojando: echando el
ojo. 4 hecho mostrenco: sin pedir permiso. 5 en achaque de tra-
ma: es la primera parte del dicho «En achaque de trama, ¿viene
acá nuestra ama?». Se empleaba para aludir a las viejas que
entraban sin permiso en las casas con intención de robar y, si se
encontraban con alguien, decían que buscaban al ama.

32. Localiza y explica los elementos propios del lenguaje


coloquial y las expresiones populares que aparecen en
esta intervención de Celestina.
Son usos propios de la lengua oral y coloquial las
exclamacio- nes (¡Una docena de agujetas…!) y los vocativos
(¿Qué, hijo?; Mochachas digo, bobo; ¡Más, ay,
Sempronio…!), así como el léxico (por ejemplo pájaras con
sentido de mozas). Hay expre- siones populares como hecho
mostrenco (sin pedir permiso) o en achaque de trama, que
alude a un dicho: En achaque de trama, ¿viene acá nuestra
ama?
Amiga y señ ora mía, no te maravilles, porque estos fines
con efecto me dan osadía a sufrir los á speros y
escrupulosos des- víos de las encerradas doncellas como tú .
Verdad es que ante que me determinase, así por el camino,
como en tu casa, estuve en grandes dudas si te descubriría
mi petició n. Visto el gran poder de tu padre, temía; mirando
la gentileza de Calisto, osa- ba; vista tu discreció n, me
recelaba; mirando tu virtud y huma- nidad, me esforzaba. En
lo uno hallaba el miedo y en lo otro la seguridad. Y pues así,
señ ora, has querido descubrir la gran merced que nos has
hecho, declara tu voluntad […]. Yo daré forma como tu
deseo y el de Calisto sean en breve cumplidos.
33. Comenta el uso de los adjetivos en esa intervención de
Celestina. ¿Son explicativos o especificativos? ¿Qué
valor expresivo aportan al texto?
Son adjetivos explicativos, su finalidad es enfá tica, retó rica.
Aportan al discurso la emotividad con la que Celestina
pretende embaucar a Melibea. También contribuyen a elevar
el registro.
34. Compara esta intervención con la anterior, en la que se
dirigía a Sempronio.
Celestina prescinde aquí de las exclamaciones, su tono es
populares. Al dirigirse a Melibea la expresividad la persigue
A comienzos del siglo XVI nos encontramos en un momento
mediante la adjetivació n con epítetos, o ciertos recursos retó -
de esplendor: es la Españ a imperial. Sin embargo, el
ricos como el paralelismo sintá ctico (visto el gran poder de tu Lazarillo muestra los contrastes que existen en una
padre, temía; mirando la gentileza de Calisto, osaba; vista sociedad marcada por el orgullo imperialista pero, a la vez, por
tu discreción, me recelaba…). Pero, en todo caso, obsérvese las desigualdades sociales, el hambre y la miseria en muchos
có mo en los dos fragmentos intenta poner al interlocutor de su casos. El idealismo de la literatura renacentista cede paso a
parte, con cierta actitud de victimismo. Por ejemplo, al criado una visió n más crítica sobre esos contrastes.
le dice: ¡Mas, ay, Sempronio de quien tiene que mantener la
honra y se va haciendo vieja como yo! A Melibea: Estos fines 3. ¿Cómo es la sociedad que se presenta en el relato, de
con efecto me dan osadía a sufrir los ásperos y escrupulosos acuerdo con el Tratado I?
desvíos de las encerradas doncellas. Una sociedad en la que existen el hambre y la miseria, pese
al triunfalismo existente en tiempos de Carlos I.
El texto como acto comunicativo
4. Explica el sentido que tienen estas palabras de Lázaro
CELESTINA.— ¡Oh mi señ or Calisto! ¿Y aquí está s? […] ¿Con qué
pagará s a la vieja, que hoy ha puesto su vida al tablero en el prólogo:
por tu servicio? ¿Cuá l mujer se vido en tan estrecha Y tambié n porque consideren los que heredaron nobles
afrenta como yo, que en tornallo a pensar se me menguan y esta- dos cuá n poco se les debe, pues Fortuna fue con ellos
vacían todas las venas de mi cuerpo, de sangre? Mi vida parcial, y cuá nto má s hicieron los que, sié ndoles contraria,
diera por menor precio que agora daría este manto raído con fuer- za y mañ a remando salieron a buen puerto.
y viejo. Lá zaro diferencia entre aquellos que heredaron sus bienes y
PÁ RMENO.— (Tú dirá s lo tuyo: «entre col y col, lechuga». los que han tenido que salir adelante por sus propios medios.
Subido has un escaló n; má s adelante te espero a la saya. 5. Busca información sobre la presencia de los ciegos en el
Todo para ti y no nada de que puedas dar parte. Pelechar folclore español y redacta un escrito de veinte líneas con
quiere la vieja. Tú me sacará s a mí verdadero, y a mi amo los datos que encuentres.
loco. No le pierdas palabra, Sempronio, y verá s como no
En la literatura oral de la Edad Media era habitual la pareja
quiere pedir dinero, porque es divisible.
de ciego y mozo, entre los que se establecía una rivalidad de
SEMPRONIO.— Calla, hombre desesperado, que te matará ingenio y artimañ as para afrontar el hambre. Exceptuando el
Calisto si te oye). episodio de las uvas, todos los que se narran entre Lá zaro y
35. Lee atentamente el fragmento anterior (perteneciente al el ciego tienen antecedentes en el folclore.
acto VI) y contesta: ¿quién es el interlocutor de Celesti- Intención
na?; ¿y los de Pármeno y Sempronio?
6. Investiga sobre la obra Cuarto libro del esforzado caballe-
El interlocutor de Celestina es Calisto. Pá rmeno y Sempronio
ro Reinaldos de Montalbán. ¿Qué elementos hay en ella
hablan entre ellos, sin ser oídos por los otros dos.
que podríamos asociar al Lazarillo?
36. ¿Cuál es la intención de la alcahueta? ¿Cómo la esconde? Se trata de una obra de 1542 con ciertas influencias de Apu-
¿Quién la conoce? leyo. Es un relato autobiográ fico y hay un personaje (Cíngar)
Celestina pretende que Calisto se sienta deudor con ella, que proviene de la clase baja y es criado de un ciego.
hacié ndole creer que ha puesto su vida en peligro por ayu- 7. Busca también información sobre El asno de oro de Apu-
darle. Pá rmeno se da cuenta de que la vieja quiere mejorar
leyo. ¿Encuentras alguna semejanza entre este relato y el
de estado y obtener un beneficio que no sea posible
compartir, para quedá rselo ella sola.
de Lázaro de Tormes?
Es un relato de aventuras del siglo II a. de C. que también pre-
Temas para la reflexión senta la sucesió n de diversos amos y tiene tono satírico.
37. ¿Crees que la condición de converso de Rojas influyó en 8. Explica el sentido que tienen las palabras del ciego en
su obra? ¿Cómo? este pasaje del Tratado I:
Respuesta libre. Comenzamos nuestro camino, y en muy pocos días me mos-
38. Teniendo en cuenta el contexto del siglo xv, reflexiona tró jerigonza1; y, como me viese de buen ingenio, holgá base
sobre el valor que tiene esa declaración de Areúsa y mucho y decía:
—Yo oro ni plata no te lo puedo dar; mas avisos para vivir
compáralo con las posibilidades de independencia que
muchos te mostraré .
tiene una mujer en el siglo xxı. Redacta un escrito con Y fue ansí, que, después de Dios, este me dio la vida y, siendo
tus conclusiones. ciego, me alumbró y adestró en la carrera de vivir.
Respuesta libre.
1
jerigonza: jerga que utilizaban los ciegos para entenderse entre
4 Lazarillo de Tormes. Tratado I ellos.

Localización
El ciego es pobre, pero tiene larga experiencia. Aprecia ense-
1. Investiga y explica cuáles son las referencias históricas guida la inteligencia de Lá zaro y se propone sacarle de la
que aparecen en esta obra. ino- cencia y enseñ arle lecciones realistas para sobrevivir.
En el Tratado I se menciona la batalla de los Gelves. Hubo
9. Explica cómo aparece el problema del hambre en el Tra-
dos: puede referirse tanto a la de 1510 como a la de 1520. En el
Tra- tado VII se habla de las Cortes que Carlos I celebra en
tado I.
Toledo. También en este caso hubo dos (1525 y 1538-39) y el El hambre es un tema muy importante en la novela, pero es
narrador puede referirse a cualquiera de ellas. con el ciego con quien Lá zaro conoce esta experiencia. Pasa
hambre porque el ciego es mezquino y apenas le da migajas
2. Recuerda el contexto social y cultural que comienza a
al niñ o. Pero esto pone en marcha su inventiva y su ingenio
darse a mediados de siglo y relaciónalo con el novedoso para afrontar la necesidad.
realismo del Lazarillo de Tormes.
10. Busca información sobre el resto de los capítulos de la
18. Explica cómo entra Lázaro al servicio del ciego.
novela e indica en qué otros momentos de la trayectoria
del personaje se presenta el motivo temático del El ciego lo conoce en el mesó n en el que trabaja la madre de
Lá zaro y le parece que puede ser buen criado. La madre se lo
hambre.
encomienda. Cuando el ciego decide abandonar Salamanca,
En los Tratados II y III el hambre va en aumento. Después, el niñ o se marcha con é l.
esta situació n va mejorando pero Lá zaro no llega a olvidarla
en toda su vida: al final de la novela, ya convertido en adul- 19. Localiza el momento en que se despiden Lázaro y su
to, le vemos triunfar sobre el hambre al precio de aceptar su madre. ¿Qué palabras le dirige ella?
deshonra. Hijo, ya sé que no te veré más Procura de ser bueno, y Dios
11. Investiga sobre el contenido del Tratado III y explica te guíe. Criado te he y con buen amo te he puesto; válete por
cómo aparece ahí el tratamiento del conflicto entre ti.
apariencia y realidad. 20. Al marcharse con el ciego, saliendo de Salamanca, Lázaro
El escudero está absolutamente decidido a ocultar su verdade- aprende pronto su primera lección. Resume ese pasaje y
ra situació n. Tanto es así que incluso Lá zaro se deja engañ ar explica cuál es la enseñanza que aprende el niño.
por su aspecto al principio. La presunció n del escudero es En el puente romano a la salida de Salamanca, el ciego le dice
el ú nico defecto que el protagonista deplora de este amo: su que acerque el oído al toro de piedra y escuchará un ruido.
empeñ o en aparentar lo que no es, por un orgullo fantasioso. El niñ o lo cree y, al aproximarse, el ciego le coge la cabeza y
12. La honra era un concepto que en el Siglo de Oro estaba se la golpea fuertemente contra el toro. El niñ o aprende que
relacionado con la consideración social que se tenía debe desconfiar de todos, pues está solo.
sobre una persona. Este tema se trata con frecuencia a lo 21. ¿Cómo es la vida que lleva Lázaro con el ciego? ¿Qué
largo de la obra. Al final, en el Tratado VII, Lázaro se casa recursos emplea para aliviar su necesidad?
con la criada de un arcipreste. Busca información sobre Pasa mucha hambre, pues el ciego es avaro y no comparte
el argu- mento de este capítulo y explica cómo se enfoca nada de lo que gana con é l. Lá zaro despliega todo su ingenio
ahí el tema de la honra y el sentido que tienen estas para engañ arle y poder así alimentarse.
palabras del arcipreste: 22. El episodio del jarro de vino supone un cambio impor-
—Lá zaro de Tormes, quien ha de mirar a dichos de malas tante en la relación de Lázaro con el ciego. ¿Qué senti-
lenguas nunca medrará 1. Digo esto porque no me maravillaría mientos empieza a albergar el niño hacia su amo? ¿Qué
alguno, viendo entrar en mi casa a tu mujer y salir della. Ella opinión se crea sobre él?
entra muy a tu honra y suya; y esto te lo prometo. Por tanto,
no mires a lo que pueden decir, sino a lo que te toca: digo a Empieza a tenerle animadversió n. Se da cuenta de su crueldad
tu provecho. y de que disfrutó castigá ndole.
Lá zaro vive en una situació n deshonrosa porque acepta que 23. Resume el episodio del racimo de uvas.
su mujer tenga relaciones con el arcipreste, algo que es un El ciego le propone compartir un racimo de uvas que les han
secreto a voces para todo su entorno. Acepta esta situació n dado: irá n comiendo las uvas de una en una por turnos. Pero
porque le resulta provechosa. Como le dice el arcipreste, el ciego empieza a tomarlas de dos en dos y entonces Lá zaro
debe olvidarse de los rumores, y pensar solamente en su decide comerlas de tres en tres. El ciego descubre que el
provecho, es decir, en su prosperidad. El arcipreste dice, niñ o le ha engañ ado porque, cuando é l empezó a coger dos
acudiendo al doble sentido de sus palabras, que el bienestar de gol- pe, Lá zaro no protestó .
importa mucho má s que la honra.
24. ¿Qué acontecimientos tienen lugar en Escalona?
Contenido El episodio de la longaniza.
13. Al comienzo del relato, Lázaro explica las circunstancias 25. El ciego acostumbra a hacer mofas en público de las des-
de su nacimiento. ¿Por qué tiene el sobrenombre «de venturas de Lázaro. Cita algunos ejemplos.
Tormes»? El ciego cuenta pú blicamente en el mesó n las desventuras de
Porque nació en la ribera de ese río. Su madre se puso de parto Lá zaro: la del jarro, la del racimo y la actual, la de la longani-
estando en el molino que proveían a orillas del Tormes. za. Habla de cuá nto le gusta el vino.
14. ¿Quiénes eran sus padres? ¿A qué se dedicaba su padre? 26. Lázaro toma la decisión de dejar al ciego. ¿Cuándo y
Tomé Gonzá lez y Antona Pérez, naturales de Tejares. Su padre cómo lleva a cabo su propósito? ¿Cuál es el desenlace del
era molinero. capítulo?
15. Tras su viudez, la madre de Lázaro se empareja de nuevo. Salen a mendigar por Escalona, pero está lloviendo y el ciego
¿Con quién? decide volver a la posada. Han de pasar por un arroyo.
Con un hombre morisco. Lá zaro le engañ a sobre el lugar por el que debe pasar,
supuestamente donde el arroyo se estrecha. Le dice que dé un
16. La humorística anécdota del hermanastro de Lázaro que buen salto para no mojarse los pies, pero lo que provoca es
tiene miedo de su propio padre por el color de la piel que se estrelle contra un poste y caiga al suelo. Aprovecha el
formaba parte de la tradición oral de la época. ¿En qué momento en que acude gente a socorrer a su amo para
se basa la comicidad de dicha anécdota? ¿Qué reflexión escaparse a Torrijos.
suscita en Lázaro?
Personajes
El niñ o tiene miedo de su padre negro, pero él tambié n es
negro. Lá zaro reflexiona sobre las gentes que no saben mirar- 27. En este aprendizaje, ¿son rápidos los progresos de Lázaro
se a sí mismas. en el Tratado I? Justifica tu respuesta.
17. ¿Qué castigo sufre la madre de Lázaro por tener relacio- Sí. El primer incidente (el golpetazo contra el toro en el puen-
nes con un hombre de otra religión? te de Salamanca) hace que Lá zaro comprenda que debe ser
desconfiado y le saca de su inocencia de niñ o. Al final del
Recibe cien azotes.
tratado incluso vemos cierta crueldad en el modo en que el
protagonista abandona al ciego.
28. ¿Qué idea sobre Lázaro y el ciego podemos hacernos a
33. ¿A quién se dirige Lázaro en la carta? ¿Qué sabemos de
partir del siguiente pasaje? ¿Y sobre la relación que hay
este destinatario?
entre ellos?
No sabemos nada de é l, excepto que tiene un rango superior
Y, aunque yo quisiera asentar mi corazó n y perdonalle el jarra-
al de Lá zaro, lo cual no implica que se trate de una persona
zo, no daba lugar el maltratamiento que el mal ciego dende
especialmente importante. El Vuestra Merced con que Lá zaro
allí en adelante me hacía, que sin causa ni razó n me hería,
se dirige a é l solo implica una actitud respetuosa.
dá ndome coxcorrones1 y repelá ndome2. Y, si alguno le decía
por qué me trataba mal, luego contaba el cuento del jarro, 34. Divide en partes el Tratado I de acuerdo con el conte-
diciendo: nido del capítulo. ¿Podemos hablar en dicho tratado de
—¿Pensaréis que este mi mozo es algú n inocente? Pues oíd una estructura organizada en planteamiento, nudo y
si el demonio ensayara otra tal hazañ a3. desenlace?
Sí. El planteamiento abarca el relato de la infancia de Lá zaro
1
coxcorrones: coscorrones. 2 repelándome: arrancándo- cuando aú n vivía con su madre hasta que conoce al ciego.
me el pelo. 3 ensayara otra tal hazaña: maquinara una hazaña El nudo ocupa los diversos hechos que se van encadenando
semejante. mientras permanece con su amo (episodio del toro a la
salida de Salamanca, racimo de uvas, jarro de vino,
Lá zaro tiene buen fondo y quisiera perdonar al ciego por su longaniza, etc.). El desenlace se da al narrar el día lluvioso en
crueldad, pero es imposible pues este es un hombre violento que Lá zaro abandonó al ciego y el modo en que lo hizo.
y cruel. La relació n entre ellos se hace inaguantable para el
protagonista: el ciego desconfía del niñ o, lo humilla y lo gol- Género
pea delante de todo el mundo, y el niñ o llega a tomarle 35. ¿Qué subgéneros narrativos se cultivaron en el Renaci-
mucha aversió n. miento? ¿En qué época se originaron?
29. El ciego es un hombre desconfiado, y ni siquiera se fía de Las novelas sentimentales y las de caballerías, que se habían
su propio mozo. Cita partes del Tratado I que justifiquen originado en el siglo XV, siguen teniendo é xito en el XVI. En el
esta afirmación. Renacimiento surgen la novela bizantina, la pastoril, la moris-
ca y la picaresca.
En el episodio del jarro de vino: Mas como fuese el traidor
tan astuto, pienso que me sintió, y dende en adelante mudó 36. Recuerda los rasgos y la temática de los subgéneros
propósito asentaba su jarro entre las piernas y atapábale con narrativos del Renacimiento y explica las diferencias que
la mano, y ansí bebía seguro; Tantas vueltas y tientos dio al el Lazarillo tiene con ellos.
jarro que halló la fuente. Las diferencias del Lazarillo —y de la novela picaresca— con
En el episodio del racimo de uvas: Acabado el racimo, estuvo respecto a los demá s subgéneros narrativos son el realismo y,
un poco con el escobajo en la mano y, meneando la cabeza, sobre todo, el hecho de que el protagonista sea un antihé roe.
dijo: —Lázaro, engañado me has. Juraré yo a Dios que has 37. Localiza en el Tratado I los elementos propios de la
tú comido las uvas tres a tres. corrien- te picaresca y explícalos.
En el episodio de la longaniza: Yo torné a jurar y perjurar La forma autobiográ fica y epistolar; el realismo de la narra-
que estaba libre de aquel trueco y cambio; mas poco me ció n, que ademá s está ambientada en un espacio contempo-
aprovechó, pues a las astucias del maldito ciego nada se le rá neo (véanse las alusiones histó ricas, ademá s del retrato de
escondía. la sociedad españ ola del siglo XVI); la procedencia humilde
30. Además de su desconfianza, ¿cómo se describe al ciego? del protagonista; el planteamiento de su evolució n
Resume su caracterización. psicoló gica y su aprendizaje vital, que viene apuntado ya
desde la primera de sus aventuras.
Mezquino y avariento. Cruel. Con muchas habilidades para
sacarle dinero a la gente y supuestos conocimientos curati- 38. Explica también los elementos del género narrativo, a
vos. Sagaz. partir de tu lectura de dicho tratado.
31. Ateniéndote al argumento de la obra, explica cuáles son La presencia del narrador personaje (protagonista), la locali-
las conclusiones a las que llega Lázaro al final, y en las zació n espacial y temporal (hay claras referencias en ambos
que se basa para vivir feliz y tranquilo. aspectos), la inserció n de la descripció n y el diá logo en el
discurso narrativo, la presentació n de unos personajes y su
La tranquilidad, para é l, significa no pasar hambre y haber
caracterizació n.
alcanzado una posició n social con su matrimonio, aunque esta
posició n sea humilde. Está dispuesto a mantener ese estatus 39. Recuerda lo que has estudiado sobre el desarrollo de la
aunque deba pagar el precio de su honra. corriente picaresca después de la publicación del
32. Recuerda lo que has estudiado sobre el argumento com- Lazarillo.
pleto de esta obra e indica a qué niveles sociales perte- ¿En qué época se produjo este desarrollo? Busca
necen sus personajes. informa- ción y cita algunas obras y autores de esta
corriente.
A las clases bajas: los orígenes de Lá zaro son muy humildes,
su primer amo es un mendigo, tendrá tambié n cinco amos La novela picaresca se desarrollaría durante el siglo XVII. Otras
que pertenecen al nivel inferior del clero. Al final de su vida obras picarescas de interés: Guzmán de Alfarache, de Mateo
alcanza un mínimo grado de prosperidad al entrar en el oficio Alemá n; La vida del Buscón llamado Pablos, de Quevedo;
de pregonero, pero es un oficio mal considerado. La pícara Justina, de Francisco Ló pez de Ú beda; Vida del
escudero Marcos de Obregón, de Vicente Espinel.
Estructura
Estilo
Y, pues Vuestra Merced escribe se le escriba y relate el caso
muy por extenso, paresciome no tomalle por el medio, sino 40. En el Tratado I, busca citas en las que pueda observarse
del principio, porque se tenga entera noticia de mi persona. el tono del narrador y la naturalidad del estilo.
Comenzamos nuestro camino, y en muy pocos días me
mostró jerigonza; y como me viese de buen ingenio,
holgábase mucho y decía: —Yo oro ni plata no te lo puedo
dar; mas avisos para
vivir muchos te mostraré. Y fue ansí, que, después de Dios, 47. ¿Cómo ha cambiado la sociedad en el siglo xxı? ¿Qué
este me dio la vida y, siendo ciego, me alumbró y adestró en tratamiento reciben los ciegos y las personas que tienen
la carrera de vivir. Huelgo de contar a Vuestra Merced estas
niñe- rías, para mostrar cuánta virtud sea saber los
hombres subir siendo bajos, y dejarse bajar siendo altos
cuánto vicio; Mas, por no ser prolijo, dejo de contar muchas
cosas, así graciosas como de notas, que con este mi primer
amo me acaescieron, y quiero decir el despidiente y, con él,
acabar.
41. ¿Qué palabras y usos lingüísticos propios del ambiente
marginal del protagonista aparecen en ese capítulo?
Hideputa; al triste de mi padrastro azotaron y pringaron (tor-
mento que consistía en echar tocino derretido al fuego sobre
las heridas de los azotes previos); centenario (castigo de cien
azotes); jerigonza (jerga de los ciegos); besos (para referirse a
tragos de vino); tío (es el tratamiento que Lá zaro da al ciego, y
responde a la fó rmula de respeto que solía dirigirse a las per-
sonas mayores en el ambiente popular), etcétera.
42. Localiza también ejemplos de dichos populares, refranes
y lenguaje proverbial.
Refranes y dichos populares: Echar la soga tras el caldero
(echarlo todo a perder); Más da el duro que el desnudo; No
paresciendo ellas, pudiera negar la demanda (al no aparecer
el cuerpo del delito —las narices— podría evitar las respon-
sabilidades, la «demanda criminal»), etc. El tono sentencioso,
de estilo proverbial, puede también comentarse: El mozo del
ciego un punto ha de saber más que el diablo; Cuánta virtud
sea saber los hombres subir siendo bajos, y dejarse bajar sien-
do altos cuánto vicio; Lo que te enfermó te sana y da salud,
etcétera.
43. ¿Has observado lo frecuentes que son las interjecciones
y exclamaciones en los diálogos? Cita algunos ejemplos.
¿Qué finalidad crees que se persigue con la
abundancia de exclamaciones?
¡Lacerado de mí!, ¡Oh, gran Dios!, ¡Sús! El objetivo es el rea-
lismo, la expresividad y la naturalidad.

El texto como acto comunicativo


44. Busca y anota citas en las que se aluda a este receptor.
Pues sepa Vuestra Merced, ante todas las cosas; Mas también
quiero que sepa Vuestra Merced; Y porque vea Vuestra Merced
a cuánto se extendía el ingenio deste astuto ciego; Conside-
rando lo que aquel día me dijo salirme tan verdadero como
adelante Vuestra Merced oirá, etc.
45. El emisor ficticio (Lázaro) se dirige a un lector implícito:
es decir, presupone que este conocerá el asunto del que
va a hablarle, que tiene ciertos datos sobre él y sobre su
vida.
¿Podrías citar algún pasaje en el que se observe esto?
En el pró logo Lá zaro escribe: Y pues Vuestra Merced escribe
se le escriba y relate el caso muy por extenso… No se aclara a
qué caso se refiere, pero es evidente que esta omisió n
corres- ponde a un contexto compartido por Lá zaro y el
narratario.

Temas para la reflexión


46. Explica las dificultades para la comprensión del mensaje
que puede conllevar el hecho de que el lector pertenezca
a una época distinta de la del autor.
Cambios en el idioma, tanto en su morfología como en el léxico.
Contexto social y cultural muy diferente; alusiones
relaciona- das con costumbres e ideología que necesitará n
aclaraciones para el lector actual. La visió n del mundo, la
forma de vida, las ideas, las conductas, etc., del lector del
siglo XXI son muy diferentes de las del siglo XVI.
una discapacidad? ¿Tienen menos oportunidades que el una refundició n firmada por Garci Rodríguez de Montalvo de
resto de la población? ¿Existen medios para paliar sus
limitaciones en la vida social y laboral?
En la sociedad actual, los discapacitados no son
marginados, como ocurría hace siglos. Ademá s, la sociedad
actual dispone de recursos y legislació n para garantizar sus
derechos y com- pensar sus limitaciones.
48. ¿Sería hoy posible que un niño de ocho años tuviese
unas andanzas como las que Lázaro vive con el ciego?
Redacta un escrito en el que expliques las diferencias
entre la for- ma de vida de un niño de clase social baja
en el siglo xvı y en nuestro tiempo.
Respuesta libre.
49. El aprendizaje a partir de las experiencias vitales supone
a veces un paso de la inocencia a la desconfianza, que
mar- ca un cambio entre la vida infantil y la adulta.
¿Compartes esta afirmación? Expón tus opiniones sobre
este asunto en un texto de unas veinte líneas.
Respuesta libre.

5 Miguel de Cervantes. Don Quijote


de la Mancha. Capítulos I-VI
Localización
1. Resume el argumento de los seis primeros capítulos.
Alonso Quijano, un viejo hidalgo que lleva una vida
tranquila en una aldea manchega, pasa el día leyendo
obsesivamente libros de caballerías, lo cual le lleva a la
locura. Decide enton- ces convertirse en caballero andante
y adopta el nombre de Don Quijote de la Mancha. Se
prepara unas armas y un caba- llo, sale a buscar aventuras,
se aloja en una venta que toma por castillo y allí toma la
orden de caballería. Corre entonces diversas aventuras:
libera a un criado que está siendo azota- do por su amo, se
encuentra con unos mercaderes a los que
se enfrenta, acaba maltrecho y lo encuentra un vecino de su
pueblo, quien lo devuelve a su hogar. Allí, el ama y la
sobrina le cuidan y encomiendan al cura y al barbero que
quemen los libros que han provocado la locura del hidalgo.
Estos hacen un escrutinio en el que deciden qué libros
merecen ser salvados y cuá les quemados y encienden una
hoguera en el patio.
2. Repasa lo que has estudiado sobre esta obra y contesta:
¿qué aventuras son las más destacadas en la segunda
salida?
La de los molinos de viento, la de los galeotes, la del yelmo
de Mambrino (bacía de barbero que don Quijote toma por
yel- mo legendario), el encuentro con otros personajes en la
venta (Fernando y Dorotea, Luscinda y Cardenio), etcétera.
3. Recuerda lo que has aprendido sobre la segunda
parte del Quijote e indica qué nuevos personajes
aparecen en el Quijote de 1615. ¿Cuáles son en esta
segunda parte las aventuras más destacadas?
Es muy importante la figura de Sansó n Carrasco, el
bachiller que decide hacerse pasar por caballero andante para
desafiar a don Quijote y, venciéndole, ponerle la condició n de
que regrese a casa (Sansó n Carrasco tomará la identidad del
Caballero de Los Espejos y de La Blanca Luna), los duques,
Á lvaro Tarfe (personaje de la novela de Avellaneda, que
Cervantes utilizará para desautorizar a los falsos don
Quijote y Sancho), etcé tera.
4. ¿Qué obras pertenecientes a dicha corriente conoces?
¿Quiénes son sus autores?
Entre otras, Amadís de Gaula, editada en 1508 (aunque es
ediciones anteriores), Palmerín de Oliva, publicada en 1511, Quesada; Este sobredicho hidalgo, los ratos que estaba ocioso
seguidas ambas por sus continuaciones: las Sergas de Esplan- (que eran
dián, el Palmerín de Inglaterra, etc.
5. ¿Cuáles eran los elementos argumentales y temáticos
propios de los libros de caballerías?
Las aventuras de un caballero andante, enamorado de una
dama idealizada a la que dedica todas sus hazañ as. El prota-
gonista se rige de acuerdo con el có digo de la caballería, que
implica fidelidad y devoció n a su amada, honestidad, honor y
aceptació n de las condiciones que le imponga otro caballero
que le derrote en justa lid. En los relatos caballerescos hay
siempre elementos y seres fantá sticos, ambientes guerreros
y, a la vez, corteses, donde se celebran justas y combates
por- tentosos. Las narraciones sobre caballeros andantes
siempre son presentadas como si fuesen recopilaciones de
supuestos cronistas, como si se tratase de hechos histó ricos.
6. Recuerda lo que sabes sobre el contexto histórico de Cer-
vantes. ¿Por qué definimos ese tiempo como un periodo
de crisis? ¿En qué terrenos?
La época imperial ha entrado en decadencia, es ya un pasado
glorioso que suscita el orgullo de muchos, pero no puede
evi- tar una realidad de crisis econó mica y social y de
corrupció n política.
7. Las etapas de crisis suelen dar lugar a expresiones artísti-
cas orientadas a la sátira, la caricatura, la ironía y, en
gene- ral, al humor crítico. ¿Por qué crees que los artistas
acuden tan a menudo al humor para expresar sus
desengaños o sus críticas?
La ironía e incluso el sarcasmo, la caricatura y la sá tira son
vehículos que hacen llegar la crítica má s lejos y suscitan
reflexiones en los receptores de estas obras.
Intención
8. ¿Qué queremos decir cuando hablamos de «tema meta-
literario»?
La literatura adopta como tema la literatura misma.
9. Como novela de caballerías, el Quijote es una parodia.
¿Por qué?
Porque presenta todos los elementos de los libros de caballería
deformá ndolos con la técnica de la caricatura, a través de la
ironía y la sá tira.
10. Los héroes de la narrativa anterior a Cervantes eran
perso- najes heroicos e idealizados. Localiza en los seis
primeros capítulos tres momentos que nos muestren el
contraste entre don Quijote y aquellos personajes
literarios.
La propia caracterizació n que se nos hace al principio del
relato: el caballero manchego es un hombre ya anciano,
desconocido, no es un héroe sino un hombre destinado a una
vida monó tona e invisible y de é l nos dice Cervantes que se le
secó el cerebro. En segundo lugar puede citarse todo el
pasaje en que recibe la orden de caballería, y se describen el
desconcierto y las mofas de los personajes que contemplan
la escena. Otro pasaje sig- nificativo es el que narra el estado
lamentable en que queda el protagonista tras el encuentro
con los mercaderes.
Contenido
11. En el capítulo I se nos presenta al personaje. ¿Cómo se
le caracteriza? Contesta a la pregunta citando pasajes
concretos.
Un hidalgo de los de lanza en astillero, adarga antigua,
rocín flaco y galgo corredor; Era de complexión recia, seco
de car- nes, enjuto de rostro, gran madrugador y amigo de
la caza. Quieren decir que tenía el sobrenombre de Quijada o
los más del año) se daba a leer libros de caballerías con tar la ceremonia y darle la orden de caballería a don Quijote
tanta afición y gusto, que olvidó casi de todo punto el cuanto antes.
ejercicio de la caza, y aun la administración de su
hacienda; Con estas razones perdía el pobre caballero el
juicio, y desvelábase por entenderlas y desentrañarles el
sentido.
12. ¿Cuáles eran sus costumbres? ¿Cuáles son sus armas?
¿Cómo escoge a su dama?
Tenía las costumbres propias de un hidalgo ya anciano, aco-
modado y de vida monó tona, entregado a la lectura hasta
que perdió el juicio. Sus armas habían pertenecido a sus
bisabue- los, estaban oxidadas y enmohecidas y olvidadas
en un rin- có n. Las limpió y aderezó la celada con cartones y
barras de hierro. A su dama la escoge recordando a una
labradora del Toboso de la que estuvo enamorado antañ o, y
que se llamaba Aldonza Lorenzo. Le puso de nombre
Dulcinea del Toboso.
13. En el capítulo II, don Quijote emprende su aventura
y se imagina que, algún día, los historiadores contarán
sus hazañas. Localiza el pasaje concreto en el que
fantasea con estas ideas.
¿Quién duda sino que en los venideros tiempos, cuando
salga a la luz la verdadera historia de mis famosos hechos,
que el sabio que los escribiere no ponga, cuando llegue a
contar esta mi primera salida tan de mañana, desta
manera?: Apenas había el rubicundo Apolo; Dichosa edad
y siglo dichoso aquel adonde saldrán a luz las famosas
hazañas mías, dignas de entallarse en bronces; ¡Oh, tú,
sabio encantador, quienquiera que seas, a quien ha de tocar
ser el coronista desta peregrina historia, ruégote que no te
olvides de mi buen Rocinante.
14. Don Quijote llega a una venta. ¿Dónde imagina que se
halla? ¿Qué personajes encuentra allí?
En un castillo. Dos mozas, un porquero, el ventero, un
castra- dor de puercos.
15. ¿Cómo se comporta don Quijote con ellos? ¿Y ellos con
él? Don Quijote se comporta con ellos de acuerdo con la
idea que se forma: piensa que está en un castillo, que ellos
le ven como un caballero andante. Ellos le tratan con una
mezcla de asom- bro y sorna, y le siguen la corriente.
16. Explica las diferencias que existen entre la imagen que
don Quijote se forja de dichos personajes y la verdadera
condición que estos tienen.
Ha tomado a las mozas por bellas damas; al porquero que
toca un cuerno, por enano que le recibe haciendo señ al de
su venida; al ventero, por alcaide de la fortaleza, y al
castrador de puercos que toca un silbato, por alguien que
ameniza su cena con mú sica.
17. Indica qué acontecimientos tienen lugar en el capítulo III
mientras don Quijote está velando sus armas.
El ventero confirma que don Quijote está loco de remate
pero le sigue la corriente y le dice que oficiará la ceremonia
para que sea armado caballero. Explica a los demá s
personajes la locura del hidalgo y todos le observan
mientras este vela sus armas en el patio. Durante la noche,
unos arrieros quieren ir a dar agua a su recua y necesitan
retirar las armas de don Quijote, que está n apoyadas en una
pila. Se originan enfren- tamientos sucesivos, el
protagonista golpea primero a uno y luego a otro con la
lanza en la cabeza. Sus compañ eros le apedrean aunque al
final le acaban tomando miedo.
18. El curso de los acontecimientos crea ciertos temores
en el ventero. ¿Cuáles? Para evitar lo que teme, ¿qué
con- ducta adopta?
Teme que se produzca otro altercado semejante y decide acor-
19. Resume la ceremonia en que don Quijote es armado días fuera de casa y que antes de marcharse no dejaba de decir
caballero. ¿Quiénes intervienen en ella? dis- parates. El labrador entonces comprende la locura del
El ventero trae un libro de asientos, hace arrodillar a don hidalgo y llama para que les abran, hablando y
Quijote, finge pronunciar una oració n y le da el consabido comportá ndose como un personaje caballeresco para seguirle
espaldarazo. Ordena a una de las mozas que le ciñ a la espada la corriente al anciano.
y ella lo hace, muerta de risa. La ceremonia se desarrolla con
tremenda rapidez. Don Quijote trata con galanura a las mozas,
les pone sobrenombres (doñ a Tolosa y doñ a Molinera), se des-
pide con gratitud del ventero y se marcha.
20. Cuando don Quijote abandona la venta, el ventero no le
cobra los servicios que le ha ofrecido. ¿Por qué?
Porque quiere evitar nuevos conflictos y desea que don Qui-
jote se marche lo antes posible.
21. Explica qué sucede en el capítulo IV cuando don Quijote
se encuentra con el labrador que está azotando a su
criado.
Don Quijote ordena al labrador (Juan Haldudo) que libere a
su criado (Andrés) y que le pague su sueldo. El labrador
responde que Andrés vaya a su casa, que allí le pagará , y don
Quijote orde- na al mozo que así lo haga. Este se resiste, pero
el caballero le asegura que basta el juramento de Haldudo para
saber que cum- plirá , pues así lo mandan las leyes de la
caballería. Cuando don Qujiote se marcha, el labrador
continú a castigando a su criado.
22. Explica el contraste que se observa en este episodio
entre el mundo que concibe don Quijote —idealizado— y
el real, en el que viven los demás personajes.
En el mundo literario de don Quijote, el mozo azotado está
sufriendo un agravio y el labrador es alguien que abusa de
un ser indefenso. En el mundo real, el criado es un bellaco
des- cuidado que recibe el castigo de su amor. Ademá s, el
idealismo del protagonista le lleva a creer que el labrador
actuará como é l le ha ordenado porque le obliga el
juramento de la caballe- ría. Para don Quijote ese juramento
es la ú nica garantía que necesita. El mozo André s le advierte
que Juan Haldudo no es un caballero, le describe la realidad,
pero don Quijote no la ve.
23. En cierto momento, don Quijote llega a una encrucijada.
¿Cómo decide el camino que toma en ella?
Se queda pensativo un buen rato, imitando lo que ha visto
hacer a los hé roes de sus libros. Después, suelta la rienda de
Rocinante, dejando que este tome el rumbo que quiera.
24. Tras la encrucijada, don Quijote se encuentra con unos
mercaderes. ¿Por quiénes los toma el hidalgo? ¿Qué les
reclama?
Por caballeros andantes. Les reclama que confiesen que Dul-
cinea del Toboso es la doncella má s hermosa del mundo.
25. ¿Cómo reaccionan ellos? ¿Cómo termina este incidente?
Responden que para hacer ese reconocimiento tienen que ver
a la dama, lo cual enoja a don Quijote. Ofendido, arremete
contra ellos con su lanza, pero se cae del caballo y un mozo
de mulas comienza a apalearle.
26. Señala con quién se encuentra el viejo caballero en el
capítulo V.
Se encuentra con un labrador y vecino de su pueblo, que al
verle molido en el suelo, le ayuda y le lleva de vuelta a su
casa.
27. Explica cómo y por qué regresa don Quijote a casa.
Regresa ayudado por el vecino, que espera a que se haga de
noche para que la gente del pueblo no vea al viejo hidalgo en
tan deplorable estado. Al llegar, oyen có mo el ama y la
sobrina
les cuentan al cura y al barbero que don Quijote lleva tres
28. Explica quiénes llevan a cabo el escrutinio de libros anó nima y anodina para llevar adelante grandes hazañ as.
en el capítulo VI. ¿Qué actitud tienen el ama y la sobrina
ante lo que le ocurre al protagonista? ¿Qué opiniones
manifiestan sobre el destino que han de correr los
libros?
El cura y el barbero. El ama tiene miedo de los
encantadores que ella cree que hay en los libros. Tanto ella
como la sobrina son partidarias de quemarlos todos porque
todos les parecen causa del dañ o.
29. ¿Qué libros se quemaron? ¿Cuáles se salvaron? ¿Por
qué? Se queman: las Sergas de Esplandián, Amadís de
Grecia, porque no les agradan las continuaciones del
Amadís de
Gaula; Don Olivante de Laura, Florismarte de Hircania,
El Caballero Platir, El Caballero de la Cruz, Bernardo del
Carpio, Roncesvalles, Palmerín de Oliva, La Diana
segunda del Salmantino, El pastor de iberia, Ninfas de
Henares, y Desengaños de celos.
Se salvan: Amadís de Gaula, porque es el mejor y primer
libro de caballerías; Espejo de Caballerías, aunque lo que
parece bien al cura es el original en italiano (Orlando
Furio- so) y no la traducció n; Palmerín de Inglaterra,
porque es muy bueno y lo compuso un rey de Portugal; Don
Belianís, porque, si se eliminan algunas cosas que deberían
enmendarse, mere- ce la pena conservarlo; Tirante el
Blanco, de Johanot Marto- rell, porque es el mejor libro del
mundo; La Diana, de Jorge de Montemayor, por su parte
en prosa y por ser el primero en su género, aunque
recomiendan eliminar la parte en verso; Diana enamorada,
de Gil Polo; Los diez libros de Fortuna de amor, de Antonio
de Lofraso, porque al cura le parece lo mejor de su género; El
pastor de Fílida, de Luis Gá lvez de Montalvo, porque al cura
le parece una joya preciosa; Tesoro de varias poesías,
porque su autor —Pedro de Padilla— es amigo del cura y
aunque ese libro no le parece de los mejores, siente un gran
respeto hacia otras obras suyas; El Cancionero de López
Maldonado, porque su autor es amigo del cura y es un gran
poeta; La Galatea, porque Miguel de Cervantes ha prometido
una segunda parte y es amigo del cura; La Araucana, La
Austríada y El Monserrate, de Cristó bal de Virués, porque
los consideran las má s ricas muestras de poesía en Españ a;
y Las lágrimas de Angélica, porque su autor (Luis Barahona
de Soto) es uno de los má s famosos poetas del mundo.
30. En el escrutinio de los libros, aparece el propio Cervan-
tes y se menciona una obra suya. ¿Cuál? ¿Se salva de la
quema? ¿Qué se dice sobre esta obra y sobre su autor?
La Galatea. Dice el barbero que Cervantes es amigo suyo,
que le han ocurrido muchas desdichas, que el libro no es
una gran obra pero que merece la pena esperar a la
segunda parte que el autor ha prometido.

Personajes
31. Teniendo en cuenta esta idea de la reinvención, explica
el contenido del siguiente fragmento del capítulo V.
A esto respondió el labrador:
—Mire vuestra merced, señ or, pecador de mí, que yo no soy
don Rodrigo de Narvá ez, ni el marqués de Mantua, sino Pedro
Alonso, su vecino; ni vuestra merced es Valdovinos, ni Abin-
darrá ez, sino el honrado hidalgo del señ or Quijana.
—Yo sé quié n soy —respondió don Quijote—, y sé que pue-
do ser no solo los que he dicho, sino todos los doce Pares de
Francia, y aun todos los nueve de la Fama, pues a todas las
hazañ as que ellos todos juntos y cada uno por sí hicieron, se
aventajará n las mías.
Alonso Quijano decide convertirse en don Quijote, es decir,
decide ser quien él quiere ser. Salta por encima de su vida
32. A partir de tu lectura de los seis primeros capítulos, loca- tales, Sancho Panza, que así se llamaba el labrador, dejó su
liza y explica los elementos concretos que contraponen la mujer e hijos y asentó por escudero de su vecino.
caracterización de Alonso Quijano con la de don Quijote
38. Siempre se ha presentado a Sancho Panza como el perso-
de la Mancha. Acude a citas concretas de la obra para
naje realista por oposición con el idealismo de don Qui-
responder.
jote. Investiga sobre el argumento de la novela y justifica
Alonso Quijano era un hidalgo que se dedicaba a la caza y a esta afirmación.
su hacienda y llevaba una existencia doméstica y rutinaria
Sancho siempre expone ante su señ or la realidad en que se
(vé ase la descripció n que se hace al comienzo de la novela).
mueven, advirtiéndole que é l no ve gigantes, sino molinos, ni
Don Quijote no quiere estar en el hogar, sino al aire libre, y
ha encontrado a Dulcinea, sino a una tosca labradora. Ade-
vivir aventuras, no en las rutinas (casi todo aquel día caminó
má s, siempre está pensando en cosas materiales, especial-
sin acontecerle cosa que de contar fuese, de lo cual se deses-
mente en la comida.
peraba, porque quisiera topar luego luego con quien hacer
experiencia del valor de su fuerte brazo). Quiere vivir una 39. Pese a dicho realismo, Sancho Panza es incondicional de
vida digna de ser narrada en una gran cró nica. su señor, le acompaña fielmente, confía siempre en su
33. ¿Qué otros personajes aparecen en estos capítulos ini- palabra y su criterio y evoluciona junto a él,
ciales? ¿Qué relación tienen con don Quijote? ¿Cuál es contagiándose de su idealismo. Busca información sobre
su función en la trama de dichos capítulos? esta evolución de San- cho y redacta un escrito con los
Ama, sobrina, cura, barbero, Pedro Alonso (su vecino del
datos que encuentres.
pueblo): representan la vida que don Quijote ha dejado atrá s, Pese a que él no ve el mundo como don Quijote, Sancho
la vida de Alonso Quijano. Ventero y demá s personajes de la consi- dera que es é l el equivocado, porque no tiene el
venta, labrador Juan Haldudo y mozo Andrés, mercaderes: elevado inge- nio de su señ or. Para é l don Quijote es un
marcan la realidad contra la que choca el mundo imaginario modelo. Así, poco a poco va aprendiendo a ver la realidad
de don Quijote, contribuyen así a la parodia y la comicidad. desde la perspectiva del caballero, va conociendo el có digo y
el contexto de ese mundo literario. En la segunda parte esta
34. ¿Qué personajes y libros de la época se citan? Entre ellos, evolució n es decisiva. Por ello, al final de la novela, cuando
¿quién constituye el ideal para don Quijote? don Quijote agoniza, el fiel escudero intenta convencerle
Sobre todo, Amadís de Gaula. para que se levante de la cama y se disponga nuevamente a
35. ¿Qué información se nos da acerca de los personajes vivir en libertad la vida de un caballero andante.
secundarios? ¿Se les caracteriza psicológicamente? Estructura
No. De estos personajes solo conocemos el contraste que pro- 40. De acuerdo con el concepto tradicional de narración,
vocan con respecto a don Quijote.
¿podrían ser estos primeros seis capítulos una novelita
36. Entre los diversos personajes con quienes don Quijote se independiente? Justifica tu respuesta.
topa en estos seis capítulos, encontramos muy diversas Sí. En estos seis capítulos encontramos una presentació n de la
conductas: unos se burlan de él, otros le siguen la situació n y los personajes, un motivo temá tico central (la sali-
corrien- te, otros intentan ayudarle, otros responden da de don Quijote), un desarrollo de una trama y un desenlace
agresiva- mente a lo que consideran ataques del que devuelve al protagonista al punto de partida.
caballero. Pon un ejemplo de cada caso. 41. Normalmente, una narración suele estructurarse en tres
Se burlan de é l las mozas de la venta. Le sigue la corriente el partes: planteamiento, nudo y desenlace. Divide la trama
ventero. Intenta ayudarle Pedro Alonso, su vecino. Responde de los seis primeros capítulos en estas tres partes.
agresivamente a don Quijote el mozo de mulas que iba con
Planteamiento: primer capítulo, donde se presenta la situació n
los mercaderes.
y a los personajes. Nudo: capítulos II-V, que recogen las
37. ¿Cómo es Dulcinea? Localiza en estos capítulos iniciales aven- turas del caballero, desde su salida hasta su regreso, y
los pasajes que muestran la imagen que don Quijote las consecuencias a que dichas aventuras le conducen.
tiene de ella. Desen- lace: capítulo VI, en el que el escrutinio de libros
No existe, es solo un ser imaginario, una invenció n de don Qui- constituye toda una reflexió n metaliteraria que explica el
jote. Por ello es una dama idealizada, típica doncella del amor sentido que Cervantes da a su relato.
cortés. Y fue, a lo que se cree, que en un lugar cerca del suyo Género
había una moza labradora de muy buen parecer, de quien él
42. Indica qué subgéneros narrativos aparecen en el Quijote
un tiempo anduvo enamorado (aunque, según se entiende,
ella jamás lo supo ni se dio cata dello). Llamábase Aldonza de 1605.
Lorenzo, y a esta le pareció bien darle título de señora de La novela pastoril (episodio de la pastora Marcela), la sen-
sus pensamientos; y buscándole nombre que no desdijese timental (narració n de El curioso impertinente), la novela
mucho del suyo, y que tirase y se encaminase al de princesa bizantina y la morisca (relatos de aventuras, como el del cau-
gran señora, vino a llamarla Dulcinea del Toboso, porque era tivo en la venta).
natural del Toboso. 43. Teniendo en cuenta dicho tratamiento del narrador,
En este tiempo solicitó don Quijote a un labrador vecino expli- ca el sentido que adquiere el siguiente fragmento
suyo, hombre de bien (si es que este título se puede dar al del capí- tulo I:
que es pobre), pero de muy poca sal en la mollera. En Quieren decir que tenía el sobrenombre de Quijada, o Quesa-
resolució n, tan- to le dijo, tanto le persuadió y prometió , que da, que en esto hay alguna diferencia entre los autores que
el pobre villano se determinó de salirse con é l y servirle de deste caso escriben; aunque por conjeturas verosímiles se deja
escudero. Decía- le, entre otras cosas, don Quijote que se entender que se llamaba Quejana. Pero esto importa poco a
dispusiese a ir con é l de buena gana, porque tal vez le podía nuestro cuento; basta que en la narració n dél no se salga un
suceder aventura que ganase, en quítame allá esas pajas, punto de la verdad.
alguna ínsula, y le dejase a é l por gobernador della. Con
estas promesas y otras El narrador se limita a recoger la informació n que encuentra
en las cró nicas, incluso cuando es imprecisa (como
ocurre
con el nombre del hidalgo). Las inexactitudes no le preocupan de Narváez, que esta hermosa Jarifa que he dicho es ahora
porque lo que interesa es la veracidad de los hechos narrados.
44. Cita otros momentos en los seis primeros capítulos en
que se pueda hablar de alusiones semejantes.
Al cabo se vino a llamar don Quijote; de donde, como queda
dicho, tomaron ocasión los autores desta tan verdadera his-
toria que sin duda se debía llamar Quijada, y no Quesada,
como otros quisieron decir; Y fue, a lo que se cree, que en un
lugar cerca del suyo había una moza labradora de muy
buen parecer, de quien él un tiempo anduvo enamorado
(aunque, según se entiende, ella jamás lo supo): obsérvese
có mo en esta alusió n el narrador afirma que ni siquiera el
origen de la inven- ció n de Dulcinea parece ser del todo seguro
para los cronistas.
45. Investiga y explica quién es ese cronista que aparecerá a
partir del capítulo IX.
Cide Hamete Benengeli.
46. En todo momento, se persigue transmitir al lector la idea
de que se narra una historia real. Localiza pasajes en los
que se aprecie este objetivo.
Puede observarse en todas las alusiones a los cronistas, o en el
uso de expresiones como según se cree, según se entiende, que
dan a entender que el narrador está recopilando unos hechos
reales de los que hablan otros. A esta impresió n contribuye
definitivamente el realismo del relato, con sus topó nimos y
tó picos totalmente reconocibles para el lector de la época.

Estilo
47. Localiza pasajes donde se observe el estilo literario y alti-
sonante del protagonista.
Hay muchos. Entre otros, pueden citarse los siguientes:
En el capítulo II: ¡Oh princesa Dulcinea, señora deste cautivo
corazón! Mucho agravio me habedes fecho en despedirme
y reprocharme con el riguroso afincamiento de mandarme
no parecer ante la vuestra fermosura. Plégaos, señora, de
membraros desde vuestro sujeto corazón, que tantas cuitas
por vuestro amor padece!
En el capítulo III: No esperaba yo menos de la gran magnifi-
cencia vuestra, señor mío —respondió don Quijote—; y así,
os digo que el don que os he pedido y de vuestra liberalidad
me ha sido otorgado es que mañana en aquel día me habéis
de armar caballero.
—¡Oh, tú, quienquiera que seas, atrevido caballero, que
llegas a tocar las armas del más valeroso andante que
jamás se ciñó espada! Mira lo que haces, y no las toques, si
no quieres dejar la vida en pago de tu atrevimiento.
¡Oh señora de la fermosura, esfuerzo y vigor del debilitado
cora- zón mío! Ahora es tiempo que vuelvas los ojos de tu
grandeza a este tu cautivo caballero, que tamaña aventura está
atendiendo.
En el capítulo IV: Por el sol que nos alumbra que estoy por
pasaros de parte a parte con esta lanza. Pagadle luego sin
más réplica; si no, por el Dios que nos rige que os
concluya y aniquile en este punto.
Del sahumerio os hago gracia —dijo don Quijote—; dádselos
en reales, que con eso me contento; y mirad que lo cumpláis
como lo habéis jurado; si no, por el mismo juramento os
juro de volver a buscaros y castigaros; y que os tengo que
hallar aunque os escondáis más que una lagartija. Y si
queréis saber quién os manda esto, para quedar con más
veras obli- gado a cumplirlos, sabed que yo soy el valeroso
don Quijote de la Mancha, el desfacedor de agravios y
sinrazones, y a Dios quedad, y no se os parta de las mientes
lo prometido y jurado, so pena de la pena pronunciada.
En el capítulo V: —Sepa vuestra merced, señor don Rodrigo
la linda Dulcinea del Toboso, por quien yo he hecho, hago y andante en
haré los más famosos hechos de caballerías que se han
visto, vean ni verán en el mundo.
48. Busca ahora citas donde se aprecie el realismo de los
personajes secundarios.
En el capítulo IV: ¿Irme yo con él —dijo el muchacho— más?
¡Mal año! No, señor, ni por pienso; ¿porque en viéndose
solo me desuelle como a un San Bartolomé?
En el capítulo V: Señor Quijana (que así se debía de llamar
cuando él tenía juicio y no había pasado de hidalgo
sosegado a caballero andante), ¿quién ha puesto a vuestra
merced desta suerte?; Mire vuestra merced, señor, pecador de
mí, que yo no soy don Rodrigo de Narváez, ni el marqués de
Mantua, sino Pedro Alonso, su vecino; ni vuestra merced es
Valdovinos, ni Abindarráez, sino el honrado hidalgo del
señor Quijana; ¿Qué le parece a vuestra merced, señor
licenciado Pero Pérez (que así se llamaba el cura), de la
desgracia de mi señor? Tres días ha que no parecen él, ni el
rocín, ni la adarga, ni la lanza, ni las armas.
¡Desventurada de mí!, que me doy a entender, y así es ello
la verdad como nací para morir, que estos malditos libros de
caballerías que él tiene y suele leer tan de ordinario le han
vuelto el juicio; que ahora me acuerdo haberle oído decir
muchas veces, hablando entre sí, que quería hacerse
caballero andante; ¡Mirá, en hora maza —dijo a este punto
el ama—, si me decía a mí bien mi corazón del pie que
cojeaba mi señor! Suba vuestra merced en buen hora, que
sin que venga esa hurgada le sabremos aquí curar; ¡Ta, ta!
—dijo el cura— ¿Jayanes hay en la danza? Para mi
santiguada que yo los queme mañana antes que llegue la
noche.
49. Localiza y explica el uso de la ironía por parte del
narrador. La ironía está presente ya desde la primera línea:
En un lugar de La Mancha, de cuyo nombre no quiero
acordarme. En todo momento el narrador da la impresió n
de estar divirtiéndose con su propio relato. Por ejemplo,
obsérvese el pasaje en que cuenta có mo don Quijote arregla
sus viejas armas y se prepara una celada de encaje con cartó n
y unas barras de hierro; o el momento en que sale para correr
sus aventuras y va pensando en có mo los cronistas contará n
algú n día su historia (con estos iba ensartando otros
disparates); o cuando ha sido armado caballero y sale de la
venta (el gozo le reventaba por las cin- chas del caballo).
Puede comentarse tambié n la propia ironía de los persona-
jes. Por ejemplo, las palabras de Juan Haldudo cuando se ha
marchado don Quijote y el labrador vuelve a encontrarse a
solas con su criado: Llamad, señor Andrés, ahora —decía el
labrador— al desfacedor de agravios.
También contribuyen a la ironía los frecuentes dobles sentidos
que se aprecian en algunas palabras. Por ejemplo, de Roci-
nante se dice que tenía más cuartos que un real, dá ndole a
la palabra cuartos a la vez el significado de moneda y el de
enfermedad de los caballos. O cuando se habla de las dis-
traídas mozas de la venta, que don Quijote toma por
nobles doncellas, se alude a su ligereza moral.
Por ú ltimo, hay que comentar el doble plano en que se articula
siempre el relato, de manera que se presenta el contraste
entre la visió n del mundo de don Quijote y su correlato
humorístico con los elementos de la realidad tosca y obvia.
El narrador incide con marcado empeñ o en ese contraste, lo
cual supone la clave má s importante para la ironía.
El texto como acto comunicativo
50. Localiza en los seis primeros capítulos pasajes en los que
se pueda apreciar el conocimiento que tienen los perso-
najes sobre las costumbres de los caballeros andantes.
El ventero es un buen ejemplo. Obsérvese el pasaje en que
dice a don Quijote que él había ejercido la caballería
su juventud, y luego explica al protagonista los recursos que so; polimetría; personajes provenientes de las clases sociales
puede poner en prá ctica para no descuidar las cuestiones mate-
riales (dinero, ropa limpia, etc.). Y, por supuesto, la ceremonia
en que don Quijote es armado caballero. Estos pasajes está n
llenos de alusiones a elementos y motivos temá ticos que
solo pueden ser citados por quien conoce los lugares comunes
de la literatura de aventuras y caballeresca. También las
interven- ciones del cura y del barbero (sobre todo del
primero) demues- tran ese conocimiento de las costumbres
de la caballería.

Temas para la reflexión


51. ¿Qué géneros literarios o cinematográficos podríamos
parodiar si hoy día quisiésemos contar una historia como
la de don Quijote? ¿Quiénes serían los protagonistas?
Podríamos parodiar las películas americanas producidas
para la televisió n con personajes y argumentos tó picos
(a veces lacrimó genos, a veces de terror fá cil, a menudo con
historias sentimentales muy manidas) cuyo devenir es
previ- sible desde el primer momento. También podríamos
parodiar las interminables sagas televisivas, que van
complicando de forma inverosímil las tramas de los
personajes a lo largo de muchísimos capítulos, y que han
llegado a constituir el género que popularmente se conoce
como «culebró n». Los pro- tagonistas de las parodias
contemporá neas serían los seres menos idealizados de
nuestra sociedad: por ejemplo, los adoles- centes feos y
marginados; las amas de casa que no participan de la
igualdad de oportunidades; los personajes fracasados en el
trabajo y en los negocios, etcé tera.
52. El Quijote es consecuencia de un momento en el que
se consideran agotados ciertos valores y esquemas no
solo artísticos, sino también sociales. ¿Crees que el
contexto que vivimos en el siglo xxı propiciaría el relato
de una his- toria así? Busca ejemplos de ello en la
literatura contem- poránea o en el cine.
Vivimos tambié n una é poca de crisis, de transformació n de
los valores. En consecuencia, también los arquetipos y los
tó picos de la ficció n quedan cuestionados. Falta dar el paso
de cuestionar la correcció n política, esquema de pensamien-
to que se impone en nuestros días, tanto en lo social como
en lo artístico, y que no puede obviarse fá cilmente sin sufrir
duras críticas o provocar fuertes polémicas. En la literatura y
en el cine contemporá neos a veces encontramos historias que
les dan la vuelta a los arquetipos y presentan protagonistas
antiheroicos. Las novelas de Luis Landero son un buen ejem-
plo (Juegos de la edad tardía, El mágico aprendiz, ademá s
Landero rinde un claro homenaje a Cervantes en buena
parte de su obra). En el cine, se llevan cuestionando muchos
de los modelos desde hace añ os. Sin perdón (Clint Eastwood,
1992) presentaba al típico forajido de los westerns como un
persona- je ya mayor, venido a menos, convertido en padre
de familia y granjero. En la actualidad, películas como
Birdman (Alejan- dro Gonzá lez Iñ á rritu, 2014) nos ofrecen
también un ejemplo de antihé roe cuyas andanzas se
desarrollan en una sociedad sumida en la crisis.

6 Pedro Calderón de la Barca.


La vida es sueño. Escena II de la jornada I:
Rosaura conoce a Segismundo
Localización
1. ¿Cuáles eran las características generales del teatro ba-
rroco?
Mezcla de elementos trá gicos y có micos; utilizació n del ver-
elevadas y de las populares, con estilos lingü ísticos que se Contenido
ajustan a la condició n de cada uno; divisió n de la obra en
6. Antes de escuchar el monólogo de Segismundo, Rosaura
tres actos o jornadas; ruptura de la regla de las tres
unidades. describe la torre. ¿Qué dice acerca de ella?
Que es una prisió n oscura y tenebrosa, sepultura de un cadá -
2. ¿Cómo relacionas el título de la obra y su tema filosófico ver vivo. Que su breve luz la hace má s tenebrosa todavía.
con la concepción barroca de la vida?
Se relaciona con el pesimismo de la é poca, que concibe la
vida como algo inconsistente.
3. Investiga sobre el argumento de la obra y responde.
¿Por qué Rosaura ha emprendido ese viaje? ¿Por qué
Segis- mundo vive encerrado? ¿Quién heredará el trono
de Basilio?
Porque busca a Astolfo, que la ha deshonrado, para reparar
su honra.

Intención
4. Explica cómo se presenta el tema del libre albedrío
en el monólogo de Segismundo que aparece en la
escena que has leído.
En este monó logo se presenta el dolor de Segismundo al verse
en inferioridad con respecto a las criaturas de la naturaleza,
que son libres pese a no tener alma ni albedrío, al
contra- rio que él.
Sueñ a el rey que es rey, y
vive con este engañ o
mandando, disponiendo y
gobernando;
y este aplauso, que recibe
prestado, en el viento
escribe y en cenizas le
convierte
la muerte (¡desdicha fuerte!):
¡que hay quien intente
reinar viendo que ha de
despertar en el sueñ o de la
muerte!
Sueñ a el rico en su
riqueza, que má s cuidados
le ofrece; sueñ a el pobre
que padece su miseria y
su pobreza;
sueñ a el que medrar
empieza, sueñ a el que afana
y pretende, sueñ a el que
agravia y ofende, y en el
mundo, en conclusió n, todos
sueñ an lo que son, aunque
ninguno lo entiende. Yo
sueñ o que estoy aquí, destas
prisiones cargado;
y soñ é que en otro
estado má s lisonjero me
vi.
¿Qué es la vida? Un frenesí.
¿Qué es la vida? Una
ilusió n, una sombra, una
ficció n,
y el mayor bien es pequeñ o;
que toda la vida es sueñ o,
y los sueñ os, sueñ os son.
5. ¿Cuál es el sentido que tiene la existencia humana para
Segismundo? ¿Cómo relacionas estas palabras con la
mentalidad barroca?
No tiene má s sentido que el de un vano sueñ o. Estas
palabras se relacionan con la idea barroca de la
inconsistencia de la existencia humana.
7. Tras este momento, se inicia el monólogo del protago- hay otro que lo es má s.
nista, quien dice que desea saber algo. ¿Qué es eso que
desea saber?
Qué delito ha cometido contra los cielos con el mero hecho
de nacer, pues no ha podido hacer ninguna otra cosa, ya que
lleva toda su vida encerrado.
8. Segismundo entiende que su existencia es un castigo.
¿Por qué cree que ha sido castigado? ¿Piensa que es jus-
ta su situación?
Su nacimiento debe de ser un delito puesto que recibe el cas-
tigo de esa dura prisió n.
9. El protagonista comienza una exposición en la que va
pre- sentando diversos seres vivos y objetos con los que
se compara. Indica de qué seres se trata y qué
comparación establece Segismundo en cada caso.
El ave, que es bella y libre, pero no tiene alma. El bruto, que
tiene el instinto de la crueldad. El pez, que no tiene albedrío.
El arroyo, que tiene el campo abierto para su carrera. Segis-
mundo, en cambio, tiene alma, buen instinto, albedrío, pero
no tiene libertad.
10. Esa comparación suscita invariablemente en Segismundo
una pregunta: explica cuál es.
Qué leyes o argumentos confieren a esos seres má s libertad
que a los hombres.
11. Al terminar su monólogo, el joven se da cuenta de
que alguien le ha escuchado. ¿Cuál es su primer impulso
entonces? ¿Cómo relacionarías ese impulso con la situa-
ción en la que vive el personaje?
El impulso de matar al que le ha oído. Es un impulso ló gico
en un hombre que siempre ha vivido privado de relació n con
otros seres humanos.
12. La presencia de Rosaura modifica el impulso de Segis-
mundo. ¿Qué sentimientos se despiertan en este?
Se enternece y se sobrecoge. Siente fascinació n al mirarla.
13. Para expresar esos sentimientos, Segismundo alude
antes a una serie de circunstancias que podrían
obstaculizar lo que experimenta, encadenando una larga
hilera de oracio- nes concesivas («aunque yo aquí tan
poco del mundo sé…»;
«aunque nunca vi ni hablé…»; «Y aunque en desdichas tan
graves…»). ¿Cuáles son esas circunstancias? Pese a ellas,
¿qué efecto ha conseguido provocar Rosaura en él? ¿Tie-
nen algo de contradictorio estos nuevos sentimientos?
No sabe nada del mundo; siempre ha vivido encerrado en
esa torre como un muerto viviente; nunca ha tenido más
compañ ía que la de un solo un ser humano, que es quien le trae
noticias del mundo; las aves y las bestias han sido sus maestros.
Rosaura le provoca fascinació n y admiració n y mientras má s la
mira, má s desea mirarla. Sí son contradictorios sus
sentimientos: Viendo que el ver me da muerte / estoy
muriendo por ver.
14. También Rosaura explica lo que siente al escuchar el
lamento de Segismundo. ¿Qué dice?
Que ella, siendo muy desdichada, ha conocido en
Segismundo a alguien mucho má s desventurado que ella.
15. En dicha intervención de Rosaura, esta alude a una histo-
ria popular. ¿Cuál es? ¿Qué pretende transmitir Rosaura
al contarla?
Habla de un hombre que se lamentaba de su pobreza porque
solo tenía para comer las yerbas que cogía. Pero al mirar atrá s
vio a otro que comía lo que él iba arrojando. Pretende trans-
mitir que, por muy desafortunado que se sienta uno, siempre
16. Dicha historia puedes relacionarla con uno de los Sí, pues encierra el conflicto que preocupa a Calderó n: la
cuentos de El conde Lucanor: «De lo que aconteció a un pre- destinació n o el libre albedrío.
hombre que por pobreza y mengua de otra vianda
comía altramu- ces». Investiga sobre el contenido de
ese cuento y rela- ciónalo con el relato que hace
Rosaura.
En ese cuento se habla de un hombre que solo tenía
altramu- ces para comer y se lamentaba por ello, hasta que
descubrió a otro que iba comiéndose las cá scaras de los
altramuces que é l tiraba. Es el mismo planteamiento que
tiene la narració n de Rosaura.
17.Al final de la escena, Rosaura se dispone a narrar su his-
toria, pues dice que esta puede tener cierta utilidad
para Segismundo. ¿Cuál es esa utilidad?
Ha comprendido que es consuelo de un desdichado ver que
existe otro que es má s desdichado aú n. Ahora piensa que
sus penas tal vez puedan aliviar en parte a Segismundo de
las suyas.
18. Teniendo en cuenta el argumento global de La vida es
sueño, ¿en cuál de estas dos posturas crees que se sitúa
Calderón? ¿El destino del ser humano está decidido de
antemano por las estrellas o puede este vencer a
cualquier designio con su prudencia?
Calderó n apuesta por la importancia de la prudencia. El
hombre puede decidir su propio destino al elegir entre el
bien y el mal.
19. Los conflictos sentimentales que se han planteado en la
trama quedan resueltos al final, sin transgredir el orden
social de la época. Investiga y aclara cómo se resuelven
dichos conflictos en el desenlace.
Los conflictos sentimentales venían dados por las
diferencias sociales entre Astolfo y Rosaura: él es un príncipe,
y no se siente obligado a cumplir su palabra con ella, porque
ella no conoce padre. Al final se descubre que Rosaura es
hija de Clotaldo y entonces Astolfo se casa con ella. La
infanta Estrella no queda desairada, pues se casa con
Segismundo, que es rey, y en con- secuencia la unió n entre
ambos entra de lleno en el orden social.
Personajes
20. ¿A qué clases sociales pertenecen los diferentes
perso- najes de este drama?
A la nobleza, exceptuando a Clarín.
21. ¿Crees que Rosaura podría responder al tipo de la dama
habitual en el teatro del xvıı? ¿Cómo la caracterizarías
tú, teniendo en cuenta la acción con que se inicia la
obra?
No es lo habitual en la é poca que una mujer se convierta en
artífice de su propio destino. Sin embargo, tuvo éxito en el
siglo XVII cierto modelo de dama hermosa y decidida, al que
responde Rosaura.
22. Investiga sobre los rasgos que definen al tipo del
gracioso y relaciónalos con el personaje de Clarín.
Se convierte en contrapunto de los momentos de tensió n y
dramatismo, introduce la comicidad. Es un personaje
realista y conformista, que no está dispuesto a dar la vida
por nadie, sino a guardarla.
23. Busca también información sobre el resto de los perso-
najes de la obra y relaciónalos con los diversos tipos
que hemos mencionado.
Basilio se identifica con el poderoso, Astolfo con el galá n,
Estrella responde al arquetipo de la dama, Clotaldo es el
viejo.
24. A tu juicio, ¿podríamos caracterizar a Segismundo
como un personaje simbólico? Justifica tu respuesta.
Estructura entre los personajes, hasta casi el final de la escena, es decir,
25. Investiga y explica el argumento de cada una de las jor- entre los versos 173 y 272. En el 273 comienza el esquema
nadas de La vida es sueño. mé trico del romance (versos octosílabos con rima asonante
en los pares), que continuará en la escena siguiente.
Jornada primera: Rosaura, que ha emprendido un viaje para
recuperar su honor, conoce a Segismundo, que vive encerrado Género
y no sabe por qué. Clotaldo reconoce a Rosaura y comprende
que es su hija, pero mantiene aú n el secreto. El rey Basilio 30. A partir de la lectura de la escena II, justifica la adscrip-
revela la existencia de su hijo, Segismundo, a quien encerró ción del texto al género dramático localizando en él sus
para evitar el augurio que dictaba que el heredero habría de elementos.
ser un tirano. Pero se siente culpable y decide darle una La acció n se presenta íntegramente a través del diá logo,
opor- tunidad, pensando que tal vez los augurios fuesen predomina la funció n apelativa del lenguaje, se introduce el
erró neos. Si la prueba sale bien, Segismundo heredará su monó logo, se emplean las acotaciones.
trono y si no, le sucederá Astolfo, que se casará con Estrella.
31. ¿Qué elementos de la tragedia y de la comedia se mez-
Jornada segunda: Segismundo, dormido, es llevado a palacio. clan aquí?
Allí despierta y, desconcertado, se conduce como un salva-
je: intenta forzar a Rosaura (que ha entrado en palacio como De la tragedia, el hecho de que los personajes sean reyes, prín-
doncella de Estrella, ocultando su identidad), se enfrenta con cipes o nobles, la gravedad del tema filosó fico. De la come-
Clotaldo y Astolfo, agrede a un criado. De esta conducta el dia, los elementos populares que vienen introducidos por el
rey Basilio concluye que los augurios eran ciertos y ordena gracioso, Clarín.
dormir de nuevo a Segismundo y devolverlo a la torre. En 32. ¿Cómo se enfocan las tres unidades en La vida es sueño?
cuanto a la acció n secundaria, Astolfo corteja a Estrella, y
No se respetan. Hay dos espacios distintos: la torre y el pala-
se descubre la identidad de Rosaura. Por su parte,
cio. Hay dos acciones: la central (conflicto de Segismundo) y
Segismundo despierta de nuevo en la torre y piensa que
una secundaria, que se superpone a ella (historia de Rosau-
todo lo que ha vivido ha sido un sueñ o.
ra y triá ngulo sentimental entre ella, Astolfo y Estrella). En
Jornada tercera: el pueblo, al saber que existe un príncipe cuanto al tiempo, hay saltos temporales, aunque no abarcan
heredero, se revuelve y libera a Segismundo. Este se enfren- un segmento amplio.
ta a su padre y vence, pero hace gala de prudencia y honor al
decirle a su padre que está dispuesto a aceptar que lo mate,
33. A Calderón le interesaban mucho la escenografía y la tra-
puesto que se ha levantado contra é l. Basilio comprende moya. ¿Qué elementos escenográficos y especialmente
entonces que su hijo es sensato y sabe comportarse como espectaculares citarías en el comienzo de La vida es sue-
un verdadero príncipe. Segismundo establece la unió n entre ño? (Recuerda la entrada de Rosaura en la acción).
Astolfo y Rosaura y, ante las dudas de Astolfo, Clotaldo La obra tiene un arranque espectacular y de complicada pues-
desve- la entonces ante todos que la joven es su hija. El ta en escena: Rosaura va bajando a caballo desde lo alto de
protagonista se promete en matrimonio con Estrella. La obra un monte.
termina con la admiració n de todos por la conducta de
Segismundo, que recupera sus derechos. Estilo
26. ¿Constituye la primera jornada una presentación de los 34. En el monólogo en que Segismundo expresa los senti-
temas, conflictos y personajes de la obra? ¿Por qué? mientos nuevos que le inspira la presencia de Rosaura
Sí, se presentan los conflictos de los personajes, el pasado de (versos 190-242) tenemos un buen ejemplo del estilo
Basilio que ha provocado la situació n de Segismundo. Quedan calderoniano. Localiza y comenta en dicho monólogo la
planteadas las dos acciones de la obra. presencia de los recursos que hemos mencionado.
27. Localiza en ese monólogo las mencionadas partes. Es muy visible la estructura paralelística en ese fragmento,
donde se suceden las oraciones concesivas: aunque yo aquí
En las dos primeras estrofas se presenta el conflicto del per-
tan poco del mundo sé; aunque desde que nací solo advier-
sonaje y su lamento. La argumentació n se desarrolla entre las
to…; aunque nunca vi ni hablé…; Y aunque en desdichas tan
estrofas tres y seis. La séptima constituye la recapitulació n
graves… A su vez, en ese largo periodo sintá ctico hay otros
y la conclusió n.
paralelismos: siendo un esqueleto vivo / siendo un animado
28. Explica cómo se produce la recapitulación final y a qué muerto; la pasión a mis enojos / la suspensión a mis ojos,
conclusión llega Segismundo en ella. / la admiración al oído. Además, encontramos un particular
Segismundo recapitula mencionando todos los seres en los que uso del paralelismo en el retruécano: soy un hombre de las
ha ido deteniéndose anteriormente, aunque en orden inverso a fieras / y una fiera de los hombres.
como los ha mencionado. Los recoge en los dos ú ltimos versos Se juega con el sentido de las palabras y con su reiteració n:
(a un cristal / a un pez, a un bruto y a un ave). La conclusió n, viendo que el ver me da muerte / estoy muriendo por ver.
en realidad, es la pregunta sin respuesta del personaje: en La paradoja puede ejemplificarse en esqueleto vivo y animado
nombre de qué ley todos esos seres gozan del privilegio de la muerto.
libertad, que se les niega a los hombres.
35. ¿Sabes lo que es un retruécano? Infórmate sobre este
29. ¿Qué tipo de versos se emplean en la escena II de la recurso y explica su uso en los versos «soy un hombre de
segun- da jornada? ¿Cuál es la rima? las fieras / y una fiera de los hombres». ¿Qué sentido
En el primer diá logo (entre Clarín y Rosaura, vv. 80-101) se tienen estas palabras de Segismundo?
alternan versos de siete y de once sílabas, que se distribu-
El retruécano es un paralelismo cruzado, en el que ademá s
yen en pareados rimando en consonante. En el monó logo
se repiten las palabras, como en el ejemplo mencionado. La
de Segismundo (vv. 102-172), los versos son octosílabos, se
afir- mació n de Segismundo tiene el sentido de que no
agrupan en décimas (siete en total) cuyo esquema métrico
pertenece a ninguno de los dos mundos. Vive como las fieras,
es abbaaccddc. Continú an las décimas en el posterior
pero es un hombre. No tiene sitio en ninguna parte.
diá logo
36. ¿Qué es la paronomasia? Explícala en el verso: «fuera Ese planteamiento habría resultado provocador y no habría
más que muerte fiera». sido comprendido por la mayoría en el siglo XVII. Hoy día, por
Es la repetició n de palabras semejantes en cuanto a su forma el contrario, lo que resultaría inadmisible sería que un hom-
y su foné tica. En el citado ejemplo existe esa semejanza bre dejase a su prometida por el mero hecho de que no tie-
entre fuera y fiera, y ademá s se reitera el diptongo —ue— en ne padre conocido, para prometerse en matrimonio con una
fuera y muerte. mujer de su posició n social.
37. El verbo morir se ha utilizado desde siempre en la poe- 43. Reflexiona: teniendo en cuenta lo particular que es el
sía con muy diversas connotaciones, especialmente en la géne- ro teatral como acto comunicativo, ¿por qué crees
temática amorosa. Explica el juego de significados con que Cal- derón inserta una trama de este tipo en medio
que se utiliza este verbo y el sustantivo muerte en los de un drama tan grave y filosófico como La vida es
versos 229-242. sueño?
Segismundo, mediante la metá fora de la hidropesía, habla Para que el argumento tenga má s interés para el pú blico.
de có mo sus ojos desean beber esa belleza que, sin embargo, 44. Al hablar del estilo calderoniano, hemos mencionado
le hace enfermar (de amor, de fascinació n). El verbo morir el gusto de este escritor por la artificiosidad ingeniosa
se emplea con el sentido del amor corté s: sufrir por lo del lenguaje. ¿Crees que este rasgo contribuye a la bue-
inalcan- zable que, a la vez, provoca deleite. Con la palabra
na acogida de la obra entre el público o que la dificulta?
muerte se refiere al afá n que experimenta ante la
contemplació n de la dama y al dolor que provoca su
Razona tu respuesta.
ausencia, un dolor que sería mucho peor que la muerte, Seguramente contribuye a la buena acogida, porque al pú blico
ahora en sentido literal. le deslumbran los usos ingeniosos y los há biles giros del estilo
calderoniano. El estilo de Calderó n, por artificioso que sea,
38. Comenta los elementos culteranos que aparecen en la no es enrevesado ni hermé tico.
intervención de Rosaura al comienzo de la escena (versos 45. También hemos hablado del gusto de Calderón por los
85-90). monólogos. ¿Qué función comunicativa tienen estos en
Obsérvense los adjetivos (explicativos en su mayoría): breve, el conjunto de una obra de teatro? ¿Y en La vida es
caduca, pálida, trémulos, oscura, que inciden en el efecto sueño, concretamente?
sensorial de la luz que se describe. Por otro lado, el enuncia-
Transmitirnos los pensamientos y sentimientos de un perso-
do que abarcan esos versos está estructurado en hipérbaton.
naje, o su historia y sus circunstancias. En La vida es sueño,
39. Localiza los hipérbatos que aparecen en el fragmento los monó logos tienen varias funciones: las dos que acabamos
que has leído. de mencionar, y tambié n la de exponer reflexiones didá cticas
Ademá s del que se produce en los versos mencionados en la sobre las ideas de la obra.
actividad anterior, hay hipérbatos en los versos 94, 97, 98, 99, Temas para la reflexión
103, 106, 135, 137-138, 157-158, 168, 173-174, 183-185, 200, 205,
206-207, 214-218, 224, 226, 250-251, 263-264, 268, 271-272. 46. En la experiencia del siglo xxı y en la sociedad en la que
vivimos, ¿cómo responderíamos a la gran pregunta sobre
40. Indica con qué reiteración sella el personaje cada una de el destino del ser humano? Es decir, ¿somos libres para
esas razones. Explica qué figura retórica encontramos en elegir entre el bien y el mal o estamos predestinados?
esa reiteración. En el siglo XXI prevalece la idea de la libertad para elegir
¿Y teniendo yo más alma, / tengo menos libertad? […] ¿Y yo, entre el bien y el mal.
con mejor instinto, / tengo menos libertad?) […] ¿Y yo, con
47. En la obra de Calderón, el mensaje final es que la
más albedrío, / tengo menos libertad? […] ¿Y teniendo
prudencia debe imponerse a los instintos y a la pasión.
yo más vida / tengo menos libertad? Son interrogaciones
retó ricas.
¿Compartes esta idea? Redacta un texto expositivo sobre
este particu- lar, en el que expongas tus opiniones.
41. ¿Qué elementos lingüísticos dan al monólogo el tono de
Respuesta libre.
lamento?
48. ¿Se corresponde esa caracterización del personaje con la
Las exclamaciones, la interjecció n ¡ay!, la interrogació n retó -
conducta ideal de la mujer en el siglo xvıı? ¿Y en nuestra
rica, el léxico que connota compasió n (mísero, infelice, des-
velos, castigo). época? ¿Supone alguna novedad que una mujer se con-
duzca de esta forma?
El texto como acto comunicativo No era lo habitual en el siglo XVII. Normalmente, eran los
42. ¿Crees que el espectador de aquella época habría aco- padres, maridos o hermanos los que reparaban el honor de
gido una obra en la que Astolfo se casase con una joven las mujeres cuando había alguna afrenta, por considerarlo
que no perteneciese a la nobleza? ¿Cómo contrasta esto parte de su propio honor. En nuestra época, por el contrario,
no es ninguna novedad que una mujer resuelva por sí misma
con la mentalidad de hoy?
sus propios problemas.

También podría gustarte